Sei sulla pagina 1di 330

Fran^ois Guéry, Didier Deleule

yPierre Osmo

COMENTARIO DE TEXTOS
DE FILOSOFÍA
Colección Teorema
Fran^ois Guéry - Didier Deleule
Pierre Osmo

Comentario de textos
de filosofía

Con un anexo de textos de filosofía española,


por Carmen García Trevijano

SEGUNDA EDICIÓN

CÁTEDRA
TEOREMA
Título original de la obra:
Le commentarie de textes de pbiiosophíe

Traducción: Elisa Lucena, quien también ha realizado


el comentario 36 sobre Wittgenstein

Reservados ledos los derechos. De conformidad con lo dispuesto


en el art. 534-bis de Código Penal vigente, podrán ser castigados
con penas de multa y privación de libertad quienes reprodujeren
o plagiaren, en todo o en parte, una obra literaria, artística
o científica fijada en cualquier tipo de soporte
sin la preceptiva autorización.

© 1990 by Editíons Nathan, París


Ediciones Cátedra, S. A., 1995
Juan Ignacio Luca de Tena, 1.3. 28027 Madrid
Depósito legal: M. 39.622/1995
ISBN: 84-376-1170-9
Printed in Spain
Impreso en Fernández Ciudad, S. L.
Catalina Suárez, 19.28007 Madrid
4

índice

INTRODUCCIÓN 11
TEXTOS Y COMENTARIOS 23
1. PUITÓN: En el principio, ¿fue la Causa de todas las
cosas? 25
2. PLATÚN: ¿Cuá) es la vida mejor? 31
3. PLATÓN: ¿Ser y / o no ser? 37
4 . ARISTÓTELES: Sobre Platón, que tiene ideas pere-
grinas 44
5 . ARISTÓTELES: Que la Ciudad de Platón es demasia-
do ideal 50
6. DESCARTES: Que es una cuestión de principios de-
sembarazarse de la Escuela 57
7. DESCARTES: Que no nos podemos contentar con
lo que dicen los antiguos para ser dichosos 64
8. DESCARTES: Mens sana in corpore sano 71
9. SPINOZA: Sobre Descartes, quien ha errado sin em-
bargo en sus principios 76
10. SPINOZA: Sobre cómo los filósofos y los políticos
conciben la naturaleza humana
11. LEIBNIZ: Que Descartes se ha equivocado al tratar el
movimiento 90
12. LEIBNIZ: Que Hobbes y Spinoza han formado la ne-
cesidad 97
13. LOCKE: Que no hay ni ideas innatas ni consenti-
miento universal 105
14. HUME: Que la sociedad suple las deficiencias del in
dividuo aislado 112

7
15. ROUSSEAU: Sobre Hobbes, quien ha falseado la pers-
pectiva del derecho 119
16. KANT: Que el problema de la causalidad determina
el tipo de metafísica 126
17. KA NT: Sobre Leíbniz, que no podia defender la
causa de Dios 134
18. KANT: Que Rousseau da mucho que pensar sobre el
porvenir del hombre 142
19. HEGEL: Que la Crítica de Kant no ha logrado tras-
pasar los límites 151
20. HEGEL: Que el Estado de Rousseau y Fichte resulta
demasiado abstracto 159
21. HEGEL: Realidad del derecho, racionalidad de la
filosofía 167
22. COMTE: Que es necesario encadenar A las ciencias
para dirigir el orden social 175
2 3 . SCHOPENHAUER: Que la metafísica es la compren-
sión del mundo 183
24. MARX: Átomo y libertad 187
2 5 . MARX: EL idealismo y Alemania 193
26. KJEKKEGAARD: El saber y el pecado 199
27. NIETZSCHE: Que el ateísmo filosófico abre la cues-
tión del sentido de la existencia 204
28. NIETZSCHE: Que el filósofo y el artista son tropie-
zos de la naturaleza 208
29. FREUD: El psicoanálisis frente a la tradición filo*
sófica y psicológica 212
30. SAUSSURE: Filosofía de la expresividad o ciencia de
los signos 219
31. DURKHEIM: Que el determinismo social garantiza
la eficacia sociológica 226
32. WEBEK: Que las singularidades de la historia re-
quieren tipos ideales 234
33. BBRGSONT Que la vida debe dar precisión a la fi-
losofía 242
34. HUSSERL: Que la ciencia de la naturaleza es ciencia
del espíritu 246
35. HUSSERL: Producciones filosóficas y producciones
tradicionales 250

8
36. WITTGENSTEIN: Sobre las proposiciones de la ética,
que carecen de sentido 255
37. HEIDEGGER: Presencia del presente en el pasado,
ausencia de la presencia en el presente 266
38. HEIDEGGER: ¿Omnipresencia o ausencia del pro-
pio hombre? 271
39. SARTRE: Saber antropológico y comunidad humana . 277
4 0 . LÉVI-STRAUSS: Una diversidad de pueblos sin his-
toria 282
41. PÍAGET: Duración infantil y tiempo espacial 288
42. FOUCAULT: Ciencia del hombre o finitud 292

APUNTES BIOGRÁFICOS 299


BIBLIOGRAFÍA 307
ANEXO. COMENTARIOS DE TEXTOS DE FILOSOFÍA ESPA-
ÑOLA 311
1. SÉNECA: El valor del tiempo 313
2. VIVES: El pacifismo 315
3. SUÁREZ: La guerra justa 318
4. SANTAYANA: La esencia de la poesía 322
5. UNAMUNO: La ¡ntra-historia 326
6. ORTEGA: Teoría de la vida: el yo y la circunstancia. 328

9
Introducción

i
La filosofía tiene una historia. Esta historia está viva y
se asienta sobre un proyecto común, la búsqueda de un
acuerdo entre los espíritus entendido como condición de
posibilidad de la paz en la Ciudad. Podremos convenir en
que tal proyecto constituye una apuesta y que es quizás en
ello donde reside la especificidad de la empresa filosófica.
En este sentido, la filosofía es política de parte a parte, no
porque se encuentre sujeta por fuerza a una ideología o
porque sea la expresión de una opción partidista, sino
más bien porque el tema del «vivir en común» se halla en
el corazón de sus preocupaciones.
En consecuencia, es bastante natural que esta inten-
ción engendre, en su realización proyectada, divergen-
cias; es decir, antagonismos, relacionados con la elección
de los medios apropiados para alcanzar el fin deseado.
Las querellas de las Escuelas, los enfrentamientos entre
los sistemas, en ocasiones incluso desgarramientos, son
otras tantas manifestaciones de la vitalidad de la discipli-
na. La ambición de la presente compilación es la de sugerir
las múltiples direcciones en las que se bifurca la filosofía,
de modo que esta historia es también nuestra historia.
El principio que ha dirigido la elección de los textos ha
sido, pues, el de la oposición o, al menos, el de la discu-
sión. Cada texto, a su manera, entra en polémica explícita
o implícita con un filósofo o una corriente de pensamien-
to anterior o contemporánea. Es así que desde el primer
texto (Platón, crítica de Anaxágoras) hasta el último
(Foucault, crítica de) humanismo formal) se establece el
cauce de la trayectoria de una reflexión acerca de los fun-
damentos del conocimiento de la acción. Pero aún es ne-

13
cesario precisar que la presentación cronológica no co-
mulga con ningún prejuicio historicista. Lejos de ser acu-
mulativos, los argumentos elaborados y desarrollados por
cada filósofo dan testimonio, por encima de todo, de la
presencia de un diálogo (aun cuando se encuentre inscri-
to en el registro de la polémica) que se persigue sin cesar y
que, por ser inagotable, puede no terminar nunca. Diálo-
go con el pasado, desde luego, pero también diálogo
transversal. Puesto que Locke, por ejemplo (texto 13), re-
chaza el consentimiento universal, pasa revista tanto a
ciertos autores de su tiempo (insertos en herencias dife-
rentes) como a la doctrina estoica. Del mismo modo, des-
de el momento que Marx (texto 24) escogió a Epicuro
contra Demócrito, podemos suponer que ni Leibniz ni
Spinoza escapan totalmente al debate. Si es verdad que la
transmisión de un saber, sea cual fuere, está destinada a
servir de alimento a la reflexión, esto es, a la posible de-
marcación, a la eventual reacción, a retomar la cuestión
inicial con nuevos aires y realizar incluso una especie de
nueva revolución intelectual, la referencia al pasado pró-
ximo o lejano no sabría ser un mero respeto fijado en la
hagiografía o en la sempiterna repetición de un único dis-
curso. Está abocada a la profun d izad ón, a la refundición
o a la franca ruptura. Entre filiación y «parricidio» se di-
buja un paisaje que oscila entre el modelo de un jardin «a
la» inglesa, un desorden aparente finalmente bien dis-
puesto, pensado, y el de un jardín «a la» francesa, con la
poda de los setos regulada y geométricamente ordenada,
donde fa razón puede exhibir su potencia. Pero con inde-
pendencia del punto de vista que se adopte, siempre serán
la fuerza y la plasticidad del pensamiento los que se afir-
man y depuran a través del arte de una controversia re-
sueltamente manifiesta o discretamente desarrollada. En
suma, si hay tradición filosófica, ésta no puede ser otra
cosa que critica.
Y a h emos señalado que esta compilación pretende ser
sugerente, lo que desde liiego le prohibe aspirar a una ilu-
soria exhaustividad. Dos ejemplos; ni Epicuro ni los es-
toicos se hallan presentes textualmente hablando, pero el

14
texto 7 (de Descartes) hace referencia directa a estas dos
escuelas helenísticas desde un punto de vista crítico, y el
texto 13 (de Locke) se dirige indirectamente a la doctrina
estoica del consentimiento universal. Asimismo, Hobbes
está ausente en apariencia, pero los comentarios de los
testos 12 (de Leibniz) y 15 (de Rousseau) permiten rees-
tablecer la importancia del filósofo, así como su influen-
cia. El lector podrá, en la medida de sus posibilidades, re-
parar otras ausencias-presencias de este tipo.
Esta compilación querría ser sugerente también de
otra forma. Los comentarios propuestos se orientan fun-
damentalmente a captar de nuevo la atención de un texto
y a restituir su movimiento argumentativo. En cierto
sentido, éstos se asemejan más bien a planos desarrolla-
dos cuya vocación explicativa se destina a prestar ayuda a
los estudiantes, a servirles de guía paso a paso, sin preten-
der, en ningún caso, sustituir a su sagacidad ni a su pers-
picacia, como tampoco, por otra parte, a su talento para
la retórica. Provistos de su equipaje intelectual, alimenta-
dos con su propia cultura, fecundados por su inspiración,
cada cual podrá, sin prisas, adentrarse en los textos de la
mano de las directrices, suficientemente precisas, para así
evitar equivocaciones o contrasentidos, que hemos indi-
cado aquí métodicarnente.
Lo que queremos decir es que esta obra requiere la co-
laboración de los estudiantes. La labor realizada no es ex-
haustiva. Sin embargo, y en la medida que los puntos
esenciales aparecen señalados en cada momento y las eta-
pas de cada razonamiento fuertemente marcadas, la ruta
debería haber quedado despojada de escollos con un rigor
tal que ahorrara al lector el riesgo de cometer errores. En
sentido estricto no se trata, por tanto, de un manual, sino
más bien — e n consonancia con el título de la colec-
c i ó n — de un ABC, la serie albafética antes de ser dele-
treada por los usuarios, siempre desde una deseada buena
inteligencia con los autores. En pocas palabras, un ins-
trumento de trabajo.

15
E L MODO D E E M P L E O

¿Cómo utilizar este instrumento de trabajo? Nosotros


sugerimos los pasos siguientes:

1." Después de una lectura completa, lenta, reflexio-


nada y repetida del texto por comentar, convendría esbo-
zar, a título de ejercicio y por escrito, una primera tentati-
va de explicación con los medios de los que se disponga,
esforzándose por captar la intención que subyace al texto
y reconstruir los diferentes argumentos que permiten
arrojar las conclusiones. Este primer ejercicio de lectura
razonada no exige más requisito que un mínimo esfuerzo
de reflexión o, simplemente, un poco de «juicio».
En efecto, ¿qué quiere mostrar el autor? ¿Cómo en-
tiende él que debe mostrarlo? Y todo ello con indepen-
dencia de las alusiones o sobreentendidos que impregnan
toda obra filosófica sin ser, no obstante, absolutamente
imprescindible, al menos en un primer momento, entrar
en sus pormenores.
2.° Una vez cumplida esta labor preliminar, el lector
estará ya en disposición de leer el comentario propuesto en
su totalidad y confrontar las diferentes etapas en que se
desarrolla con las notas metódicamente redactadas y or-
denadas en el cutso de la primera parte de la operación.
Desde este momento, será posible completar el esbozo de
explicación mediante las indicaciones del comentador al
prolongar tal análisis, profundizar en el sentido de tal
ejemplo o delimitar con mayor precisión el alcance de
tal otra imagen o comparación cuya función sea la de
apuntalar un determinado argumento. Para llevar a cabo
esta tarea se permite el «visualizar» los diferentes elemen-
tos que entran en juego y recurrir a diferentes colores que
podrán repartirse, según los gustos, entre los propósitos,
los argumentos, las imágenes las comparaciones de los
textos.
3." Cumplido lo anterior es posible, además, buscar
apoyo en las notas del comentario e imponerse el lector las

16
lecturas complementarias (textos paralelos del mismo au-
tor, textos explicativos, artículos, monografías) que clari-
fican el texto y permiten situarlo mejor dentro de la tradi-
ción filosófica. Ni que decir tiene que, desde una perspec-
tiva a más largo plazo, es recomendable, en ciertos casos
indispensable, la lectura de la totalidad de la obra de la
que se haya extraído el texto por comentar,
4.° Puede darse el caso, a pesar de lo que venimos di-
ciendo, de que uno se encuentre relativamente desampa-
rado ante la presencia de un autor cuyo proyecto filosofi-
co se ignota por completo. De ocurrir esto, es convenien-
te hacerse con una visión, por así decirlo, panorámica de
la obra mediante la consulta de alguna Historia de la Filo-
sofía, algunas de las cuales son excelentes. Señalamos a tí-
tulo indicativo:

a) Histoire de la philosophk, París, Gallimard, Bibliothé-


que de la Pléiade, en tres volúmenes. (Traducción es-
pañola, publicada en México, siglo xxi, 1974, con el tí-
tulo de Historia de la filosofía):
Tomo I (Oriente, Antigüedad, Edad Media), dir.
B. Parain, 1969.
Tomo II (del Renacimiento a la revolución kantiana),
dir. Y. Belaval, 1973.
Tomo III (de) siglo xix a nuestros días), dir. Y. Bela-
val, 1974.
b) E. Bréhier, Histoire de la pbilosopbie, en 7 fascículos y 2
fascículos suplementarios (La filosofía en Oriente, de
P. Masson-Oursel, y Lafilosofía bizantina, de B. Tatakis),
París, Alean, 1926-1932, regularmente reeditados por
la editorial PUF; última reedición PUF «Cuadriga», en
tres volúmenes, sin los dos fascículos suplementarios
(Traducción española, Historia de la filosofa, Buenos Ai-
res, Ed. Sudamericana, 1944-56. Editada en 3 volúme-
nes y con prólogo de Ortega y Gasset).
c) Histoire de la pbilosopbie: idees, doctrines, bajo la dirección
de F. Chátelet, París, Hachette, ocho volúmenes,
1972-1973. Nueva edición abreviada y puesta al día

17
bajo el título de La filosofía, en 4 volúmenes, colección
Marabount-Université, 1979, (En castellano existe una
versión traducida con el título original de Historia de la
filosofía: ideas, doctrinas, publicada en 4 volúmenes en
Madrid, Espasa-Calpe, 1976.)

5.° Una vez cumplidas todas Jas fases del procedimien-


to señalado, lo ideal sería que el estudiante regresara al
propio texto para esforzarse, en esta ocasión, por pro-
veerse de un comentario compuesto, íntegramente redactado,
en el que hará uso de todos los conocimientos adquiridos
a lo largo de lecturas anteriores consignadas previamente
en una serie de fichas metódicamente confeccionadas.
Este trayecto-retorno debería permitir calibrar el progre-
so alcanzado tanto en la apreciación del contexto como
en la agudeza de la lectura.

¡Qué se tranquilicen nuestros lectores! A partir de


unos cuantos preceptos metodológicos de base que todo
enseñante tiene la costumbre de comunicar a sus alum-
nos (situar el texto, extraer las ideas principales del mis-
mo, indicar sus articulaciones argumentativas, prestar
atención —en el análisis de los detalles— a las imágenes,
a los ejemplos y a las comparaciones, subrayar las dificul-
tades, delimitar con exactitud el propósito o intención
general y sus implicaciones), cada cual posee una manera
particular de comentar, pues no existe algo así como un
comentario-tipo o canónico. La riqueza de un texto es, en
cierto sentido, inagotable, porque induce a lecturas varia-
das, siempre renovadas de acuerdo con la época y la idio-
sincrasia del comentador. Pero también, y al mismo
tiempo, tales lecturas son coherentes y legítimas en tanto
que se respeten los principios elementales que acabamos
de recordar al lector.
Podríamos, en el momento oportuno, haber emulado
el arte del comentario de la práctica de la exégesis bíblica
y sacar a la luz, desde la pasión irrefrenable y nunca satis-
fecha de poner al día el verbo divino, un presupuesto in-
herente a nuestra herencia cultural. Sin necesidad de vol-

18
ver a cuestionar la eventual pertinencia de la compara-
ción, deberíamos, a! menos, recordar que la fuerza crítica
de la exégesis —aquella que aparece reflejada en el Trata-
do teológko-polítko (1670) de Spinoza o en la Historia critUa
del Antiguo Testamento (1678) de Richard Simón (1638-
1712)— representa una conquista, que en ocasiones ha
costado muy cara, sobre la mera autoridad de la Escritu-
ra, a la que, por definición, repugna toda exploración y
nunca ha sufrido discusión alguna.
Comentar no es, en ningún caso, una empresa comple-
tamente inocente, puesto que consiste en clarificar un
texto, en proyectar plena luz sobre su sentido y su alcan-
ce. Pero las constricciones aparentemente formales que
sostienen a la técnica ordinaria del comentario, a la vez
que ofrece/i la garantía de un ejercicio crítico, constitu-
yen el marco dentro del cual se desarrolla, contra todo ar-
gumento de autoridad, la posibilidad de un pensamiento
líbre de ataduras.

EL USUARIO-TIPO

La presente compilación está especialmente destinada


a los estudiantes universitarios de primer ciclo (sobre
todo de filosofía), así como a los alumnos que siguen las
clases preparatorias de COU. A pesar de todo, y sea cual
sea el público a quien concierne esta obra, es conveniente
señalar que el arte del comentario de texto no conoce
modificaciones reguladas en función del nivel de los es-
tudios.
En principio, tanto si se trata de un alumno de bachi-
llerato como de un candidato a la docencia (CAP, Agre-
gaduría), las exigencias son las mismas. Podremos tomar
en consideración solamente que eJ aprendizaje continua-
do permite, de hecho, progresar en la calidad de la lectura
de los textos, en parte debido a que la cultura adquirida
facilita la identificación rápida de, pongamos el caso, al-
guna alusión hasta entonces enigmática, inasible. Y en
parte también porque la repetición del ejercicio tiende a

19
perfeccionar esa agilidad de espíritu que sabe dar en el
blanco, ir a lo esencial del propósito del texto sin caer en
las redes de lo accesorio, o sin dejarse envolver por la se-
ducción de la palabra inductora por la cual el texto se
convierte en pretexto para comenzar a hablar de otra
cosa...
¿Gimo podría fallar su tiro el arquero cuando el blan- >
co es una puerta?, inquiría Aristóteles. Variante trivial:
¿cómo no acertar cuando la diana es una vaca en el fondo
de un estrecho corredor? En efecto, el arquero tiene todas
las posibilidades de hincar su flecha en una confortable
superficie de madera pero le será mucho más difícil al-
canzar el centro de un objeto colgado de esa misma super-
ficie. De igual modo, la vaca no escapará a la intención
asesina de quien la persigue, pero atravesarle el corazón
en lugar de la panza o el lomo es harina de otro costal.
Aquí, como en cualquier otro lugar, para dar de lleno en
el blanco es indispensable el entrenamiento. Nuestra úni-
ca ambición es la de contribuir a mejorar la ejecución, te-
niendo siempre presente en el ánimo que la puntería, en
sí misma, no es menos importante que el resultado conta-
bilizado, que lo anterior se denomina placer de la lectura,
aunque sin perder la vista que tal placer se ve notable-
mente acrecentado desde el momento en que el lector ha
debido hacerse con los medios para apropiarse, al menos
en parte, de las riquezas de un texto. De este modo es
como el lector, en el verdadero sentido del término, se
enriquece.
Añadiremos que en lo que venimos diciendo reside
también el interés del ejercicio. La exigencia de un méto-
do a la hora de abordar el estudio de un sistema de pensa-
miento no excluye, sino más bien al contrario, ese aspee- ,,
to de sueño e imaginación sin el cual no existirían más
que palabras sin vida. Comentar un texto consiste en des-
lindar lo que en él hay de vida, para mayor beneficio del
autor y del lector.
Dirigido a todos pero quizás de mayor utilidad para la
labor más específica de los estudiantes de COU, quienes
por medio de un sistema de «pasarelas» y de «puesta a ni-

20
vel», pueden, si lo desean, seguir los estudios especializa-
dos de filosofía y así reunirse con sus camaradas que ya
cursan la licenciatura. Para ellos nos hemos cuidado de
incluir en nuestra compilación comentarios de textos que
se interesan más directamente por las relaciones entre la
filosofía y ías ciencias humanas (cfr., sobre todo, los tex-
tos 22, 29, 30, 31, 32, 38, 39, 40, 41).
Nuestra elección se ha orientado, principal pero no ex-
clusivamente, hacia textos que podríamos considerar
fundadores en la medida en que pretenden (ya se trate de
la psicología, del psicoanálisis, de la lingüística, de la so-
ciología o de la antropología) marcar una ruptura, apa-
rente o discreta, con la herencia filosófica. En cualquier
caso, de acuerdo con el principio propuesto y evocado al
comienzo de la introducción, nuestra meta es la de poner
de manifiesto la continuidad de un debate que, a despe-
cho de las veleidades y estrategias de ruptura, plantea con
nuevos aires —en el registro modulado de nuevas disci-
plinas que anhelan la independencia— un cierto número
de cuestiones perfectamente integradas ya en la tradición,
inaugurando con ello un diálogo a menudo polémico,
pero siempre fecundo, entre la filosofía y las ciencias hu-
manas.

21
Textos y comentarios
1. Platón

E N E L P R I N C I P I O , ¿ F U E LA CAUSA
D E T O D A S LAS COSAS?

Ahora bien, he aquí que un día oí hablar a alguien


mientras leía dé un libro, de Anaxágoras, según dijo, en
los términos siguientes: «En definitiva, es la mente la que
pone todo en orden y la causa de todas las cosas.» Una
causa tal me llenó de regocijo, pues me parecía que, en
cierto sentido, era una ventaja que fuera la menté la causa
de todas las cosas. Si esto era así, pensé, esa mente orde-
nadora, que justamente realiza el orden universal, dis-
pondría cada cosa en particular allí donde mejor estuvie-
ra. Así, pues, sí alguno quería descubrir la causa de cada
una de las cosas, según la cual nace, perece o existe, debía
encontrar, a tal efecto, cómo es mejor para ella existir, pa-
decer o producir la acción que fuere (...) ¡Y bien!, ¡adiós
mi maravillosa esperanza, oh compañero! Mientras avan-
zaba, en efecto, en la lectura, vi que mi hombre no usaba
para nada la mente, ni le imputaba parte alguna entre las
causas particulares del orden de las cosas, sino que, por el
contrario, asignaba las causas al aire, al éter, al agua y a
otras muchas explicaciones desconcertantes. Su caso, a
mi entender, era algo sumamente parecido al de alguien
que, tras haber dicho que todo lo que Sócrates hace lo
hace con la mente, intentara a continuación enumerar las
causas de cada uno de mis actos afirmando en primer lu-

25
gar que estoy sentado en este lugar porque mi cuerpo se
compone de huesos y de músculos; que los huesos son du-
ros y poseen articulaciones que los separan los unos de los
otros, en tanto que los músculos tienen la propiedad de
tensarse y relajarse, y cubren a los huesos juntamente con
la carne y la piel que los mantiene unidos; que, en conse-
cuencia, la oscilación de los huesos en sus coyunturas, la ¿l
distensión y la tensión de los músculos hacen que sea yo
capaz ahora, por ejemplo, de flexionar los miembros.
¡Esa es la causa en virtud de la cual yo estoy aquí sentado
con las piernas dobladas! (...) Dar el nombre de causas a
cosas semejantes es excesivamente extraño. Al contrario,
si alguno dijera que sin la posesión de huesos, de múscu-
los y de todo lo demás que tengo, no sería capaz de llevar
mis decisiones a la práctica, bien, diría la verdad. Pero
decir que es a causa de esto por lo que hago lo que hago, y
que al hacerlo obro con la mente y no en virtud de la
elección de lo mejor, sería quizás tomarse demasiadas li-
bertades con el lenguaje. Como lo es el no poder distin-
guir que una cosa es, en efecto, la causa real de algo, y
otra aquello sin lo cual la causa jamás llegaría a ser causa.
Y esto, a mi parecer, es a lo que la mayoría, andando
a tientas como en las tinieblas, designa, mediante un
término cuyo empleo es impropio, como siendo una causa.

Fedón, 97 b-c, 98 b-d, 99 a-b.

COMENTARIO

1. No sabríamos apreciar plenamente este extracto


del Fedón de Platón sin delimitar, en pocas palabras, su
contexto y las circuosiancias que evoca:
"a) "TEil relato de Sócrates (Ahora bien, he aquí que un
día...1 tiene lugar en una discusión cuyo objeto constituye

1 Con el objeto de facilitar la inmediata localización de ciertos párra-

fos del texto, en los comentarios aparecerán escritos con los mismos ca-

26
sin duda el motivo más constante y universal de la in-
quietud humana: ¿existe algo asi como un alma inmortal
que sobreviva a la muerte del cuerpo?
b) La discusión mencionada se desenvuelve durante
las horas precedentes a la muerte de Sócrates, condenado
a beber cicuta por el crimen de impiedad de que es acusa-
do por el Tribunal de Atenas. Así, las cuestiones filosófi-
cas que aborda el relato de Sócrates, aparentemente aje-
nas al contexto e indiferentes a las circunstancias, adquie-
ren todo su sentido en el momento crucial en el cual son
planteadas. Tales cuestiones no pueden desligarse de la
vida de un hombre que se enfrenta a la muerte como filó-
sofo, en presencia de sus compañeros y discípulos, ni de
la historia de una ciudad que hace uso de su derecho de
vida y de muerte al encuentro de una filosofía que la pone
en cuestión. Pero también, la vida de la filosofía que ilustra
el texto presente es inseparable de la historia de la filosofía
que el autor hace entrar en escena. Sócrates necesita para
razonar de un «antesocrático», Anaxágoras. La actividad
filosófica se nutre de la continua referencia al pasado que
la anima. Ella se efectúa, con la mediación de libros que
actualizan sin cesar las sucesivas lecturas, en el seno de una
auténtica tradición: una transmisión de pensamientos o de
problemas fundamentales sólo se conserva al renovarse,
al mismo tiempo que a través de un diálogo (consigo mis-
mo o con sus interlocutores) donde es posible poner a
prueba las contradicciones observadas.
2. La estructura v.Jmp^gjffligq¡to del texto se pueden
reestablecer Sn dificultad. A un primer «momento» (...la
acción que fuere), donde se expresa una esperan^, la que Só-
crates pusiera a prueba en el enunciado de la tesis de Ana-
xágoras —esperanza de llegar finalmente a la explicación
última de todas las cosas—, le sigue un segundo «mo-
mento» (...sentado con las piernas dobladas1.) que constata una
decepción, la de descubrir que Anaxágoras reduce su explica-
ción a causas materiales que guardan un silencio irrisorio

racteres que en el extracto original todas las palabras o fragmentos que


de él procedan.

27
sobre lo esencial. Un tercer «momento» (hasta el final del
texto) pone su esfuerzo, aparte de las reacciones subjeti-
vas de Sócrates, en proponer la objetividad de un punto
de vista critico acerca de aquello que sea causa (la propie-
dad de los términos y la pertinencia de una distinción, si
se trata de clarificar lo que en un principio aborda el pen-
samiento de forma algo oscura). Desde esta perspectiva,
el sujeto singular del relato (yo, Sócrates) ambiciona
constituirse en el sujeto impersonal de una experiencia
universal. Y el ejemplo que se ofrece, donde Sócrates se
toma a sí mismo como objeto para poner de relieve lo ab-
surdo de una acción y de una situación conducidas al te-
rreno de la puta mecánica y la sola posición de un cuer-
po, pone también sus miras en el establecimiento de la
materia de esta experiencia. Tales aspectos, entre otros,
son característicos de la ironía socrática, consistente
en hacer surgir la posibilidad de un auténtico discurso
verdadero al desvelar la insostenible ingenuidad de la expe-
riencia.
3. Analizaremos, a continuación, con mayor detalle,
lo que constituye fl flfrn d L ' T U s r é f f v l r > En primer
lugar, pasaremos a considerar ja^ pfffffllffífci'SPí;'» f p T ' -
fícameg&iUosóficas que allí aparecen reflejadas. En se-
gundo lugar, expondremos aquello que, en respuesta a lo
anterior, configura el esbozo de la^so^iaciÓD propiamente
platónica que el texto presenta.
"a) La experiencia universal de la que hablábamos no
es otra que la que cualquier individuo puede constatar
por sí mismo; esto es, que las cosas nacen, perecen, exis-
ten. Dicho de otto modo, tenemos el ser y el cambio, un
movimiento de generación y de corrupción (de ahí el
problema de la inmortalidad del alma). Y en conocer sus
causas consiste, precisamente, el problema de la física, dis-
ciplina que, en el origen de la filosofía, no se diferenciaba
de esta última. Saber es poder explicar el cómo y quizás, so-
bre todo, comprender el por qué de la acción o de la pasión de
cada cosa. El filósofo (físico) es el hombre de! sentido, noción que
el texto incita a definir: ¿acaso no se trata de concebir a
cada cosa según su disposición (su lugar y su destino de He-

28
gada —única manera de fijarla, de apren henderla, en
suma, en su «movimiento») en un orden que le confiere su
perfección, concibiéndose a sí mismo como a un universal
donde se unen (¿o unifican?) las diferentes particularidades
de las cosas? Ello exige un logas, un lenguaje que sea entera-
mente razó», es decir, presencia siempre activa del lazo
formado entre el universal y el particular.
Si las cosas no son inteligibles por su(s) causa(s), sólo
una mente que todo lo cause podrá hacer inteligible desde sí
misma la causalidad. De hecho, Platón inaugura su pro-
pio sistema filosófico llevando a sus últimas consecuen-
cias este pensamiento de Anaxágoras, objetivo que logra
radicalizándolo. Sólo una mente (en griego, un noús; una
inteligencia capaz de elegir y proyectar con absoluta sabi-
duría) puede querer el orden y producir el principio ade-
cuado para ordenar todas las cosas; a saber, el principio de lo
mejor. Asi se anuncia lo que más tarde aparecerá en la Re-
pública y las Leyes, si bien ya purificado de sus aspectos
«psicológicos». Nos referimos a la forma rigurosa de la
idea de Bien1. En el fondo, ¿no nos está invitando Platón a
meditar al sugerir que una cosa se comprende verdadera-
mente a sí misma siempre que esté justificada? Y, ¿qué
puede ser «la causa real de algo», sino su razam de ser? Y, de
nuevo, ¿qué es lo que da razón de una existencia, acción o
pasión, sino su fin? Y , por último, ¿qué es aquello que jus-
tifica todo fin, sino el Bien? En cualquier estado de causas
el requisito mínimo para un filósofo, en consecuencia, es
el de no confundir el orden de la causa y la necesidad de
la condición (aquello sin lo cual la causajamás llegaría a ser cau-
sa), que, en este caso, es toda la maquinaria del cuerpo,
tan irónicamente desmontada por Sócrates, sin la cual es
indudable que no sólo no sabría qué acción tiene lugar en
él, sino que también desconocería qué sentido otorgarle.
4, En lo sucesivo estaremos en mejor disposición de
calibrar la importancia del texto que nos ocupa y que
Leibniz, posteriormente, reprodujo en su totalidad a lo

2 Textos paralelos: República, VI, S04 a-509 b; Ltyts, X, 895 a-905 d (y

libro V).

29
largo de un capítulo de su Discurso de metafísica1'. Para em-
pezar, Platón nos suministra, al principio de una historia
de la racionalidad filosófica, un buen ejemplo del núcleo
central de la actividad del filósofo. Ella consiste, funda-
mentalmente, en conducir al pensamiento desde las tinie-
blas a la claridad a través de la práctica, en el seno de una
crítica de todas las confusiones, de las distinciones con-
ceptuales. Una vez aceptado esto, la victoria teórica que
constituye, al término del texto, la oposición entre la cau- •
sa y la condición se inscribe en una problemática más
amplia a la que Platón contribuye al definir sus términos.
En efecto, es imposible interrogarse acerca de la causali-
dad sin interrogarse al mismo tiempo acerca de la finali-
dad, sobre todo si se trata de pensar la acción. También
imposible es el no establecer una oposición entre la teleo-
logía (la inteligencia de los fines) y el mecanismo (la inte-
ligencia de los efectos), sin que la primera excluya, sin
embargo, la necesidad del segundo término de la oposi-
ción, pues, de hecho, y durante siglos de continuada re-
flexión filosófica, pensar la Naturaleza no ha sido otra
cosa que intentar concebir la justa relación entre los tér-
minos señalados. E imposible, finalmente, es el no llegar
a presentir, bajo la mención de la Mente Causa de Todas las
Cosas, una posible definición de Dios, tanto como imposi-
ble el no plantearse la cuestión, en esta misma línea, de la
distinción entre filosofía y física, así como de la conexión
existente entre la filosofía y la teología4. Para concluir no
podemos dejar de señalar que la historia de la filosofía se
presenta aquí como un auténtico conflicto entre dos pun-
tos de vista antagónicos que Anaxágoras parece ya conte-
ner en potencia. Nos referimos a la concepción que abo-
ga por las causas materiales (anclada a la evidencia, en el
campo de la física) y la que se decanta por las causas idea-
les (sustentada por su lógica frente al cielo de la teología).
Antes de que pasaran a denominarse materialistas e tdeaiis-

Leibniz, Discurso de Metafísica, 520.


3

Para estas cuestiones acudir al Time», también de Platón, sobre


4

todo 29 6-31 b.

30
tas, Platón mismo los situará en escena, en El Sofista, me-
diante el enfrentamiento que tiene lugar en un combate
de gigantes (una «gigantomaquia») entre los Hijos de ¡a
Tierra y los Amigos dé las Formas.

2. Platón

¿CUÁL ES LA VIDA MEJOR?

SÓCRATES.—En efecto, es con la mira puesta en los


bienes que todo debe hacerse. Tal era nuestra opinión de
Polos y de mí mismo, como sin duda recuerdas. ¿Estás tú
también de acuerdo en decir con nosotros que el bien es
el fin de toda acción, que todo lo demás se ha de encami-
nar hacia ese objetivo, y que el bien no debe subordinarse
a nada? ¿Añades tu voto al de nosotros dos?
CALICLES.—Sí, estoy de acuerdo con vosotros.
SÓCRATES.—Así, pues todo lo demás, sobre todo las
cosas agradables, debe hacerse por los bienes, en lugar de
hacer el bien por las cosas agradables.
CALICLES.—Desde luego.
SÓCRATES.—Sin embargo, ¿puede cualquier hombre
seleccionar, de entre las cosas agradables, aquellas que
son buenas y aquellas que son malas? ¿No hace falta recu-
rrir, en cada caso, a un experto?
CALICLES.—Sí, se necesita un experto.
SÓCRATES.—Traigamos, pues, de nuevo a la memoria
lo que decían Polos y Gorgias. Yo les decía, como sin
duda recuerdas, que hay ciertas prácticas cuyo objetivo es
el placer y nada más procuran, en una ignorancia total de
lo que es mejor o peor. Pero, añadía, hay otras que saben
distinguir entre lo bueno y lo malo. Así, en el grupo de
prácticas relativas al placer yo colocaba a la culinaria,
pues consiste en un saber hacer, mas no es un arte, mien-
tras que, por el contrario, situaba a la medicina entre las
disciplinas que buscan el bien. Y te ruego, Calicles, en
31
nombre de Zeus, dios de la amistad, que no creas que tie-
nes derecho de bromear conmigo, ni respondas al azar,
en contra de tus propias opiniones, lo primero que te
venga a las mientes, ni tampoco tomes mis afirmaciones
como si pensara en otra cosa que en bromear. ¿No ves
que el asunto del cual estamos discutiendo es, precisa-
mente, el problema que más puede interesar a todo hom-
bre, con sólo que tenga dos dedos de frente? En efecto,
¿de qué modo hay que vivir? ¿Hay que hacerlo como tú
aconsejas? ¿Observando la conducta que, según tú, le co-
rresponde a un hombre, tratando los asuntos propios del
hombre, hablando ante el pueblo, cultivando la retórica e
interviniendo en los asuntos públicos como nosotros, o
bien, según vivo yo, según la filosofía? Y, por último, ¿en
qué supera cada una de estas vidas a la otra? Quizás lo
mejor que podemos hacer es distinguir bien entre la vida
que Calicles aconseja y la que aconseja Sócrates; por ejem-
plo, con la definición que yo he emprendido hace un mo-
mento. Debemos tratar de hacer una discriminación y,
una vez puestos de acuerdo sobre si en verdad existen es-
tos dos géneros de vida, examinar qué los distingue y cuál
de ellas merece ser vivida.

Gorgtas, 499 e - 500 d.

COMENTARIO

1. Nos encontramos en el texto con Sócrates frente a


Calicles, uno de sus interlocutores más célebres1 de todos
los diálogos platónicos, a causa de la violencia de sus con-
vicciones, en las antípodas de las de Sócrates, y porque és-
tas aluden a un orden de cosas de importancia capital a
los ojos de todo griego. Nos referimos al orden social y
político y, en íntima conexión con él, como enseguida
veremos, al sentido de la vida. El texto menciona igual-

1 Otro interlocutor de igual categoría es Trasímaco, Ver Rtpáblúa, I,

336 b-354 c.

32
mente la presencia, en situación, de Gorgias y de Polos2.
A través de éstos podemos ver qué intenta Platón intro-
ducir en escena; a saber, dos tipos de especialista en el
discurso, el retor y el filósofo; dos géneros de actividad
«lingüística». Así, pues, tenemos la retórica (el arte de la
oratoria, tan estrechamente ligado, en el seno de las cos-
tumbres de la polis griega, a la conquista o al ejercicio del
poder y, por tanto, a la política) y la filosofía, cuya defini-
ción podrá, quizás, extraerse a partir de su oposición con
la retórica. En suma, dos modos de legos aparecen enfren-
tados en el texto que nos ocupa, sobre el telón de fondo
de un fenómeno histórico que conviene evocar: la apari-
ción en Atenas, en el siglo v a.C., del pensamiento sofista
y de un cierto uso interesado de la ciencia y de la palabra
que plantea el problema de las relaciones entre política y
moral, precisamente porque la fuerza del logas se verá
acompañada en adelante de una duda sobre su valor.
De nuevo, la vida de la filosofía aparece unida a la his-
toria.
2. El texto comienza con un diálogo cuya función no
parece ser otra que la de precisar, repitiéndola con insis-
tencia, una tesis y necesariamente fundamental que, a la
vez, es teórica (el bien es elfinde toda acción) y práctica (todo lo
demás se ha de encaminar hacia ese objetivo)• Para ello el autor
trae a la memoria las distinciones que la tesis pudiera per-
mitir (entre las prácticas ligadas, precisamente, a formas de
bien, como son la culinaria o la medicina). Diálogo falso, al
menos en apariencia, pues Calicles se contenta con dar su
consentimiento. Pero el corto intermedio que sigue a
continuación (Yte ruego, Calicles, ... ...en otra cosa que en
bromear) disipa nuestras dudas al sugerir que la aproba-
ción de Calicles bien pudiera no ser más que cinismo, una
desenvuelta indiferencia ante el esfuerzo orientado hacia
la verdad. Y tanto más insoportable (moralmente) cuan-
to que el objeto, más o menos enmascarado, del debate,
tras el inicio del diálogo, se revela ahora, al término de

1 Otros sofistas, frente a Sócrates: Pródico, Hippias y Protágoras mis-

mo en el Protágoras,

33
una patética súplica, como el más serio de todos los pro-
blemas: ¿De qué modo hay que vivirt La oposición entre retó-
rica/política y filosofía se inscribe en la urgencia de este
interrogante de naturaleza existencial que exige una deci-
sión, aunque sobre la base de una «jerarquízacíón» (de va-
lores) (¿No ves que... —*... ¿en qué supera cada una de estas vidas
a la otra?). Nada más normal, en tales condiciones, que la
tesis definida en un principio sea aplicada en la práctica
como una norma: quizás lo mejor que podemos hacer es....; se
esboza entonces un programa de reflexión rigurosa que
no es otro que la señal distintiva de la filosofía, diferen-
ciándola así de la retórica (final del texto),
3. Algo que llame especialmente la atención en este
texto, aparte de la reivindicación de una cierta seriedad
(contra quienes bromean) y de un cierto derecho (o de una
cierta rectitud, contra el cinismo), es el rigor de una exi-
gencia que aparece, como es evidente, conectada con el
sentido de la vida,
a) Exigencia de un acuerdo acerca de los principios y
definiciones, conquistado por virtud del diálogo. Acuer-
do que se pretende ampliar poco a poco, de persona a
persona (¿Estás tú también de acuerdo en decir con nosotros...), y,
por último, general izado n lo verdadero debe ser el bien
de todos.
b) Exigencia de una competencia, pues se necesita un ex-
perto, se necesita, por decirlo de otro modo, disponer de
herramientas de juicio, saber definir criterios y cómo darles
empleo siempre que se trate de decidir, en cada momen-
to, entre lo que está bien y lo que está mal; competencia del
filósofo, a quien corresponden exigencias diferentes de
las del retor, así como otra ciencia de los principios.
Aparte de la virtud propia del diálogo, no estará de
más apreciar aquí la virtud de un método (desde donde se
debe entender la virtud como el derecho y la fuerza reuni-
dos). Es, en efecto, tal método el que confiere especificidad
a la filosofía en cuanto al género del logos y al género de vida.
Preocupación siempre por re-definir, por re-memorar
sus logros, etapas y momentos más decisivos; preocupa-
ción (de suyo distintiva de la filosofía) siempre por dis-

34
tinguir (los géneros entre sí y las especies en cada género),
identificar y jerarquizar (con el objeto de poder elegir con
conocimiento de causa). Preocupación teórica cuyo indi-
sociable significado práctico se manifiesta cuando se diri-
ge hacia las prácticas mismas (¿qué es arte y qué no es más
que mero saber hacer?).
Para comprender todo el alcance de este método sería
necesario escudriñar entre todos los párrafos del diálogo
que clarifican el sentido de la relación culinaria/medi-
cina\ Veríamos entonces cómo Platón transfiere a la fi-
losofía (aquello que da especificidad al método platónico
en «la» filosofía) la racionalidad de la analogía que con tan-
to rigor y fecundidad opera en el terreno de las matemáti-
cas (a partir de tres términos conocidos es posible hallar
un cuarto término; la filosofía es a la retórica lo que la
medicina a la culinaria, al igual que 12 es a 9 lo que 4 a 3,
etc.). Sin embargo, a partir de aquí se vendría a plantear,
a lo largo de toda la historia de la filosofía, la cuestión de
su relación con las matemáticas (esa ciencia cuyo objeto
es característicamente ideal, cuyas verdades parecen ser
eternas...).
4. No podemos descuidar el «fondo» de la cuestión
que constituye el centro de gravedad de nuestro texto. El
gran asunto, tanto para el cocinero, el médico, el retor, el
político o el filósofo, no es otro que el de conocer el Bien.
Aquí se actualiza, de forma más concreta que nunca, en el
campo de las practicas humanas, el principio de lo mejor. Lo
esencial es no confundir la idea del Bien (piedra angular
del sistema de las ideas platónico), que en la acción buena
jamás podría ser relativizada como medio para algún fin
(hacer el bien per las cosas agradables), con sus apariencias sen-
sibles (lo agradable, el placer)*. Nada hay peor que subordi-
nar la acción a tales fines, necesariamente transitorios,
subjetivos y particulares, fines que por sí mismos son
equívocos (lo agradable, como tal, puede ser bueno y

1 Ver, por ejemplo, 464b-465e; 500e-501c.


4 Acerca del placer y del Bien en Platón, acudir, sobre todo, al
Fikbo.

35
malo; el placer de beber es, al mismo tiempo, sufrimiento
por tener sed), cuando no simplemente risibles, obscenos
(rascarse todo el tiempo y por todos lados, ¿en eso consis-
te la felicidad de la vida?, se pregunta Sócrates)5. Ello su-
pondría ignorar la intemporalidad, objetividad, universa-
lidad, univocidad y unidad del Bien en sí, supondría
no ser ya capaces de distinguir al hombre bueno del
malvado.
Las prácticas que se orientan a la satisfacción de mi de-
seo (el placer de comer, seducir o dominar «bien») nos re-
miten a la arbitrariedad de las pasiones, aparte de hallarse
expuestas a lo equívoco de su resultado. A la inversa, las
acciones buenas, morales, son acciones desinteresadas
que no comportan ningún contrario, pues no suponen
atender al Bien como meta más que en tanto reposen en
el conocimiento como criterio. La filosofía, según Pla-
tón, consiste, precisamente, en el método que le permite
hacerse con ese saber y orientarse a tal fin. Pero esto no es
todo, porque la moral implícita en el texto que estamos
comentando, cuyo tema es la oposición entre filosofía y
retórica, sugiere que la filosofía bien podría ser otro tipo de
política, mejor desde todo punto de vista. No debemos ol-
vidar que el fin más elevado, para los griegos (Aristóteles
lo repetirá después de Platón), es el de la pida buena, objeti-
vo que sólo es posible y realmente valioso en el marco de
una comunidad política.
Teniendo en cuenta lo anterior, podemos comprender
ahora que la mejor manera de vivir el Todo que supone la
polis (la ciudad griega), para cada individuo (afort'tori para
todo gobernante), es la de conocer el Bien como Uno
(causa de todas las cosas). Ello explica que, al menos en
las obras de Platón, desde la República al Político, la cien-
cia de la política y la ciencia del Bien constituyen una
unidad.

5 Gorgias, 494 c-d.

36
3. Platón
¿SER Y/O NO SER?

EXTRANJERO.—Entonces, te haría aún una petición


más apremiante.
TEÉTETO. —¿Cuál?
EXTRANJERO.—Que no me consideres como una espe-
cie de parricida.
T E É T E T O . — ¿ E n qué sentido?
EXTRANJERO.—Necesariamente, y en nuestra defensa,
deberíamos poner a prueba la tesis de nuestro padre Par-
ménides y establecer, por fuerza mayor, que, en cierto as-
pecto, el no ser es y que el ser, por su parte, en cierto
modo no es.
TEÉTETO.—Es ahí, evidentemente, donde tendrá que
recaer el peso de nuestra argumentación.
EXTRANJERO.—¿Cómo no habría de ser evidente y
evidente hasta para un ciego, como suele decirse? Hasta
que estas proposiciones no sean afirmadas o refutadas, no
se podrá en justicia hablar de discursos ni de opiniones
falsas, ni de imágenes, copias, imitaciones o simulacros,
al menos no más que de cualquiera de las artes de las que
se ocupan, sin convertirse en algo digno de risa al verse
obligado a contradecirse a sí mismo.
T E É T E T O . — E s muy cierto.
EXTRANJERO.—He ahí p o r qué a h o r a h a llegado el
momento de poner irrespetuosamente las manos sobre la
tesis paterna o, en caso de que algún escrúpulo nos deten-
ga, de abandonar por completo este asunto.
TEÉTETO.—No debemos tener escrúpulos de ninguna
clase.
(...)
EXTRANJERO.—Se me ocurre que Parménides y quie-
nes con él se lanzaron ala empresa de determinar cuántas
cosas reales hay y de cuáles son, nos endilgaron discursos
de formas un tanto extemporáneas.

37
T E É T E T O . — ¿ C ó m o es eso?
EXTRANJERO.—Todos ellos parecen habernos estado
contando cuentos, cada cual el suyo, como si fuésemos
niños. Según uno, hay tres cosas reales, algunas de las
cuales mantienen entre sí una especie de lucha para lue-
go, formando amistad, hacernos asistir a sus desposorios,
al nacimiento, crianza y educación de sus retoños. Otro
de ellos se detiene en dos — l o húmedo y lo seco, o lo ca-
liente y lo frío— a las cuales une en matrimonio y las co-
loca luego en la misma casa. Para nosotros, el pueblo
eleático, que se remonta ajenófanes y más atrás todavía,
no percibe sino unidad en lo que llamamos el Todo, y es
en este sentido que desarrolla la exposición de su cuento.
Posteriormente, ciertas musas de Jonia y de Sicilia se die-
ron cuenta de que la salvación consistía en entrelazar las
dos tesis y afirmar que lo real es a un tiempo una y mu-
chas cosas, y se mantiene unido mediante la amistad y el
odio. Su mismo desacuerdo es un eterno acuerdo. (...)
En todo esto ¿quién podría saber quién dijo la verdad y
quién no?

El Sofista, 241 d - 242 a; 242 c - 243 a.

COMENTARIO

1. El diálogo de El Sofista pertenece a la serie de Jos


grandes diálogos platónicos, donde Platón da el giro más
sistemático a la puesta en práctica de su método y, al mis-
mo tiempo, a la elaboración de sus tesis. Para captar toda
su importancia tendríamos que remitir este diálogo al
Parménides (consagrado a la tan difícil, por abstracta, cues-
tión de las relaciones existentes entre el Ser y el Uno) y al
Teéteto (que se ocupa de definir la esencia de la ciencia).
Asimismo, convendría situarlo en aquel orden de
preocupaciones que el Gorgtas ya había ilustrado: ¿A
quién llamamos sofista (nuestro diálogo se esfuerza por
responder a esta pregunta)? ¿A quién llamamos filósofo?
¿Cuáles son las notas que les diferencian (y, en conse-

38
cuencia, en qué se asemejan)? ¿Por qué la necesidad de es-
tas indicaciones preliminares? Porque nuestro texto, den-
tro de sus límites, ilustra, como ya veremos, el modo
como convergen y se entreveran los diversos temas men-
cionados al formular, de manera eminentemente dramá-
tica, aquel que es, en último término, el interrogante de
los interrogantes para todo filósofo: ¿qué es el Ser? Este
«ser» al que sin cesar se lo presupone conocido, en cual-
quier respecto, cuando decimos que una cosa es, que es
esto o lo otro, que no es «nada» o que no es, ese «ser», repe-
timos, en verdad ¿qué es?
2. El texto formula esta cuestión de dos modos y en
dos contextos diferentes, aunque ligados entre sí:
a) ¿Acaso no hay necesidad de establecer porfuerza mayor,
que, en cierto aspecto, el no ser esy que el ser, por su parte, en cierto
aspecto no es? Esta indispensable hipótesis concerniente al
ser del Ser ocupa la primera parte del texto (No debemos te-
ner escrúpulos de ninguna clase). Ella es introducida con gran-
des precauciones (¿será, por tanto, escandalosa?) y remiti-
da principalmente a la tesis de Parménides, que así apare-
ce como su «anti-tesis» —su propia demostración implica la
refutación de aquélla, y (¡el colmo de la paradoja!) condi-
ciona, aunque en sí misma sea contradictoria, la persecu-
ción de una palabra filosófica depurada de todo lo que
pudiera convertirla «en algo digno de risa al verse obliga-
do a contradecirse a sí mismo».
b) En el marco, muy general en esta ocasión, de un
«inventario» de todas las tesis acerca del Ser previamente
enunciadas (entre las cuales se encuentra precisamente la
de Parménides, a quien debemos incluir entre el «pueblo
eleático») (final del texto), la interrogación que recae so-
bre el ser del Ser es conducida a su simplicidad original, a
sus aspectos cuantitativo y cualitativo. En efecto, ¿cuántas
cosas reales hayy cuáles son? O dicho de otro modo, el ser que
son las cosas, el Ser que son los setes, ¿es Uno o Múltiple?
¿Es en si mismo el Mismo o es Otro que sí mismo? ¿O todo
ello a un tiempo y siempre? En consecuencia, la cuestión
aparece planteada la primera vez de un modo netamente
polémico, mientras que la segunda adquiere un cariz más

39
«histórico» y, en los dos casos, crítico. Esto es así porque
existe otra cuestión que lo anima desde el interior, pres-
tando unidad a sus dos contextos. Se trata de si es posible
prescindir de un criterio (pero ¿cuál sería?) de lo verdade-
ro y de lo falso ante tal profusión de discursos acerca del
Ser, discursos que presuponen ya la «verdad» del ser.
Cuestión de importancia crucial cuya solución condicio-
na necesariamente el estatuto de la filosofía, si ésta no es
sofística, pero también la posibilidad de la sofistica como
tal, si es que consiste en este uso del discurso que hace pa-
sar lo falso por lo verdadero.
3. Debemos prestar atención a la manera como se
dramatiza la presentación de los interrogantes que veni-
mos señalando. La forma, ciertamente, está en gran me-
dida ligada al fondo. Todo comienza con una petición, la
de no tomar por un parricida a quien no ¡o es; no tener por
verdadero lo que sería falso. Ahora bien, este comienzo
debe ser sometido a la duda (o a la perplejidad), tal y
como aparece expresado al final del texto: ¿quiénpodría sa-
ber quién dijo la verdad y quién no? De hecho, la continuación
del texto sugiere que habría, en efecto, un parricida —al
rechazar todo escrúpulo posible. Así, la verdad de) texto
se anuncia en una de-negación, una manera de negar aquello
que es para mejor afirmar eso que es (el no-ser es).
Vemos de este modo cómo se va perfilando lo esencial
del problema al plantear el texto, por una parte, qué rela-
ciones es necesario establecer entre el ser y el no-ser, pero
también (y sobre todo) entre éstos y los contrarios de lo
verdadero y de lo falso (o, si se quiere, entre el ser del Ser y
el Ser del discurso). En este punto podemos sopesar el
grado tanto de sutilidad como de rigor que exige una con-
cepción tal de la verdad. Practicados, en el límite de la
confusión entre lo verdadero y lo falso, por el placer de
confundir al adversario, la sutilidad y el rigor darán ori-
gen a la eristicatan combatida por Platón. Consagrados a
la distinción entre los términos señalados, y con el pro-

1 Sobre la eristica, ver Rtpúbiiiü, 537 c-539 d y Eutidemo.

40
pósito de lograr un diálogo desinteresado, aquellos ele-
mentos conformarán, en sentido amplio, la dialéctica pla-
tónica2. Señalemos por último que la posibilidad misma
de la verdad parece poner en tela de juicio, de una parte,
el estatus de la tradición y, de otra, el de la palabra:
a) La tradición aparece enmarcada en la autoridad de
Parménídes, el padre. Es posible, sin duda, entender la
historia de la filosofía en términos de un «asunto de fami-
lia», y el asesinato del padre como el acto requerido sim-
bólicamente para asegurar la liberación del hijo. Sin em-
bargo, los «padres» (fundadores) son en el caso que nos
ocupa maestros, y los hijos, discípulos. El conflicto que
instaura el texto (en nuestra defensa, poner irrespetuosamente ios
manos sobre la tesis paterna) no pone en juego otra cosa que el
advenimiento de la verdad en el orden de las «ideas reci-
bidas». En efecto, el coraje escandaloso del parricida es el
prejuicio de la radícalidad, es la exigencia de una verdad
autónoma, no entiende más ley que ella misma (y ésta será,
en este caso, el coraje de afrontar la contradicción en
el corazón del Ser).
b) La palabra es el mythos de los antiguos. Aquí po-
dríamos interpretar la actividad filosófica, de nuevo,
como si se tratara de relatos que los abuelos contaran a
sus nietos, relatos de «historias de familia» (disputas, des-
posorios, nacimientos de sus retoños). Pero desde los unos a los
otros (a las musas de Joniay Sicilia, entendiendo por tales a
Heráclito y Empédocles) y a medida que los relatos «pro-
gresan», sugieren, bajo la ironía de la intención, que bien
podía haber una verdad de la palabra mítica. El propio mito
>arece algo i ngenuo. Toma sus modelosde la experiencia de
Ía vida. Y si esto es así, ¿no lo será acaso para volver sen-
sible el sentido oculto del «ser» de las cosas, impensable
como tal? ¿Y si la verdad del Ser consiste en estar en
disputa consigo mismo? En cualquier caso, desde el mo-
mento en que es dicho, se contradice: El ser es a un tiempo
una y muchas cosas (pero permanece siendo uno cuando se

2 Sobre la dialéctica en sentido restringido, ver República, libro VII y

Fedm, 270 a-272 c.

41
contradice: su mismo desacuerdo es un eterno acuerdo).
Y en orden a recordar lo anterior, a aventurarse a asumir
todo el «manifiesto» de la contradicción, ¿no sería nece-
saria una filosofía que encarnara a su manera «la unidad
de los contrarios»? Justamente, el extranjero que carga
con el peso del «combate», que reconoce al padre al refu-
tarlo, es alguien de la familia (él mismo es un eléata). Sin
embargo, viene de lejos...
4. Toda la dramatizadón del texto, con un simbolis-
mo propio, tiende por tanto a hacer admitir en lo funda-
mental, en ruptura y a la vez en continuidad con las de los
«padres» fundadores, la radical novedad de la tesis plató-
nica, sobre la cual puede edificar su teoría de las ideas y
fundar, «legitimar», también sobre la tesis mencionada, la
posibilidad del discurso falso (la sofística, diríamos nos-
otros).
Para Platón, el no-ser es un género del Ser o si se desea,
hay una incuestionable «realidad» del no-ser (podríamos
añadir algo más: de lo «negativo»). Sobre tal realidad re-
posa, en líneas muy generales, la posibilidad del error (de
las opiniones falsas). Pero debemos ir todavía más lejos y
afirmar que la realidad de todas las «falsificaciones», ope-
raciones y obras que alteran la realidad del Ser al reprodu-
cirlo (por arte o artificio), imágenes, copias, imitaciones o simu-
lacros, también descansa sobre la realidad del no-ser. Más
radical aún que la relación que mantienen con el Ser estas
páginas sensibles (su «participación» de las ideas de las
cuales son el reflejo), el nuevo objeto filosófico que elabo-
ra El Sofista, necesariamente abstracto, es la «participa-
ción de las ideas entre sí y, para empezar, de los «géneros
del Ser» (el Movimiento, el Reposo, lo Otro, el No-Ser,.,).
Por este medio se cumple y perfecciona cierto tránsito,
aquel que va del mythos al iogos^, de la palabra mítica a la
esencia lógica del pensamiento. Precisamente, el trabajo

1 Sobre el mythos y el ¡ttgps, ver J, P. Venían t, Mjlhe et s/xiété en Grite an-

(¡ennt, 1974, y Mytbe et pernee cbtz tes Crea, La Découverte/Maspéro, 1960.


{Mito y sociedad en ¡a Crecía antigua, Barcelona, Ariei, 1985; y Mito y pensa-
miento en las griegos, México, siglo xxi, 1982.)

42
de esta esencia lógica sobre sí misma ha constituido desde
entonces la historia toda de la filosofía.

PARA SABER MÁS

Sobre Platón (textos 1, 2y i)

COPLESTON, F., Historia de la filosofía, vol. I, Grecia y Roma, trad.


J. M. García de la Mora, Barcelona, Ariel, 1986.
GRUBE, G. M. A., El pensamiento de Platón, trad. Tomás Calvo
Martinez, Madrid, Gredos, 1976.
MAGEE, B., Los gandes filósofos, trad. de Amaia Barcena, Madrid,
Cátedra, 1990, págs. 15-35.
Ross, D., La teoría de ¡as ideas de Platón, trad. José Luis Diez Arias,
Madrid, 1986.
STEVENSON, L., Siete teorías de ¡a naturaleza humana, cap, 1, «Pla-
tón: el gobierno de los sabios», trad, E. Ibáñez, Madrid, Cá-
tedra, Col. Teorema, 1984.

Sobre ios presocrátkos

BARN ES, J., Lospresocrátkos, trad. Eugenia Martín López, Madrid,


Cátedra (Teorema Mayor), 1992,
JAEGER, W., La teología de los primeros filósofos griegos, trad. José
Gaos, México, F.C.E., 1982.
KIRK, G . S . , R A V E N , J . E . y SCHOFIELD, M . , Los filósofos presocráti-
cos, trad. Jesús García Fernández, Madrid, Gredos, 1987.

Sobre los sofistas

BARNES, j . , Los presocrátkos, cap. X X I , trad. Eugenia Martin Ló-


pez, Madrid, Cátedra, 1992.

43
GUTHRIE, W. K. C., Historia de lafilosofogriega, t. III, «Siglo v:
Ilustración», trad. Joaquín Fernández Feo, Madrid, Gre-
dos, 1988.

Otros

VERNANT, J. P., Mitoy sociedad en la Grecia antigua, trad. J. D. Ló-


pez Bonillo, Barcelona, Ariel, 1985.
— Mito y pensamiento en los griegas, trad, C. Cázquez, México,
Siglo X X I , 1982.

4. Aristóteles

S O B R E P L A T Ó N , QUE T I E N E IDEAS PEREGRINAS

Pero lo más importante que uno se preguntaría es qué


aportan las Ideas a los seres sensibles, tanto a los eternos
como a los que se generan y corrompen. En efecto, no
son para estos seres causa de ningún movimiento ni de
ningún cambio. Por lo demás, tampoco sirven de nada
para la ciencia de los demás seres (pues aquéllos no son
substancia de éstos, pues de lo contrario, estarían en és-
tas), ni para explicar su ser, pues no son inmanentes a las
cosas que de ellas participan. SI fueran inmanentes, pare-
cerían quizá ser causas como lo blanco es causa de la
blancura en el ser blanco al entrar en su composición.
Pero este concepto, que Anaxágoras expresó el primero,
y posteriormente Eudoxo y algunos otros, es por demás
refutable, pues es fácil reunir objeciones insolubles contra
una opinión tal. Por otra parte, tampoco se puede decir
que las demás cosas provengan de las Ideas, en ninguno
de los sentidos en que de ordinario se entiende la expre-
sión de. En cuanto a afirmar que las Ideas son paradigmas
y que las otras cosas participan de ellas son palabras va-
cías y metáforas poéticas. ¿Qué es lo que actúa, pues, al

44
dirigir la mirada sobre las Ideas? Puede ocurrir, en efecto,
que exista o se haga semejante a otra cosa sin ser modela-
do según ella; de modo que Sócrates, existiendo o no, po-
dría nacer alguien como Sócrates; y es evidente que lo
mismo podría ocurrir aun cuando hubiera un Sócrates
eterno. Además, habrá numerosos paradigmas de la mis-
ma cosa, y, por tanto, numerosas Ideas; por ejemplo, del
hombre sería lo Animal y lo Bípedo, y, al mismo tiempo,
el Hombre en sí. Además, las Ideas no serán solamente
paradigmas de las cosas sensibles, sino también de sí mis-
mas, y, por ejemplo, el género, en tanto que género, será
paradigma de las Ideas contenidas en el género; la misma
cosa será, pues, paradigma e imagen. Además, parece im-
posible que la substancia esté separada de aquello de lo
que es substancia; por consiguiente, ¿cómo podrían las
Ideas, que son substancias de las cosas, estar separadas de
ellas? En el Fedán, sin embargo, se dice que las causas del
ser y del devenir son las Ideas. Pero, aun admitiendo la
existencia de las Ideas, no se engendran las cosas partici-
pantes sin la intervención de la causa motriz.

Metafísica, A, 991, a9 - 991 b5.

COMENTARIO

1. Que sea necesario, para comprender lo que aquí


está en juego, remitirnos, entre Anaxágoras y Eudoxo, a
Platón mismo y, en concreto, a su Fedón, parece evidente
a juzgar por la conclusión del texto. Nos encontramos,
pues, como referencia explícita, con la cuestión que allí
planteaba Sócrates (de alcance que va más allá de los sim-
ples límites de una moral): ¿cuáles podemos decir que
sean las causas del ser y del devenir?, así como con su contex-
to, el contexto de una física. Pero en el caso de Aristóteles,
la nueva cuestión (la más importante que uno se preguntaría) se
dirige antes que nada a la respuesta elaborada por Platón
(su teoría de las Ideas): ¿qué valor tiene ésta para el cono-

45
_ cimiento filosófico?1 Podemos apreciar de este modo el
estatus de la Metafísica aristotélica, pues, más allá de su sig-
nificado anecdótico (lo que se puede leer a continuarán
de la Física, denominación que corresponde a la clasifica-
ción concebida por el primer editor de las obras de Aris-
tóteles2 con vista a su publicación), este conjunto de ca-
torce tratados constituye, sin duda, una profundización
en los problemas de la filosofía primera y, al mismo tiempo
e irremediablemente unido con lo anterior, un examen
histórico-critico de la pertinencia y validez de los siste-
mas ya existentes.
2. Al pasar del diálogo platónico al tratado aristotélico
no se puede evitar percibir el cambio de «estilo» filosófico
que en este tránsito se produce. En el segundo caso, del
que aquí nos ocupamos, tenemos que vérnoslas, tras el
impulso ejercido por la introducción de la cuestión ini-
cial, con una sucesión (ni... ni, por lo demás, por otra parte, en
cuanto a, además) de observaciones argumentadas (en efecto,
pero, pues, sin embargo) cuya función es la de agotar, en el
seno de un tejido denso y apretado, todos los aspectos de
una tesis, con el propósito de establecer finalmente su ca-
rácter inadmisible o, al menos, poco satisfactorio. Desde
Platón a Aristóteles, la manera socrática de filosofar va
siendo paulatinamente abandonada. El debate se traslada
del ágora, donde aquél tenía lugar entre subjetividades in-
dividuales en continua confrontación de su ingenuidad e
inquietudes, del grado de sus convicciones y el valor de
sus opiniones, siempre a la búsqueda de un acuerdo, al es-
pacio propiamente abstracto y puramente lógico de la teo-
ría como tal y de su objetividad, donde las objeciones juegan
un papel decisivo en tanto que comienzan a surgir desde
el interior del discurso —del logos— y dan testimonio de
una existencia de coherencia, precisamente la de un siste-
ma, más que de un esfuerzo común por alcanzar el con-
senso.
Desde esta perspectiva, pues, la doctrina platónica de las

1 Textos paralelos: Metaffíisa, M, 4-8 y N (sobre todo 4).


2 Se trata de Andrónico de Rodas (hacia -60 a.C).

46
ideas quizás no resista la crítica. Podemos distinguir los
dos ángulos de los que Aristóteles se sirve para ponerla a
prueba:
a) A través del tema inicial de qué aportan ¡as Ideas a ios
seres sensibks, se trata de evaluar globalmente la utifíad de la
introducción de las Ideas para explicar los seres sensibles y,
en consecuencia, evaluar también la eficacia de su acción.
Ello implica lanzar un ataque a la teoría de Platón desde
el ángulo de lo que más ambicionaba.
b) Por otro lado, se trata de atacar la especificidad de
su solución («núcleo» del platonismo, de su «metafísica»,
que le confiere su originalidad), tal y como se desprende
del paradigmatismo* y de su complemento, la doctrina de la
participación. Conviene ahora, por tanto, analizar con ma-
yor atención las dos vías de exploración que acabamos de
señalar.
3. El primer punto nos remite, simplificando algo la
cuestión, al problema siguiente: suponiendo que su «na-
turaleza» es conocida, que son puramente inteligibles, no
sensibles por tanto y, sin embargo, tan reales como las co-
sas, ¿de qué sirven entonces las Ideas? Se puede apreciar
que este problema debe ser examinado desde el plano de
la ciencia, por un parte, y, de otra, desde el plano de la exis-
tencia, Lo que aquí se pretende es acudir a las Ideas, consi-
deradas como principios de inteligibilidad de los seres
sensibles, cuya esencia inmutable permite aprehender
algo estable y fijo en la naturaleza móvil y cambiante de
los seres que se generan y corrompen, de todo lo que nace y
perece. Se trata de hacer referencia a las Ideas en tanto
que origen, fuente generadora de aquellos mismos seres
sensibles (ello implica que tas Ideas tienen la capacidad
de engendrarlos). Lo que aquí se pretende, en suma, es
explicar las cosas, hacer brotar su existencia sensible y su
esencia inteligible a partir de una realidad superior que
les precede.
Si hay alguna dificultad en ello, ésta no parece residir

1 Ver V. Goldschmidt, Lt puradigtne dais ¡a d¡a¡citique platonicicnnt,

Vrinreprise, 1985.

47
en la relación causal que asi se establece entre la inmate-
rialidad de las Ideas y la materialidad de los seres sensi-
bles. La dificultad surge más bien, y de forma más dramá-
tica, de la mano de la necesidad de situar a las unas y a los
otros en un espacio común que haga posible su mutua co-
nexión —pero, ¿cuál? Ahora es el momento de que la ló-
gica imponga su derecho. Una de dos, o bien las Ideas es-
tán fuera de los seres sensibles, separadas de ellos, o bien
están en ellos, les son inmanentes. Pero en el primer caso
cabría preguntarse si las Ideas, de por si inmóviles e in-
materiales, pueden generar fuera de ellas mismas tanto el
movimiento como el cambio. En efecto, no podríamos
provenir de las Ideas como un hombre provenir de otro
hombre, o una cama de su forma proyectada, de su mate-
ria (la madera) y de las herramientas del carpintero...
Y en el segundo caso la objeción sería la de cómo las Ideas,
por ejemplo, no perecen al tiempo que los seres sensibles,
ya que entran en su composición y se mezclan con su materia.
SÍ se tornan corruptibles, ¿continuarán siendo Ideas? Po-
demos apreciar cómo se refuerzan entre sí, hasta hacer a
cada supuesto impensable, el rigor de una necesidad con-
dicional (si a, entonces b) y el imperativo de la no contra-
dicción, imperativo que Aristóteles convirtió en pieza
clave de la ciencia y, aún más fundamental, del discurso,
de la comunicación y de la comunidad de pensamiento4.
4. En cuanto al paradigmatismo y la participación (la
misma que permi te denominar cosasparticipantes a Jos seres
sensibles), la severidad de Aristóteles aparece expresada
en una fórmula memorable: «palabras vacías», todo lo
más, «metáforas poéticas», lo que supone decir que las
nociones centrales de la teoría platónica no constituyen
ningún saber de la esencia singular (ni de la substancia) de
las cosas «existentes». Recordemos en qué reside el que,
para Platón, las Ideas sean paradigmas. Este término signi-
fica «modelo», razón por la cual habremos de entender
por Ideas las formas ejemplares cuyas copias serían los seres

4 Ver Mttajiuca, G (en especial 4).

48
sensibles, fabricadas por un artesano o «demiurgo» que
dirige su mirada hacia el «original»5. Y afirmar que las
cosas sensibles participan de las Ideas es sostener que la
substancia de las primeras conservan algo esencial (de ahí
que sean cognoscibles) de la esencia de las segundas, por
efecto mismo del acto de reproducción. Mediante este ro-
deo, y gracias a la participación recíproca de los «géneros
del Ser», cuya teoría aparece en El Sofista, Platón creyó ha-
ber resuelto la dificultad más grave (y que atenta fuerte-
mente contra el principio de no contradicción); a saber,
que los seres sensibles difieren de las Ideas y están separa-
dos de ellas (como el modelo de la copia), y aun tiempo son
la misma cosa (por la semejanza reproducida en !a copia).
Y bien, Aristóteles, en el texto que venimos comentando,
desafia a este aspecto tan fundamental del platonismo por
el efecto conjugado de cuatro observaciones críticas:
a) La similitud no prueba que haya reproducción.
Cualquier tipo de semejanza puede tener su origen en el
solo hecho del a&ir —argumento que alcanza a la «factu-
ra» de las cosas participantes.
b) Argumento que aborda, en esta ocasión, la «es-
tructura» de la propia participación: la pluralidad de as-
pectos que presenta una cosa no puede significar nada
más que una pluralidad de participaciones de una plurali-
dad de Ideas distintas, de modo que algunas de ellas no
podrán englobar a algunas otras (como el género engloba
a la especie, y ésta al individuo). Pero ocurre que el Hom-
bre en si es el paradigma del hombre-ser sensible, y, al mis-
mo tiempo, la imagen del Animal en si. Así, pues, una Idea
que, de forma «idéntica», es imagen y paradigma no podrá
ser otra cosa que una contradicción en sí...
c) Y aún más grave: la condición de posibilidad de la
ciencia de las cosas existentes, la separación que condiciona
a la participación, es igualmente contradictoria, pues im-
plica que aquello por lo que «una» cosa es una (su substan-
cia) no forma una unidad con ella...

5 Ver Platón, Timen, 28 3-29 c.

49
d) Y para terminar, otra observación, la que se
plantea cómo prescindir de una causa motriz, algo que las
Ideas no podrían ser, para pasar de su inmovilidad/
inmutabilidad al movimiento y al cambio de los seres
sensibles.
Podemos ya vislumbrar cuál es la tarea que atañe a
Aristóteles, si el objetivo es el de poner de manifiesto y
escapar de las contradicciones que hacen incoherente la
coherencia propia del platonismo. Para renovar, de una
forma consecuente, la reflexión acerca del Ser y la doctri-
na de la ciencia, conviene elaborar otra teoría de la causa-
lidad (que sitúe a la noción de motor, junto a la de fin, de la
forma y de la materia, el lugar que le corresponde de he-
cho y de derecho) y, conjuntamente, del movimiento
(Aristóteles la desarrollará por medio de la distinción del
acto y de la potencia6). Pero, además, también conviene
elaborar otra teoría de la substancia individual desde
donde poder dar una solución al dualismo de la Materia y
la Forma diferente del dualismo platónico de lo Sensible
y lo inteligible7. Y es, precisamente, en razón de tales ne-
cesidades teóricas que la Metafísica se impone, para toda la
filosofía posterior, como una empresa de filosofía pri-
mera.

5. Aristóteles
Q U E LA CIUDAD DE PLATÓN ES DEMASIADO IDEAL

Asi, los unos gobiernan, los otros son gobernados,


como si alternativamente se convirtieran en otros hom-
bres. Igualmente, entre quienes gobiernan, los unos de-
sempeñan una función, los otros otra. Según esto, es cla-
ro entonces que la naturaleza de la ciudad no consiste en

\et Fiska, II, III, V-VIII. También: De iagenenuióny dt la comtpttón; y


Melufhud.
7 Ver Mttafhka, 2 y H.

50
ser unitaria en este sentido, como algunos dicen, y que lo
que llaman bien supremo de las ciudades las destruye en
realidad, cuando por el contrario el bien propio de cada
cosa la conserva (...) Pero aun admitiendo que el bien su-
premo de una comunidad política consista en ser lo más
unitaria posible, no es indicio de ello el que todos digan
al unísono «mío» y «no mío»; tal es, en efecto, según Só-
crates, la señal de la unidad perfecta de una ciudad. Como
sea que la palabra «todos» tiene un doble sentido, si se le
entiende como «cada uno» quizás nos aproximemos más
a lo que Sócrates pretende, pues cada uno llamará al mis-
mo individuo su propio hijo o a la misma persona su pro-
pia mujer, e igual ocurriría con la hacienda y todo lo de-
más. Pero no lo dirán en este sentido, de hecho, los que
tienen en común las mujeres y los hijos; hablarán de «to-
dos colectivamente» y no de «cada uno de ellos en parti-
cular». De igual modo ocurre con la hacienda, que será de
«todos» y no en el sentido de «cada uno». Es evidente, por
tanto, que es un paralogismo decir simplemente «todos».
Términos como «todos, ambos, pares e impares», por
causa de su doble acepción, dan origen, incluso en los ra-
ciocinios, a silogismos erísticos. Así, que todos llamen
«mío» al mismo objeto, en un sentido es una fórmula be-
lla, pero inaplicable, y, en otro, no conduce en absoluto a
la concordia.
Pues la fórmula propuesta ofrece, además, otro incon-
veniente, pues es objeto de menos cuidado lo que es co-
mún a un gran número de personas; todos se preocupa-
rán más por los bienes propios y menos por los comunes,
a no ser en la medida de un interés personal; se desentien-
den de lo demás, entre otras razones, porque piensan que
otro se preocupará de ello, al igual que, cuando se trata
del servicio doméstico, muchos criados sirven en ocasio-
nes peor que pocos. Si cada ciudadano tiene mil hijos, és-
tos no serán hijos de cada uno, sino que cualquiera será
por igual hijo cualquiera, de tal modo que todos se
desentenderán por igual de él.

Política, II, 3, 1261 b4 - 1262 al.

51
COMENTARIO

1. Extraído, como su título indica, de una de las


obras de filosofía política más significativas y célebres de la
tradición, este texto nos invita a redescubrir, en el seno
de la tradición señalada (y deberíamos ser sensibles a la
actualidad y modernidad de la misma, sea cual sea su an-
tigüedad), el momento decisivo de una oposición eminente-
mente instructiva. Sigue siendo Platón el autor a quien
Aristóteles se enfrenta, a través, en esta ocasión, del Só-
crates de la República, y, como ya tendremos oportunidad
de ver con mayor precisión, de su «espíritu de sistema»,
en un dominio (la Ciudad o el Estado, traducción de polis;
esto es, organización de una comunidad de individuos)
cuya complejidad requiere, sin duda, la mayor «pruden-
cia», tanto teórica como práctica, es decir, una auténtica
virtud de sabiduría y de justa medida en la apreciación de
medios y fines, de causas y efectos. Pero ello también im-
plica que la reflexión política no puede estar separada del
examen «metafísico» de los principios. En cierto sentido,
si el uno constituye un problema en el marco de la Ciu-
dad (tal es el tema del presente extracto), esto será así en
la medida que la filosofía sea ella misma una. El análisis
del texto nos permitirá verificarlo.
2, El texto se articula de forma manifiesta, según los
tres párrafos que lo componen;
a) Primerpárrafo: bajo la autoridad de un principio co-
mún (el bien propio de cada cosa la conserva* —principio al que
Aristóteles, quien en esta materia es discípulo de Platón,
alude con fuerte ironía), se afirma con toda claridad una
diversidad de puntos de vista a propósito de la naturaleza
de la Ciudad (aprehendida aquí desde la perspectiva del
ejercicio del poder político) para concluir, finalmente,
con la invalidación de la tesis adversa que algunos sostie-

i Política, II, 1261 b9.

52
nen, entendiendo por «algunos» Sócrates y Platón. En
efecto, esta tesis tendrá que ser anulada al contradecirse
en sus resultados, pues, de «ponerse en práctica», signifi-
caría la muerte de las ciudades en lugar de su salvación.
La argumentación que sigue a este párrafo se ocupará de
la demostración de lo anterior a través de dos puntos par-
ticularmente «sensibles» en esta materia de litigio.
b) Segundo párrafo: el primero concierne a las relacio-
nes de propiedad, tan fundamentales en la teoría política.
Desde la versión «colectivista» (que todos digan al unísono
mío 7 no mío), ¿qué propone Sócrates? ¿Qué significa
exactamente para la unidad de la ciudad? ¿Cómo repercu-
te en su bien? Centrándose, sobre todo, en el párrafo que
nos ocupa, en la «propiedad» de las mujeres y los hijos (lo
que se relaciona, evidentemente, con la preocupación
más profunda del derecho de la época), Aristóteles em-
prende el examen de uno de los aspectos de lo que se ha
convenido en denominar «comunismo» platónico. En
cualquier caso, es fácil comprobar algo de suma impor-
tancia, que el ángulo de ataque no es en absoluto «jurídi-
co», sino lingüístico y lógico. La denuncia aristotélica de un
equívoco (el doble sentido de «todos») permite señalar la pre-
sencia del vicio del paralogismo (de un razonamiento falso)
y el riesgo de la eristica.
c) Tercer párrafo: El segundo punto concierne al «mo-
tor» del interés personal, tan fuertemente ligado a las rela-
ciones de propiedad y tan fundamental como estos para
el análisis de los «hechos sociales». ¿En qué se convierte
aquel «motor» del interés personal dentro del llamado
«comunismo»? De su estatus depende, sin embargo, lo
que podríamos denominar «gestión» del bien común. El
ángulo de ataque, aquí, es psicológico, y parece orientarse
hacia el irrealismo de la postura platónica.
3. Dirigiremos nuestra mirada para empezar, hacia la
oposición doctrinal que indica, sin duda muy elíptica-
mente, el primer párrafo. En último término hay, cierta-
mente un principio común a Platón y Aristóteles, pues en
ambos encontramos que la naturaleza de lo «político»
sólo es inteligible cuando se lo piensa según la racionali-
53
dad del Bien 2 . Las implicaciones de un principio tal son
considerables, incluso de importancia vital, y ello porque
en él le va la salvación de una ciudad, su «salud», la conser-
vación de sí misma: en un sentido, no cabe esperar de un
buen régimen político otra cosa que lo que podemos espe-
rar de un buen régimen de vida. Asegurar las mejores con-
diciones de su propia duración; ésta debe ser, para el Es-
tado, la meta de una política razonable (¿o racional?). En
este punto Aristóteles se separa de Platón, y podría decir-
se, a manera de anticipación, que la política de este últi-
mo es, a los ojos de aquél, demasiado racional y no lo bas-
tante razonable. Según Platón, la identidad del Bien y del
Uno (como causa inteligible de todas las cosas) exige que
la Ciudad sea una, para lo cual esa unidad deberá reflejar-
se en su propio interior. A la pregunta planteada en la Re-
pública: «¿Cuál es, en la organización del Estado, el mayor
bien? ¿Y el mayor mal?» la respuesta es: «El Estado mejor
gobernado es aquel que más se aproxima al modelo del
individuo», lo que implica que «todos los ciudadanos di-
gan al mismo tiempo las mismas palabras, esto es mío,
esto no es mío». Que todo se posea en común en la comu-
nidad política (salvo el cuerpo de cada cual), que ésta se
asemeje al hombre, cuerpo y alma unidos, que sufra \z to-
talidad incluso la herida de un solo dedo, y los ciudadanos
«serán liberados de todas las querellas motivadas por el
dinero, los hijos y las personas allegadas». La unidad, por
tanto, es condición de la armonía del todo1'. Enseguida vere-
mos que Aristóteles se opone a esta idea. Pero habremos
de recordar antes su propia concepción de lo que deba ser
la mejor forma de gobierno, resumida esquemáticamente
en las dos primeras líneas del texto: «La conservación de
los Estados es la igualdad por la reciprocidad», lo cual su-
pone que se mantenga la diferencia entre los individuos
(de ahí la insistencia en los unos y los otros), así como la alter-

2 Ver Platón, Politice, 292 d sq., y 311 b-c; y Aristóteles, Etica a Nicó-

mato, I, 1-8; Potinca. VII, 1-2.


3 Acerca de estas cuestiones y los fragmentos citados, ver Platón, Re-

pública, V, 462 i-465 b.

54
nancia (como si alternativamente...) y la «substituibilidad» de
las responsabilidades y funciones, elementos que instau-
ran la igualdad, precisamente, en el juego mismo de dife-
rencias. Aristóteles ofrece, pues, otra idea de armonía del
todo que no implica, como hemos visto, que la plurali-
dad sea reducida a la unidad1.
4. Ahora bien, la reducción generada por Platón, «des-
truye en realidad las ciudades», aún cuando se dirija a su
bien supremo. Los párrafos segundo y tercero ponen bien de
manifiesto la razón de que esto sea así. Haciendo abstrac-
ción, por una parte de la ambigüedad constitutiva de la pa-
labra «todos», y, por otra, de la fuerza del interés perso-
nal, el «modelo» platónico no puede más que conservar
en sí el fermento mismo de la discordia (y no el de la con-
cordia) y producir el desinterés (indiferencia) de todos (en
lugar de una simpatía y solidaridad activas). Así, paradó-
jicamente, está abocado a su contrario, como cuando mu-
chos (criados) (sirven) peor que pocos (criados). La comu-
nidad de sentido que Sócrates Imagina, «señal de la unidad
perfecta» de la comunidad política, no es en realidad
nada más que una diversidad conducida a la fuerza a una
identidad (un mismo juicio/un mismo sentimiento que
da como resultado una misma propiedad/un mismo inte-
rés) contraria al sentido común... Si «ambos» es equívoco
(¿querrá decir: los dos juntos como un todo?, o, ¿en el
todo, el uno y el otro, bien diferenciados?), y, de igual
modo, «pares e impares» [si 5 = 2 (par) + 3 (impar), ¿no
es acaso la operación entera a la vez par e impar?], ¿qué
podemos decir entonces de «todos», puesto que significa
cada uno y también la totalidad del (los) «cada uno(s)»? So-
bre todo si partimos del doble sentido de uno, una cosa es
la unidad (aritmética) en un conjunto, o el particular que
se singulariza como tal (cada «uno»), y otra cosa la unidad
(orgánica) de un todo que se individualiza como todo.
En este sentido, para clarificar los equívocos mencio-
nados tendremos que preguntar:
4 Textos paralelos: Etica a Nkómato, V, 8; VII], 8-16; Política, VII,

3; V.l.

55
a) ¿Qué armonía (que no sea ficticia, o «totalitaria»),
y entre qué individuos, podrá significar «un colectivo»
donde «cada uno» se encuentre anulado?
b) ¿Qué armonía (que no sea «utópica»; bella, aunque
irrealizable) entre qué propietarios puede resultar de unas
relaciones de propiedad en adelante indiferenciada (se
aplica indistintamente a todos y a cada uno, a cualquiera
desde el primer momento), cuando su función es, preci-
samente y en conexión con el interés, la de «personali-
zar» a los individuos y a la relación que entre ellos man-
tienen?
Puesto que el interés personal (a la vez motor y medida de
la acción individual frente a un gran número de personas) es,
precisamente, aquello que liga a los individuos entre sí
como personas (el padre al hijo, el esposo a la esposa, el
amo al esclavo y el ciudadano a su igual, en una «micro-
sociología» cuyo origen podemos atribuir a Aristóteles),
ese interés personal, paralelamente, identifica a «cada
uno» como sujeto. En tales condiciones, según la fórmula
platónica, «unificar» (de manera puramente formal, des-
preciando la «materia» de las relaciones humanas) consis-
te simplemente en suprimir («destruir») el objeto mismo
de la teoría platónica.
La lección de Aristóteles no puede ser más clara. En
efecto, es preciso saber lo que quiere decir hablar, sobre
todo cuando el logos dice el derecho/la ley; es preciso sa-
ber también qué es lo que quiere tener cada uno en particu-
lar, sobre todo cuando la ley garantiza el disfrute de los
bienes. En suma, es preciso saber analizar lo concreto de
la experiencia y de la historia. Este es el sentido del lla-
mado empirismo aristotélico (y su realismo). Y vemos ya
cómo se opone al idealismo platónico, que debemos enten-
der en el doble sentido que le es propio; el de un dogmatis-
mo teórico que deduce el ideal a partir de la Idea y el de un
«moralismo» ingenuo que oculta lo real de la Idea.

56
PARA SABER MÁS

Sobre Aristóteles (textos 4 y 5)

AUBENQUE, P., El problema del ser en Aristóteles, trad. Vidal Peña,


Madrid, Tauros, 1987.
BARNES, J „ Aristóteles, trad. Marta Sansigre Vidal, Madrid, Cáte-
dra, 1987.
JAEGER, W., Aristóteles, trad. José Gaos, México, FCE, 1946.
MAGEE, B., Los grandesfilósofos, trad. Amaia Barcena, Madrid, Cá-
tedra, 1990, págs. 35-59.
Ross, W, D., Aristóteles, trad. Diego F. Pro, Buenos Aires, Char-
cas, 1981.

6. Descartes

Q U E ES UNA CUESTIÓN D E PRINCIPIOS DESEMBARAZARSE


DE LA E S C U E L A . . .

(...) La mayor parte de quienes, a lo largo de los últi-


mos siglos, quisieron ser filósofos han seguido ciegamen-
te a Aristóteles, de manera que, a menudo, corrompieron
el sentido de sus escritos al atribuirle diversas opiniones
que nunca reconocería como suyas si volviera a este
mundo. En cuanto a quienes no lo siguieron, y muchos
de ellos se cuentan entre los espíritus más preclaros, no
escaparon a la influencia de sus opiniones durante su ju-
ventud, pues son las únicas que se enseñan en las escue-
las, causándoles tanta preocupación que no logran ele-
varse al conocimiento de los verdaderos principios.
Y aunque a todos los estimo, y no quisiera parecer odioso
por reprenderles, quizás pueda ofrecer una prueba de mi
cosecha (y creo que ninguno de ellos la condenaría); a sa-
ber, que todos supusieron como principio algo que no co-

57
nocían perfectamente. Así, por ejemplo, no sé de ningu-
no que no haya supuesto la pesantez en el caso de los
cuerpos terrestres, pero, aún cuando la experiencia nos
muestra con gran claridad que los cuerpos que denomi-
namos pesados descienden hacia el centro de la Tierra,
ello no nos da el conocimiento de cuál sea la naturaleza
de eso que llamamos pesantez, es decir, de la causa o el
principio que los hace descender de ese modo, luego ten-
dremos que hallarla en otra parte. Otro tanto cabe decir
del vacío y de los átomos, y de lo caliente, y de lo frío, de
lo seco, de lo húmedo, y de la sal, del azufre y del mercu-
rio, y de todas las cosas semejantes que algunos han su-
puesto como sus principios. Ahora bien, las conclusiones
que se deducen de un principio que no es evidente tam-
poco pueden ser evidentes, y ello aunque su deducción sí
lo fuera; de lo que se sigue que todos los razonamientos
que aquéllos apoyaron sobre tales principios no pudieron
proveerles de ningún conocimiento cierto acerca de cosa
alguna, ni, por tanto, hacerles avanzar en la búsqueda de
la sabiduría. Y, de haber encontrado algo verdadero, no
habrá sido por medio de alguna de las cuatro vías deduci-
das más arriba. Sin embargo, no quisiera disminuir en
nada el honor al que cada uno de ellos pudiera aspirar;
tan sólo estoy obligado a decir, para el consuelo de quie-
nes no han estudiado, que, como en un viaje en el que
volviéramos la espalda al lugar a donde queremos llegar,
nos alejaríamos de nuestro destino tanto más cuanto más
prolongada y rápida sea nuestra marcha, de manera que,
aún cuando nos encontremos luego colocados en el recto
camino, no lo alcanzaremos tan pronto como si anterior-
mente no hubiésemos caminado en absoluto. Así, cuando
los principios son inadecuados, cuanto más los cultive-
mos por encima de otros principios y nos apliquemos
con mayor cuidado a extraer de ellos consecuencias di-
versas, pensando en que en esto consiste el buen filosofar,
tanto más nos distanciaremos del conocimiento de la ver-
dad y de la sabiduría; de donde habremos de concluir que
los que menos han aprendido de esa que ha venido
llamándose filosofía hasta nuestros días, son también

58
los más capacitados para aprender la verdadera filo-
sofía.
«Los principios de la filosofía. Carta del autor a quien
tradujo el libro, la cual puede servir aquí a modo de pre-
facio» (1647),

COMENTARIO

1. Este texto presenta una singularidad que debemos


señalar de entrada, pues está en relación con uno de los
aspectos más relevantes del mismo (la convicción de la
que Descartes da prueba al asegurar un nuevo punto de
partida para la filosofía). Nos encontramos con el postíl-
elo, escrito en francés, de una obra aparecida en latín
en 1644 y traducida a) francés por el abad Picot en 1647, que
hace las veces de /wfacio donde se resume el espíritu de
una obra considerada como completa. De este modo,
Descartes «intemporiza» su filosofía al realizar la decons-
trucción de las relaciones del antes y del después, a través,
precisamente, del mismo gesto que, repitiendo el aconte-
cimiento que supuso el Discurso del Método, lo inscribe en
una historia, a partir de entonces secular, bajo el aspecto
de una ruptura que tiene lugar en la tradición; esto es,
desembarazarse, para filosofar, de la lengua de los doctos,
del latín, e impugnar desde esta actitud un cierto modo de
autoridad. ¿Se trata de un nuevo «parricidio», veinte si-
glos después de Platón? Sin embargo, el «padre», en esta
ocasión, es menos una figura (¿Aristóteles?) que -uniforma.
2. El texto aparece escrito en primera persona, algo
que el autor emplea con una insistencia significativa
(¿cómo no nos habría de recordar en adelante esta omni-
presencia del «yo» a la experiencia cartesiana de la subjeti-
vidad, a su función irreductible en el cogito y la búsqueda de
verdades metafísicas en las Meditaciones, al papel que juega
esta experiencia en la constitución de la filosofía de Des-
cartes y el advenimiento de una «modernidad» filosófi-
ca?). En efecto, el texto se presenta como una acusación
inequívoca dirigida a la «Filosofía de la Escuela», la deno-

59
minada escolástica. Ésta designa una realidad a la vez histó-
rica (pues supone el conjunto de problemas legados por
Aristóteles a la posteridad, herencia de ideas expresadas
en su debida forma a lo largo de la Edad Media y a través
de las controversias que enfrentaron entre sí a las «sectas»
de filósofos-teólogos) y pedagógica (ya que el corpas así
constituido era enseñado en la universidad y las escuelas
como un sistema de verdades dogmáticas acordes con la
razón y la fe, y, según una serie de «métodos» fijados bajo
el control y monopolio de la iglesia). Descartes emprende
su ataque desde dos ángulos simultáneamente. Así, en
nombre del conocimiento de los verdaderos principios (intempo-
rales), la historicidad de la filosofía se encuentra plena-
mente asumida en la reivindicación de otra «cultura» y de
otro «aprendizaje».
La primera parte del texto (verdaderos principios), de-
nuncia, por tanto, la ceguera de los secuaces de Aristóte-
les, o, lo que aún es peor, el modo como traicionaron su
pensamiento al enfeudarse en su doctrina; y el prejuicio
intelectual que experimentan todos aquellos que han es-
tado sometidos a sus lecciones. En suma, por un lado te-
nemos interpretaciones falseadas, por otro, los efectos del
desconocimiento. Lo anteriorconformaun «orden»depre-
\\xvc\o(de opiniones) históricamente condicionado que obsta-
culiza el camino h acia ¿i verdad y la sabiduría. Tal es la materia
que Descartes pone en juego en el fragmento que ofrecemos.
Elfinal del texto (a partir de Sin embargo, no quisiera dismi-
nuir en nada...) lo Índica con toda claridad en ese concen-
trado de retórica que el autor ofrece al lector y que consta
de algunas, breves, precauciones oratorias, de argumen-
tos que retoma de líneas superiores, elementos que sirven
de introducción a la metáfora, cuya pertinencia ya esti-
maremos, de un método (puesto que esta palabra, etimoló-
gicamente, remite a la noción de camino y marcha) para
producir, en una comparación muy estructurada (al igual
como/Así) y bien fundamentada en la generalidad del con-
cepto de método, entendido, precisamente, como arte de
conducir y desplazamiento regulado desde un punto de
partida hasta un punto de llegada determinados. Ello im-

60
plica toda una «dinámica» del caminar, una conclusión
adecuadamente razonada, si bien algo provocadora: la ig-
norancia de la (falsa) filosofía es, sin embargo, la condi-
ción de su ejercicio (verdadero).
Entre la primera parte del texto y el final (y, aunque a todos
ellos los estimo... —* ... ¿as cuatro vtas deducidas más arriba)
acontece lo esencial de la critica, segura de su prueba, la
cual hace que la restricción inicial aunque... parezca una
verdadera restricción mental, procedimiento familiar a los
casuistas que, por su parte, Pascal combatiera en sus Pro-
vinciales... En efecto, la insuficiente evidencia de los princi-
pios, en todos aquellos autores, al llevar consigo, por vía
de consecuencia, la insuficiente certeza de las conclusio-
nes, no ha permitido que progrese la búsqueda de la sabiduría
en las cuestiones principales de la física (aquí, el ejemplo
privilegiado es la pesantez). Ello indica suficientemente
que el conocimiento de los principios es un factor deter-
minante para la búsqueda de aquélla. De ahí los Principia
pbilosopkiae, de ahí la forma escolar, que no escolástica,
que Descartes a querido otorgarles.
3. Ciertamente, cuando surge el Renacimiento, a for-
tiori después de la Edad Media, el contexto del saber ha
cambiado. Descartes no puede ignorar las revoluciones
cope mi can a y galileana, como tampoco la importancia
que han adquirido en la «física» las matemáticas por una
parte, por otra la experimentación. De la geometría y el
álgebra, el magnetismo, pasando por la cosmología y la
óptica, Descartes acepta la parte que le corresponde en la
labor investigadora, o toma partido en los debates relati-
vos a tales materias. Los Principios, a lo largo de sus tres úl-
timas partes, retoman cuestiones ya abordadas con ante-
rioridad, por ejemplo, en los tratados del Mundo o del
Hombre. En estas obras, o en sus cartas, Descartes con-
fronta u opone sus propias tesis o sus «soluciones» a las de
Copérnico, Tycho Brahe, Galileo, Torricelli, Fermat,
Roberval, Gilbert o Harvey. Pero no nos confundamos,
pues, cuando Descartes escribe que tendremos que baílatela)
en otra parte la causa o el principio de la pesantez (lo mismo
cabe decir en cuanto a lo que debemos pensar del vacío y

61
los átomos, unidos tras el atomismo de Demócrito; o de
las cualidades características de los cuatro elementos, de
tal sal o metal, que han servido de «principios» en tantas
conjeturas de aristotélicos y alquimistas), no está subordi-
nado el conocimiento filosófico al saber de las ciencias de
la naturaleza. Lo que Descartes hace es más bien lo con-
trario, pues ese en otra parte no debe buscarse fuera de, sino
en la filosofía, allí donde el método, precisamente gracias
a su rigor y novedad, la ha renovado al poner en libertad
los verdaderos principios mediante la prueba de la duda y
la demostración de la existencia de Dios1, garantizando
así toda clase de verdad, incluidas las verdades matemáti-
cas, En efecto, la matemática suministra el modelo de
una ciencia del orden y la medida, y de cadenas de razo-
nes que transfieren la evidencia desde los principios a las
conclusiones, dentro de la continuidad de la deducción.
La física cartesiana será, en consecuencia, deductiva,
aunque el valor de las conclusiones dependerá, en un úl-
timo análisis, del valor de los principios, que sólo la meta-
física puede establecer (del modo como lo hace en el libro
primero de los Principios) desde la raíz del árbol del cono-
cimiento (cuyo tronco es la física, según la conocida me-
táfora2). En el fondo, toda deducción tiene doble filo, y
ello porque, si los principios son inadecuados, conducirán
al extravío de una forma tan segura como cierto el cami-
no hacia la verdad cuando aquéllos son evidentes. Des-
cartes, al hacernos ver «todas las nociones claras y simples
que están en nosotros» (a) mismo tiempo que «los princi-
pales atributos de Dios») y la metafísica, fortalecida por
su método y convertida, más que nunca, enfilosofíaprime-
ra, confirman el (nuevo) comienzo de la filosofía.
4, Pero el cumplimiento de este objetivo exige aún
algo más, que la filosofía (que por su conocimiento de los
principios engloba, pues, a todos los demás saberes, con-
1 Acerca de la duda, ver las Meditaciones Primeraj Segunda; sobre las de-

mostraciones de la existencia de Dios, la Tercera y la Quinta, Un trata-


miento conjunto de las dos cuestiones se encuentra en los Principios,
1.* parte, SS 1 " 2 1 *
2 Cfr. Carta-Prefacio, op. cit., pigs, 779-780.

62
siderados así como consecuencias de su metafísica) se ele-
ve a un quinto grado, en su marcha en pos de la sabiduría,
sobre los otros cuatro cuyas vías han sido deducidas más arri-
ba (la intelección inmediata, la experiencia de los senti-
dos, lo aprendido de oídas y la lectura, siendo de un or-
den distinto la revelación divina)3. La exigencia no es
otra que la de volver al método (ahí debemos recuperar,
sin lugar a dudas, la originalidad de Descartes) y a la im-
portancia de las Regias para ia dirección del Espíritu. Al me-
nos, esto será así si, como el texto parece sugerir, se trata
en el fondo de saber distinguir el conocimiento verdade-
ro de la mera experiencia; de saber, pues, más radical-
mente, aprehender con toda su fuerza las condiciones de
la evidencia en el punto de partida mismo de cualquier
certeza; de saber suponer (libremente, poner debajo, como
soporte y fundamento) con total conocimiento en orden
a ser capaces de razonar «sabiamente». Tal es el precio de
una filosofía entendida como «ciencia rigurosa» para la
que las verdades metódicas no son, ni segundas, ni secun-
darias. Ello implica también regresar a la pedagogía, por-
que filosofar (otros lo volverían a decir, más tarde)" no es
aprender un saber, sino aprender a pensar. Quizá se com-
prenda ahora la aparente paradoja de] final del texto. La
ignorancia, al menos la ingenuidad, no es contraria al
ejercicio filosófico, pues ofrece una tabula rasa, que no
puede sino favorecer la «revolución» necesaria ante la
fuerza de los prejuicios.

PARA S A B E R MAS

Sobre escolástica
LIBERA de A., Laphilosopbie du Mayen Age, París, PUF, Que sais-
fe? núm. 1044, 1989.

3 Jfo'd,, pág. 772.


4 Por ejemplo, Kant, ver Armonctptmr U tours stmestrit! d'hieer 1765-66,
en CEuvwphjiasophtqut! (Obras filosóficas), Bib). de la Pléiade, 1980, t. 1,
pigs. 514-515.

63
Sobre Descartes

ALQUIÉ, F , , Descartes, Fbonme et l'oeuvre, París, PUF, 1956.

PARA PROFUNDIZAR

GOUTHIER, H., La pensét métaphysique de Descartes, París, Vrin,


1962; y también Laporte J., Le ratienisme de Descartes, París,
PUF, 1945.

7. Descartes
Q U E NO NOS PODEMOS C O N T E N T A R
CON LO Q U E DICEN LOS ANTIGUOS PARA S E R DICHOSOS...

Ahora bien, entre los filósofos paganos han existido


tres opiniones principales en lo tocante al soberano bien
y al fin de nuestras acciones, a saber: la de Epicuro, quien
lo identificó con la voluptuosidad; la de Zenón, quien
quiso que lo fuera la virtud; y la de Aristóteles, quien lo
compuso a partir de todas las perfecciones, tanto del
cuerpo como del espíritu. Estas tres opiniones pueden,
me parece, ser aceptadas como verdaderas y concordan-
tes entre sí, siempre que se las interprete favorablemente.
Puesto que Aristóteles tomó en consideración el sobera-
no bien de toda la naturaleza humana en general, esto es,
aquel que puede encontrarse en el más cabal de los hom-
bres, estaba en lo cierto al componerlo de todas las per-
fecciones de las que el hombre es capaz. Pero lo anterior
en nada sirve a nuestros própositos,
Zenón, por el contrario, consideró aquel bien que cada
hombre en particular puede poseer. Es por ello que tam-
bién él tuvo una muy buena razón para decir que consiste
nada más que en la virtud, porque, entre los bienes que
podemos poseer, tan sólo ella depende enteramente de
nuestro libre albedrío. Sin embargo, representó esta vir-
tud tan severa y enemiga de la voluptuosidad, al hacer

64
1

iguales a todos los vicios, que sólo los melancólicos, o los


espíritus enteramente despegados del cuerpo, pudieron, a
mi parecer, contarse entre sus seguidores.
Por último, Epicuro, considerando en qué consiste la
felicidad y cuál es su motivo o el fin al que nuestras accio-
nes tienden, no se equivocó al decir que consiste en la vo-
luptuosidad en general, esto es, en la satisfacción del espí-
ritu; porque, si bien el solo conocimiento de nuestro de-
ber podría obligarnos a realizar acciones buenas, no nos
proporciona, sin embargo, el disfrute de felicidad alguna
si con ello no obtuviéramos ningún placer. Pero, como
sea que a menudo se atribuye el nombre de voluptuosidad
a los falsos placeres, que vienen acompañados o seguidos
de inquietud, de tedio y arrepentimiento, muchos han
creído que la opinión de Epicuro practicaba la enseñanza
del vicio; y, en efecto, no enseña la virtud (...) Es por esto
que creo poder concluir aquí que la felicidad no consiste
nada más que en la satisfacción del espíritu, es decir, en la
satisfacción en general, pues, aunque existan satisfaccio-
nes que dependen del cuerpo, y otras que en nada depen-
dan de él, no hay, sin embargo, ninguna que no lo sea en
el espíritu. Para que la satisfacción sea sólida es necesario
seguir la virtud, o, lo que es lo mismo, tener una voluntad
firme y constante en orden a ejecutar todo aquello que
juzguemos como lo mejor, y emplear toda la fuerza de
nuestro entendimiento con el objeto de juzgar correcta-
mente.

(Carta a Elisabeth del 18 de agosto de 1645).

COMENTARIO

1. No puede sernos indiferente que el presente texto,


que trata de filosofía moral, posea una destínataria (pues
está extraído de una carta). En esta materia, más que en
cualquier otra, la generosidad y la aparente abstracción
de teorías concernientes, en última instancia, a la muy

65
concreta singularidad de las personas, así como a sus elec-
ciones, fines y razones (o desatinos) en ta conducta de sus
vidas. La cuestión que el autor aborda, cuestión clave en
todas las filosofías morales (si la totalidad de nuestras ac-
ciones se definen por su fin, y si el supremo no puede ser
otro que el Soberano Bien, ¿cómo podremos, o tendremos
que definirlo?), desborda los límites de la mera relación
epistolar para actualizar el verdadero interés de la investi-
gación teórica: ¿acaso no es un objetivo promordial el de
instaurar una comunidad de pensamiento y sabiduría que
sea vivida en el ejercicio de una práctica efectiva? Ha lle-
gado el momento, quizás, de llamar la atención sobre una
de las funciones de las «cartas filosóficas», entendiéndolas
no como género literario, sino como vehículo de la prác-
tica mencionada. Por ellas o en ellas se enfrentan, tras la
apariencia y a pesar de la fuerza de las convenciones, las
diferencias de edad, de sexo y de estatus social (el filósofo
conversa, en una completa reciprocidad, con una prince-
sa veinte años menor que él), como también la distinción
entre la esfera de lo privado y la de lo que es de dominio
público. En efecto, aquello que está destinado a un uso
particular, reaparece inseparablemente unido a una «uti-
lidad» universal (¿no procede la publicación de las Pasio-
nes del Alma, fechada en 1649, de una profundización en
las cuestiones suscitadas por Elisabeth a raíz de la presen-
te correspondencia privada?).
2. El contenido del texto se despliega con toda clari-
dad, tanto en su economía general como en cada una de
las partes en que se diferencia. Tras un primer párrafo
que anuncia el objeto del examen aún por iniciar^ y que no
es otro que una comparación entre tres grandes doctrinas
morales de la antigüedad, identificadas del modo más
conciso posible según su concepto de soberano bien, y
tras señalar su objetivo (un ensayo de síntesis subordinado
a una «buena» lectura), se suceden (2, 3 y 4) una serie de
apreciaciones, más o menos argumentadas, pero siempre
contrastadas entre sí {no se equivocó o estaba en lo cierto res-
ponde en cada paso a un pero o sin embargo que limita los
elogios e introduce las críticas que condicionan la sínte-

66
sis). A través de tales consideraciones se va construyen-
do, progresivamente, por el hecho mismo de sus oposi-
ciones, la solidaridad entre el aristotelismo, el estoicismo
de Zenón y el epicureismo. Cabe preguntarse, sin embar-
go, sí, al llegar a la conclusión del último párrafo las opi-
niones mencionadas «concuerdan» realmente entre sí,
pues, tras el velo de su «composición», ¿no se estará for-
mulando quizás la moral puramente cartesiana? Los tér-
minos en los cuales intervienen, entre comillas, la duali-
dad de entendimiento y la voluntad invitan a creerlo asi.
Debemos señalar, sin embargo, que tal dualidad no apa-
rece expresada aquí con la misma seguridad como en
otros lugares de la obra de Descartes, donde las conclu-
siones son deducidas a partir de principios evidentes. Al
introducirlas mediante una fórmula «prudente» (creo poder
concluir aquí...), Descartes reinscribe su moral en el mismo
registro que la de losfilósofospaganos, el registro de la opi-
nión. A pesar de hacerla remitir a las verdades eternas
creadas por Dios, a pesar de que constituya la última
rama (la más elevada y perfecta) del árbol filosófico, ta
«ciencia moral», para Descartes, continúa siendo esen-
cialmente problemática. Volveremos sobre este punto al
término del presente comentario.
3. En lo que sigue tendremos que examinar aquello
que Descartes retiene y destaca de las doctrinas objeto de
su evocación. Pero conviene señalar, en primer lugar,
que el autor las presenta desde dos órdenes inversos:
a) En el párrafo introductorio pasamos de Epicuro a
Zenón y Aristóteles según grados que parecen ser de mo-
ralidad creciente, pues ¿no es acaso la virtud más moral
que la voluptuosidadf?, y, ¿no es cierto que son más valiosas
todas las perfecciones que la sola virtud?
b) Y bien, que la jerarquía mencionada se desglose en
la exposición del texto a lo largo de los tres párrafos si-
guientes (de Aristóteles y Zenón a Epicuro), indica, qui-
zás, que es necesario deshacerse de las ideas recibidas,
tanto en esta materia como en otras...
Debemos, sin duda, partir de las restricciones expresa-
das por Descartes, en cada caso, en orden a juzgar correcta-
67
mente en lo que a nosotros toca (así concluye el texto), si
queremos obtener una moral a partir de esta confronta-
ción de morales.
a) El caso de Aristóteles es despachado de un modo
seco y frío. En efecto, sin el soberano bien carece de utili-
dad práctica alguna, pues su autor lo concibe desde la abs-
tracción de la generalidad. Es cierto que la Etica a Nicóma-
co no facilita reglas de acción para los individuos, y ello
porque su propósito fundamental (de nuevo contra la se-
paración operada por Platón entre la idea del Bien o las
situaciones particulares donde puede manifestarse su rea-
lidad) 1 no es otro que el de realizar un inventario del con-
junto de condiciones de las cuales depende su realización.
b) El caso de Zenón es objeto de «sospecha»... Su defi-
nición del Soberano Bien es reduccionista y unilateral, pues
excluye la voluptuosidad, quedando únicamente la virtud
en oposición radical y desprovista de matices a todos los
vicios2. Su mérito, ciertamente, consiste en poner en co-
nexión la virtud con el poder real y particular de cada in-
dividuo, así como en preparar la distinción, tan operativa
en el orden práctico, entre lo que depende y lo que no de-
pende de nosotros1. No obstante, tan sólo quienes poseen
un humor mórbido o un total despego por la carne pue-
den amar la virtud del estoicismo, que, de este modo, po-
dría considerarse como un vicio de constitución. Cuando
la disociación entre el cuerpo y el espíritu es tan acusada
como en esta doctrina, es difícil comprender cómo po-
dría no resultar dañada la dignidad del individuo, siendo
entonces imposible la concordancia entre la virtud estoi-
ca y la aspiración a la plenitud del bien,
c) Y, por último, Epicuro. La crítica a su doctrina es
breve. Al contrario que Zenón, en efecto, no enseña la virtud...
Pero esta brevedad, algo irónica, es posible que nos esté

1Ver Aristóteles, Etica a Nkimaco, 1, 4-12.


2 Ver Séneca, La vida feliz, 'rad. Julián Marías, Madrid, Alian-
za, 1980,
1 Ver Epicteto, Pláticas per Amano, Irad. de Urriesy Azara, Barcelona,

Alma Mater, 1957-1965.

68
advirtiendo que se debe, más bien, elogiar a Epícuro (una
vez rectificadas las falsas interpretaciones de su hedonismo,
ya que el verdadero placer, para Epícuro, es aquel que
nace de un espíritu necesariamente satisfecho con su
cuerpo, él mismo en paz dentro del equilibrio y armonía
de sus partes), y elogiarlo por hablar de la esperada felici-
dad, de beatitud, entendida como meta razonable de la ac-
ción moral, proponiendo un eudemonismo consecuente que
no hace abstracción del placer en el «juicio»". En este
punto conviene advertir la cautela con la que Descartes
aborda el epicureismo: Epkuro... no se equivocó al decir...
aquello para lo que Zenón tuvo una muy buena razón...
4. Las consideraciones anteriores permiten precisar
las líneas generales de la moral cartesiana que aparece,
apenas esbozada, en la conclusión.
Descartes privilegia, de forma incontestable, la versión
epicúrea del Soberano Bien según la lectura que de ella
hace. En efecto, Descartes retoma la felicidad como fin,
felicidad que es satisfacción del espíritu y satisfacción en general.
No nos confundamos; de ser posible la existencia de satis-
facciones puras del alma (en tanto que inmaterial) y re-
sultado, por ejemplo, de la sola actividad del pensamien-
to, no podría haber en ellas satisfacción alguna (es decir,
en el sentido tanto físico como moral, de completitud,
plenitud y contento5) del cuerpo sin la satisfacción del es-
píritu, pues el cuerpo está siempre unido al alma.
De este modo, todas las especies de placer, dependan o
no del cuerpo, participan del género espiritual. De ello re-
sulta que no hay lugar (ni medio) para dejar fuera de jue-
go a las pasiones, como hacen los estoicos con su «apa-
tía»6, si se pretende alcanzar el contento del espíritu. Al
contrario, de lo que se trata es de hacer un buen uso de ta-
les pasiones. Aquí interviene la virtud, tomada en aparien-

4 Ver Epicuro, Carta a Menéete jr Máximas capitulares, trad. R. Ojeda y

A. Olabenaga, Madrid, Alhambra, 1985.


5 Texto paralelo: Carta a Elisabttb del 4 de agosto de 1645.

' Ver Cicerón, Cuestiones académicas. Tratadofilosófico,trad. Agustín Mi-


llares Cario, Madrid, Espasa-Calpe, 1919.

69
cia de Zenón, aunque en un sentido que tampoco es estoi-
co, porque ya no es el fin que se confunde con el Sobera-
no Bien, sino el medio de llegar a ella y fortalecerla, medio
que conduce a la fuemp de nuestro entendimiento (instruido o
movido por los verdaderos principios, si es que se trata,
ante todo, de juzgar correctamente), aplicada a orientar en la
buena dirección, todo el tiempo que sea necesario, la firme-
za de una voluntad constante. Sin embargo, es comprensible
que no pueda haber una ciencia de esta clase de improba-
ble «mecánica»: en su origen encontramos el libre albe-
drio de un ser finito, para el cual lo mejor no deja de ser
un circunstancial, de manera que para acceder a él ten-
dremos siempre la urgencia de comprometernos y arries-
garnos a suspender (en exceso) el juicio. En el origen des-
cubrimos, de igual modo, ese compuesto imposible de
descomponer que es la unión de cuerpo y alma, verdade-
ro sujeto de la acción moral y no tan fácil de conocer
como la simplicidad del pensamiento o la extensión. De
ahí la importancia, sea ello lo que fuere, para Descartes y
sus lectores, de una moral provisional cuyas reglas aparecen
definidas en el Discurso del método''.

PARA SABER MAS

De Descartes

Lettres sur la moraie, ed J. Checalier, Hatier-Boivin, 1935 (Corres-


pondencia con Elisabeth, Cristina de Suecia, Chamut).

Sobre Descartes

RODIS-LEWIS, G., La morak de Descaries, París, PUF, 1957,


3.' ed., 1970.

7 Discutió dt¡ método, 3.a parte.

70
Sobre estoicismo y epicureismo

BRÉHIER, E., Éiudes depbihsopbie anticue, 1955; también: G. RO-


dís-Lewis, La moraie Stokienne, París, PUF, 1970.

8. Descartes
M E N S SANA IN CORPORE SANO

Pero tan pronto como hube adquirido algunas nocio-


nes generales relativas a la física y comprobado, al co-
menzar a ponerlas a prueba en diversas dificultades parti-
culares, hasta qué punto difieren de los principios hasta
ahora usados, creí que conservarlas ocultas era grandísi-
mo pecado contra la ley que nos obliga a procurar el bien
general de todos los hombres, en cuanto esté en nuestro
poder. Puesto que tales nociones me han hecho ver que
es posible llegar a conocimientos muy útiles para la vida y
que, en lugar de la filosofía especulativa, enseñada en las
escuelas, es posible hallar una práctica, mediante la cual,
conociendo la fuerza y las acciones del fuego, del aire, de
los astros, de los cielos y de los restantes cuerpos que nos
rodean, tan distintamente como conocemos los diversos
oficios de nuestros artesanos, podríamos emplearlas del
mismo modo, en todos los usos a los que sean propias, y
asi hacernos como maestros y poseedores de la naturale-
za. Ello es deseable, no sólo por la invención de una infi-
nidad de artificios que nos permiten disfrutar sin esfuer-
zo los frutos de la tierra y de todas las comodidades que
en ella se encuentran, sino también, principalmente, por
la conservación de la salud, que, sin duda, es el primer
bien y el fundamento de los demás bienes de esta vida,
porque el propio espíritu depende tanto del temperamen-
to y de la disposición de los órganos del cuerpo que, si es
posible algún medio que haga a los hombres comúnmen-

71
te más sabios y más hábiles de lo que hasta ahora han
sido, creo que es en la medicina donde se debe buscar.

Discurso del método. Meditaciones metafísicas, Sexta


parte. Traducción, prólogo y notas de Manuel
García Morente, Madrid, Espasa-Calpe, Selec-
ciones Austral, 1980, págs. 85-86.

COMENTARIO

1. Este texto tan bello ha sido más comentado que


leído, más traicionado que lealmente descifrado. Se ha
creído ver en él el proyecto de la dominación planetaria
que «la técnica» habría terminado por establecer. En oca-
siones, se buscan circunstancias atenuantes en la pruden-
cia de ciertas expresiones (como maestros). Tan sólo el exa-
men del propósito mismo que encierra el texto puede ha-
cer justicia a la cuestión de la intención. Descartes se
siente obligado, afirma, a comunicar al público el resulta-
do de sus pensamientos y razonamientos en el terreno de
la física. Para ello, Descartes procede desde el bien gene-
ral, de «la utilidad de la vida», del progreso de la humani-
dad en sabiduría y habilidad que cabe esperar de las apli-
caciones de la ciencia de la naturaleza, hasta llegar a la
medicina. El secreto de estas mejoras reside en la posibili-
dad, ofrecida por un conocimiento más adecuado de los
principios de la filosofía natural, de emplear la naturaleza
misma en servicio de nuestros fines. El texto se divide en
tres consideraciones encadenadas:
— El sentimiento de obligación del autor de comuni-
car sus pensamientos en una materia que atañe al bien co-
mún (Pero tan pronto... -* ...En cuanto esté en nuestro po-
der).
—La naturaleza particular de una filosofía práctica de-
rivada de la física que permite tratar a la propia naturale-
za como si fuera propia al uso humano, a la manera de los
oficios de nuestras artesanos (Puesto que tales nociones... ~~* ...de la
naturaleza).

72
—Por último, la facilidad y comodidad así adquiridas
en el disfrute de las cosas de la naturaleza tienen su mejor
uso en la medicina, que permite alcanzar la salud del
cuerpo y del propio espíritu.
2. El primer propósito parece reclamar una preci-
sión, la de determinar cuál sea la ley que nos obliga a contri-
buir al bien común, pues Descartes no insiste, en el texto
que nos ocupa, en el contenido del conocimiento, sino en
la orientación de sus principios y la solidaridad humana.
Ninguna ley civil puede obligarlo a divulgar sus pensa-
mientos. Se trata necesariamente de un problema moral,
de una ley que no está escrita. Se podría pensar en la cari-
dad sí estuviera en juego la salud del alma, pero la conti-
nuación del texto excluye esta posibilidad. Es la relación
entre la humanidad (todos los hombres) y cada individuo
lo que obliga a Descartes a procurar el bien general a través de
su aportación particular; es decir, a través de, en este
caso, un esfuerzo filosófico cuyo principio no es ninguna
curiosidad individual incomunicable, sino que ya en sí
misma comprende la realización de un interés universal.
Es necesario, por tanto, plantear la solidaridad de todos
los hombres en término de Ley de los pensamientos y ac-
ciones particulares, como lo afirma explícitamente Des-
cartes algunas líneas más abajo (que no aparecen en el
texto): yjuntando así las vidas y los trabajos de parios, (llegásemos)
todosjuntos mucho más allá de donde puede llegar uno en particular.
Este párrafo es indisociable de un debate entablado algu-
nas líneas más arriba, a propósito del interés general en lo
que respecta a los pensamientos de los individuos. Los
«soberanos» y los profetas «se encargan del bien público,
no porque siempre tengan razón, sino, al contrario, por-
que cada cual cree ser, en todo momento, el mejor juez en
materia de reforma social, de suerte que es necesario ca-
llar, y dejarles hacer, guardando para sí sus pensamientos
reformadores propios. Descartes explica aquí por qué
rompe su silencio cuando se trata del bien público, mate-
ria para la cual no bastan los pensamientos comunes, in-
cluidos los de los soberanos y profetas. ¿Cómo podrían
adivinar ellos aquello que una vida consagrada a la ínves-

73
tigación apenas sí alcanza a entrever? En este punto, Des-
cartes plantea un problema que Galíleo y Giordano Bru-
no afrontaron dolo rosamente.
3. Habiendo rechazado una cobarde prudencia que
sería pecado, Descartes indica cuál, en substancia, es su
descubrimiento. No trata, en efecto, de la cuestión de los
profetas (la vida eterna) o de los soberanos (el orden pú-
blico), sino del interés de los particulares en la medida
que la vida es su preocupación. Una filosofía, en el senti-
do del siglo xvu, es práctica si comporta alguna utilidad
para el ser humano viviente. Aquí Descartes razona por
analogía; nuestros artesanos contribuyen con el ejercicio
de sus oficios tanto a las necesidades como a hacer la vida
más agradable. Conocemos tales oficios (una de las notas
características del método cartesiano es el de considerar
el oficio como ejemplo de tarea regulada y de conoci-
miento «distinto», cfr. Regulae, regla X).
Si conociésemos la naturaleza en sus procesos del
modo como conocemos los oficios, los elementos circun-
dantes se convertirían en otras tantas fuentes de posibili-
dad de utilidad para el hombre, quien, así, estaría en una
relación con la naturaleza igual que con aquellos oficios
que los artesanos poseen y de los que son «maestros».
Pero aquel que conoce el oficio de los artesanos, ¿es
maestro, lo posee? Cabría dudarlo, pero Descartes no pa-
rece albergar tales reservas.
4. Dentro de los límites de lo que ni tan siquiera es
un proyecto, sino una simple extrapolación (podríamos...),
Descartes examina finalmente las ventajas que cabe espe-
rar de una orientación que apunta hacia el conocimiento
de la naturaleza. Haciendo caso omiso en esta materia a la
ley divina y a los profetas, nuestro autor examina en este
mundo inferior el final de una existencia penitenciaria
consagrada a un trabajo artificialmente hecho punible
por decreto superior. Si pudiéramos disfrutar de un cono-
cimiento científico de las fuerzas que componen el medio
ambiente (aire, agua,...), esos «artificios» harían benefi-
ciarse al hombre de los usos que les sean propios (no se trata
de modificar la naturaleza, sino de sacar provecho de

74
ella) pero sin esfuerza, algo que no atenta contra las leyes de
la naturaleza ni contra el interés del viviente, tan sólo
acaso contraría a las Escrituras en el dominio secular al
que Descartes pretende aplicar los resultados de sus pen-
samientos. Si sus palabras son escandalosas no es porque,
tres siglos y medio después, una «política» de desarrollo
tecnocientífico haya planteado problemas relacionados
con el medio ambiente. Es, desde la perspectiva de su
tiempo, a causa de la contumaz rivalidad establecida en-
tre filósofos y profetas en torno a la definición del «bien
común».
Pero Descartes en modo alguno se detiene en esta pri-
mera parte de su extrapolación; juzga que también, princi-
palmente, ello beneficiaría a la conservación de la salud
que interesa a todo el mundo y condiciona los demás bie-
nes de esta vida. Entre ellos, Descartes cita el espíritu mis-
mo, a su sabiduría y su habilidad, en un siglo de locuras
homicidas y de guerras civiles que acortan la vida y alte-
ran la salud, pero que, sobre todo, proceden de un espíri-
tu ya alterado por las pasiones. Ciencia y técnica contra
las pasiones reformadoras y la locura supersticiosa; tal es
el programa filosófico de Descartes (un espíritu sano en
un cuerpo sano), que parece seguir manteniendo toda su
actualidad.

PARA SABER MÁS

Sobre Descartes (Textos 6, 7y S)

CLARKE, D. M., La filosofía de la ciencia en Descartes, trad. Eloy


Rada, Madrid, Alianza, 1986.
GARIN, E., Descartes, trad.J. Martínez Gázquez, Barcelona, Críti-
ca, 1989.
GÓMEZ PIN, V., Conocer DescartesJ su obra, con la colaboración de
Michel Le Doeuf y javier Echevarría, Barcelona, Dope-
sa, 1979.
MAGEE, B., Los grandesfilósofos, trad. Amaia Barcena, Madrid, Cá-
tedra, 1990, págs. 53-100.

75
Sobre la filosofía medieval

GILSON, E-, El espíritu de ¡afilosofíamedieval, trad. del francés, Ma-


drid, Rialp, cop. 1981.
WEINBERG, J., Breve historia de la filosofía medieval, trad. Carlos La-
guna, Madrid, Cátedra, Col. Teorema, 1987.

Sobre estoicos y epicúreos

ELORDUY, E . y PÉREZ ALONSO, J . , Biestoicismo, Madrid, Gredos,


1972, vols. 1 y 2.
EPICURO, Carta a Mernceo y Máximas capitales, trad. R. Ojeda y
A. Olabuenaga, Madrid, Alhambra, 1985.
LONG, A. A., Filosofía helenística: estoicos, epicúreos, escépticos, trad.
P, Jordán de Urríes, Madrid, Revista de Occidente, 1977.
SÉNECA, La vidafeliz, trad. Julián Marías, Alianza, Madrid, 1980.

9. Spinoza

S O B R E D E S C A R T E S , QUIEN HA ERRADO SIN EMBARGO


EN SUS PRINCIPIOS

Pero, puesto que podemos unir a una voluntad cual-


quiera un movimiento cualquiera de la glándula y, en
consecuencia, de los espíritus animales, y que la determi-
nación de la voluntad depende de nuestra sola potestad,
si determinamos a nuestra voluntad mediante juicios fir-
mes y ciertos siguiendo a los cuales queremos dirigir las
acciones de nuestra vida, y añadimos a tales juicios los
movimientos de las pasiones que queremos tener, adqui-
rimos un imperio absoluto sobre nuestras pasiones. Tal es

76
el punto de vista de este hombre tan célebre (según puedo
conjeturar a partir de sus palabras), y apenas sí hubiera
podido creer que provenían de un hombre tan eminente
si fueran menos sutiles. En verdad no puedo dejar de
asombrarme ante un filósofo que, tras haber resuelto fir-
memente no deducir nada sino principios evidentes por
sí mismos ni afirmar nada que no percibiera clara y dis-
tintamente, tras haber reprochado con tanta frecuencia a
los escolásticos el querer explicar cosas oscuras por cuali-
dades ocultas, admite una hipótesis aún más oculta que
cualquier cualidad oculta. ¿Qué entiende, me pregunto,
por la «unión» del alma y del cuerpo? ¿Qué concepto cla-
ro y distinto tiene de un pensamiento tan estrechamente
ligado a una cierta, pequeña porción de cantidad?. Qui-
siera que hubiera explicado esta unión por su causa próxi-
ma. Pero había concebido el alma como distinta del cuer-
po, y de tal forma que no puedo asignar causa singular al-
guna ni a esa unión ni al alma misma, y le fue necesario
recurrir a la causa de todo el universo; es decir, a Dios.
Quisiera, además, saber cuántos grados de movimiento
puede imprimir el alma a aquella glándula pineal y qué
fuerza la mantiene suspendida, (...) Y, ciertamente, no
habiendo ninguna medida común entre la voluntad y el
movimiento, tampoco puede haber ninguna compara-
ción entre la potencia — o fuerzas— del alma y la del
cuerpo. Por consiguiente, las fuerzas de este último no
pueden estar dirigidas por las de aquélla. Añádase a lo an-
terior que se busca en vano una glándula situada en me-
dio del cerebro de manera que pueda ser mutada de un
lado a otro con tanta facilidad y de tantos modos, y que
no todos los nervios se prolongan hasta las cavidades del
cerebro. Dejo de lado, por último, todo lo que Descartes
afirma acerca de la voluntad y su libertad, pues ya he
mostrado sobradamente que es falso. Así, por tanto,
como el alma se define, según lo he mostrado más arriba,
por la sola capacidad de conocer los remedios contra los
efectos, remedios de los que todos tienen, creo, alguna
experiencia, pero que no observan con cuidado ni ven
con distinción, por el solo conocimiento del alma los de-

77
terminaremos, y de ahí deduciremos todo lo que concier-
ne a su felicidad.
Etica (1677). Quinta parte, De la potencia del
entendimiento o de la libertad del hombre.
Prefacio.

COMENTARIO

1, No podemos dudar ni de las intenciones ni de la


importancia de un texto como éste. Situémoslo antes de
comenzar. Nos disponemos a introducirnos en la última
parte de uno de los esfuerzos más extraordinarios del
pensamiento en toda la historia de la filosofía: Todo se ha
puesto en su lugar (de more geométrico: según el método de-
mostrativo de los geómetras) con el objeto de que la filo-
sofía pueda desarrollar su mayor potencia, en la unidad
de sus aspectos teórico y práctico, de modo que permita
al conocimiento su realización como libertad (que nos re-
mite al título de este libro V) con la misma eficacia. No se
trata (Spinoza lo había precisado poco antes del comien-
zo del texto) de medicina ni de lógica, ni —añadimos no-
sotros— tampoco de teología, pues el desarrollo de la
obra progresa desde el saber (Libro I: De Deo) hasta el
amor intelectual de Dios. Se trata más bien de la consecu-
ción de la felicidad (última palabra del texto), y en conse-
cuencia, de una Ética (al igual que en Aristóteles) que im-
plica necesariamente cuestiones (instauradas por Platón)
de la metafísica (¿cuál es la Causa? ¿el Bien, el Ser, lo Uno,
la substancia, el acto y la potencia, el actuar y el padecer,
el movimiento?...) así como las exigencias, impuestas por
Descartes, del método concerniente al valor de los princi-
pios y de las consecuencias. Estamos, en suma, en el cora-
zón del cartesianismo, pues se trata al fin y al cabo de ver
distintamente y deducir. Mas he aquí que se desvela todo el
asunto, pues también se trata de ir contra Descartes (¡en
nombre de sus principios!). Momento decisivo donde el
cartesianismo se prolonga hasta convertirse en su contra-
rio. No podemos entonce^ adentrarnos en el texto sin to-
mar medidas de precaución.

78
2. a) El punto de partida (... según... sus palabras) es
íntegramente cartesiano, una reexposición fiel de lo que
Descartes (ese hombre tan eminente) describiera en las
Pasiones del Alma (1.a parte, artículos 27 a 50) respecto al
dominio de las pasiones y, en conexión con esta meta, las
relaciones entre alma y cuerpo mediante la glándula pineal,
y también las relaciones entre voluntad y el entendimien-
to (ya dentro del alma).
b) En el movimiento mismo de la frase aparece, sú-
bitamente la emoción: difícilmente hubiera podido creer que pro-
venían de un hombre tan eminente..,—*... no puedo dejar de asom-
brarme (la «admiración» en el sentido cartesiano es «la pri-
mera de las pasiones»...).
c) Pero Spinoza no se deja llevar por la admiración,
como lo confirman las palabras que siguen en el texto
{que unfilósofoque había decididofirmementeno deducir nada sino
de principios evidentespor sí mismos, ni afirma nada que no percibie-
ra clara y distintamente... —*•.., hasta las cavidades del cerebro),
que no son polémicas, sino expresión sincera de una crí-
tica rigurosa. Spinoza señala a continuación*.
— Que Descartes ha faltado a la más fundamental de
las reglas del método (no deducir nada... etc), lo que le hace
acreedor de las mismas críticas que aquél había dirigido a
los «filósofos de la Escuela» (explicar cosas oscuras mediante
cualidades ocultas).
— El ejemplo más sospechoso de este recurso a lo
oculto, que exige en efecto que uno se interrogue acerca de
la inteligibilidad de la unión entre alma y cuerpo (tal que
además se concentra y particulariza en la región de la
glándula pineal) que no es ni clara ni distinta y, sin embar-
go, juega un papel de central importancia en la problemá-
tica de las Pasiones del Alma.
— Las razones de esta oscuridad pueden hallarse en
un defecto de la explicación (no hay causa próxima de este
modo de existencia), que aparece disfrazado por el exceso
(el recurso a Dios) y la ausencia de toda explicación (al
nivel de la glándula pineal) de la esencia mecánica del fe-
nómeno descrito (los grados del movimiento, la fuerza).
d) A continuación y como por casualidad, pero muy
79
a propósito (Por último omito todo lo que afirma...-^*... que es fal-
so), Spinoza nos hace recordar el punto más importante
(que el autor sólo decide omitir en orden a invitarnos a re-
hacer todo el camino): la tesis cartesiana sobre la voluntad y
la libertad (núcleo central de su teoría moral) es falsa.
e) La conclusión (Asipues... —hasta el final) se im-
pone al señalar que no se puede elaborar una Etica com-
pleta que no conciba como único fundamento el conoci-
miento distinto del Alma (y, a través de él, el conocimiento
distinto de la voluntad). Según esta condición, la filosofía
es con respecto al Alma lo que la medicina al Cuerpo (reme-
dios contra los efectos, fu ndados enlaexperiencia común/ en la
razón). Reactualización, en suma, de una antigua analogía.
Podemos comprobar que son muchas las dificultades
que entraña el comentario de un texto como el presente,
pues pre-supone una familiaridad más o menos sólida
con la teoría cartesiana, así como la comprensión de los
cuatro libros precedentes de la Etica, anteriores a este
«prefacio» y a los que hace referencia explícita. Así, no te-
nemos más remedio que contentarnos con limitar nues-
tra exégesis a lo esencial, clarificando algo más los aspec-
tos y elementos principales propuestos por Spinoza en su
crítica a Descartes.
3. La respuesta de Spinoza consiste en extraer argu-
mentos, de forma bien evidente, a partir de todo lo que
ha establecido ya su propia deducción desde las definicio-
nes y axiomas que sirvieron de inicio a la Etica (y a cada
una de sus partes). La glándula pineal (y así comenzamos
por el elemento más característico), tal y como Descartes
lo concibe y hace «funcionar»', es, diríamos, una pura fic-
ción imaginada a propósito de la exigencia de la propia
teoría. Hemos de subrayar que, según Spinoza, la imagi-
nación surge del registro de ideas inadecuadas —correlati-
vas de afecciones corporales que implican la acción de
cuerpos exteriores, de modo que el conocimiento que
originan es confuso2—, al igual que en el caso de las otras

1Cfr. Las Pajionts itt Alma, ar. 31 y 55.


2 Cfr. Éíuo, libro II, 5 XVI-XXV1, y Parte III, §111.

80
pasiones. En cualquier caso, el lugar teórico que le pres-
cibe el sistema cartesiano, esto es, la acción real del alma
sobre el cuerpo y recíprocamente, el hecho de la unión
tan estrecha que forman en el hombre, unidad substan-
cial que Dios ha querido a pesar de la distinción real en-
tre substancia pensante y substancia extensa3, es, a los
ojos de Spinoza, inadmisible. Explicitar la coherencia de
la substancia una (Dios) implica necesariamente hacer del
pensamiento y de la extensión los dos únicos atributas de
la substancia a través de los cuales ésta puede ser concebida;
significa admitir que no puede haber relación causal de
una sobre otra, que toda explicación por la causa próxima4,
indispensable para todo lo que existe por sí, sólo puede
cumplirse de un modo a otro modo (de la substancia),
desde el pensamiento o desde la extensión. En suma, ia
concepción spinoziana de la unión entre alma y cuerpo
que resulta de tales exigencias lógicas, su paralelismo5, no
puede llevarse a cabo si no es rechazando toda determina-
ción del alma (en el pensamiento) por el cuerpo, o del
cuerpo (movimiento o reposo) por el alma6. Pero he aquí
lo esencial (pues la metafísica, aquí, se hace inmediata-
mente práctica): de lo anterior resulta también que la li-
bertad de la voluntad es una ilusión y con ello se desvanece el
sentido de la tesis recogida en las primeras líneas del tex-
to. Las relaciones entre la voluntad y el entendimiento
(sobre la naturaleza de las cuales Descartes levanta toda
su teoría del error que implica la esencia de la libertad7),
redefinidas por Spinoza, vuelven caduca la voluntad
entendida como facultad o poder distinto (donde está
comprendida la voluntad de Dios, ese «asilo de la igno-
rancia»8): ello no es más que causa necesaria o competida? y re-

3 Cfr. i 7 i Meditación y Respuestas a las cuartas Objeciones,


4 Cfr. Ética, Parte 1, § XXVIII, escolio.

5 m . , P, II, § VII, Esc.; XIII, Corol. y Esc.


6 m . , P. III, 5 II, Esc.
1 Cfr. IV Meditaciones y Principios, Parágrafos 32-43.

' Ver Ética, P. I, Apéndice, el texto más revelador de la filosofía de


Spinoza,
' Cfr. Ética, P. I, S XXII.

81
mite a voliciones particulares, siempre causadas, que no son
más que las ideas en tanto que entrañan afirmación o ne-
gación10. La voluntad se confunde con el esfuerzo ten-
dente a perseverar en el ser (conatus) del Alma11.
4, Tenemos, pues, que delimitar la significación
práctica de estas tesis («teoremas») en apariencia tan con-
trarias no solamente al punto de vista cartesiano sino
también al sentimiento común que experimentamos
acerca de nuestro libre albedrío y de su importancia mo-
ral. La ilusión de la libertad consiste en creer que pode-
mos alcanzar un dominio absoluto sobre nuestras pasiones. Para
Spinoza esto sería confundir el orden verdadero de la na-
turaleza: el hombre, precisamente, «no es un imperio
dentro de un imperio»'2, y su «impotencia o su inconstan-
cia» no pertenece sino a la «potencia común de la natura-
leza». Pero si la libertad es ilusoria (pues no significa nada
más que la universal necesidad), ello no conduce a nues-
tro autor a negar que el hombre pueda llegar a ser libre a
pesar de que no nazca libre o la identidad de la voluntad y
el entendimiento, que parece suprimir la esencia de la li-
bertad, prepara, al contrario, el camino de la liberación.
«Un sentimiento que es una pasión cesa de ser pasión des-
de el momento en que nos formamos de ella una idea cla-
ra y distinta»13. Cuanto más incrementamos nuestra inte-
ligencia de las cosas (de la naturaleza o de Dios, pues ex-
presan la misma necesidad), mayor se hará la potencia del
entendimiento, más intenso el esfuerzo de la voluntad,
mayor la actividad, menor la pasividad; nos vamos ha-
ciendo menos «esclavos». Tal es la «virtud soberana» para
Spinoza, y significa lo mismo que «el supremo esfuerzo del
alma» (o del espíritu —mens en latín), que consiste en
«comprender las cosas por el tercer género de conoci-
miento14». Es un camino arduo, como el mismo Spinoza

i» M., P. II, S XLVII y Esc; S XI.IX, C y E.


M., P. III, S IX, Esc.
u Ibid., P. III, Prefacio,
lí/tóí, P. V, § II],
m., P. V, S X X V ; y también, P. IV, Def. VIII y P. II, XL.
R. II.

82
reconoce al concluir su Ética, pero la satisfacción del
alma que aparece como resultado (la beatitud) es también
«la auténtica salvación».

10. Spinoza

S O B R E CÓMO LOS FILÓSOFOS Y LOS POLÍTICOS


CONCIBEN LA NATURALEZA HUMANA

§ 1. Los filósofos conciben los afectos que libran ba-


talla en nosotros como vicios en los cuales caen los hom-
bres por su culpa. Por ello suelen reírse, quejarse de ellos,
criticarlos o, cuando quieren parecer más virtuosos, de-
testarlos. Creen así hacer una obra divina y elevarse al pi-
náculo de la sabiduría, prodigando toda suerte de alaban-
zas a una naturaleza humana que no existe en parte algu-
na y vituperando con sus dichos la que realmente existe.
Conciben a los hombres, en efecto, no como son, sino
como ellos quisieran que fuesen. De ahí que, las más de
las veces, hayan escrito una sátira en lugar de una ética, y
que no hayan ideado jamás una política que pudiera ser
llevada a la práctica. La política, del modo como la con-
ciben, debería ser considerada como una quimera o sólo
podría ser instaurada en el país de la Utopía o en la edad
de oro de los poetas, esto es, allí donde ninguna institu-
ción es necesaria. Entre todas las ciencias que poseen una
aplicación, pues, es en la política donde la teoría pasa por
diferir en mayor grado de su práctica, y se considera que
nadie es menos idóneo para el gobierno del Estado que
los teóricos o filósofos.
§ 2. En cambio, en el caso de los políticos les cree-
mos más ocupados en tender trampas a los hombres que
en dirigirlos con vistas a lo mejor, y se les juzga más bien
hábiles que sabios. La experiencia, en efecto, les ha ense-
ñado que mientras haya hombres habrá vicios, razón por
la cual se esfuerzan en prevenir la malicia humana utili-

83
zando medios de eficacia demostrada a través de una lar-
ga experiencia y que los hombres acostumbran a emplear,
cuando son guiados por el miedo más que por la razón.
Actúan así de un modo que parece contrario a la religión
y, sobre todo, a los teólogos, ya que éstos creen, en efecto,
que el soberano debe administrar los asuntos públicos
conforme a las mismas reglas de la piedad que los parti-
culares deben observar. Pese a ello, no cabe duda de que
esos políticos hablan de la política en sus escritos con ma-
yor fortuna que los filósofos; pues, como tomaron la ex-
periencia por maestra, no enseñaron nada que se apartara
de la práctica.

Tratado político (1677) capítulo I, §§ 1-2.

COMENTARIO

1. Obra inacabada, redactada originalmente en latín


y publicada en 1677 tras la muerte de su autor; al mismo
tiempo que la Ética, el Tratado político introduce sin ro-
deos, en el tratamiento de la cosa política, una confronta-
ción entre filósofos (aún denominados teóricos) y po-
líticos. La recusación global de la competencia de los
filósofos en materia de política (§ 1) parece dar paso al re-
conocimiento de la competencia de los politicos en un
ámbito que es el suyo propio y que emerge, ante todo, de
la experiencia (§2). Se trata, sin embargo, de una simetría
engañosa, pues, como ya veremos, la denuncia de la erró-
nea evaluación de la naturaleza humana efectuada por los
filósofos, y cuya consecuencia inmediata es la escisión
entre teoría y práctica en el terreno político, no implica
la necesidad de proponer la práctica política efectiva
como modelo puro y simple de gobierno de las ciu-
dades,
2. El parágrafo 1 da cuenta de las actitudes de evalua-
ción moral erróneas y sugiere el esbozo de una tipología
(irrisorio, lamentable, reprensible, detestable1) que des-

Textos paralelos: Etica, libro III, comienzo; Ética, libro IV, prop.

84
r
[
¡
;
cansa sobre una percepción aberrante de la naturaleza
humana, dando origen a una serie de proyecciones (qui-
¡ mera, utopía, edad de oro) donde la imaginación triunfa
i en el lugar que le es propio a la razón. La totalidad del
texto aparece edificada sobre la oposición entre ética y
moral. La idea general es la siguiente: no es la moral (es
decir, una percepción edificante de la naturaleza humana
que culmina en la religión) la que debe hacer las veces de
fundamento de h política, sino que más bien es la ética,
tal y como Spinoza la concibe, la que debe prestar ese ser-
vicio. En efecto, a la errónea evaluación de las virtudes y
los vicios se opone la correcta apreciación (ética) de los
afectos, lo que, dicho con otras palabras, no es otra cosa
i que la ubicación de los mecanismos naturales en la inte-
racción de los individuos, porque los hombres, al igual
que el resto de los seres, son pane constitutiva de la natu-
raleza. Parece así que la ciencia política no debe ser cons-
truida en función de normas trascendentes y preestableci-
das, sino determinada a partir de la puesta en evidencia
de una red de interacciones inmanentes entre los cuer-
pos. El efecto nos devuelve a la potencia de actuar propia
del cuerpo en tanto que pueda ser incrementada o dismi-
nuida, favorecida u obstaculizada2. Sin lugpr a dudas, la
visión moral tiende, en cuanto tal, a la debilitación, y
obstaculización por tanto, de aquella potencia, sobre
todo al suscitar sentimientos tales como el de culpabi-
lidad.
a) El esbozo de tipología sugerido por Spinoza (Los
filósofos... —*•... realmente existe) remite, a través de una serie
de registros modulados, a un mismo y único error; el de
promover una naturaleza humana inexistente al despre-
ciar sus manifestaciones reales. En lugar de esforzarse por
comprender los mecanismos que actúan en el seno del
comportamiento humano, los moralistas se contentan
con la mera condena, condena que en sí misma descansa

35, esc.; prop, 50, esc; Coria 30 a Oldenburg (Cvrrtsp<mitnc¡a, trad.


A. Domínguez, Madrid, Alianza, 1988),
2 Cfr. Etica, Libro III, definición III.

85
sobre una confusión entre ser y deber ser. Atravesamos,
así, una gradación que se extiende desde lo más inofensi-
vo (la irrisión) hasta lo más represivo (el criticar, el detes-
tar), pasando por la lamentación melancólica. La tipología
se despliega, en consecuencia, al seguir lo que podría-
mos designar como un acrecentamiento del resentimien-
to, cuya nota común continúa siendo el rechazo de lo que
es en nombre de lo que no es. La moral, en efecto, es por
esencia negativa, mientras que la ética quiere ser afirma-
tiva.
b) Aunque no pretendamos poner, a toda costa, eti-
quetas bajo los diferentes tipos sugeridos por Spinoza, no
estará de más que, retomando las diferentes proyecciones
descritas a lo largo de la continuación del párrafo ante-
rior (conciben,.. —...necesaria) situemos las líneas generales
de una tabla de correspondencias. La Sátira, ciertamente,
es el género literario moral por excelencia que pretende
atacar, esto es, ridiculizar, los vicios y costumbres de la
época. La Sátira, pues, mantiene una relación directa con
la irrisión (Menípo, filósofo cínico del siglo 111 a.C., fue
uno de los fundadores del género satírico). En contrapo-
sición, la Etica, que, según las palabras de Spinoza, trata
los efectos como si fueran «líneas, superficies y sólidos»,
quiere ser, en sentido estricto, a-moral. Desde que los
moralistas se interesan por la política, la han concebido
fundamentalmente como una quimera, es decir; al mis-
mo tiempo como un animal fabuloso y como un proyecto
vano. Así se efectúa la primera proyección (negativa),
donde la quimera podría parecer correlato de la irrisión:
la censura de los asuntos de la ciudad en nombre, por
ejemplo, de un necesario regliegue del individuo sobre sí
mismo como única vía de acceso a la sabiduría y la vir-
tud. Irrisión, provocación e invectiva forman parte efec-
tiva del arsenal de la tradición cínica después de que Dió-
genes de Sínope declarara, desde el fondo de un tonel, no
pertenecer a ninguna ciudad.
La utopía y la edad de oro, por el contrario, constitu-
yen proyecciones positivas en correspondencia con la la-
mentación, por una parte, y con la represión y el rechazo

86
de lo detestable (dicho de otro modo, la actitud teológica,
pero también el fanatismo de quienes desean sobresalir
en materia de santidad). La Utopía (se trata de la Utopía
de Tomas Moro, 1516, de la que Spinoza poseía un ejem-
plar en su biblioteca) es una proyección en un más allá
que no existe en ninguna parte, suponiendo el confina-
miento de una naturaleza humana ilusoria en un espacio
diferente. La Utopía aparece como correlato de la lamen-
tación en la medida que se trata de volver la espalda al
mundo actual para instalar la «verdadera» naturaleza hu-
mana en otro lugar puro, carente de toda contaminación.
La edad de oro, por su parte, representa la proyección de
la naturaleza ideal en un tiempo diferente, el tiempo que
precede a la historia. Podemos considerarlo como el co-
rrelato de la reprimenda en la medida que la edad de oro
siempre estará quebrantada por una falta que entraña el
castigo divino.
A la vista de lo anterior, tendríamos una correspon-
dencia, casi término a término, en la negación de la ver-
dadera naturaleza humana y la promoción, en contrapar-
tida, de una naturaleza humana ficticia que, o bien impli-
ca el desprecio de la política, o bien conduce a proyectar
su ejercicio en un más allá o en otro tiempo. En cualquier
caso de que se trate, quimera, utopía y edad de oro son el
fruto de la imaginación y no de la razón.
c) La conclusión del párrafo que venimos analizando
(Entre todas las ciencias... —» ...filósofos), al ampliar la pers-
pectiva, aparece dotado de un abierto carácter antiplató-
nico. Lo que aquí se rechaza es la idea del filósofo-rey3. El
teórico, en efecto, en función de los prejuicios examinados,
no es apto para gobernar la ciudad. El problema político
no es, sin duda, ni el de la innovación ni el de la normali-
zación. La cuestión del poder (político) es, probablemen-
te, el de la compatibilidad de las potencias (individuales).
3. El párrafo 2 aborda el modo como de ordinario se
percibe a los políticos, la astucia supera la sabiduría, el

3 Cfr. Platón, RepMia, VI, 501 b-502 c.

87
arte de la caza ocupa el lugar de la vocación gestora. En
suma, la primera figura que entra en escena es la captura
y no la protección. Sin embargo, este cálculo tiene en su
base un presupuesto que invoca la autoridad de la expe-
riencia. Spinoza retoma en este punto y a su manera el pre-
cepto del realismo político, tal y como lo enuncia la má-
xima de Maquíavelo: hay que tratar a los hombres como
bribones. Este realismo antropológico, que contrasta con
el irrealismo de los teóricos, introduce, empero, una pun-
tualizacíón según la cual, para los teóricos, la proyección
de la sanción moral acompaña a la negación de lo real.
En contraposición, el realismo de los políticos se inspira
en la experiencia en sentido amplio, en la constatación, e
implica una acción preventiva (... de la malicia humana) que
frustra de antemano toda sanción moral. El político es
«realista» porque trabaja en el campo de la inmanencia de
los afectos, de los que se presenta acta al huir de toda
perspectiva trascendente. Ahora bien, el presupuesto al
que nos referíamos estipula que es el miedo, y no la ra-
zón, quien dirige a la humanidad. Del mismo modo que
el miedo —como, por otra parte, lo indica Spinoza— es
la causa que engendra, conserva y alimenta la supersti-
ción, ese temor también es el resorte de la dependencia
mutua entre los individuos, miedo que, en todos los ca-
sos, no puede ser superado sino a través de un proceso
cognitivo (la singladura misma de la Etica) y no mera-
mente social. De ahí que los políticos utilicen los vicios
con la mirada puesta en la eficacia de la acción y que el
miedo sea el motor de toda empresa humana. No obstan-
te, y a diferencia de los teólogos, la actitud mencionada
no es «moral», y ello porque el ser de la política no es nin-
gún tipo de ilusión, sino que debe tomar a los hombres
por lo que son y como son; es la especificidad de la políti-
ca con respecto, sobre todo, a la religión la que abre un
espacio donde el gobierno de sí mismo sirve de norma de
conducta en los asuntos públicos. Tal rechazo del modelo
«privado» hipertrofiado por los teólogos implica una re-
lativa autonomía del terreno político en cuya delimita-
ción consiste la tarea que asume el Tratado político.

88
4. El rechazo global e implacable de la actitud de
los teóricos no supone, sin embargo, que se esté propo-
niendo como modelo puro y simple la actitud realista de
los políticos, pues su competencia yace esencialmente en
la experiencia, a título de la cual sus modos de acción son,
antes que nada, indicativos de eficacia. Es posible supo-
ner, a pesar de todo, que el precio de la eficacia es la au-
sencia de una fundamentación sólida de la propia prácti-
ca. Es por ello que la Etica, que concierne en primer lu-
gar a la labor del individuo sobre sí mismo, representa el
preámbulo de toda reflexión política. Sin embargo, el
tránsito de esa labor sobre sí mismo al gobierno de los
hombres no es evidente, de modo que la ética, no obstan-
te su condición de preámbulo, difícilmente puede apare-
cer como modelo para la organización política. Es indu-
dable que toda reflexión política sana debe tener siempre
presente los conocimientos plasmados en la Ética, pero la
política no puede ser percibida como simple aplicación
de la ética. I^a experiencia constituye el terreno inmediato,
la sala de operaciones políticas, pero esa misma experien-
cia, en el orden del conocimiento, no es nada más que el
punto de partida de la reflexión, y debe ceder el testigo,
en adelante, a la razón. Esta fundamentación racional de
la política, que debiera permitir la conciliación de la se-
cularidad del estado con la preservación de la libertad de
los ciudadanos, debe descubrir su propio camino apoyán-
dose en todo momento en las exigencias propias de la
ética.

PARA SABER MÁS

Sobre Spinoza (textos 9 j 10)

ALBIAC, G., La sinagoga varía: un estudio de las fuentes marranas del es-
pirtozismo, Madrid, Hiperión, 1987.
DELEUZE, G., Spino&ty el problema de la expresión, trad, Horst Vo-
gel, Barcelona, Muchnik, 1975.
— Spinoza: filosofía práctica, trad. A. Escohotado, Barcelona,
Tusquets, 1984.

89
HAMPSHIRE, S., Spme&t, trad. Vidal Peña, Madrid, Alian-
za, 1982.
MACEE, B., Las grandesfilósofos, trad. Amaia Barcena, Madrid, Cá-
tedra, 1990, págs. 103-127.
VIDAL, PEÑA, El materialismo de Spinoza, Madrid, Alianza, Revista
de Occidente, 1974.

11. Leibniz

Q U E D E S C A R T E S SE HA EQUIVOCADO
AL TRATAR EL MOVIMIENTO

El autor termina esta segunda parte, consagrada a los


principios generales de las cosas materiales, con una ad-
vertencia que exige, según me parece, una restricción.
Pretende que, en Ta explicación de los fenómenos de la
naturaleza, no son necesarios más principios que los que
se pueden extraer de la matemática abstracta, es decir, de
la doctrina que concierne al tamaño, las figuras y el mo-
vimiento, y no reconoce más materia que aquella que es
el objeto de la geometría. Coincido plenamente con él en
que todos los fenómenos particulares de la naturaleza po-
drían ser explicados desde la mecánica si hubieran sido
explorados suficientemente, y en que no hay otro medio
de conocer las causas de las cosas materiales. Pero lo que
a mi parecer se debe tener siempre presente en el espíritu
es que los principios mecánicos mismos, esto es, las leyes
generales de la naturaleza, nacen de principios más eleva-
dos y no podrían ser explicados por la mera cantidad y
por consideraciones geométricas. Estos principios impli-
can, bien al contrario, algo de orden metafísico, indepen-
diente de las nociones que nos provee la imaginación, y
que deben ser atribuidas a una substancia desposeída de
extensión, puesto que, aparte de la extensión y sus modi-
ficaciones, está, inherente a la materia, la fuerza misma
de actuar que permite el paso de la metafísica a la natura-

90
Jeza y de las cosas materiales a las inmateriales. Esta fuer-
za obedece a sus propias leyes, de las cuales no derivan
únicamente principios de necesidad absoluta y, si se nos
permite, ciega, como las leyes matemáticas, sino también
principios de la razón perfecta. Desde el momento en que
se establecen esos principios mediante una investigación
preliminar general, todo lo demás puede ser explicado a
continuación mecánicamente en los fenómenos de la na-
turaleza. E igualmente inútil es, por tanto, recurrir a las
percepciones e inclinaciones de un arjé, o a las ideas operan-
tes, a las formas substanciales o incluso a las almas; sería inútil
apelar, para explicar las cosas particulares de la naturale-
za, a la simple voluntad de la causa universal de todo lo
que hay, como a un Deus ex machina, al igual que lo hiciera
el autor de la Pbilosophia Mosaica. Si se tienen en cuenta,
con gran cuidado, estas observaciones, se evitarán los ex-
tremos en filosofía, dando satisfacción a los teólogos tan-
to como a los físicos.

Observaciones sobre ta parte general de ios principios de


Descartes (1692). Sobre la Segunda parte, sobre
el arte. 64.

COMENTARIO

1. El opúsculo del que ha sido extraído este texto


data de 1692. Por esta época, el sistema de Leíbniz ya está
plenamente constituido, y, sin embargo, podemos ver
por estas observaciones que el mejor modo de consolidarlo
es el de asegurar sus bases al volver sobre sus principios;
que habrá que re-marcar y des-marcarlos lo más distinta-
mente posible de aquellos mismos principios que, medio
siglo antes, fueran presentados como una nueva funda-
mentación de la ciencia filosófica en los Principios de Des-
cartes. Es evidente que este último es el autor a quien se
pone en cuestión desde las primeras líneas del texto. El tí-
tulo del mismo es la traducción del vocablo latino ani-
madversiones, que significa, aparte de observaciones resul-

9t
tantes de la atención del espíritu, algo así como amones-
taciones. En consecuencia, tenemos, al hilo de una lectu-
ra razonada de los sucesivos artículos del tratado de Des-
cartes, un modelo de comentario crítico que muestra,
bien el acuerdo, bien las objeciones (sin embargo) o las re-
futaciones (bien al contrario) en relación con tal o cual pun-
to de la doctrina o del método, así como una notable
preocupación por las consecuencias extraíbles (B igual-
mente inútil es, por tanto,...) o sus implicaciones (dando satis-
facción a...). De todo ello emerge, sin lugar a dudas, la víc-
tima inconfesada, quizá inconfesable, de un sistema sobre
otro. En efecto, ¿cómo no ser consciente de su superiori-
dad cuando se está capacitado para formular los principios
más elevados e, incluso, los principios de la razón perfecta? En
otra polémica, dirimida sobre un terreno cercano al que
nos ocupa, ¿acaso no se había embarcado Leibniz, junto
con Newton, y a propósito del cálculo diferencial, «en
una ardiente lucha por la supremacía en la cual» nos dice
E. Boutoux «Leibniz no está exento de amor propio»?
2. Hay cierta ironía, es cierto, al recordar que Leib-
niz fue un gran matemático, pues de existir una lección
última que aprender de este texto, quizá no sea otra que la
de poner de manifiesto que la filosofía no puede recurrir
a la matemática para comprender el análisis, o, dicho de
otro modo, que la racionalidad de la matemática es insa-
tisfactoria para tal propósito. En efecto, la oposición a
Descartes se maniñesta en el aspecto concreto de la fun-
ción y valor de la geometría en el orden de la explicación
de losfenómenos de la naturales; esto es, en la física y, especí-
ficamente, en la mecánica. Podemos reconstñiir sin
dificultad las diferentes etapas de la discusión. Esta
consta de:
— Una llamada de atención sobre la tesis cartesiana
por la cual se completa la segunda parte de los Principios
(—''el objeto de la geometría), resumida en los términos si-
guientes en la Tabla de los principios de la filosofía: «Que no
apruebo más principios físicos que los aprobados tam-
bién por la matemática, con el fin de poder probar por
demostración todo lo que de ellos deduciría, y que estos
92
principios bastan, en tanto que todos los fenómenos de la
naturaleza puedan ser explicados por ellos»1. De ahí que
pasemos a:
— La crítica de lo anterior elaborada por Lcibniz
(Coincido plenamente...—*-... consideraciones geométricas), donde
reconoce la necesidad de la explicación mecánica, pero
reivindicando también (y, a pesar de las apariencias: a mi
parecer..., no se trata de una opinión) la necesidad de su
subordinación a una razón diferente de la geométrica.
— A continuación se introduce la exposición de ¡a te-
sis propiamente leibnmana (Estos principios implican... —* ...déla
razón perfecta), que, ateniéndonos a lo más esencial, substi-
tuye claramente la matemática por la metafísica en orden
a dar razón de las leyes físicas de modo que las propieda-
des de la fuer%# pasan a ocupar el lugar de la extensión en
el terreno de la comprensión de la materia en su generali-
dad.
— La conclusión (Desde el momento en que se estable-
cen. ..—*•.. .fin del texto) se presenta con toda naturalidad como
un balance, pues Leibniz destaca lo que se gana al perder, si se
nos permite decirlo así, ciertos hábitos de pensamiento;
esto es, al desembarazarnos de nociones superfluas o re-
nunciar a facilidades en exceso cómodas.)
3. De haber un aspecto clave del texto que requiera
ulteriores puntualizaciones es esa «metafísica de la fuer-
za» ausente en la filosofía cartesiana, de donde podemos
entender que para Leibniz sea indispensable formular
una restricción. Y, para empezar, tendremos que preguntar-
nos lo que significa esa transposición de la fuerza, una
realidad tan física en apariencia, al orden de la metafísica.
Y preguntarnos, también, cuáles son las razones, las justi-
ficaciones y, asimismo, las consecuencias que ello repre-
senta, en la filosofía de Leibniz, para la coherencia de su
sistema. Conviene repetir, antes de continuar con nues-
tro análisis, que Leibniz no está intentando negar la
irreemplazable utilidad de la explicación mecánica, pues no
hay otro modo de conocer las causas de las cosas materia-
1 Descartes, (Euvrts, Bíbl. de la Pléiadt, pág. 677.

93
fes; pero tampoco pretende negar su universalidad, ya
que todos los fenómenos particulares de la naturaleza po-
drían ser explicados mecánicamente. Así, pues, no cabe
hablar de la física (de la creencia de la pesantez, de los choques, de
¡a elasticidad o del magnetismo) sin hablar al mismo tiempo de las le-
yes generales de ¡a naturaleza, o, dicho con otras palabras, sin
principios mecánicos. En efecto, esta es la única manera de es-
capar a las ilusiones contenidas en las explicaciones pura-
mente formales que hasta entonces ofrecieron, mediante
diversas cualidades ocultas (Descartes, recordémoslo, ya
las había combatido), los antiguos, los escolásticos y aún
los newtonianos (con su «atracción»). Sin embargo, la
mecánica, aunque permite formular adecuadamente las
relaciones de causalidad establecidas entre las cosas mate-
riales, no alcanza a dar cuenta de la eficacia de la causa.
Este es todo el problema que nos ocupa,
O bien, ¿cómo explicar que un cuerpo material tenga
capacidad de actuar o reaccionar, si la materia, según su
acepción tradicional, es puramente pasiva? Y, de forma
todavía más radical, ¿cómo explicar que pueda nacer el
movimiento de la materia y que del movimiento puedan
también nacer todas las diferencias que constituyan las
particularidades de ¿os fenómenos? Se trata, precisamente, de ir al
fondo de las cosas, lo que podría ser un modo de definición
de la metafísica según Leibniz, esto es, poner en eviden-
cia el exterior de los fenómenos a partir del interior de las subs-
tancias (ello exige que consideremos a estas últimas como
el preámbulo de toda física). En este sentido, la tentativa
de Descartes fue notable, aunque su error continúa resi-
diendo en la versión que ofreciera de la materia («cuya
naturaleza consiste tan sólo en que es extensión»), o, tam-
bién, de la extensión misma (que «en longitud, anchura y
profundidad, constituye la naturaleza de la substancia
corporal»)2. Al identificar así el objeto de la física con el
objeto de la geometría, Descartes contribuyó sin duda a hacer
de la física una ciencia de la cantidad, restando, sin em-
bargo, la cuestión de si es posible comprender la acción

2 Descartes, Principies, I, § 53; II, S 22.

94
por las -abstracciones del tamaño, de la figura y del movi-
miento. Ciertamente este autor se proveyó de los medios
necesarios para deducir, en física, los efectos de las causas
con la misma certeza que en la matemática se deducen las
consecuencias de los principios. No obstante, surge la
duda de si la necesidad física es absoluta (como la necesi-
dad matemática, enteramente determinada por el único
principio de contradicción).
Leibniz, de hecho, tiene buenas razones para oponerse
a Descartes. En primer lugar, y en el mismo plano donde
este último se sitúa, es posible demostrar matemática-
mente que lo que se conserva no es la cantidad de movi-
miento mv (como pretendía Descartes), sino la cantidad
de fuerza viva mv2 (que Leibniz todavía denomina canti-
dad de potencia activa), lo cual índica que en la naturale-
za hay, en concreto, otra esencia del movimiento y otro
sentido desde el que comprender la inercia. En segundo
lugar, y aún cuando sus nociones nada expliquen, Para-
celso y su atjé, Marci de Kronland y sus ideas operantes, los
escolásticos y sus formas substanciales expresan una misma
intuición, la de una materia viva, dotada de energía y es-
pontaneidad (tales como esos corpúsculos animados, los
infusorios, que el microscopio revela al naturalista Leeu-
wenhoek). La razón y la experiencia conducen, pues, a
postular como inherente a la materia, la fuerza misma de actuar,
que Leibniz aún llama entelequia primera (término que
toma prestado de Aristóteles) y que define (en un opúscu-
lo algo más tardío: De la naturaleza en sí misma, 1698, «la
fuerza motriz primitiva que se añade a la extensión (o a lo
puramente geométrico) y a la masa (o a lo puramente ma-
terial) y que es siempre activa». Así, ese «algo más» viene
a cualificar, para Leibniz, a la materia primera (pasiva)
como materia segunda (activa), y por su unidad distinta con
la materia, a individualizar las substancias en mónadas. Im-
primida en estas últimas por «decreto divino», la fuerza
mencionada no requiere (como es el caso de R. Fludd) la
intervención constante y extraordinaria de Dios para ex-
plicar lo ordinario de la naturaleza. Intermediaria entre
la potencia desnuda y el acto, ella supone un esfuerzo por

95
existir (en cierta medida, deseo del efecto) de donde se
pueden extraer las leyes del movimiento (es decir, una
Dinámica) y fenómenos de los cuerpos. Es por ello que
la fuerza permite el paso de la metafísica a la naturaleza (de la
profundidad a la superficie, de las esencias a los seres
existentes), y, reciprocamente, de las cosas materiales (cuer-
pos, fenómenos) a las cosas inmateriales (el alma de cada
substancia). En suma, en la metafísica de la fuerza se ope-
ra efectivamente la inversión que Leibniz hace sufrir al
cartesianismo, pues, lejos de ser substancia, la extensión
no es sino un ser de imaginación engendrado por las per-
cepciones (más o menos confusas) y las apetíciones de las
substancias, movidas por su fuerza al contacto de unas
con otras. Es por ello que esta metafísica exige (y por la
misma razón es, precisamente, metafísica) que hemos de
desprendernos de la imaginación a la hora de representar
la fuerza, que sólo es concebible cuando se la atribuye a la
substancia —desposeída de extensión.
4. Teniendo en cuenta lo anterior veremos cómo se
debe comprender la metafísica «en general»: como una
investigación preliminar que no es general más que en tanto se
dirige sobre los principios y asume la tarea de distinguirlos,
jerarquizarlos, subordinarlos y unirlos en una razón perfec-
ta. Así, la necesidad que destila de la imposibilidad del
contrario (principio de contradicción, como en la matemáti-
ca), pudiendo ser ciega, porque al no estar determinada
por su fin no tiene la necesidad de «verlo», no podría go-
bernar a los seres existentes, como tampoco a sus cuer-
pos, movimiento y leyes. A propósito de éstos, cabría
preguntarse cómo es que son así y no de otro modo, cues-
tión que inevitablemente nos hace remontarnos a la pri-
mera de todas: ¿por qué hay algo y no más bien nada? A
estas preguntas sólo se puede responder mediante otro
principio, el de razón suficiente. De este modo, filosofía pri-
mera se erige en una investigación de la razón última de las
cosas y su razón completa. El dinamismo se muestra, en
este terreno, intrínsecamente ligado al finalismo (por lo
cual el «cartesianismo» de nuestros días se encuentra rela-
cionado con el platonismo y el aristotelismo). Por tanto,

96
hay otros principios (el de lo mejoro el del máximo y del mí-
nimo) que, por su parte, pueden ser formulados con plena
eficacia. Y, para terminar, ¿cómo no advertir que tales
principios son los que dirigen el movimiento mismo del
pensamiento liebniziano? ¿Acaso no se trata de fomentar
una verdadera «economía racional»; de producir con un
mínimo de (buenos) principios (bien ordenados), un má-
ximo de verdades al eliminar lo superfluo; de instaurar la
armonía (a igual distancia del materialismo y del formalis-
mo, cfr, final de De la naturaleza en sí misma) entre los físicos
dedicados a analizar el reino mecánico de la naturaleza, y
los teólogos ansiosos por comprender el reino moral de la
Gracia?

PARA SABER MAS

Textos paralelos de Leibntz

De la production origineíle des cbosesprise a sa racine (1697) y también:


«De la reforme de la phiiosophie premié re et de la norion de
substance» (1694) en Opmcules philosopbiques cboisis (opúsculos fi-
losóficos escogidos), op, rit.

12. Leibniz
Q U E H O B B E S Y S P I N O Z A HAN FORMADO
LA N E C E S I D A D

(...)Hobbes quiere que incluso la sola presencia divina


baste para establecer una necesidad absoluta de los acon-
tecimientos, creencia que también compartía Wiclef y
aún Lutero, cuando escribió De servo arbitrio, o, al menos,
hablaban en tales términos, Pero se ha llegado a recono-
cer en nuestros días que esta especie de necesidad deno-
minada hipotética y procedente de la presciencia o de
otras razones anteriores, nada tiene que nos deba alar-
mar. Algo totalmente distinto ocurría si la cosa fuera ne-
cesaria por ella misma, de suerte que su contrario impli-

97
cara contradicción. Hobbes tampoco quiere oír hablar
más de una necesidad moral porque, de hecho, todo ocu-
rre por causas físicas. Sin embargo, existe una buena ra-
zón para establecer una gran diferencia entre la necesidad
que obliga al sabio a actuar bien, llamada moral, y que
tiene lugar incluso en relación con Dios, y entre esa nece-
sidad ciega por la cual Epicuro, Estratón, Spinoza y qui-
zás Hobbes han creído que las cosas existían sin inteligen-
cia ni opción y, en consecuencia, sin Dios, del cual —se-
gún ellos— no se tendría necesidad alguna, puesto que
según esta necesidad todo existiría por su propia esencia,
tan necesariamente como dos y tres hacen cinco. Y esta
necesidad es absoluta, pues tódo lo que ella comporta
debe ocurrir con independencia de lo que se haga, mien-
tras que aquello que acontece merced a una necesidad hi-
potética, acontece partiendo del supuesto de que esto o lo
de más allá ha sido previsto o resuelto, o hecho de ante-
mano, presuponiendo que la necesidad moral lleva consi-
go una obligación de razón que produce siempre su efec-
to en el sabio. Esta especie de necesidad es dichosa y
deseable, y ello porque nos ofrece buenas razones para ac-
tuar como lo hacemos; pero la necesidad ciega y absoluta
acabaría con la piedad y la moral.
El discurso de Hobbes contiene mucha razón al acor-
dar que nuestras acciones se hallan bajo nuestro poder, de
suerte que hacemos lo que queremos, cuando tenemos el
poder de hacerlo y no se interpone en nuestro camino
ningún impedimento, sosteniendo, sin embargo, que
nuestras voliciones mismas no se encuentran bajo nues-
tro control, de tal modo que podríamos abandonarnos
sin dificultad, de acuerdo con nuestro capricho, a las in-
clinaciones y voluptuosidades que pudiéramos desear (...).
La verdad es que contamos con algún control sobre nues-
tras voliciones, si bien de una manera oblicua y no abso-
luta e indiferentemente.

«Reflexiones en torno a la libertad, la necesi-


dad y el azar, a propósito de la obra que Hobbes
ha publicado en inglés», §J 3-4 (1710),

98
COMENTARIO

1. Sin lugar a dudas, el breve ensayo del cual hemos


extraído el presente texto constituye, entre los que com-
ponen la Teodicea de Leibniz (publicada en 1710), una crí-
tica dirigida hacia ciertos aspectos de la filosofía de Hob-
bes, cuestiones que habían sido debatidas (de 1655
a 1658) entre el propio Hobbes y un pastor inglés, John
Bramhall. Y, de hecho, el problema aquí abordado inte-
resa tanto a los teólogos (Wycliffe, llamado Wiclef por
Leibniz y Lutero) como a los filósofos (antiguos y moder-
nos, ejemplos de los cuales son Epicuro, Estratón, de un
lado, y Spinoza o Hobbes del otro), porque concierne a
las relaciones entre lo «necesario» y lo «libre» en el hom-
bre y a las que se establecen entre este último y Dios (de la
criatura con respecto a su creador) en la creación. Proble-
ma de todas las épocas, pues, y digno de ser temido, e
identificado con aquellos «dos famosos laberintos donde
nuestra razón se extravía con tanta frecuencia», por reco-
ger la fórmula de Leibniz en el Prefacio a sus Ensayos de Teo-
dicea (siendo el otro el del «continuo y los indivisibles en
que los elementos se manifiestan, y donde debe entrar la
consideración del infinito»)1. Problema, en efecto, capi-
tal, y no sólo para la especulación, sino también para la
práctica, resultándole «embarazoso» tanto a los filósofos
como a la «práctica totalidad del género humano». Pode-
mos enunciarlo así: ¿es posible, en relación con una cier-
ta idea de Dios, concebido con omnipotente y omnis-
ciente, con una Providencia capaz de presciencia y, por esto
mismo, de pre-determinar (en lenguaje filosófico) todas las
cosas o de predestinar (en sentido teológico) a todos los in-
dividuos, o en relación con su negación (una naturaleza
sin Dios, entendida como «mecanismo» estricto), conci-

1 Eniayos dt Teodicea, pig. 29.

99
liar, por una parte, la necesidad que parece deber resultar
tanto de la una como de la otra (y, en ambos casos, ¿pue-
de ser distinta de absoluta?), y, de otra parte, la libertad
que parece debe ser postulada en el hombre (de otro
modo, ¿qué sería de la piedad y la moral, sobre qué fun-
damentar la responsabilidad, la culpabilidad, la recom-
pensa y el castigo?). La respuesta de Leibniz, sutil como
lo es, no es menos equívoca, pues, según él, sí que es posi-
ble, pero a condición de realizar una adecuada distinción
en el seno de la necesidad misma,
2. Como etapa preliminar nos dirigiremos a exami-
nar cómo se concatenan en el texto los puntos que se dis-
cuten. La libertad, a decir verdad, no es analizada de fren-
te, sino indirectamente tan sólo, a través de la mención
del tratado de Lutero acerca del «esclavo albedrío» (el
contrario del «libre arbitrio») y, hacia el final del texto, de
nuestro capricho. Sin embargo, el fondo del problema no es
otro que su posibilidad, y ello mediante el rodeo de aque-
llo que supuestamente se opone a la misma, esto es, la ne-
cesidad, materia que ocupa la totalidad del primer párrafo.
En el segundo párrafo, mucho más breve, el examen se
centra en lo que normalmente se acepta como manifesta-
ción de la voluntad o la realiza: la voluntad. Tales son, por
tanto, las dos vertientes del problema que Leibniz afronta
al confrontar sus tesis con lo que Hobbes quiere o no quiere,
acuerda o sostiene. De un lado, en efecto, tenemos la potencia
de la necesidad, aunque no sepamos cómo entenderla. ¿Se-
ría «todopoderosa» o estaría «en potencia», sólo actualiza-
da bajo alguna condición? De otro lado, nos encontramos
con el poder de la voluntad, aunque no sepamos si está
acompañada de un poder «sobre» la voluntad. En un pri-
mer momento (—*• implicara contradicción') el autor opone la
noción de necesidad hipotética a la necesidad absoluta que se
desprende de la presciencia divina, tal y como Hobbes la re-
presenta (y también Wiclef, e incluso Lutero). A conti-
nuación (Hobbes tampoco quiere... —•- ... hacen cinco) el texto
introduce la gran diferencia que existe entre la necesidad
moral y, de nuevo, la necesidad absoluta, en este caso ligada
a la esencia de las cosas (de una realidad física concebida

100
como un ente matemático), según el punto de vista, que
podríamos denominar materialista, de Epicuro, Estra-
tón, Spinoza y, quizá, Hobbes (en su De Corpore)... Resta,
dentro del primer párrafo {Yesta necesidad es disoluta... -*
...la piedad y la moral), reflexionar sobre y definir por últi-
ma vez las anteriores distinciones internas al orden de la
necesidad. Nos encontramos, pues, con tres tipos de «de-
terminaciones»: la inevitable (o inexorable) de la necesi-
dad desnuda (y ciega); la condicionada de la suposición; la
indispensable (y deseable) de la obligación. Esta última sería
beneficiosa, mientras que la primera sería dañina (anu-
lando toda posibilidad de actuar bien).
En un segundo momento, Leibniz acude a Hobbes a
propósito de la acción voluntaria, reuniéndose con él jus-
tamente en la distinción entre «poder de actuar» y «poder
de querer» (El discurso de Hobbes contiene mucha razón...-*
...pudiéramos desear), pero con la intención de aportar una
rectificación (la palabra que mejor la expresa es la última
del texto; indiferentemente) que orienta la crítica de Leibniz
en otra dirección. Volveremos sobre este punto.
3. Regresemos ahora a lo que constituye el telón de
fondo de Ta discusión en la parte principal del texto. La
preocupación filosófica de Leibniz se focaliza aquí en el
denominado necesitarismo, sea teológico o filosófico. Los
pensadores modernos reactualizan, a su manera (hacien-
do derivar, de su concepto del absoluto divino o de la
«máquina» del Universo, o de los dos a la vez, una conse-
cución infalible de cosas y acontecimientos), las tesis an-
tiguas de las que no ha cesado de alimentarse un sofisma
que los propios antiguos llamaban la razón perezosa, y ello
porque incita a no actuar verdaderamente. Este sofisma
concluye, partiendo de que «hay una verdad en el evento
futuro que está determinada por causas» y de aquello que
«Dios ha preestablecido al establecer sus causas (así lo es-
cribe en el Prefacio citado más arriba), que nada hay que
hacer (sino abandonarse al presente). Porque, siendo el
porvenir necesario, «lo que deba acontecer acontecerá sin
importar lo que yo pueda hacer». A este fatalismo, en el
cual Lebniz distingue un factum mahumetanum y un factum

101
stoicum2t se le opone un factum cbristiamm que restituye su
razón a la acción.
Tal es, pues, la tesis que Leibniz, a lo largo de diversos
textos, se esforzará por acreditar todo lo que acontece lo
es por nuestra voluntad, voluntad que, como el resto de
las cosas, está prevista por Dios, aunque tan sólo como
algo posible y no absolutamente necesario, y en este sen-
tido lo contrario podría producirse sin contradicción. En
el último aspecto que acabamos de señalar reside lo esen-
cial de la tesis leibniziana, ya que le permite escapar a la
jurisdicción del principio de contradicción (única fuente
de necesidad absoluta) para someterse a la regla de la com-
posibilidad (y esta es otra cuestión por completo diferente).
La acción por llegar (que pertenece, desde el decreto ini-
cial de Dios, a la definición del individuo como el predi-
cado pertenece al sujeto) podrá ser afirmada como cierta,
mas no por ello será necesaria: «no hay nada necesario
cuyo opuesto sea posible»'. Sería «cierto», pero «no nece-
sario en sí mismo», que César atravesara el Rubicón 4 , que
Adán tendría esta posteridad5 o, incluso, que el mismo
Spinoza moriría en La Haya y no en Leiden6... La idea
más brillante de Leibniz, en este terreno, es la de aceptar
la existencia de una presciencia/previsión de la provi-
dencia que inclina sin necesidad desde el momento que acce-
demos a la existencia, es decir, a un orden que de suyo es
contingente porque depende del libre albedrío de Dios (los
decretos de su voluntad), y no solamente de su entendi-
miento. De este modo, es posible ser a un tiempo (pre)
determinado y libre (sin que por ello quede en entredicho
el principio de contradicción). Asimismo, carece de sen-
tido alarmarse (por el tipo de moral y piedad), pues «sea lo
que sea aquello que está encerrado en mi noción, haré
este viaje, y tan encerrado en ella se encuentra que lo haré

^ Ibid., págs. 30-31.


3 Discurro de metafísica, § 13.
4 Ibid.
5 Correspondencia con Arnauld, IX.

<> Ensayos de Teodicea, II, §§ 173-174,

102
libremente»7. Nos encontramos aquí con otra clase de de-
terminismo, el de la necesidad moral. Y, si es preciso acceder a
su plena legitimidad, tendremos que concebirla como
conveniencia, referirla al principio de razón suficiente y de
ahí al principio de lo mejor (del optimum) y a la armonía
preestablecida. Se comprende así que el sabio esté tanto
más perfectamente (y siempre) determinado por esa obliga-
ción de razón cuanto más capacitado para captar distinta-
mente la «naturaleza».
4. La observación anterior nos introduce en una últi-
ma consideración acerca de la voluntad. Hobbes se mues-
tra riguroso en su concepción de las implicaciones del
querer y el poder podemos lo que queremos; por tanto,
queremos lo que podemos —en la medida de las fuerzas
de cada cual y siempre que no se halle limitado por cons-
tricciones externas. Este autor llega a escribir, en su Le-
viathan: «Si un hombre me hablara de un cuadrado redondo,
(...) o de un sujeto libre, o de una voluntad libre, o de cual-
quier cosa libre, con un sentido distinto del de liberado del
impedimento constituido por una oposición, no diría
que está en un error, sino que sus palabras no quieten de-
cir nada, y, en otros términos, son absurdas»8. En el «me-
canismo» de las determinaciones no podemos, por consi-
guiente, forzar una volición a otra cosa que no sea a lo
que se ve impelida cuando no se encuentra con ningún
obstáculo; nada hay de arbitrario en el libre albedrío posi-
ble. Pero Leibniz no propone ninguna objeción a este ra-
zonamiento, pues, también para él, el mecanismo gobier-
na a todos los cuerpos. Sin embargo, no es universal más
que cuando es pensado desde el ángulo del finalismo y se-
gún el principio de lo mejor. «No sabríamos querer lo que
consideramos como bueno», escribe Leibniz en sus Nuevos
ensayos sobre el entendimiento humano, «y según que la facultad
de entender esté avanzada, la elección de la voluntad será
mejor»9. Con el método y con el tiempo, es posible llegar a
7 Correspondencia con Arrmuld, X.
® H oh bes, Leviathan, ed. Sirey, cap. V, pág. 40; cap. XIV (sobre la li-
bertad).
5 Nuevos ensayos, libro III, Cap. 21, 5 17.

103
«hacer lo que queramos hacer y la razón ordene»10. De la
mano de tales mediaciones, pues, podemos hablar de una
vía directa (y no absoluta) y cierta (aunque no necesaria)
de hacer presa, en razón de lo mejor, sobre nuestras voli-
ciones, de dirigir esas que Leibníz llama «voluntades con-
secuentes». Lo más esencial para él, en el asunto que nos
ocupa, es que la libertad no puede ser entendida como li-
bertad de in-dijerencia (esta última conduce a la indiferen-
cia de equilibrio). El principio de los indiscernibles se
opone, esta vez, a la concepción de libertad que ofrece el
propio Leibniz, mientras, bajo la concepción rechazada,
dirige su crítica a los discípulos de Escoto y a los de Moli-
na, pero también a los cartesianos. De cierto modo, Leib-
niz predica con el ejemplo: aquello que él podría «querer»
como Hobbes lo rectifica por medio de una inteligencia
que distingue siempre de antemano, siempre mejor, en el
equívoco de las nociones. Es por ello que se autoriza a de-
cir, desde una razón perfecta: hay razpn para... o ¿a verdad es
que...

PARA SABER MAS

Sobre Leibniz (Textos 11 y 12)

DILTHEY, W., Hombre y Mundo en ks siglos XVI y XVII, trad. E


Imaz, México, FCE, 1978.
MAGEE, B., Los grandesfilósofos, trad. Amaia Barcena, Madrid, Cá-
tedra, 1990, págs. 103-127.
ORTEGA Y GASSET, J., Ni vitalismo ni racionalismo, Obras Comple-
tas, III, Madrid, Alianza, Revista de Occidente, 1983, 12
vols.

Otros

HOBBES, Leviathan o la materia,formay poder de una república eclesiásti-

111 m „ § 47,

104
cay civil, trad. M. Sánchez Sarto cedida por FCE, Madrid,
Sarpe, 1983, 3 vols.
LEIBNIZ, Nuevos ensayos sobre el entendimiento humano, trad. R Eche-
verría Ezponda, Madrid, Editora Nacional, 1977.

13. Locke

Q U E NO HAY NI IDEAS INNATAS


NI C O N S E N T I M I E N T O UNIVERSAL

§ 2. No hay opinión más comúnmente aceptada que


la que establece que hay ciertos principios, tanto para la
especulación como para la práctica (pues contamos con estos
dos tipos) de la verdad en los cuales todos los hombres
convienen en general. De lo anterior se infiere la necesi-
dad de que estos principios sean otras tantas impresiones
que el alma del hombre recibe junto con su existencia y
trae al mundo con ellas de un modo tan necesario y real
como las propiedades que le son inherentes.
§ 3. En primer lugar, he de destacar que este argu-
mento, extraído del consentimiento universal, está sujeto a un
inconveniente, pues, aunque fuera cierto que de hecho
hubiese unas verdades acerca de las cuales todo el género
humano estuviera de acuerdo, aquel consentimiento uni-
versal no probaría en absoluto que tales verdades son in-
natas, si pudiéramos mostrar otra vía por la cual los hom-
bres hayan podido llegar a esta uniformidad de pareceres
sobre las cosas en las que todos convienen, lo que, si no
me equivoco, es por completo factible.
§ 4. Pero, lo que aún es peor, la razón que extrae del
consentimiento universal para hacer ver que hay princi-
pios innatos es, así me lo parece, una prueba demostrati-
va de que no existe ningún principio semejante, y ello
porque, efectivamente, no hay principio sobre el cual to-
dos los hombres se muestren en general de acuerdo. Y,
comenzando por las nociones especulativas, he aquí dos

105
de esos principios célebres a los cuales se concede, con
preferencia a cualquier otro, la cualidad de principios in-
natos: Todo ¿o que es, es y Es imposible que una cosa seay no sea ai
mismo tiempo. Estas proposiciones han pasado tan a menu-
do por ser máximas universal mente aceptadas que se en-
contrará, sin duda, muy extraño que cualquier otra cosa
pretenda disputarles su titulo. Sin embargo, me tomaré la
libertad de decir que mucho falta todavía para que estos
dos principios reciban el consentimiento general, pues
hay una gran parte del género humano que ni siquiera
tiene noción de ellas.
§ 5. Porque, en primer lugar, está claro que los niños
y los idiotas no tienen la menor idea de qué son tales
principios ni pueden pensar en modo alguno, lo que es
suficiente para destruir aquel consentimiento universal
que todas las verdades innatas deben producir necesaria-
mente. Porque decir que hay verdades impresas en el
alma que el alma no percibe o no entiende es, según creo,
una suerte de contradicción, pues la acción de imprimir no
puede apuntar a otra cosa (suponiendo que ella signifique
algo real en este caso) que a hacer apercibir ciertas verda-
des. Porque imprimir lo que sea en el alma, sin que el
alma lo aperciba, es a mi juicio algo apenas inteligible. De
existir impresiones tales en el alma de los niños y los idio-
tas, será necesario entonces que los niños y los idiotas
aperciban estas impresiones, que conozcan las verdades
grabadas en su espíritu y les den su consentimiento. Pero
como esto no es lo que sucede, es evidente que las impre-
siones señaladas no existen en absoluto. Ahora bien, si no
son nociones impresas naturales en el alma. ¿Cómo po-
drán ser innatas? Y, si, en efecto, están impresas en ella.
¿Cómo es que le son desconocidas? Decir que una noción
está grabada en el alma y sostener al mismo tiempo que el
alma no la conoce ni ha tenido todavía ningún conoci-
miento de aquélla, es hacer de esta impresión una pura
nada.

Ensayo Filosófica sobre el entendimiento humano


(1.» ed., 1689-1690), Libro I.cap. 1, §5 2-3-4-5. §

106
COMENTARIO

1. Locke es bien conocido por haber recogido la ima-


gen aristotélica del espíritu concebido como una tabula
rasa (el filósofo — L o c k e — dice, en realidad, «hoja en
blanco»), esto es, como una tablilla de cera sobre la cual
aún nada ha sido escrito. El origen primero de todo co-
nocimiento debe entonces buscarse en la experiencia (la in-
formación procedente de los sentidos, pero también la
percepción de las operaciones de nuestro espíritu sobre
las ideas que le han sido comunicadas por los sentidos).
De ahí el ataque frontal dirigido contra la tesis del inna-
tismo de las ideas y principios. De hecho, el mayor inte-
rés de Locke se dirige a los principios, y, sobre todo a los
principios especulativos, de lo cual da testimonio nuestro
texto que, por su parte, inaugura el Ensayo sobre el entendi-
miento humano con la intención de denunciar un prejuicio
continuamente admitido bajo el título de consentimiento
universal.
2. El consentimiento universal estipula que cada
cual puede reconocer inmediatamente como verdades pri-
meras, tanto lógica como temporalmente, a ciertos prin-
cipios que son el objeto de un acuerdo sin restricciones
entre los hombres, y que están grabados en el espíritu des-
de el nacimiento. Cualquiera que sea el origen (en el caso
estoico) de esta posición, de ella resulta siempre que la
reputada universalidad se erige en autoridad, siendo abe-
rrante, en consecuencia, discutir acerca de si está bien
fundamentado el reconocimiento de un valor de verdad
comandado por un instinto natural. Es, sin lugar a dudas,
este argumento de autoridad el que, desde el comienzo
del párrafo, sirve de motivo para la crítica lockeana. En
efecto, tras el espectro de la especulación se perfila la rei-
vindicación de la tolerancia, verdadero motor de la empre-
sa. Nuestro texto lo anuncia a su manera: Tanto para la es-
peculación como para la práctica (§2), expresión que hay que
entender en los términos siguientes: lo que vale para la

107
especulación vale igual, cuando no principalmente, para
la práctica en la medida en que los principios prácticos
(rectores de la vida común) son de ordinario menos segu-
ros que los especulativos (rectores del conocimiento). Un
pensamiento de tipo «empírista» tiene por finalidad la
práctica efectiva; y el aprendizaje de la vida en común pasa
por el rechazo de los dogmatismos.
Y bien, el primer dogmatismo reside precisamente, y
según Locke, en la aceptación de un consentimiento uni-
versal. La opinión admitida (la del consentimiento univer-
sal) toma el aspecto de un dogma al servir de base al argu-
mento del innatismo de los principios. Es, precisamente,
sobre este aspecto particular que Locke lanza su primer
ataque (§3), pues, aún cuando se admita la realidad del
consentimiento universal (que Locke, por su parte, nie-
ga), no por ello sería posible concluir el innatismo de las
verdades que conforman el objeto mismo de ese consenti-
miento. Dicho de otro modo, de la aceptación del con-
sentimiento universal al reconocimiento del innatismo
de los principios, la conclusión no es válida; al confundir
los dos planos se efectúa un saito que hipoteca de antema-
no toda posibilidad de establecer de otra forma (por la vía
de la experiencia, sobre todo) la eventual consistencia del
consentimiento universal (la víapor ia cual los hombres hayan
podido llegar a esta uniformidad...). En efecto, si la opinión del
consentimiento universal, punto de apoyo para el argu-
mento del innatismo, puede ser reconocido como el fruto
de una génesis, de una adquisición, el pretendido argumen-
to pasa por ser lo que verdaderamente es; a saber, un pre-
supuesto, un prejuicio, y no el resultado concluyeme de un
razonamiento bien ejecutado.
3. Sin embargo, hagámoslo notar, este primer ataque
hace una cesión al permitir suponer que podría haber un
consentimiento universal sobre ciertos principios, por lo
cual el único punto que discutir concierne al estatus de
las verdades objeto de aquéllos (¿son innatas? ¿son adqui-
ridas?).
El segundo ataque es más decisivo (§4), puesto que
vuelve a poner en cuestión la posibilidad misma de un

108
consentimiento general cualquiera a propósito de los
principios especulativos aparentemente más indiscuti-
bles: el principio de identidad y el principio de contradic-
ción (ios mismos que Aristóteles denominara «principios
comunes»), que son inmediatamente operativos en el in-
terior de todo pensamiento y de todo discurso. Locke,
por otra parte, es consciente de su atrevimiento (se encon-
trará, sin duda, muy extraño.,,), aunque tampoco vacila al to-
marse la libertad de afirmar que los susodichos principios
escapan a una buena parte del género humano.
El argumento (§5) que permite establecer las etapas de
esta radical acusación contra el consentimiento universal
acerca de los grandes principios lógicos, se funda en el he-
cho de que los niños y los simples de espíritu ignoran to-
talmente qué sean tales principios. La experiencia común
bastaría entonces para rechazar la opinión de un acuerdo
universal. Y, desde luego, podríamos atenernos a ello y
dar por cerrado el debate. Sin embargo, en las líneas que
siguen Locke responde, en realidad, a una objeción digna
de ser considerada: ciertas verdades están impresas en el
alma de cada cual, pero cada cual no siempre está en dis-
posición de tomar consciencia de lo anterior. Es posible
imaginar, por ejemplo, que hasta que un niño no haya al-
canzado la edad de la razón, el uso de principios especula-
tivos no estará del todo en su poder; dicho de otro modo,
estas verdades están en potencia en su espíritu (verdades... —*•
...que ti aima no percibe), y no estarán en acto mientras sus fa-
cultades intelectuales no se hayan desarrollado. En este
sentido, cabe imaginar que el simple espíritu posee en po-
tencia (como cualquier ser humano) los principios de
identidad y contradicción, aunque circunstancias parti-
culares (enfermedad, entorno cultural, etc.) le impidan,
quizá para siempre, actualizarlos plenamente (verdades...
—» ...que el alma no entiende). Con un espíritu similar, ciertos
autores consideraban, en el momento de la conquista del
Nuevo Mundo, que los principios de la ley moral estarían
grabados naturalmente en el alma de los salvajes, si bien
Ta especificidad de su cultura constituiría un obstáculo
para la actualización de esos mismos principios (el mis-

109
mo argumento, paralelo, del niño y el simple de espíritu
fue utilizado para esta precisa circunstancia). ;
En respuesta a la objeción señalada, que permite man-
tener conjuntamente la tesis del ¡nnatísmo de los princi- :
pios y la afirmación del consentimiento universal jugan- j
do con las nociones (de origen aristotélico) de potencia y i
acto, Locke pone de manifiesto una contradicción. En efec-
to, si todo lo que se halla impreso en el alma tiene que ser !

percibido, los niños y los simples de espíritu no pueden ¡


ser una excepción a la regla; si, no obstante, son una ex-
cepción (como es el caso), ello prueba tan sólo que no ;
hay verdades impresas en el alma ni, por tanto, ideas in- j
natas. No se puede afirmar ai mismo tiempo que una idea -
está impresa en el alma y que el alma no puede llegar a ;
conocerla. De sostener estas dos afirmaciones simultá- í
neamente, se estará reduciendo la impresión (en un sentido i
más primario; el hecho de imprimir algo) a una pura nada,
lo que, como es evidente, destruye la tesis del innatismo. 1
Los partidarios del argumento de la potencia y el acto |
(ciertos escolásticos y sus herederos) no han visto, según A
Locke, que ese mismo argumento pone en peligro la tesis %
que ellos, por otra parte, mantienen y pretenden así de- |
fender. Con el objeto de captar adecuadamente el alcance |
del ataque frontal que Locke dirige contra el consentí- g
miento universal y el innatismo de los principios especu- i
lativos debemos, para concluir, subrayar la diferencia que ¿
existe entre conocimiento empírico y conocimiento ra- -i
cional: Locke es el primero en reconocer que un niño 1
sabe perfectamente bien que su nodriza no es «el gato con 1

el cual juguetea», del mismo modo que no confundirá la


mostaza con el azúcar, ni aflorará en él duda alguna llega-
do cualquiera de los dos casos. Lo que Locke rechaza, sin
embargo, es la idea de que el niño pueda tener conoci-
miento de estas distinciones en virtud del principio lógico de
contradicción, cuya posesión activa sea lo que determine la
localización de las distinciones empíricas. Ni los niños,
ni los simples de espíritu detentan aquellas máximas abs-
tractas que se enseñan en las Escuelas.
4. A lo largo de todo el libro del Ensayo, Locke perse-

110
r - guirá, paso a paso, el cumplimiento de su expresa critica
con respecto a la tesis del innatísmo de los principios e
ideas. El único filósofo citado en la obrarserá Herbert de
Cherbury (1582-1648), cuyas obras adelantan, dentro de
la herencia estoica, las teorías del consentimiento univer-
sal y de las nociones comunes, blancos privilegiados de
los embates de Locke. No obstante, aparte de Cherbury,
podríamos considerar igualmente que Locke se dirige, de
un modo algo desordenado, a los platónicos de Cambrid-
; ge (Ralph Cudworth en particular, autor del Verdadero sis-
tema intelectual de! universo, 1678) y, hacia atrás en el tiempo,
[|
al propio Platón (a su teoría de la reminiscencia), así
como a la teoría cartesiana de las ideas innatas e, incluso,
a una cierta herencia escolástica. Poco importa aquí. Lo
I esencial reside en el hecho de que este «empirismo meta-
¡ físico», retomando la expresión hegeliana, se esfuerza por
| poner al día, en el seno de toda una tradición racionalista
| (en el sentido fuerte del término), cierto irracionalismo
ft más secreto que, al fundamentar las verdades de razón en
| un instinto natural cuya señal sería el pretendido consen-
í timiento universal, nos devuelve finalmente a un argu-
I mentó de autoridad que viene a desconcertar hasta la más
: mínima experiencia ordinaria. Este retorno a la génesis
i de las representaciones reactiva, en la época moderna,
f una tradición cómoda y, quizás, impropiamente denomi-
I; nada «empjrista» de la cual aún en nuestros días, a través
del vehículo del círculo de Viena y la filosofía analítica
anglosajona, se oyen ecos cada vez más amplificados. De-
bemos retener en la mente que Locke lanza prioritariamente
sus golpes contra el bastión especulativo y que éste sea un
buen método. En efecto, si bien la demostración crítica
de Locke atenta contra la máxima de la universalidad e
innatismo de los grandes principios lógicos, a fortmri será
válida para los principios prácticos (como, por ejemplo,
«no hacer al prójimo lo que no quisiéramos que nos hi-
cieran a nosotros mismos»), cuya certeza nunca será tan
indiscutible como la de los principios especulativos. En
este terreno, el dogma de la infalibilidad podría entrañar
consecuencias ruinosas para la vida en común.

111
PARA SABER MÁS

Sobre Locke

CASSIRER, E., Elproblema del conocimiento en lafilosofíay la ciencia mo-


derna, vol. II, trad. W. Roces, México, FCE, 1973-74,
GONZÁLEZ GALLEGO, A., Locke: empirismo y experiencia, Barcelo-
na, Montesinos, 1984,
MAGEE, B., Los grandesfilósofos, trad. Amaia Barcena, Madrid, Cá-
tedra, 1990, págs. 127-155.
M EL EN DO, T.,John Locke: ensayo sobre el entendimiento humano, Ma-
drid, Magisterio Español, 1978.
TIPTON, I. C., Lockt y el entendimiento humano, Ensayos Escogidos,
Tipton, I. C. (Compilador), trad. J. Ferrero Santana, Méxi-
co, FCE, 1981,

14, Hume

Q U E LA SOCIEDAD SUPLE LAS DEFICIENCIAS


D E L INDIVIDUO AISLADO

De todos los seres animados que pueblan el globo no


hay uno solo, al menos eso parece a simple vista, contra
el que la naturaleza se haya aplicado con tanta crueldad
como en el caso del hombre, y ello por la cantidad infini-
ta de necesidades corporales con las cuales lo ha provisto
y la precariedad de medios que ella ha dispuesto para sa-
tisfacerlas, En el resto de las criaturas estas dos circuns-
tancias se compensan generalmente la una a la otra. SÍ
consideramos al león en tanto que animal voraz y carní-
voro, descubriremos sin dificultad que está sometido a
randes necesidades; pero, si volvemos nuestra mirada
f acia su constitución y temperamento, hacia su agilidad,
coraje, garras y fuerza, encontraremos que sus ventajas
son proporcionales a sus necesidades. El carnero y el
buey se hallan privados de tales ventajas, pero sus apetitos

112
son moderados y fácil de obtener su alimento. Es en el
hombre únicamente donde se puede observar, en su más
alto grado de realización, la unión monstruosa de debili-
dad y necesidad. No sólo el alimento imprescindible para
su subsistencia huye de su búsqueda y acceso, o, al menos
reclama un penoso esfuerzo para su producción; sino que
es preciso aun que el hombre se provea de vestidos y de
una habitación para defenderse contra la intemperie. Sin
embargo, considerándolo solamente en sí mismo, no está
provisto ni de garras, ni de fuerza, ni de ninguna otra ca-
pacidad natural que pudiera responder en cualquier gra-
do a tantas necesidades.
Es por el solo concurso de la sociedad que el hombre es
capaz de suplir sus deficiencias, elevarse a un rango de
igualdad con respecto a sus compañeros de creación e, in-
cluso, adquirir superioridad sobre ellos. La sociedad
compensa todas sus imperfecciones. Aun cuando sus ne-
cesidades se multiplican a cada momento en este nuevo
estado, sus capacidades, sin embargo, aumentan, deján-
dolo, en todos los sentidos, tan satisfecho y feliz como
nunca le hubiera sido posible en su estado de salvajismo y
soledad. Cuando cada individuo trabaja aisladamente y
sólo para sí mismo, sus fuerzas son demasiado débiles
como para ejecutar una obra de importancia, pues, al em-
plear su trabajo en sobreponerse a sus diversas necesida-
des, no atiende jamás a la perfección en cualesquiera artes
particulares; como sus fuerzas y éxitos no siempre perma-
necen iguales a sí mismos, el menor logro en el uno o el
otro de sus aspectos se acompaña necesariamente de una
inevitable catástrofe, así como de infelicidad. La sociedad
ofrece un remedio para las tres desventajas mencionadas.
La unión entre las fuerzas acrecienta nuestro poder, la di-
visión de las tareas aumenta nuestra capacidad; la ayuda
mutua hace que estemos menos expuestos al azar y a la
casualidad. Es este suplemento de fuerza, capacidad y se-
guridad lo que hace más ventajosa a la sociedad.

Tratado de la naturaleza humana (1739-1740), Li-


bro Tercero, parte II, sección II.
COMENTARIO

1. En esta sección del Tratado, Hume se interroga so- j


bre el origen de la justicia y de la propiedad; esto es, sobre É,
los motivos que condujeron a los hombres a inventar re- m
glas que fijan los derechos y obligaciones apropiadas para £
la preservación del interés de cada uno y para promover M
así el interés común. Desde este punto de vista, nuestro S
texto pretende, ante todo, arrojar luz sobre la particular |f
situación de la especie humana en la economía general de %
la naturaleza; a la crueldad de la misma, el arte social debe |
responder en calidad de remedio. Podemos apreciar de un f
solo golpe de vista que la cuestión tratada no es la del ori- 1
gen de los gobiernos (materia que Hume abordará más %
tarde), sino la de la constitución del vínculo social.
2. El primer párrafo subraya la cualidad restrictiva V
de la naturaleza. Esta restricción es doble, siendo la pri-
mera la escasez relativa de los bienes disponibles (el alt- J
mentó imprescindible para su subsistencia huye de su búsqueda), la
precariedad de los medios puestos a disposición del hom-
bre. Esta cualidad restrictiva entra en contradicción con
la cantidad infinita de necesidades corporales con las que la natu-
raleza ha abrumado al hombre. Aunque generoso en cier-
tos aspectos, en otros la naturaleza se ha mostrado parsi-
moniosa; la alianza de esta generosidad perversa y de su
cruel parsimonia culmina en lo que Hume denomina la
unión monstruosa de debilidady necesidad cuyo desdichado be-
neficio sería el hombre. Finalmente, la naturaleza es una
madrastra, una mala madre. Alimentos, vestidos, habita-
ción; es decir, las tres necesidades fundamentales vincu-
ladas con la subsistencia y la protección, requieren pena-
lidades, esfuerzo, en una palabra, trabajo. Hemos de des-
tacar que el propósito de Hume se inscribe en una doble
herencia. De un lado, Platón, quien indica que las necesi-
dades de alimentos, vestidos y alojámiento son fundamen-
tos mismos de la organización social, recordándonos que
la ciudad debe su nacimiento a «la impotencia en la que

114
se halla el individuo a la hora de bastarse a sí mismo, así
como a la menesterosidad que demuestra en miles de co-
sas1». De otro lado, el epicureismo, que, al insistir en la
crueldad de la naturaleza, intenta aniquilar toda visión fi-
nalista y providencialista ansiosa por definir al hombre
como el feliz beneficiario de un plan divino.
A diferencia del mundo animal que, sea por un desa-
rrollo interno de las capacidades apropiadas para la satis-
facción de sus necesidades (ejemplo del león), sea por la
limitación de sus necesidades mismas (ejemplo del carne-
ro y el buey), consigue una relativa adecuación entre los
medios disponibles y las necesidades experimentales,
mientras que el mundo humano se caracteriza por un
profundo desequilibrio natural entre las aspiraciones y
los medios a los cuales acudir para alcanzar las primeras.
En la naturaleza, el hombre se encuentra desnudo. El en-
frentamiento del hombre con la naturaleza exterior en-
trañaría así un proceso de adiestramiento que nada debe a
la providencia y sí todo al arte y la industria. En efecto, es
de la confrontación entre la abundancia de las necesida-
des, los deseos humanos y la escasez de bienes en el seno
de una naturaleza hostil (fieras, variaciones climáticas,
etc.), lo que hace surgir la necesidad de la acción. Podría-
mos imaginar que la indolencia, constitutiva de la felici-
dad humana, se desplegaría con total plenitud en un
mundo donde se diera un perfecto equilibrio entre nece-
sidades y bienes; sería el patrimonio de una especie de
edad de oro.
Pero hablando desde los hechos mismos, es la impo-
tencia la que exige una ejercitación, necesidad de la ac-
ción en la forma de un proceso de transformación. Las
penalidades, el trabajo, aparecen en ios hechos, en razón de
constricciones a las que se encuentra sometida la natura-
leza humana en su relación con la naturaleza exterior,
como la dirección primera a la cual se compromete la ac-
ción humana so pena de ver amenazada la supervivencia

' Cfr. Rtpübiica, II, 369 b y ss.

115
misma del individuo y de la propia especie. La actividad
económica deviene de este modo prototipo de la acción.
La definición del hombre en términos de un ser para el
consumo (cantidad infinita de necesidades), calibrada conjun-
tamente con la escasez de los bienes inmediatamente da-
dos para su satisfacción y con la constancia de la amenaza
exterior, implica una definición del hombre como ser
productor a través de la puesta en escena de la invención
y del artificio, considerados como características esencia-
les de la naturaleza humana.
3. Así se justifica (segundo párrafo) la instauración
de la sociedad, cuyo primer paso aparece constituido por
la unión entre las fuerzas y la división del trabajo. La ac-
ción colectiva es requerida como rodeo de la satisfacción
individual del mismo modo que la distribución de tareas
en el seno de la colectividad está destinada a facilitar la
obtención de aquella satisfacción. La unión monstruosa
de debilidad y necesidad debe poder ser equilibrada, com-
pensada, mediante la unión benéfica entre los individuos,
débiles en medios y ricos en necesidades. La adición de
fuerzas representa una verdadera amplificación de la po-
tencia.
En cualquier caso, esta amplificación no puede ser
realmente eficaz más que cuando se acompaña de una di-
visión de tareas destinada a acrecentar la habilidad, la ca-
pacidad. Esta visión es bastante acorde con la representa-
ción general de las ventajas de la división del trabajo, tal y
como lo podemos descubrir desde Platón a Adam Smith,
aunque, en este caso, el punto de vista de Hume hace me-
nos referencia al perfeccionamiento de la actividad eco-
nómica que a su necesaria emergencia. La división es
coextensiva de la unión, y ello porque no hay unión que
sea duradera sí no se da allí donde existe división. Unióny
división pasan por ser garantes de la ayuda mutua que sinte-
tiza, así, los dos términos. En todos los casos se obtiene un
suplemento que, al permitir el acrecentamiento de los re-
cursos comunes, asegura, a través de este rodeo, el au-
mento de la satisfacción individual. A las carencias natu-
rales responde la astucia cultural, ella misma inscrita en

116
la naturaleza humana, con el posible destino de sus pasio-
nes: si el fin de la acción humana es consumir, la capaci-
dad de producción susceptible de poner en marcha los
medios adecuados para alcanzar este fin emerge del artifi-
cio, de las astucias, de la técnica.
No obstante, resulta paradójico que Hume sugiera sin
vacilar que la unión entre las fuerzas y la división de las
tareas exigidas en vista de la satisfacción de las necesida-
des, suscitan, por su propio movimiento, la multiplica-
ción de estas últimas (sus necesidades se multiplican a cada me-
mento en este nuevo estado). En lugar de estabilizar el consu-
mo, la sociedad no hace sino amplificar sus exigencias y
reforzar la definición del individuo mediante el consumo
infinito. Este, aparente, inconveniente, que resulta de las
ventajas de la división del trabajo, se sitúa en un proceso
donde las relaciones naturales sufren una inversión, pues,
si bien es cierto que la cantidad infinita de necesidades (ac-
tualización de ciertas necesidades, creación de otras nue-
vas) no experimenta alteración alguna, en otro sentido
será necesario reconocer que la precariedad natural de los
medios es substituida por la fuerza artificial de aquellos
medios que engendran un aumento continuo de la capa-
cidad, de la fuerza y de la seguridad. El suplemento así
adquirido reduce, por tanto, el distanciamiento entre los
fines y los medios de forma tal que, una vez problemati-
zada la satisfacción absoluta, subsiste la posibilidad de un
cálculo elemental en más o en menos que ahonda, de un
modo decisivo, la fosa abierta entre el estado de sociedad
y el de soledad. La virtud terapéutica del estado social
está ligada a la introducción de un suplemento que hace
las veces de remedio; la condición solitaria ignora, sin em-
bargo, toda posible mediación. Esta sencilla diferencia es
suficiente para hacer aflorar las ventajas de la sociedad. La
unión monstruosa entre debilidad y necesidad queda des-
plazada por la feliz unión de la fuerza/capacidad y de la
necesidad; la conversión de uno de los términos en juego
permite un nuevo equilibrio, relativo, en el seno del cual
las virtudes adapta ti vas tienen un empleo más justo. AI
pretender reparar las aberraciones naturales (la unión

117
monstruosa), el objetivo de Hume es de orden teratológi-
co; la práctica que suscita, y de la que a la vez da cuenta,
es, en esencia, un reajuste.
4. Antes de ser abordada como perfeccionamiento
de la producción, la unión entre las fuerzas y la división
del trabajo son aprehendidas como fundamento mismo
de la organización social. Ello significa que la única justi-
ficación de la sociedad es la necesidad económica en sí
misma. Su motivación determinante no es una inclina-
ción natural al intercambio (como Adam Smith, por
ejemplo), ní tampoco un designio providencial adecuado
para provocar las mejoras naturales de la capacidad hu-
mana, en pos de la consumación y cumplimiento de la
obra. Se trata, en suma, de la conservación del individuo
a la que se encuentra sometida, de una vez, toda realiza-
ción artificial, técnica u organizativa.

PARA SABER MAS

Sobre Hume

AYER, A. J., Hume, trad. J. C. Armero, Madrid, Alianza, 1988.


DELEUZE, G., Empirismo y subjetividad, trad. Hugo Acevedo, Bar-
celona, Gedisa, 1981.
MAGEE, B., Los grandesfilósofos,trad. Amaía Barcena, Madrid, Cá-
tedra, 1990, págs. 155-181.
NOXON, La evolución de ¡a filosofía de Hume, ed. Carlos Solís, Ma-
drid, Alianza, 1987.

118
15. Rousseau

S O B R E H O B B E S , QUIEN HA FALSEADO
LA PERSPECTIVA D E L D E R E C H O . . .

No vayamos sobre todo a concluir con Hobbes que por


no tener idea alguna de la bondad, el hombre es natural-
mente malvado, que es vicioso porque desconoce la vir-
tud, que rehusa siempre a sus semejantes servicios que
cree que no son su deber, ni que en virtud del derecho
que se atribuye con razón a las cosas que necesita, se ima-
gina neciamente a sí mismo como el único propietario de
todo el universo. Hobbes ha visto muy bien el defecto de
todas las definiciones modernas del derecho natural; pero
las consecuencias que extrae de la suya muestran que la
toma en un sentido no menos falso. Al razonar acerca de
los principios que establece, este autor debiera decir que,
siendo el estado de naturaleza aquel en el que el cuidado
de nuestra conservación es el menos perjudicial para la
del prójimo, este estado sería, en consecuencia, el más
adecuado para la paz, así como el más conveniente para el
género humano. Él dice precisamente lo contrario por
haber introducido fuera de propósito en el cuidado de la
conservación del hombre salvaje la necesidad de satisfa-
cer una multitud de pasiones que son obra de la sociedad
y hecho necesarias las leyes. El malvado, afirmo, es un
niño robusto, Y aun cuando estuviéramos de acuerdo,
¿qué se concluiría de ello? Que si, cuando es robusto, este
hombre es tan dependiente de los demás como cuando es
débil, entonces no hay clase de excesos a los que no se en-
tregue, que no golpee a su madre cuando ésta tarde dema-
siado en darle el pecho, que no estrangule a alguno de sus
hermanos menores si le llega a incomodar, que no muer-
da la pierna de otro cuando tropiece con él o esté enfada-
do. Pero son dos suposiciones contradictorias en el esta-
do de naturaleza ser robusto y dependiente; el hombre es

119
débil cuando es dependiente, y se emancipa antes de ser
robusto. Hobbes no ha visto que la misma causa que im-
pide a los salvajes el uso de su razón, como pretenden
nuestros jurisconsultos, les impide al mismo tiempo abu-
sar de sus facultades, como el propio Hobbes pretende; de
forma que se podría decir que los salvajes no son precisa-
mente malvados porque no saben lo que es ser buenos;
porque no lo es ni el desarrollo de las luces, ni el freno de
la ley, sino la calma de las pasiones y la ignorancia del vi-
cio lo que les impide obrar mal; Tanto plus in i/lis profitit m-
tiorum ignoratio, quam in bis cognitio virtutis*.

«Discurso sobre el origen y los fundamentos de


la desigualdad entre los hombres» (1755).

COMENTARIO

1. El problema que constituye el telón de fondo de


este texto es clásico. Los siglos xvn y xvm le ofrecieron
un amplio espacio en sus tratados de «filosofía política»,
en ausencia de una verdadera teoría de la historia. Nos re-
ferimos al estado de naturaleza, que Rousseau menciona ex-
presamente. En el caso que nos ocupa, es conveniente si-
tuar esta cuestión en una doble perspectiva. De una parte,
el problema del derecho natural, tan estrechamente liga-
do a aquélla, donde coinciden juristas y teólogos, confor-
ma el eje de referencias al que es necesario remitir ciertas
alusiones del texto con el objeto de clarificarlo. De otra
parte, es inseparable de un interrogante que recorre toda
obra de Rousseau: ¿cómo comprender la naturaleza huma-
na, si consideramos que tiene una historia, que es la histo-
ria de la razón (no confundir con la conciencia), desde el
niño al adulto, desde el salvaje al ciudadano? En esta gran

* «Tan provechosa es en ellos la ignorancia de los vicios como en


éstos el conocimiento de la virtud.» Cita de Justino, Historial (Libro II,
capítulo II, número 15) referida a los escitas. Es probable que Rousseau
ia descubriera en la obra de Grocio De jure beiti et paeis, II, cap. II.

120
empresa, los interlocutores de Rousseau no son menos
importantes: Hobbes, en primer lugar, cuyo De ave
(1642) y, sobre todo, el Leviatban (1651) no han cesado,
desde el instante mismo de su publicación, de alimentar
las más ásperas controversias. Los jurisconsultos siguen
en orden al anterior, y entre ellos, por no citar más que
los dos más célebres, se encuentran Grocío (con su De jure
beti't et pacis, 1625) y Pufendorf (con su De jure naturae etgen-
tium, 1672). Aparte, no podemos olvidar a Locke (y su Se-
gundo tratado del gobierno civil, 1690), ni tampoco a Spinoza
(y su Tractatus Theologko-politicus, 1670).
2. Una vez traído a la memoria lo anterior, nuestro
texto se presenta, en lo esencial, como una refutación del
«hobbismo», argumentada con un notable ingenio pero,
al mismo tiempo, construida con gran solidez. Al servi-
cio del movimiento y ritmo del pensamiento aparece
toda un retórica reflejada a lo largo de comparaciones por
oposiciones, de paralelismoo inversiones. Alimentadopor
las lecciones de la antigüedad, de las que sirve de muestra
la cita latina del final del texto, procedente del historia-
dor Justino, nos encontramos ante el arte de convencerse
a sí mismo, seguro de la imposibilidad de separar la belle-
za de la forma con la verdad del contenido, seguro, de
nuevo, de que cualquier propósito estará tanto más justi-
ficado cuando más «ajustado» sea. En el «sentido de la
fórmula» donde se concentra la totalidad de una cultura,
la filosofía se practicará en lo sucesivo como estilo que une
en un mismo amor por el lenguaje la exigencia estética y
el imperativo racional. Todo ello, por lo demás, se halla
presente en el principio y en el centro del texto, reivindi-
cado expresamente como su preocupación fundamental:
Rousseau razona acerca de razonamientos, acerca de conse-
cuencias extraídas de principios. Con el propósito de indicar
qué debemos concluir, el autor establece una comparación
entre aquello que Hobbes debiera haber concluido (este
autor debiera decir...), lo que él mismo, de hecho, ha dicho (El
dice precisamente lo contrario...) y lo que además pudiera ha-
ber concluido siguiendo la misma lógica (Yaun cuando es-
tuviéramos de acuerdo, ¿qué se concluiría de ello?), a partir de pre-

121
misas erradas (por haber introducido fuera de propósito...) o
contradictorias (¿por no haber pisto?). Con respecto a este
punto, podemos analizar formalmente la continuidad del
texto del modo que señalamos a continuación:
a) (—*•...de todo el universo.) No es necesario concluir como o o o
con Hobbes (que el hombre es naturalmente malvado)..,
b) (Hobbes ha visto muy bien... ~ » ...hecho necesarias las le-
yes)... porque ha ras&nado mal (a causa, ya volveremos sobre
ello, de su definición del derecho natural)...
c) (Elmalvado, afirma... ...antesde ser robusto)... como
lo atestigua una contradicción que es posible sacar a la luz
(extendiendo al salvaje, el hombre en el estado de natura-
leza, su definición del malvado...)...
d) (Hobbes no ha visto... ~~*• ...final del texto). La conclu-
sión que se debe extraer es la contraria de la de Hobbes;
esto es, que los salvajes no son precisamente malvados porque no sa-
ben lo que es ser buenos.
Más exactamente, la tesis que Rousseau sugiere en la
conclusión mencionada (deforma que se podría decir...) es re-
sultado de una doble oposición, tanto a los jurisconsultos
como a Hobbes, quienes, por su parte, también se oponen
entre sí (como el uso de su razón se opone al abuso de susfacul-
tades). De este modo vemos cómo se trama la necesidad
de una consecuencia que parece actualizar una vieja his-
toria. En efecto, Justino ya la escribía, referida a los esci-
tas, bajo la dinastía de los Antonínos, con las palabras que
el propio Rousseau reproduce en la cita latina que pone
fin al texto. Su traducción es la siguiente:
«Tan provechosa es en ella la ignorancia de los vicios
como en éstos el conocimiento de la virtud».
3. Sobre este fondo de debate podemos ya ver todo lo
que se debe clarificar y cómo proceder. No es posible
comprender la severidad de Rousseau con Hobbes más
que a la luz de aquello que separa a Hobbes de los defen-
sores del derecho natural clásico (los jurisconsultos).
Aquí reside el punto más sensible, pues Rousseau elogia a
Hobbes por la perspicacia crítica de la que ha dado prue-
ba al considerar aquellas definiciones (ha visto muy bien el
defecto de todas las definiciones modernas de derecho natural), pero

122
le reprocha el haberlas sustituido por otra no menos de-
fectuosa, tal y como se puede juzgar a raíz de «las conse-
cuencias que extrae de la suya».
a) Para la escuela del derecho natural, se trataba de eman-
cipar la política de la teología, de liberar de la tutela de la
iglesia a la ciencia del estado, sin necesidad, por tanto, de
separar la política de la moral (como podía haberlo he-
cho Maquiavelo). De ahí la suposición de un estado de na-
turales que precede al estado civil (cuya función es la de
unir con lazos más sólidos, en los términos de una con-
vención o de un contrato, lo que ya estaba unido en el es-
tado de naturaleza) y caracterizado por las siguientes pro-
piedades: todos los hombres son libres e iguales por natu-
raleza, las luces de la recta razón vuelven de inmediato
perceptibles las reglas de detecho y el deber que emanan
de la inmanencia de las leyes naturales y, por último y fun-
damentalmente, todos los hombres poseen, aunque aisla-
dos, una inclinación natural a la benevolencia, un «ins-
tinto» de sociabilidad que los empuja a entrar en asociación
(idea heredada de Aristóteles y de los estoicos). Así, pues,
en el estado de naturaleza, todos los hombres poseen,
aunque aislados, una inclinación natural a la benevolen-
cia, un «instinto» de sociabilidad que los empuja a entrar en
asociación (idea heredada de Aristóteles y de los estoi-
cos). Así, pues, el estado de naturaleza es un estado de
paz
b) En contraposición, para Hobbes, en el comienzo
que supone el estado de naturaleza, ningún hombre es na-
turalmente sociable, sino que cada cual se mueve impul-
sado únicamente por el instinto de conservación y un
instinto de apropiación que él mismo es función de las
necesidades naturales. Así, los hombres, o bien «buscan
muchos al mismo tiempo la misma cosa», como escribe el
propio Hobbes', o bien cada uno de ellos quiere ser más
que los otros. De ello resulta, como consecuencia de un
derecho que todos poseen sobre todo (tal es el derecho natural re-
definido por Hobbes), un estado de rivalidad y temor

1 Hobbes, De Gvt, I, capítulos 1 y 2; Levimbm, cap. XIII.

123
generalizados que hacen de todo salvaje un ser malvado,
precisamente porque el estado de naturaleza no es, en sí
mismo, más que un estado de guerra de todos contra todas. La
función del «contrato social» es la de poner fin a este esta-
do de guerra, pues las leyes civiles garantizan la seguridad
de cada individuo.
4, ¿A qué se dirige, entonces, la crítica de Rousseau?
En el punto de partida del texto se puede ver con claridad
que nuestro autor no discute las «cualidades» primeras por
medio de las cuales Hobbes define al Salvaje (ignora la
bondad, la virtud, el sentido del deber y tan sólo reconoce
un derecho fundamentado racionalmente en sus meras
necesidades). Pero pone en tela de juicio las característi-
cas segundas que se infieren de las anteriores, de modo que,
en consecuencia, el salvaje sería por naturaleza malvado, vi-
cioso, egoísta; se erigiría, ¡qué desatino! en propietario ex-
clusivo y absoluto. De las cualidades primeras atribuidas
al salvaje, una vez referidas a las necesidades relacionadas
con la autoconservación, la lógica hubiera querido que
Hobbes no extrajera nada más que las condiciones racio-
nales de un estado de naturaleza pacífico, fundamentado en
la coexistencia de necesidades moderadas y donde la sa-
tisfacción de los unos no entrañara daño alguno pata los
otros. Sin embargo, no es este el caso, y Rousseau explica
por qué: Hobbes ha puesto en juego, desde el estado de natu-
raleza, motivos (pasionales, como el deseo de gloria o de
riqueza, reflejos de la figura del propietario) que se co-
rresponden con necesidades en absoluto naturales, nece-
sidades que sólo la sociedad (que, sin embargo se trata de
engendrar o explicar a partir del estado de naturaleza) ha
podido engendrar. He aquí lo que Hobbes ha introducido
fuera de propósito en el cuidado de la conservación. La crítica es
decisiva. En efecto, Hobbes se ha hecho culpable de una
teoría deficiente porque ha sido víctima de una ilusión
retrospectiva, favorecida por su método analítico. En el
supuesto principio, Hobbes ha proyectado algo que única-
mente podía llegar a ocurrir, como una consecuencia, a lo
largo del tiempo y por efecto de toda suerte de cambios.
El estado de guerra no se origina con independencia de la

124
sociedad2, no existe previamente en el estado de naturale-
za en la forma de algo así como su «desnudez original». Es
necesario, por consiguiente, invertir la tesis hobbesiana,
recurriendo para ello, siempre que sea posible, al método
genético que Rousseau, precisamente, ha intentado aplicar
en su Discurso,
Si alguno desea una prueba (suplementaria) que no es-
cape a la crítica, tomemos en lo sucesivo la definición del
malvado que aparece en De Cive. Diderot ! ta encontraba
«sublime» y falsa solamente en el estado de la sociedad.
Rousseau la juzga incoherente desde el estado de natura-
leza. ¿Cómo podría ser salvado el salvaje, si aquél es un
niño robusto? ¿Cómo concebir al salvaje como un niño
robusto? Según la lógica de Hobbes, la ausencia de razón
en el niño, aliada con la fuerza del hombre, no puede dar
como resultado más que un monstruo de violencia. Para
Rousseau, suponiendo que el salvaje es tan dependiente
como un niño, no puede ser, lógicamente, otra cosa que
débil (lo que excluye que sea robusto). Suponiendo que
sea robusto como un hombre, el salvaje no puede ser, cro-
nológicamente, otra cosa que un individuo emancipado (lo
que excluye que sea dependiente).
Así, nada autoriza a afirmar que el hombre, en el esta-
do de naturaleza, sea malo por naturaleza. Hobbes extrae
de ahí su justificación de la soberanía absoluta pero, en el
caso de Rousseau, lo capital era invalidar una tesis que le-
gitima el despotismo. Dicho lo anterior, hay que tener
cuidado de no equivocarse con la tesis del propio Rous-
seau, como ya hiciera, deliberadamente, Voltaire: «a uno
le entran ganas de andar a cuatro patas cuando lee vuestra
obra»'1. La sola calma de las pasiones, que se desprende de
la moderación de las necesidades naturales, explica que el
salvaje no pueda ser malvado y que sea posible rechazar,

2 Texto paralelo: «Que l'éta: deguerre nait de l'état social», en CEsuw

Complitts, Bibl. de la Pléiade, t. III, págs. 601 y ss.


1 Diderot, articulo «Hobbismo», de La Emúhptiia,
4 Carta de Voltaire a Rousseau del 30 de agosto de 1755.

125
espalda contra espalda, las pretensiones simétricas y contra-
rias de Hobbes y los jurisconsultos. Sin embargo ello no
significa que sea bueno por naturaleza, ni tampoco que el
estado de naturaleza sea el ideal hacia el que hay que re-
gresar. La «bondad» del hombre natural no es más que
«estupidez»5. Es en sociedad que el hombre deviene bueno o
malvado, y esto recibe el nombre de progreso.

PARA SABER MAS

Sobre Rousseau

DERATHÉ, R-,/.-/ Rousseau et la science de son temps, París, Vrin,


1970.
GRIMSLEY, R., La filosofía de Rousseau, trad. Josefina Rubio, Ma-
drid, Alianza, 1988.
MOREAU, J., Rousseauy la jundamentación de la democracia, trad. Juan
del Agua, Madrid, Espasa-Calpe, 1977.

PARA PROFUNDIZAR

GOLDSCHMIDT, V., Antbropologe et Politique, Les principes du sisteme


de Rousseau, París, Vrin, 1974.

16. Kant
Q U E EL PROBLEMA DE LA CAUSALIDAD DETERMINA
EL TIPO D E METAFÍSICA

El punto de partida de Hume era, en esencia, un único


aunque importante concepto metafísico, a saber, la rela-
ción de causa a efecto (y, por consiguiente, también los con-
ceptos de fuerza, acción, etc. que de él dependen), y exigía
5 Ver Contrate social, I, cap. VIII.

126
a la razón, que pretende haberlo engendrado en su seno,
que le explicara con qué derecho piensa que una cosa
pueda ser de tal naturaleza que, una vez dada, se siga de
ella necesariamente que otra tenga también que estar
dada, pues no es otra cosa lo que afirma el concepto de
causa. Probó este autor, de forma irrefutable, que es por
completo imposible para la razón pensar a priori y me-
diante conceptos una relación tal, ya que ésta encierra
una necesidad; no es posible concebir cómo, porque una
cosa es, otra tenga también que ser, ni tampoco cómo es
posible introducir, a priori, el concepto de esta relación.
Hume concluía que la razón, simple y llanamente, se en-
gaña a sí misma acerca de esta noción, considerándola
falsamente como su propio vástago cuando no es más que
un bastardo de la imaginación que, fecundado por la ex-
periencia, ha reunido ciertas relaciones bajo la ley de aso-
ciación, haciendo pasar la necesidad subjetiva que de ella
se deriva, esto es, un hábito, por una necesidad objetiva
fundamentada en el conocimiento. De ahí concluía que
la razón no posee la facultad de pensar tales relaciones, ni
aun en general, porque sus conceptos no serían entonces
sino puras ficciones; y que todas sus pretendidas nociones
a priori no son nada más que experiencias comunes erró-
neamente etiquetadas, lo cual viene a decir que en abso-
luto hay, ni tampoco podría haber, metafísica.
Ahora bien, por muy precipitada e inexacta que fuera
su conclusión, al menos se fundamentaba en una investi-
gación, y esta investigación poseía el mérito de haber reu-
nido a los mejores espíritus de la época para resolver, en
la medida de lo posible, este problema más felizmente y
en el sentido por él propuesto; de ello resultó poco des-
pués una radical reforma de la ciencia.
Pero el destino de todo punto desfavorable de la meta-
física quiso que Hume no fuera comprendido. Nadie pue-
de evitar sentir cierto pesar al ver cómo sus adversarios
(...) se alejaron tanto del núcleo del problema. (...) No se
trataba de saber sí el concepto de causa era exacto, prácti-
co e indispensable para el completo conocimiento de la
naturaleza, pues esto es algo que Hume jamás puso en
duda. Se trataba más bien de si era concebido por la razón
a priori y de si poseía, así, una verdad interna, indepen-
diente de cualquier experiencia, y, por tanto, una utilidad
más amplia sin estar limitada a los objetos de la experien-
cia. Era acerca de esto que Hume esperaba comunica-
ción. La cuestión en juego no era otra, en suma, que la del
origen de este concepto y no la de su indispensable utili-
dad. (...) Lo confieso con franqueza: ésta fue la adverten-
cia de David Hume que, ya hace muchos años, interrum-
pió mi sueño dogmático para, en adelante, dar a mis in-
vestigaciones en el campo de la filosofía especulativa una
dirección por completo diferente.

Prolegómenos para toda metafísica futura que podrá


presentarse como ciencia (1783).

COMENTARIO

1. Hemos de acercarnos a la lectura de este texto en-


tendiéndolo como el retorno de Kant al «problema» de
Hume, considerado como decisivo en el marco de una re-
flexión acerca de la posibilidad de la metafísica. La reme-
moración, planteada de un modo muy conciso, de la ar-
gumentación de Hume a propósito de la causalidad (pri-
mer párrafo), sacando a ta luz sus reservas (sugeridas en el
comienzo del segundo párrafo) que serán desarrolladas
más adelante, conduce a una quasiapología del plantea-
miento del problema (y no de sus conclusiones) en relación
con la impresión manifestada por los primeros lectores
de Hume (tercer párrafo). En esta página de la obra kan-
tiana, el autor emprende del modo señalado la defensa de
Hume contra sus detractores, culpables de una lectura
errónea; no será sino en lo que sigue al presente texto que
Kant desarrollará la crítica de las conclusiones del esco-
cés para introducir, y situar, las suyas propias. Por el mo-
mento, a pesar de la rápida indicación de lo que supone el
punto central del litigio (por muy precipitada e inexacta que
fuera su conclusión), el hilo conductor de la discusión es el

128
homenaje, preparando así la llegada de la célebre «confe-
sión»: no es otro sino Hume quien habría interrumpido
el sueño dogmático de Kant.
2. El texto aborda, pues, la cuestión de la causalidad,
considerada como piedra de toque de la metafísica. En
este sentido, Hume habría arruinado la metafísica al mos-
trar que la razón es, en sí misma, impotente a la hora de
captar la relación de causa a efecto desde el ángulo de la
necesidad objetiva. En el primer párrafo, Kant retoma lo
esencial del propósito que guió a Hume a lo largo de las
secciones quinta y séptima de su Investigación sobre el entendi-
miento humano1. Hume, en efecto, centró toda su atención
en la relación de causa a efecto (un único aunque impor-
tante concepto...), en la medida en que el conjunto de los
razonamientos acerca de los hechos se fundamenta en esta
relación; ella sola desborda la evidencia de la memoria y
los sentidos, llevándonos a inferir, a partir de un evento
dado (por ejemplo, el movimiento de una bola de billar),
la aparición de otro evento (la comunicación del movi-
miento a una segunda bola). Ahora bien, el conocimiento
de esta relación no puede, en ningún caso, ser establecido
a priori, por intuición o por demostración; está entera-
mente fundamentada en la experiencia (la conjunción
constante de dos objetos particulares).
Una vez entendido que los razonamientos que versan
sobre hechos se fundamentan en la relación de causa a
efecto, y que todos nuestros razonamientos acerca de esta
relación se fundamentan en la experiencia, la vuelta de
tuerca de Hume consiste en denunciar el dogmatismo
metafísico (la razón presume de haber engendrado en su seno el
concepto de causalidad), problema que nunca había aflo-
rado de una preocupación por interrogarse acerca del va-
lor y legitimidad de los conceptos que ese mismo dogma-
tismo emplea sin inmutarse. Al subrayar que no hay más

1 Los Ensayos filosóficos sobre el entendimiento humano vieron la

luí en 1748 y fueron rebautizados en 1758 con el cítulo de Investigaciones


sobre et entendimiento humano, obra que fue leída por Kant en la traducción
alemana de Sulzer.

129
ideas oscuras en metafísica que las de poder, fuerza, ener-
gía y conexión necesaria, las cuales, por otra parte, son
prácticamente sinónimas. Hume tuvo el enorme mérito
de llamar la atención sobre la sospecha de una pseudoevi-
dencia y de exigir un replanteamiento de la pregunta so-
bre el origen del concepto causa. Sin embargo, según Kant
—y en ello consistiría la originalidad de Hume—, la
cuestión banal del origen queda inmediatamente traduci-
da al lenguaje jurídico: es preciso exigir a la razón que •
diga con qué derecho enlazar, con un vínculo necesario, a
una cosa con otra que no está intrínsecamente contenida
en la primera. Kant expresa, pues, en su propio idioma
(el del tribunal de la razón pura), la trayectoria seguida •
por Hume: se exige a la razón, se emprende una investiga-
ción, se aporta una prueba, se muestran las conclusiones. La
cuestión del origen de los conceptos queda, por tanto, li-
gada a la de la legitimidad del uso de los conceptos.
El problema que Hume presenta es, en efecto, de una
importancia crucial: detrás de la experiencia permanece
siempre la suposición de la constancia de la experiencia
(el futuro será semejante al pasado); ¿cómo podrá enton-
ces el pasado erigirse en regla para el futuro? ¿Qué nos
asegura la repetición de los acontecimientos? ¿Qué
nos garantiza que nuestra espera no será defraudada?
¿Cuál es aquí el principio que guia nuestros pasos? La res-
puesta parece simple, pues esa guía que buscamos es la
costumbre (custom) o el hábito (habit), principio de la na-
turaleza humana un i versal mente admitido y bien conoci-
do por sus efectos. La experiencia de la conjunción cons- 1
tante de dos objetos (por ejemplo, la llama y el calor, la í
nieve y el frío) nos hace esperar, de la mano del hábito, el |
calor o el frío toda vez que se presente de nuevo ante '
nuestros sentidos la llama o la nieve, así como también »
nos hace creer en la existencia de esa cualidad. Tal opera- -i
ción es, según Hume, inevitable; es una especie de «ins-
tinto natural» que no depende en absoluto del raciocinio.
La creencia es para Hume una manera de sentir (Jeeling)
más intensa que la que acompaña de ordinario a las sim-
ples ficciones de la imaginación. Se trata de una verdade-

130
ra convicción cuyo origen es, precisamente, la conjun-
ción habitual del objeto que se presenta ante nosotros
junto con el que continúa presente en nuestra memoria o
en nuestros sentidos. En todos los casos se da que, por
una transición habitual (asociación de ideas e impresio-
nes), pasamos de un objeto presente a la idea de otro obje-
to que hemos adquirido la costumbre de unir al primero.
Tal es, por consiguiente, la operación general y de exis-
tencia: el hábito, la costumbre, es el principio general que
guía el decurso de la operación. El concepto de causali-
dad no es, pues, el vastago legítimo de la razón, sino un
bastardo (esto es, un hijo ilegítimo) de la imaginación, en
la medida en que la experiencia de la conjunción frecuente
de dos objetos no puede, en ningún caso, permitir el legí-
timo acceso a la idea de una conexión necesaria entre los mis-
mos. La conexión entre acontecimientos es factual (aun
cuando se repita indefinidamente, aun cuando estemos
convencidos de que la llama quema y la nieve enfría),
pero nunca podríamos afirmar que es necesaria (no hay
contradicción en pensar que la naturaleza pueda cambiar
su curso y desconcertar así nuestras expectativas). El úni-
co origen de la idea de una conexión necesaria reside en
el hábito, que, forjado al hilo de la repetición de la con-
junción de dos acontecimientos, suscita el sentimiento de
una conexión habitual. Sin embargo, esta conexión es
subjetiva, pues la necesidad sólo existe en el espíritu, nun-
ca en los objetos mismos. Dicho con otras palabras, su
mecanismo escapa a la razón, y aquello que la metafísica
presenta como conceptos apriori, surge en realidad de una
serie de experiencias erróneamente etiquetadas. Existe,
en consecuencia, un fallo en el origen mismo de la mer-
cancía, siendo ilegítima la representación de la pretendi-
da necesidad la relación de causa a efecto apelando tan
sólo a los conceptos y sin recurrir en ningún caso a la ex-
periencia.
3. En este texto, la coincidencia entre Kant y Hume
llega lejos. De antemano, la prueba es considerada como
irrefutable. En términos kantianos podríamos decir que la
causa no procede de una simple relación lógica de prinri-

131
pío a consecuencia, que, en lo que a los hechos concierne,
la existencia no puede buscarse en la inherencia de un
predicado a un sujeto, y que, por tanto, la relación de cau-
sa a efecto no depende de ningún juicio de tipo analítico,
pues éstos son meramente explicitativos y nada añaden al
contenido del conocimiento (afirmar, por ejemplo, que
todos los cuerpos son extensos, no supone nada más que
explicitar el concepto de cuerpo, pues sería contradicto-
rio concebir un cuerpo que no ocupara espacio). En con-
traposición, en tanto que un concepto esté dado y se pre-
tenda ir más allá del mismo al ponerlo en relación con
otro concepto que no está contenido en él, se plantea el
problema de la posibilidad de un conocimiento sintético
apriori. Este es, según Kant, el problema de la posibilidad
misma de la metafísica, el mismo que Hume, a pesar de
sus conclusiones erróneas, logró sacar a la luz.
Prosiguiendo con el análisis del texto, la primera conclu-
sión (la razón se engaña acerca de la noción de causalidad)
puede, tomándola tal y como aparece enunciada y sin
prejuzgar de nuevo las críticas que Kant dirigirá contra
las conclusiones de Hume, estar relacionada con la idea
kantiana de una dialéctica trascendental, definida en con-
creto como la denuncia de un engaño, de una ilusión (por
otra parte inevitable) que emana de la razón desde el mo-
mento mismo en que afirma poder conocer algo además de
la experiencia posible.
Por último, ni siquiera la segunda conclusión (la razpn no
posee lafacultad de pensar tales relaciones, ni aún en general, porque
sus conceptos no serían entonces sino puras ficciones) escapa a la
evocación, mediada por los reajustes oportunos, de la
idea de que los conceptos puros del entendimiento (las
categorías) están irremisiblemente vacíos mientras no les
corresponda una intuición empírica (en el espacio y en el
tiempo) que los remita a una experiencia posible.
En contraste con lo anterior, el desacuerdo entre ambos
autores, que concierne a las conclusiones, no va a ser objeto
de un análisis minucioso en nuestro comentario. Tan
sólo conviene señalar que la única indicación en la cual
basarnos a la hora de abordar este tema viene ya dada des-

132
de el comienzo del texto (un único aunque importante concep-
to...). En efecto, Hume no se ocupó de otra cosa que no
fuera el concepto de causalidad, cuando, por ejemplo, el
principio de la permanencia de la substancia anticipa asi-
mismo la experiencia. Así, pues, tendría que haber gene-
ralizado el problema incluyendo en él, sobre todo, las
proposiciones de la matemática pura (que también son
sintéticas a priori y no analíticas, como él pensaba)2.
4. Antes de tomar el relevo del desafío humeano, re-
pitámoslo, la intención de Kant es la de destacar el carác-
ter fundamental del mismo y, por tanto, de asumir su de-
fensa frente a las lecturas erróneas o mal intencionadas.
El argumento del tercer párrafo juega con las nociones de
origen y utilidad. En efecto, como sea que Hume se inte-
rroga acerca del origen del concepto de causalidad, los
lectores podrían haber comprendido (o, al menos, simu-
larlo) que no estaba negando su utilidad. En realidad,
Hume quiso poner de manifiesto que, fuera de la expe-
riencia mental de la transición habitual del uno al otro,
no podemos esperar ninguna definición exacta de la cau-
sa ni de la conexión causa-efecto. Nada nos autoriza a pa-
sar de la conjunción a la conexión, a no ser la anticipa-
ción del efecto que es producto del espíritu aunque en
ningún caso aparece inscrita en el fenómeno mismo. Sin
embargo, el hábito, que dirige la representación de la re-
lación causa-efecto, es un principio práctico necesario
para la conservación del individuo y la supervivencia de
la especie, y ello porque dirige el ajuste de los medios a tos
fines y nos permite, por ejemplo, apartarnos de los peli-

2 Para el decalle de la crítica kantiana conviene acudir a Protégeme-


nos, fin de la introducción. Es de gran utilidad la lectura, en paralelo, de
la Crítica dt la razón pura, parágrafos 2, 3, 6-13, «Introducción» y «Lógica
transcendental» (apartados 1 , 2 y 3) y Libro II (sección 3. a , IV), trad. Pe-
dro Ribas, Madrid, Alfaguara, 1978; y de la Crítita a ¡a razpnpráctiia «Pró-
logo», «Introducción»), trad. E. Miñana y Villagrasa y Manuel García
Morente, Madrid, Librería general de Victoriano Suárez, 1913.
Se podrán completar esas lecturas con la obra de M. Malherbe, Km! en
Humeou taraitonelksensible (Kant o Humeóla razón y lo sensible), París,
Vrir», 1980 (en particular, cap. 1).

133
gros que nos pudieran amenazar. Este principio es, pues,
regulador de nuestras acciones en el mundo; es instintivo, y
su misma seguridad lo convierte en un mecanismo que
escapa a la razón. Hume, como parece evidente, no puso
en duda jamás ni la utilidad ni el carácter indispensable
del concepto de causa, en ausencia del cual nuestra segu-
ridad y el propio conocimiento de la naturaleza (la cien-
cia) serían tan imposibles como impensables.
5. La importancia del problema (condiciones de la
previsión, fundamento de la inducción, posibilidad de la
metafísica) es tan grande que aún continúa alterando a
los mejores espíritus de la filosofía contemporánea. Kant
nos se equivocó: independientemente del decisivo cam-
bio de rumbo efectuado por su propia «revolución coper-
nicana», el advenimiento del criticismo, concebido como
el acceso de la razón a su madurez, exige una ruptura con
el dogmatismo, infancia de la razón, ruptura que pasa por
el escepticismo cuyo heraldo es Hume, mensajero por-
tador de las exigencias que darán la señal que abra el
combate.

17. Kant

S O B R E L E I B N I Z , Q U E NO PODÍA D E F E N D E R
LA CAUSA D E Dios

Por el nombre de Teodicea se entiende la defensa de la


sabiduría suprema del autor del mundo con respecto a las
acusaciones que en su contra levanta la razón, apoyándo-
se para ello en lo que en el mundo no se nos muestra
como adecuado a ningún fin. A esta defensa se la suele
denominar defensa de la causa de Dios, si bien, en el fon-
do, no puede tratarse nada más que de la causa de nuestra
razón arrogante que desconoce sus límites. Es indudable
que la causa mencionada no es, precisamente, la mejor,
aunque puede, sin embargo, ser aprobada en la medida en

134
que (dejando de lado aquella presunción) el hombre, en
cuanto ser racional, tiene derecho a someter a examen
cualquier información y cualquier doctrina que le infun-
da respeto, antes de someterse a ella, con el objeto de que
este respeto sea sincero y no simulado.
Para esta justificación se requiere, pues, que el preten-
dido abogado de Dios pruebe: o bien que lo que juzgamos
como inadecuado a todo fin en el mundo no lo es; o bien
que, aun cuando lo fuera, no debería ser juzgado como
una consecuencia inevitable de la naturaleza de las cosas;
o bien, por último, que al menos no se lo debe considerar
como un factum del autor de todas las cosas, sino única-
mente de los seres del mundo a quienes se les puede
imputar algo, a saber, los hombres (o quizás también los
seres espirituales superiores, buenos o malos).
De este modo, el autor de una Teodicea da su consenti-
miento a que este pleito sea llevado al tribunal de la ra-
zón, comprometiéndose a asumir, en calidad de abogado,
la defensa de la parte acusada y a realizar la refutación
formal de todas las reclamaciones de la parte contraria.
No le está permitido, pues, desestimar las demandas du-
rante el curso del proceso decidiendo arbitrariamente la
incompetencia del tribunal de la razón. Dicho con otras
palabras, no está autorizado a rechazar los cargos recla-
mando al contrario un pacto sobre la sabiduría suprema
del autor del mundo, que viene a ser lo mismo que decla-
rar sin fundamento, en ausencia de todo examen, todas
las dudas que se pudieran suscitar contra ella.
Por el contrario, debe participar en el debate, arrojar
luz sobre las objeciones y reducirlas a la nada, con el fin
de hacer comprender que en absoluto atenían contra el con-
cepto de sabiduría suprema. Sin embargo, hay una empresa
a la que no necesita arriesgarse: la de tener que demostrar in-
cluso la suprema sabiduría de Dios a partir de lo que nos
enseña la experiencia de este mundo; con tales medios, en
efecto, no obtendría resultado alguno; pues es preciso ser
omnisciente para reconocer en un mundo dado (como se
da a conocer en la experiencia), aquella perfección de la
cual pueda decirse con certeza que es lo más grande posi-

135
ble y que no hay absolutamente nada, ni en la creación ni
en el gobierno de este mundo, que sea mayor.

Sobre el frataso de todas Jas tentativasfilosóficasen


materia de teodicea (1791).

COMENTARIO

1. Al culpar a la teodicea con el propósito de probar


que no puede satisfacer su ambición en una filosofía
consciente de sus tareas y poderes, Kant está culpando,
en realidad, a un hombre de quien recogió, a través de
Wolff, toda la herencia filosófica en esta materia. Se tra-
ta, evidentemente, de Leibniz, quien, conviene señalarlo,
no aparece nombrado expresamente en el corto tratado
de donde hemos extraído este texto. En efecto, es Leibniz
el creador del término, confeccionado a partir de dos pa-
labras griegas cuya combinación hace recaer en la noción
cierta ambigüedad. En ella encontramos el término Dios,
Tbéos, y la justicia/lo justo, diké/dikaios. ¿Se trata entonces
de hacer justicia en un asunto en el que Dios sería convo-
cado al estrado para rendir cuentas, o de derivar de la jus-
ticia de Dios todos los efectos esperables desde un punto
de vista lógico? Ya veremos que, para Leibniz, las dos
cuestiones se confunden. Y también que la crítica de
Kant centra su atención en esta misma confusión. Sin
embargo, habremos de comenzar por señalar que, de es-
tar defendiendo la causa de Dios', la idea no es nueva, pues
Platón se había ocupado mucho antes de disculpar a Dios
en una célebre fórmula (Dios no es causa; esto es, respon-
sable de todo...)2. Asimismo, la idea mencionada no pue-
de contrariar al buen sentido, porque, al afirmar Kant
que en ella hay arrogancia, tenemos motivos para dudar
de la bondad y de la inocencia de Dios, puesto que existe

1 La fórmula es de Leibniz, y aparece en sus Ensayos dt Teodicea, trad.

Patricio Azcárate, Buenos Aires, Claridad, 1946.


2 Ver República, II, 379 c.

136
el mal en el mundo, aun cuando esté contenido en el con-
cepto de la perfección de Dios el ser todopoderoso, y
puesto que es poder total es, necesariamente, causa de
todo lo que hay. Resta sólo señalar que, una vez surgidos
los términos del problema hay, para la razón de los filóso-
fos, un nudo que no sabrían deshacer sin dimitir de su la-
bor. ¿Se debe justificar a Dios? ¿Es posible hacerlo? No;
¿está permitido? Pero, ¿contamos con el poder y los me-
dios adecuados para intentarlo?
2. Ciertamente filosófico, el asunto se presenta, no
obstante, por sus considerandos, estilo y terminología,
como algo que, en esencia, es de índole jurídica. Se habla
en el texto de pleito, de defensa y acusación, de abogado, de tri-
bunal, de probar o refutar, de cargos, de agravios, o de objeciones; íes;
en suma, de la práctica común de los tribunales de justi-
cia. Y el problema de Kant, enfrentado a Leibniz, bien
parece ser el de definir con sumo cuidado la competencia
de este tribunal y la naturaleza del pleito, y al proceder así,
por otra parte, no hace sino reunirse con Leibniz en su
propio terreno, sometiéndolo a las «reglas del juego» que
este último ya había practicado en calidad de «especialis-
ta». No olvidemos quién fue Leibniz: «Jurista desde siem-
pre, doctor en derecho a los veinte años, juez o consejero
jurídico en diversos momentos de su vida profesional,
teórico y reformador del derecho a lo largo de innumera-
bles proyectos o propuestas»3. Pero merece la pena subra-
yar, antes de examinar la argumentación de Kant, que, al
reencontrarse aquí con la «causa», en su sentido primero,
jurídico (a saber: materia de un litigio donde se enfrentan
intereses contrapuestos, donde cada una de las partes re-
clama justicia; es decir, su merecido), la filosofía, preocu-
pada «después del origen» (recordemos a Sócrates comen-
tando a Anaxágoras, así como el interés de Leibniz por
este comentario) por la causa de todas las cosas, no está ha-
ciendo otra cosa, quizás, que redescubrir en la etimología
de la cosa (y recordemos que cosa es una palabra para ex-
presar la causa), la cuestión del derecho por encima de la

1 Ver la introducción dej. Brunschwig a los Ensayes dt Ttodiaa, op. ál.

\ 137
verdad, y el origen de la razón misma al tiempo que su in-
terés (tema recurrente en la obra de Kant, Critica de la ra-
Z'w pura).
Es necesario reconocer que la razón aparece implica-
da, en todos sus aspectos, en el denominado tribunal. Al i
mismo tiempo está acusada (por todo lo que hay de irra-
cional en lo que «no se nos muestra como adecuado a
ningún fin» que encontramos en el mundo) y es, por su-
puesto, acusadora (por las mismas razones y porque no so-
porta que se la contradiga...). La razón es a la vez parte y i
juez, tanto la instancia llamada a decidir y resolver, como |
los adversarios determinados a combatirla y someterla. '
En suma, se admitirá que, en una situación donde está i
llamada a formular el derecho la razón se encontrará en una *
posición que calificaríamos sin vacilar como falsa. En
cualquier caso, su estatus propio es netamente «crítico». j
No extraña que la razón esté en crisis, y, por tanto, encau-
sada (de igual modo, la causa de Dios sería más bien la causa 1
de nuestra razón), desde el momento en que lo inadecuado a :

todo fin (Zweckwidrig, escribe Kant, esto es, literalmente, i


lo que es contrarío al fin que uno se propone alcanzar) ha 1
dado jaque a la teología, naciendo así el mal en su acep- 1
ción más conceptual de «mal», a propósito de «mal» apro- 1
piado, de un «mal» ajuste (¿perversión? ¿subversión?) del
orden finalístico. Ahora bien, este último (teleológico: lo ;
racional ordenado por la perfección del fin), ¿no es, aca- •
so, precisamente después de Leibniz, el que mejor satisfa-
ce a la razón, dado que las causasfinalessatisfacen de igual !
modo al principio de lo mejor y, por consiguiente, al j
principio de razón suficiente y al de contradicción? 1
3. Lo inadecuado a todo fin sirve de punto de partida de |
la rebelión de la razón, del proceso iniciado y del tratado
de Kant. Surge así la pregunta de cómo conciliar todas las ;
imperfecciones de la sinrazón en los fenómenos físicos o
las acciones morales, con la santidad, justicia, bondad y ¡
perfección de la voluntad de Dios. ¿Qué «razón de ser»
tiene entonces el pretender justificar lo que hace insopor-
table a la existencia (el mal físico) u odiable (el mal
moral)? ;

138
El primer párrafo, a) tiempo que reabre el informe al
centrarlo en el «punto central de la acusación» de lo ina-
decuado a todo fin en el mundo, abre paso a dos conside-
raciones divergentes que, no obstante, se equilibran entre
sí, como para atestiguar la imparcialidad del juez, su be-
nevolencia en suma. De un lado, nos encontramos con
una reserva acerca del proceso en curso, reserva que lo es
desde un punto de vista teórico (una causa que no es, precisa-
mente, la mejor, y no por motivos de moral o piedad, sino
porque la razón reconoce en este terreno sus propios lí-
mites; tal es, para Kant, lo esencial del debate, como ya
veremos al final del comentario). De otro lado, contamos
con una aprobación de la empresa, desde un punto de vista
práctico y en nombre del principio de libre examen que
las luces han consagrado y que Kant reivindica para el su-
jeto moral (el ser razonable para quien son fundamentales
el respeto y la sinceridad).
El segundo párrafo aborda el fondo mismo del problema,
haciendo inventario de las pruebas que tenemos el dere-
cho (que es legítimo) de esperar del abogado de la defensa
(del autor de una teodicea), y que es posible determinar
con derecho (apriori) según la lógica misma (o bien, o bien)
de la cuestión (de la relación de finalidad):

1. Lo inadecuado a todo fin no es lo inadecuado a to-


do fin.
2. Lo inadecuado a todo fin no es final (no es un he-
cho, como una obra —un factum—, sino el efecto de una
causalidad eficiente).
3. ]x> inadecuado a todo fin es anti-final, pero siem-
pre como un hecho humano o sobrehumano, y nunca divi- ri-
ño.

El tercerpárrafo sitúa al abogado de la defensa frente a su


deber (aquello con lo que se compromete) y a su derecho
(aquello que no le está permitido, a lo que no está autori-
zado). Podemos observar ya que tanto el uno como el
otro aparecen marcados por el signo de la negatividad,
pues allí se trata de refutar al adversario; aquí, de someter-
se a una restricción del albedrio. Nos encontramos, de lleno,

139
emplazados en un racionalismo jurídico que funciona a la
manera de una crítica de la razón. El cumplimiento de la
obligación se dirige, de hecho, sobre un punto importan-
te: hay riesgo de provocar un «cortocircuito» en el asunto
debatido, cortocircuito que está ligado a la esencia de
aquello que está siendo «encausado», esto es, la suprema sa-
biduría de Dios. Exigir que ésta sea reconocida, y, por tan-
to, válida, significa, de entrada, invalidar los cargos y argu-
mentos del adversario. Es demasiado fácil, cuando se
quiere probar cualquier cosa referente a Dios, presuponer
(admitir o dar por sentado) lo que, precisamente, se pre-
tende concluir.
El último párrafo no hace más que prolongar los manda-
tos precedentes. Sin embargo, este punto del texto resalta
aún mejor el carácter meramente negativo del objetivo que
puede proponerse a sí mismo el autor de una teodicea.
Aunque tenga a su disposición los medios racionales
apropiados para operar motivamente (al deslindar el con-
cepto de sabiduría suprema de los argumentos en su contra)
y situar a Dios fuera de peligro, descargándolo de lo que
se le imputa, los límites de su saber le prohiben establecer
positivamente la perfección de aquella sabiduría, o, dicho,
de otro modo, no le está permitido probar que este es el
mejor mundo posible.
4. Hemos llegado, finalmente, a uno de los puntos
más importantes de los Ensayos de Teodicea de Leibniz, así
como a uno de los resultados más decisivos de la kantiana
Crítica de la razón pura.
a) Leibniz, en efecto, no soñó con defender la causa de
Dios4 de otro modo que no fuera aportando «otra idea de
la grandeza de las perfecciones divinas» y exponiendo,
pues, la esencia de su sabiduría, capaz de «sobrepasar in-
tensivamente a todos los posibles, y ello por causa de las
combinaciones infinitamente infinitas que hace con
ellos», y de comparar entre tales combinaciones «una in-

4 Ver Causa dei en Die philosophischen Schriften von Gottfríed Wil-

htlrn Leibniz, ed. de C. I. Gerhardt, 7 vols., Berlín, Híldesheim, 1960-


61, vol. 6, págs. 439 y ss.

140
í finidad de series posibles de universos, cada una de las
i cuales contiene una infinidad de criaturas». De lo ante-
[ rior sólo puede resultar, con exactitud casi matemática,
«el mejor de todos los sistemas posibles». Además, no es
posible, sin incurrir en contradicción, suponer «alguna
¡ cosa que supere en bondad a lo mejor», pues de lo contra-
rio sería, según las palabras de Leibniz en una analogía
que clarifica la lógica de su tesis, «como si alguien afirma-
í ra que Dios pudo trazar de un punto a otro una línea más
corta que línea recta»s.
, b) Para Kant, la única demostración posible de una
tal sabiduría sólo concierne a la nuestra, y siempre con el
objeto de establecer, como acabamos de ver, que debe ser
negativa; «a saber, la inteligencia de la necesidad que está
ahí para restringir nuestras pretensiones con respecto a lo
S que es demasiado elevado para nosotros»11. Esta conclu-
sión del opúsculo, a la vez que consagra, en Kant, la rup-

Í
,
tura con sus convicciones primeras, todavía muy cerca-
ñas a las de Leibniz7, confirma que la razón teórica no
podría obtener ningún conocimiento de Dios, sea cual fuere, par-
tiendo de la experiencia de un mundo que se nos aparece
como dado, pues sólo la razón práctica puede legítimamente
formar una idea de Dios. Esto es, entre otras cosas, lo que
1 la Critica de la razón pura quiso probar; es decir, que la ra-
zón especulativa tiene un interés por no atravesar los lí-
mites de la experiencia. De este modo, pues, la sensibili-
dad está perfectamente contenida entre sus propios lími-
tes y el uso moral de la razón pura (la razón práctica)
puede, así, prestar su mejor servicio8.
Lo negativo se transforma, aquí, en positivo. El principal
interés de nuestro texto es el haber intentado ilustrar que
; el progreso de la filosofía (de la metafísica depurada de
sus pretensiones abusivas) es posible, aun cuando no re-
[ sulte del todo claro que tal progreso sea correlativo del

' Ensayos de Teodicea, JS 224-226.


<> Sobre el fracase... op. cit., pág. 1403.
1 Ver Consideraciones sobre el optimismo (1759).
8 Cfr. Critica de la razó»pura, Prefacio a la 2." edición.

141
racionalismojurídico que Kant exhibe en et fragmento selec-
cionado, tomándolo como modelo una vez adoptada la
resolución de practicar, en el terreno de la filosofía, el
método de los jurisconsultos, de distinguir «en una causa,
la cuestión de derecho (quidjuris) de la cuestión de hecho
(quidfacii)», y, al igual que aquellos jurisconsultos, de le-
gitimar el uso de ciertos conceptos fundamentales para
toda «deducción»^. En cuanto a la finalidad, ocasión y
materia del debate en el proceso de la sabiduría divina,
Kant definió su estatus en la Crítica deljuicio: al hacer de
ella un principio únicamente subjetivo del juicio reflexio-
nante, nuestro autor desposeyó de toda legitimidad a un
uso de lo teológico que, en la teodicea, seria constitutivo
de un saber acerca de las intenciones de Dios1".

18. Kant

Q U E ROUSSEAU DA MUCHO QUE PENSAR


SOBRE EL PORVENIR DEL HOMBRE

Anteriormente al despertar de la razón no existían ni


prescripciones ni prohibiciones, ni, por consiguiente,
ninguna infracción. Sin embargo, una vez que ella co-
menzó a ejercer su actividad y, con todo lo débil que es, a
porfiar con la animalidad y su entera fuerza, no tardaron
en aparecer males, y, lo que es peor, en el estadio de la ra-
zón cultivada, vicios totalmente extraños al estado de ig-
norancia y, por tanto, de inocencia. El primer paso fuera
de los dominios de este estado fue, pues, desde el punto
de vista moral, una caída. Desde el punto de vista físico,
las consecuencias de esta caída fueron la aparición de una
multitud de males hasta entonces desconocidos y su con-
siguiente castigo. La historia de la naturales comienza, pues,

' Ibti., Analítica transcendental, cap. II, 1.» sección, § 13


Crítica del jwicié, Dialéctica del juicio teleológico, 5 75.

142
por el Bien, porque la naturaleza es obra de Dios; La historia
de la libertad comienza con el Mal, porque la libertad es
obra del hombre. Para el individuo, quien en el uso de la li-
bertad no piensa nada más que en sí mismo, el cambio su-
puso una pérdida; para la naturaleza, que persigue junto
al hombre su fin con las miras puestas en la especie, supu-
so una ganancia. El individuo tiene, pues, razones para
tenerse por responsable de todos los males que sufre, así
como del mal que él mismo causa, y, al mismo tiempo, en
tanto que miembro de un todo (de una especie), también
tiene razones para apreciar y admirar la sabiduría y ade-
cuación de este orden de las cosas. De este modo, es posi-
ble igualmente conciliar entre sí por medio de la razón las
afirmaciones con tanta frecuencia mal entendidas, y en
apariencia contradictorias, del célebre J.J. Rousseau. En
sus obras Sobre la influencia de las ciencias y Sobre la desigualdad
entre los hombres, este autor muestra, con total justicia, el
conflicto inevitable de la cultura respecto a la naturaleza
de la especie humana, entendida ésta como especie física
en el seno de la cual cada individuo debería atenerse a al-
canzar plenamente su destino; pero, en su Emilio, en su
Contrato social y en otros de sus escritos, Rousseau intenta
i hallar una nueva solución a un problema más grave:
¿cómo tiene que progresar la cultura para que se puedan
desarrollar las disposiciones de la humanidad, considera-
da como especie moral, en forma congruente con su des-
tino, de suerte que no se contradiga ya la especie natural?,
i De este conflicto nacen (dado que la cultura, según los
verdaderos principios de una educación que forma a la vez
hombres y ciudadanos, no ha comenzado todavía y no di-
gamos que terminado) los auténticos males que pesan so-
bre la vida humana y todos los vicios que la deshonran,
aunque los impulsos que nos empujan hacia los vicios, y a
los que se considera responsables de estos últimos, son en
sí mismos buenos, y, en calidad de disposiciones natura-
les, son también adecuados. Sin embargo, el desarrollo de
la cultura perjudica a estas disposiciones, una vez acepta-
do que estaban destinadas al simple estado de naturaleza.
Por su parte, esas mismas disposiciones perjudicarán al

143
desarrollo mencionado hasta que el arte, habiendo alcan-
zado la perfección, se transforme de nuevo en naturaleza.
Tal es el fin último del destino moral de la especie hu-
mana.

«Comienzo verosímil de la historia humana».


Anotación (1786), t. II. i

COMENTARIO

1. No hay nada como este texto (extraído de uno de


los Opúsculos históricos más apasionantes de Kant, ya vere- •
mos por qué) para tomar consciencia de todo lo que pue-
de hacer el entendimiento en su esfuerzo por discernir, de
su capacidad para variar y multiplicar las distinciones -
útiles o necesarias allí donde la materia en cuestión sea ¿
compleja o esté enmarañada. Y nada como este texto para |
quien desee entrar en el aspecto «vivo» del kantismo, de *
una filosofía siempre interesada por introducir algún or-
den en el espacio del pensamiento, y ello con el propósito I
de hacer progresar a «las luces», precisamente porque su f
mayor preocupación —como lo enuncia el final del tex-
t o — es el fin último del destino moral de la especie humana. La '
crítica de la razón especulativa tiene como destino clarificar,
desde sí misma, la razón práctica en cuanto a sus deberes, i
poderes y derechos. . t
En una primera lectura, nuestro extracto se presenta |
como un registro de oposiciones bien reglamentadas, lo '
que podemos juzgar a raíz de los binomios Dios/hombre;
especie/individuo; Bien/Mal; vicio/inocencia; razón/
animalidad; físico/moral (o, más concretamente, dispo- i
siciones/ responsabilidad); sufrir/causar; pérdida/ga-
nancía. Sin embargo, aún no hemos hablado de esa cons-
telación de parejas de términos que se organiza en torno a
un único polo, parejas que debemos considerar, por esta i
misma razón, como el «foco» de la perspectiva que Kant
se propone reflejar. Así, a naturaleza/libertad (oposición
fundamental que el propio Kant subraya) se superponen

144
naturaleza/cultora y naturaleza/arte. ¿Podemos, sin em-
bargo, darnos por satisfechos con esta apariencia de catá-
logo, sin ver en ella más que el efecto natural de un espí-
ritu analítico? ¿Cómo no sospechar que esa dualidad sis-
temática, que todo lo significa en la forma de un orden
dual, está enteramente afectada por la lucha cuerpo a cuerpo
dirimida al comienzo del texto (¿al comienzo de la histo-
ria?) entre razón y animalidad? ¿Cómo no relacionar esta
serie de «contrarios» con el pensamiento del «conflicto»
(Widerstreit, escribe Kant) que ocupa toda la segunda
parte del texto junto con (y a partir de) la referencia a J. J.
Rousseau? Resta, además, una última oposición que no
podemos omitir. Se trata, en efecto, de la oposición del
Rousseau «polemista» (autor de dos Discursos y filósofo del
conflicto, justamente, en guerra contra ta sociedad) con
respecto al Rousseau «teórico» (autor del Contrato Social y
de Emilio, preocupado por fundamentar y formar). Más
que un mero ejemplo, Rousseau es aquel pensador que
expuso en su obra, en pleno siglo de las luces, lo que ha-
bía interiorizado a lo largo de su vida; esto es, la violencia
de una «lucha de contrarios» que bien puede adoptar la
forma de la contradicción.
2. Henos aquí introducidos ya en el verdadero tema
del texto. La vida de los filósofos y la historia de la filoso-
fía terminan reflejándose reciprocamente en una filosofía
de la historia. Parece haber llegado el momento de propo-
nerse como objetivo todas aquellas crisis donde la razón
(en particular, la razón de los filósofos) exige nuevos
principios, quizás por no estar segura de los caminos que
sigue, o por haber entrado en conflicto consigo misma, o,
quizás, porque, en ámbitos diferentes pero relacionados,
la labor misma de la razón engendra aquí (en las ciencias y
las artes; es decir, las técnicas) el progreso, o, dicho de otro
modo, genera una acumulación (cuantitativa) de útiles de
conocimiento y de medios para la acción sobre la natura-
leza, así como un perfeccionamiento (cuantitativo) de to-
dos los instrumentos mencionados. Y ahí (en la solidari-
dad moral, en la paz social y en el equilibrio económico)
se originan regresiones, es decir, una alteración y una des-

145
composición de los vínculos, relaciones y proporciones,
todo lo cual podemos expresarlo con una sola palabra:
«corrupción». Tal es, en suma, lo Rousseau quiso estable-
cer en su dos Discursos, una constatación sobre la que Kant
nunca dejará de meditar. En efecto, si acudimos al párra-
fo 83 de la Critica deljuicio ideológico, encontramos allí un
resumen de la filosofía kantiana de la historia, pero tam-
bién podremos oír el insistente eco de nuestro texto. Nos
enfrentamos, pues, con un asunto enigmático para la ra-
zón en lucha contra lo que podríamos denominar sus
efectos «perversos»... Este es un objeto de estudio mal de-
finido a causa del cual y sobre el cual no conviene arries-
gar ninguna conjetura precipitada. De hecho, podríamos
preguntarnos qué quiere decir historia para Kant, en el
año 1789. Es posible distinguir tres órdenes de racionali-
dad en el proceso analizado por nuestro autor:
a) Una sucesión meramente temporal (un anteriormen-
te a que anuncia un «después de», una secuencia de apari-
ciones; una vez que... no tardaron...). Pero hemos de advertir
que enseguida aparece una notable complicación en esta
cronología, pues hay dos comienzos. Así, el primer paso se
cumple con el despertar de la razón, reíniciando de otra for-
ma lo que ya antes había comenzado. ¿Tenemos, enton-
ces, dos historias en una?
b) Una sucesión que podríamos calificar como lógica
y causal a la vez: la actividad de la razón entraña consecuen-
cias. ¿Una razón en, o de la historia?
c) Una sucesión «genética» en la cual la historia hu-
mana es aprehendida como génesis, como desarrollo de una
naturaleza, de disposiciones iniciales orientadas hacia un fin
(todos estos términos se repiten con insistencia). ¿Una
historia prescrita (ya escrita, prevista o proyectada antes
de su despliegue en el tiempo) como la de un organismo,
desde el germen a su forma adulta?
Llegados a este punto aparece uno de los «referentes»
(aparte de Rousseau) que facilitan a Kant los ejes de su re-
flexión: la historia natural de su época. Nos contentaremos
aquí con recordar la oposición (una más...) entre los de-
fensores de la preformación y los partidarios de la epigé-

146
nesis1, con el único propósito de señalar que, junto con la
emergencia de la razón, la realidad de una epigénesis pa-
rece hacer fracasar la pretendida intemporalidad de una
preformación. En este asunto no podemos olvidar a Buf-
fon (quien introdujo causas y factores a la hora de expli-
car la «degeneración» de ciertas especies animales) y, en
el lado opuesto, a Linneo (quien puso en juego la idea de
una finalidad en la creación a la hora de sistematizar el
orden y economía de la naturaleza)... Pero, ¿y el otro refe-
rente? Lo podemos obtener sin mayor dificultad a partir
de las menciones de la caída y el castigo (aunque todo lo que
precede a nuestro texto, en el Opúsculo original, se ocupa
de explicarlo expresamente). En esta ocasión se trata de
la.génesis, pero de una génesis que es menos la del relato bí-
blico que la del paraíso perdido. Así, pues, dos «escritu-
ras», la una santa, la otra profana, se entrecruzan en el
texto de la historia repensada por Kant. Y también son
dos los discursos (el de la tradición bíblica, de los «Testa-
mentos», y el de la innovación científica) que buscan el
encuentro, una unificación en la cual cada uno dé testi-
monio de un enfremamiento (la desobediencia a la vo-
luntad divina en el uno, el desacuerdo acerca del estatus y
sentido de los géneros y especies vivientes, en el otro). En
suma, nos enfrentamos con dos «antropologías», una de
ellas teológica (el hombre es una criatura de Dios) y la
otra «física» (el hombre es miembro de una especie natu-
ral'), que se esfuerzan por establecer el acuerdo entre sus
razones. Ahora bien, ¿cómo conciliar la razón teleológica
que parece gobernar con tanta regularidad la historia físi-
ca (la formación y reproducción de tipos científicos) y la
razón providencial, que parece estar requerida por la histo-
ria moral, aunque de un modo inconcebible —puesto
que todo el mal que hay en el mundo lo es por causa del
Mal...? Esta es, precisamente, la dificultad teórica.
3. Las consideraciones anteriores explican quizás
que todo nuestro texto sea una secuencia de paradojas. Una

1 Ver. J. Rostand, Esqukst i'une histoire de biolúgtt {Esbozo de una histo-

ria de la biología), Paris, Gallimard, Col. Idees, 1945.

147
vez abandonado (elprimerpaso) el estado de ignorancia y de ino-
cencia anterior al despertar de la razón (estado que parece
definido, negativamente, por sus carencias, aunque es po-
sible que se tratara, más bien, de un «pleno» aún no em-
pañado por lo prohibido), ¿con qué nos encontramos?
Con el primer «pues», nos instalamos en la primera pa-
radoja (...pues ...una caída ...—*• ...su consiguiente castigo). En
efecto, con la aparición de la razón el hombre se eleva por
encima de la animalidad pero cae en el mal; y cuanto más
se cultiva la razón, más se acrecienta el mal (cualitativa-
mente, si se nos permite decirlo así, los vicios además de #
los males; y, cuantitativamente, una multitud). La razón es,
por tanto, ambivalente en su origen, y ello porque es fuente
de la moralidad (por su lucha contra la animalidad) y, en
consecuencia, del Bien, aparte de ser causa de «decaden-
cia» debido al mal que ella causa (en el sentido más activo
del término).
Con el segundo pues, llegamos a la segunda paradoja (la
historia de la naturaleza... ~~* ...obra del hombre). Es el turno de
la libertad, que también deviene ambivalente. Es, preci-
samente, por causa de la libertad que el hombre se sustrae
a la necesidad natural (gracias a la cual puede ser moral).
Sin embargo, la naturaleza, obra de Dios, es buena, de
modo que es asimismo por causa de la libertad que el
hombre se sustrae al Bien. Por tanto, la libertad depende,
en tanto que principio y desde el principio, tanto del mal
como del Bien.
Con el tercer pues, alcanzamos la tercera paradoja, que en
esta ocasión es doble (Para el individuo... — ...este orden de las
cosas). El mismo acontecimiento (la razón, la libertad, el
mal) puede estar afectado por un signo contrario, según
que lo contemplemos desde el punto de vista del indivi-
duo (—, una pérdida) o de la especie (+, una ganancia). El
propio individuo tiene buenas razones para atribuirse este
acontecimiento como falta (es responsable de un desor-
den) y como mérito (se inscribe a través de él en un plan y
su orden propio: la sabiduría/finalidad de una orde-
nación).
Y, para finalizar, nos encontramos con la cuartaparado-

148
ja (De este modo... ...en tanto que espede natural). Todos es-
tos desafíos al sentido común deberán hacer racional la
(aparente) irracionalidad del pensamiento de Rousseau, y
ello a través de la oposición entre sus vertientes, o, dicho
de otro modo: la contradicción entre su constatación de
un conflicto inevitable entre la naturaleza (el Bien) y la cultura
(el Mal), y su proyecto de una armonización indispensable de
los fines de la naturaleza y de la cultura (el Bien y el Mal
se oponen en la cultura, pero esa oposición puede desapa-
recer con la educación). La totalidad de la última parte del
texto manifiesta la convicción de Kant de que la solución
reside en la lógica de la contradicción señalada; esto es,
en la naturaleza misma del conflicto.
4. Explicitaremos en lo que sigue la tesis de Kant.
Esta se centra, en lo esencial, en la constatación de una
«causalidad recíproca» de un género particular entre
naturaleza y cultura, cada una de las cuales perjudica el de-
sarrollo de la otra (la cultura a la naturaleza porque co-
rrompe los impulsos y disposiciones que, debido a su finali-
dad natural, son buenos en sí, pero degeneran al ser des-
naturalizados por la cultura). De ser preciso poner fin a
este doble perjuicio, será necesario entonces que el fin (la
finalidad) de la cultura coincida con el de la naturaleza, la
única que es buena por definición, pues es obra de Dios.
Será necesario que la razón suprima, entre cultura y natu-
raleza, la separación constitutiva de la primera en cuanto
a tal, separación que es obra de la propia cultura. Sólo así
se podrán cumplir los objetivos de la naturaleza (el Bien),
pero de tal modo que la cultura oriente hacia ella su per-
fección (en la moralidad). Podemos adivinar qué clase de
arte precisará la razón (y el tiempo que será necesario para
producirlo), es decir, qué extremada habilidad de medios
tendrá que ser desplegada para alcanzar una finalidad tal.
Pero ello exige que la razón se cultive hasta regresar al
«más acá» de su despertar...
Así, el comentario nos ha conducido, por última vez, a
la esencia paradójica de la razón, algo que la razón no
puede rechazar cuando reflexiona sobre su historia. Pero
¿no procede esto, en el fondo, del carácter contradictorio

149
con el que la naturaleza se presenta en el hombre? ¿Acaso el
arte humano se dirige hacia algo distinto de intentar ha-
cer viable la insociable sociabilidad que define, según Kant, a
la naturaleza humana? ¿Cómo entender esta naturales
¿Cómo explicar, desde su planteamiento, esa discordan-
cia que tiene por nombre razón? Las dos historias de nues-
tro texto; ¿no significan, quizás que existen, además de
dos razones (la de Dios y la del hombre), dos naturalezas
en una, cuya discordancia sería el único medio de una po-
sible concordancia, la única fuente de vitalidad? En La
idea de una historia universal, Kant escribía (en 1784): «El
hombre quiere la concordia, pero la naturaleza sabe me-
jor lo que es bueno para su especie: ella quiere la discor-
dia... Quiere que se lo obligue a salir de su inercia»2. Así,
contra cualquier clase de moral, lo que la naturaleza hace
del hombre parece contar más que lo que el hombre hace
de sí mismo. En esto consistiría «el orden de un sabio
creador»; la guerra, en sí misma, no sería nada más que
un medio para el perfeccionamiento del Bien; en ella po-
dríamos descifrar «la tentativa profundamente oculta e
intencional de la sabiduría suprema»1. Así, en lo referen-
te a la historia, la labor propia del entendimiento kantia-
no se encuentra cara a cara con el enigma de una razón
agazapada en una naturaleza buena, aunque astuta, o en
un Dios sabio, aunque él mismo esté oculto. Y tanto es
así que no podemos pedirle que asegure a la humanidad
su salvación, en tanto sea preciso asegurarla contra sí
misma. De ahí su Crítica.

PARA SABER MÁS

De Kant

Critica del juicio, trad. M. García Morente, Madrid, Espasa-Cal pe,


1989.
1 Ver Idtüs para una historia universal en clave cosmopolita, proposición

cuarta, trad. Carmen Roldan Panadero y Ramón Rodríguez Aramayo,


Madrid, Tecnos, 1987.
' Ver Crítica del juicio teieológico, § 83.

150
Sabnr Kant (Textos 16, 17 y 18)

CASSIRER, E., Kant, vida j doctrina, trad. Wenceslao Roces, Méxi-


co, FCE, 1974.
GARCÍA MORENTE, M., La filosofía de Kant, Madrid, Espasa-
Calpe, 1975.
HARTNACK, La teoría del conocimiento de Kant, trad. C. García Tre-
vijano y J. A. Lorentc, Madrid, Cátedra, 1984.
KÓRNER, S., Kant, trad. I. Zapata Pellechea, Alianza, Madrid,
1987.
MACEE, B., Los grandesfilósofos,trad. Amaia Barcena, Madrid, Cá-
tedra, 1990, págs. 181-203.
MARECHAL, J., Historia de la teoría del conocimiento, t. III: «El punto
de partida de la Metafísica», trad. Antonio Millán Puelles,
Madrid, Gredos, 1957.
PALACIOS, J, M,, El idealismo trascendental: teoría de la verdad, Ma-
drid, Gredos, 1979.

19. Hegel

Q U E LA C R Í T I C A DE K A N T NO HA LOGRADO TRASPASAR
LOS LÍMITES

Desde el interior del punto de vista de la antigua meta-


física se admitía que, cuando el conocimiento cae en con-
tradicciones, no se trata más que de una perturbación
contingente cuyo origen reside en un fallo subjetivo en el
encadenamiento silogístico y en el razonamiento. Según
Kant, en contraposición con lo anterior, está implícito
en la naturaleza del pensamiento mismo el zozobrar en
las contradicciones (antinomias) siempre que aspira a co-
nocer el infinito. Ahora bien, aunque la revelación de las
antinomias deba ser considerada como un notable pro-
greso del conocimiento filosófico, en la medida en que
gracias a ella se ha dejado de lado y enviado al movimien-

151
ío dialéctico del pensamiento aquel anquilosado dogma-
tismo de la metafísica del entendimiento, es preciso, sin
embargo, con respecto a este asunto, poner de relieve al
mismo tiempo que Kant, aquí también, se detuvo en el
resultado meramente negativo del carácter incognoscible
del en sí de las cosas, no habiendo penetrado hasta el co-
nocimiento del verdadero y positivo significado de las
antinomias. Ahora bien, el significado verdadero y posi-
tivo de las antinomias consiste, en general, en que toda
realidad efectiva contiene en sí misma determinaciones
opuestas, de modo que el conocimiento, y más concreta-
mente la concepción de un objeto, no significan, precisa-
mente, nada más que ser consciente de él como unidad
concreta de determinaciones opuestas. (...) Kant, en la
presentación de estas antinomias, se limitó a la cosmolo-
gía de la antigua metafísica y, en su polémica contra esta
última, tomando como base el esquema de las categorías
despejó cuatro antinomias. (...) Ahora bien, las pruebas
que Kant aporta para sus tesis y antítesis deben ser consi-
deradas, en realidad, como simples apariencias de prue-
bas, pues lo que debe ser probado está siempre contenido
en las proposiciones de partida, siendo únicamente por la
vía apagógíca, llena de redundancias, que se produce la
apariencia de una meditación. Sin embargo, el estableci-
miento de las antinomias continúa siendo, en cualquier
caso, un resultado de gran importancia y digno de ser
aprobado por la filosofía crítica en la medida que, en esas
antinomias (aunque por de pronto sólo sea de forma sub-
jetiva e inmediata), se expresa la unidad de hecho de las
determinaciones (opuestas), mientras que en el entendi-
miento permanecen firmemente separadas.

Enciclopedia de las cienciasfilosóficas,I, La ciencia


de la Lógica. Añadido al §48 (1830).

152
COMENTARIO

1. La Enciclopedia de ios ciencias filosóficas constituye,


como ya el título Índica por sí solo, más que la «suma» de
la filosofía de Hegel, una revisión de todas las cuestiones
metódicamente engendradas por la definición del Ser que
lo sitúa, desde y en su inmediatez, en la misma posición
de su contrario, la Nada, de modo que ambos devienen la
misma cosa. A lo largo de este «paseo» que, según su lógi-
ca propia, exige que recorramos sin descanso un camino
que se articula como un «círculo de círculos», hasta el
punto de llegada, lo que supone llegar, de nuevo, al punto
del cual partimos, el saber adopta la forma de un sistema
(que podríamos creer dogmático). Sin embargo, haría-
mos más justicia a Hegel si imaginamos ese recorrido
como la reorganización de un todo, aspecto este que hace
especialmente difícil el acceso a su filosofía, pues no es
posible entrar en ninguna de sus partes constitutivas sin
movilizar a un tiempo el movimiento completo de pensa-
miento que la determinó como tal y contiene su razón.
Ahora bien, la relación parte-todo nos hace ver también
la importancia del «punto de partida». En efecto, la posi-
ción del Ser como Nada revela la fuerza de lo «negativo»
en la afirmación misma, y en ello se manifiesta e\ pensa-
miento como lo que uno hace con el Ser en la actividad
que, precisamente, lo niega. La contradicción, por tanto, es
completamente inicial y rigurosamente productora de
todo el proceso enciclopédico: ella está en el origen de la
especulación hegelíana, y en ella reside su originalidad.
2. Sería necesario guardar en la memoria lo que aca-
bamos de decir para apreciar las implicaciones que nues-
tro texto comporta (texto que procede de un Añadido, de
un anexo; esto es, de un comentario «exotérico» destina-
do a ofrecer al auditorio, acerca del párrafo §48, las acla-
raciones o ampliaciones convenientes debido a la extre-
mada concentración del tema, al margen, par así decirlo,
del esfuerzo de conceptualización metódica). ¿Cuál es el

153
núcleo central del texto, lo más esencial del mismo? En el
terreno mismo de la «filosofía primera», pero en oposi-
ción a la antigua metafísica, Hegel nos propone una nueva
definición de la contradicción que entraña, como ya ve-
remos, una reinterpretación de la dialéctica, de las opera-
ciones del entendimiento y de la razón, de la esencia y de la
extensión del conocimientofilosófico,cuyo resultado es, cier-
tamente, una nueva comprensión de la naturaleza delpensa-
miento, como también correlativamente (y este es un as-
pecto de capital importancia), de lo que Hegel denomina
realidad efectiva (la Wirkiichkeit).
Pero sin Kant esta labor hubiera resultado impractica-
ble e innecesaria; antes que nada, y sobre todo, el texto
intenta hacernos entender, desde una relación critica, en el
mejor sentido del término, lafilosofíacrítica (es decir, el cri-
ticismo kantiano). En este caso, el juicio del filósofo es, a
la vez, el punto de vista del historiador: por su insistencia
en subrayar la antigüedad de la metafísica prekantiana, He-
gel nos invita a que tomemos nota de la modernidad del
«momento» kantiano. A este respecto, conviene enlazar
la importancia del resultado, tal y como señala Hegel al final
del texto, con la importancia del progreso mencionada en
las primeras líneas del mismo. Es indudable que esta con-
cepción de la historia en términos de progreso nace de una
filosofía, aunque ahora esta filosofía no hace nada más
que expresar la experiencia que ella posee de su propio
proceso de maduración (a través de los logros y crisis
de su razón), en el que Kant jugó un papel determi-
nante.
En relación con el aspecto señalado, en este extracto
nos vemos inmersos en una evaluación del kantismo más
que en su interpretación. Su última parte (Kant, en la pre-
sentación... —*• ...firmemente separadas), se propone, por otra
parte, hacer una especie de balance, acercando el aspecto
positivo (desde Sin embargo) al aspecto negativo, y ello a
pesar de ciertas reservas manifiestas (aunque por de pron-
to...). Antes de llegar a este punto del texto, Hegel ya ha-
bía ofrecido los elementos de apreciación adecuados para
sus propósitos. En primer lugar (Desde el interior... —*• ...in-

154
finito), es preciso tomar conciencia de una decisiva oposi-
ción: el punto de vista de la antigua metafísica y el de Kant son
tan irreconciliables, en lo que respecta a la relación del
pensamiento y ta contradicción, como la contingencia
que el primero ve en la contradicción y la necesidad que
ésta revela en el pensamiento. En segundo lugar (Ahora
bien aunque... ...de las antinomias), se descubre, esta ve2
entre Kant y Hegel y a propósito del estatus de las antino-
mias como tales, la misma oposición que entre lo mera-
mente negativo y lo positivo, o entre lo incognoscible y lo verda-
deramente conocido. A continuación llegamos (Ahora bien, el
significado... * ...determinaciones opuestas) a la explicación de
la tesis hegeliana, donde se indica en qué es necesario
profundizar para hacer buen uso de las antinomias; a sa-
ber, profundizar en el significado de concebir un objeto.
Y de este modo hemos alcanzado ya el corazón de la filo-
sofía de Hegel.
3. Y acceder al corazón del pensamiento hegeliano
significa pasar por las antinomias kantianas. Conviene
recordar brevemente cómo se articulan éstas y de qué
materias tratan. Desde la forma sistemática de un antago-
nismo donde cada tesis excluye a sus antítesis —y a la in-
versa— al tiempo que la requiere, Kant sostiene, sucesi-
vamente:
a) que el mundo es finito/infinito en el espacio y el
tiempo,
b) que es divisible/indivisible en partes simples,
c) que existe/no existe una causalidad libre para la ex-
plicación de sus fenómenos,
d) que el mundo implica/no implica un ser absoluta-
mente necesario.
Cada una de las afirmaciones (o negaciones) anteriores
puede ser probada, y con ellas nos hallamos frente a fren-
te con los conflictos de la razón consigo misma, resultan-
tes de su pretensión de atribuirse la serie íntegra de las
condiciones de los fenómenos, de abarcar «la absoluta to-
talidad del conjunto de las cosas existentes» —es decir,
eso que llamamos el «mundo». De ahí el marco cosmológico
en que se sitúa esta «antitética», como Hegel nos recuerda

155
de pasada', Y es cierto, como Hegel vuelve a recordarnos,
que estas «cuatro ideas cosmológicas» proceden de los
«cuatro nombres de las categorías» del entendimiento
(cantidad, cualidad, relación, modalidad), cuyo cuadro
expositivo confeccionó Kant en su Analítica trascendental1.
Hegel reconoce, con respecto a la forma general de la an-
tinomia, el mérito fundamental de haber puesto de relieve
la relación consubstancial que se establece entre la natura-
les del pensamiento y lo ineluctable de la contradicción (y
en esto consiste toda antinomia, si contradecirse es decir
que A y no A son igualmente verdaderos), y ello desde el
momento en que el pensamiento no se detiene en los lí-
mites de las cosas finitas. En una antinomia, la verdad no
es «sea A», «sea no A», separados entre sí por el principio
de (no) contradicción, sino la inseparabilidad (¿a unidad de
hecho) de dos determinaciones opuestas; y la esencia del pensa-
miento no consiste tampoco en la adhesión (fija) a una y
otra expresión, sino que se verifica en ese movimiento
necesario que es el paso de la una a la otra, a pesar de la
prueba de cada una, y en la tensión de esta coexistencia.
Se comprende ya que la in-quietud del pensamiento signi-
fique la eliminación del dogmatismo, si es que el dogma está
hecho de aserciones excluyentes de su contrario. Y si la
historia de la filosofía (después de que la metafísica dog-
mática reprimiera u ocultara la lección de Parménides y
Heráclito) se ha producido bajo la dirección de una razón
sujeta al modelo de la matemática, bajo el signo del prin-
cipio de contradicción que la gobierna, se compren-
de también que el mérito de Kant sea innegablemente his-
tórico.
4, Resta añadir que, desde el punto de vista filosófi-
co, para Hegel la obra de Kant no llegó a cumplir su pro-
pósito. La razón de esto último la ofreceremos enseguida,
tal y como aparece esbozada, de forma muy alusiva, en
nuestro texto: en su crítica de la razón, Kant ha quedado

1 Ver Critka de ¡a raspn pura, Dialéctica transcendental, libro II,

cap. 2, secciones 1 y 2.
2 Ibid., Analítica transcendental, Libro I, cap. I, sección 3.

156
prisionero del pensamiento del entendimiento, el mismo
que se «mantiene firmemente en la separación». De ello
se sigue que, al «enviar al movimiento dialéctico de la ra-
zón» el dogmatismo, Kant sólo pudo obtener una visión
«negativa» de la dialéctica como tal. De hecho, Kant sólo
quería alcanzar una «lógica de la apariencia» procedente
de «la ilusión de una extensión del entendimiento puro»
(de sus categorías, citadas más arriba) que toma «la nece-
sidad subjetiva de la unión entre nuestros conceptos exi-
gida por el entendimiento, por la necesidad objetiva de la
determinación de las cosas en sí». Esta ilusión es «natu-
ral» e «inevitable» sin embargo, lo cual hace que, según
Kant, exista una dialéctica «natural e inevitable», propia
del uso de la razón, que es necesario, en todo momento,
disipar, aun cuando jamás pueda llegar a ser anulada1.
Para comprender la crítica de Hegel, tendremos que re-
gresar a un plano más «técnico». En efecto, desde esta ló-
gica de la apariencia podríamos preguntarnos si es posi-
ble encontrar algo que no sean simples apariencias de pruebas,
junto con la apariencia de una mediación. En cierto sen-
tido, todo está ahí: en la antitética kantiana, el principio
de contradicción y su capacidad para desunir continúan
siendo la ley, y el entendimiento continúa operando al
abrigo de la razón porque la vía que sigue es apagógica (el
razonamiento se hace «por reducción al absurdo»; es de-
cir, cada argumento llega a probarse mediante la imposi-
bilidad de su contradictorio, en una circularidad que re-
duce a cada uno de ellos, en un último análisis, a nada
más que a su propia reafirmación —de ahí la redundancia
señalada por Hegel).
Debajo del movimiento dialéctico, subsiste, en Kant, la se-
paración,firmementeestablecida por el entendimiento, entre
el sujeto del conocimiento y el en sí de las cosas, en sí que,
precisamente, sólo es superable mediante una «ilusión»
(trascendental). Esta separación explica que la dialéctica,

1 Ibid., Dialéctica transcendental, Introducción, I, De la apariencia

transcendental.

157
inseparable de la actividad de la razón cuando está objeti-
vada en la revelación de las antinomias, no deje de ser subjetiva,
encerrada entre los límites de la subjetividad finita frente
a lo objetivamente incognoscible... Ya estamos en dispo-
sición de apreciar mejor el objetivo de la crítica hegeliana
del criticismo kantiano: el poder del entendimiento. Pero
también podemos vislumbrar a qué aspira: a liberar al mo-
vimiento dialéctico de esa tutela que lo mantiene sumido en
la apariencia, siendo este movimiento dialéctico lo pro-
pio de la razón y, más aún, la razón misma.
Hegel, en efecto, al negar lo que hay de meramente ne-
gativo en la posición kantiana, trata de producir la verda-
dera positividad de la dialéctica que en las antinomias
kantianas sólo aparece para criticar a la razón. Asimismo,
Hegel, al suprimir la separación entre el pensamiento y el
ser de las cosas, comenzando por la totalidad de las deter-
minaciones finitas, trata de hacer posible el conocimiento
del infinito. Y, al hacer surgir de la contradicción misma
la mediación (el paso efectivo del uno en el otro que, conser-
vando las marcas propias de la finitud, desplaza sus lími-
tes y los suprime en un encadenamiento silogístico que permite
a la razón tanto razonar como «dar la vuelta» al ser), He-
gel deja expedito el camino para concebir finalmente el obje-
to como una realidad efectiva, un ser de contenido más
pleno (auténticamente concreto). Y esto es así porque en el
objeto, una vez que las abstracciones de la subjetividad y
la objetividad, habiéndose transfigurado la una en la otra,
pasan en el objeto a formar una sola unidad a pesar de se-
guir siendo distintas (cada una sigue manteniendo su
condición de «otra» con respecto a la primera). En tales
condiciones ya no tiene sentido —si la contradicción,
por el movimiento de la mediación, se encuentra umver-
salmente en el corazón de toda realidad efectiva— pretender
hacer del razonamiento un accidente, o el signo de los lí-
mites de la razón teórica. Sin embargo, y para ser fíeles a
la lección de Hegel, si queremos concebir la filosofía
como la historia de «su» (o de la) razón, y esta historia
como una realidad concreta, ello significa también que
no tenemos derecho a separar aquellas «determinaciones
156
opuestas» que son la antigua metafísica y la filosofía críti-
ca, o el kantismo y el hegelianismo.

PARA SABER MÁS

De Hegel

Para una visión abreviada del conjunto de la lógica y el lugar


que en ella ocupa la crítica de las antinomias kantianas: Prope-
déutica filosófica, Z° curso (clase media), 2. 1 subdivisión.
Paralelamente: La ciencia de la lógica (1812), libro I, El ser,
sección II, cap. I, A, Observación 2, Buenos Aires, Clari-
dad, 1969.

20. Hegel

Q U E E L E S T A D O D E ROUSSEAU Y F I C H T E
RESULTA DEMASIADO ABSTRACTO...

Si se confunde al Estado con la sociedad civil, y si por


destino se le asigna la tarea de velar por la seguridad y
protección personal, es entonces el interés de los indivi-
duos como tales lo que constituye el fin último en vista
del cual aquéllos se unieron, y de ello se sigue que se ha
hecho depender del capricho de cada cual el convertirse
en miembro del Estado. Sin embargo, el Estado guarda
una relación completamente diferente con el individuo;
dado que el Estado es el Espíritu Objetivo, el individuo
no puede poseer, por sí mismo, una existencia objetiva y
una vida ética verdaderas más que cuando se hace miem-
bro del estado. La unión, en cuanto tal, es ella misma el
verdadero contenido y el verdadero fin, porque los hom-
bres tienen por destino llevar una vida universal; las res-

159
tantes formas que adoptan su satisfacción, actividad y
conducta tienen a este elemento substancial y universal
como punto de partida y resultado. Considerada de forma
abstracta, la racionalidad consiste, en general, en la unión
íntima de universalidad y singularidad. Considerada de
forma concreta, como aquí es el caso, consiste, en cuanto
a su contenido, en la unidad de la libertad objetiva, es de-
cir, de la voluntad universal substancial general y la liber-
tad subjetiva, en tanto que saber individual y voluntad
que persigue sus fines particulares —y por esta razón, en
cuanto a su forma, la racionalidad consiste en una mane-
ra de actuar determinándose según leyes y principios
pensados, o, lo que es lo mismo, universales. Esta Idea es
el ser eterno y necesario en sí y para sí del Espíritu. (...)
En lo que concierne al concepto [de Estado] y a su elabo-
ración, Rousseau tuvo el mérito de establecer un princi-
pio que, no por su mera forma (como es el caso de la so-
ciabilidad y de la autoridad divina), sino por su conteni-
do es un pensamiento y, a decir verdad, el pensamiento
mismo, pues él puso a la voluntad como principio del Es-
tado. Pero, como sea que no concebía a la voluntad más
que en la forma determinada de la voluntad individual
(Fichte haría lo mismo más tarde), y a la voluntad gene-
ral no en términos de lo que es racional en sí y para sí en
la voluntad, sino únicamente como un elemento común
que se desgajara de cada voluntad individual en tanto que
consciente, la asociación de individuos en el Estado de-
viene, en su doctrina, un contrato. El fundamento de este
contrato es el libre albedrio de los individuos, su opinión
y sus consentimientos explícitos, cualesquiera que sean.
El resultado de esto, por la vía de la consecuencia lógica,
es la destrucción de lo divino existente en sí y para sí, de
su autoridad y majestad absolutas. Una vez que estas abs-
tracciones llegaron al poder, nos ofrecieron el espectácu-
lo más monstruoso jamás contemplado desde que la hu-
manidad existe, a saber, la tentativa de asir enteramente y
por su comienzo la constitución de un gran Estado real,
destruyendo en él todo lo que existía y apoyándose en el
pensamiento con el fin de dar voluntariamente como

160
base a este Estado algo que sólo era supuestamente racio-
nal. Sin embargo, y al mismo tiempo, por no tratarse más
que de abstracciones sin Idea esta tentativa ha provocado
la situación más espantosa y cruel.

Principios de la filosofia del derecho, §258, Observa-


ción (1821).

COMENTARIO

1. En los principios de la filosofía del derecho de He-


gel, de donde hemos extraído el presente texto, se recoge
y reorganiza toda una herencia de tesis políticas concer-
nientes a la naturaleza de la Ciudad y a las formas de go-
bierno (desde Platón y Aristóteles a Montesquieu), así
como al origen y finalidad del derecho, o a la esencia de
la soberanía (desde Hobbes y Spinoza a Rousseau, Kant y
Fichte, como también Savigny y Haller, pasando por los
jurisconsultos, Locke y Hume). No debemos olvidar, sin
embargo, que Hegel, aún estudiante en el seminario pro-
testante de Tübingen, es el contemporáneo atento (y en-
tusiasta) de la toma de la Bastilla, de modo que, entre
1789 y 1820-21, tuvo que vivir forzosamente el desenlace
de la Revolución Francesa (La Convención y el regicidio,
el Terror, el advenimiento del Imperio, las guerras napo-
leónicas, la Restauración) como si se tratara de una histo-
ria donde la «lógica» de los acontecimientos políticos fue
sometida a prueba. El presente texto (procedente de una
observación; es decir, de un comentario destinado a ex-
plicítar, ilustrar y actualizar el contenido de un artículo, a
menudo enigmático por su concisión y densidad, locali-
zado en el párrafo 258, que define al Estado como lo racio-
nal en sí y para si, según se sigue del párrafo 257, donde se
lo presenta como la realidad efectiva de la idea de una ética)
pone de manifiesto, precisamente, la conexión que existe
entre las lecciones teóricas y las «prácticas». Pero también
destaca la implicación de la abstracción doctrinal en lo
concreto de la vivencia histórica. Bajo su aspecto dogmá-

161
tico se descubre la función crítica de la actividad filosófi-
ca, llamada en todo momento a reasumir las contradic-
ciones de la «racionalidad» con el propósito de facilitar el
progreso de la razón.
2. Comencemos, pues, por reconstruir la «problemá-
tica» en la que se inscribe la actividad crítica menciona-
da, y ello para poder, a continuación, descubrir sus mo-
mentos, fines, objetos e implicaciones.
La cuestión del Estado, tal y como la aborda Hegel en
esta ocasión, es en sí misma de una amplitud considera-
ble, pues al aspecto, clásico desde la Antigüedad, de su es-
tructura (de su organización y régimen), se viene a añadir
el aspecto de la génesis; esto es, aquel que se pregunta de
dónde procede su forma, de dónde procede su autoridad.
Y esto es así porque, en el seno de la propia historia, el in-
dividuo no ha cesado de imponerse, frente al Estado,
como una forma y una realidad tan indiscutibles como
irreductibles entre sí. En efecto, la proclamación de los
Derechos del Hombre y del Ciudadano no hace más que
registrar el advenimiento de una doble legitimidad. Aho-
ra bien, ¿cómo asegurarla, en el espacio y en el tiempo?
Así, pues, el debate tiene como tema un doble enfrenta-
miento: por una parte, la unidad de la persona individual
(uniendo en ella su naturaleza de hombre y su cultura de ciu-
dadano = miembro del Estado); y, de otra, la unidad que es la
polis en cuanto tal (unificando en ella, en un sólo cuerpo, la
multiplicidad y diversidad de los individuos). En este punto,
se plantea el problema de hallar el modo de distinguir
esta unión de esa otra unión que viene representada por la
sociedad como tal (.sociedad civil; esto es, «burguesa», bürgerli-
che Gesellscbaft en la terminología de Hegel)1, una sociedad
entendida como una asociación entre individuos que ha-
bitan en mutua vecindad en la ciudad y sus alrededores, y
unidos entre sí por toda suerte de relaciones de coopera-
ción e intercambio que satisfacen sus diversas nece-
sidades.

' Ver Principios de ¡a filosofía de! derecho, 3.a Parte, 2.a sección.

162
3. La necesidad de esta distinción constituye, de he-
cho, el punto de partida de nuestro texto.
Primera intervención critica: se refiere al núcleo del debate,
a la ¡nterrelación existente entre las tres «unidades» que
acabamos de identificar con el Estado, la Sociedad civil y
el Individuo. Palabra clave: no confundir. ¿Cuál va a ser el
instrumento de discernimiento filosófico?
Todo el primer párrafo (Sise confunde... —*• ...en si y para sí del
Espíritu) desarrolla la importante oposición (bien delimi-
tada por Sin embargp, —* ...completamente diferente, después
del primer punto y seguido) entre dos relaciones: la del
individuo con la sociedad civil, y la del individuo con el
Estado aunque sería más justo expresarlo así: la relación
del individuo con todos los demás en la sociedad civil, y
la relación del Estado con el individuo en el individuo
mismo. La diferencia irreductible de estas relaciones se
ilumina desde el interior en el momento en que se hace
aparecer lo que constituye su lógica, no siendo otra que la
finalidad, e\ fin último o verdaderofin que es su destino; dicho
con otras palabras, su lógica es aquello que las determina a
ser tales, o, para expresarlo de un modo que nos sitúe ya
en el camino acertado, aquello que quiere realizarse por su
mediación (por medio de ellas). No estamos hablando de
otra cosa que del interés de cada uno, interés que es necesa-
riamente particular en un caso, y en el otro, una vida uni-
versal. La segunda parte de este primer párrafo (a partir de
Considerada deforma abstracta) puede ser leída como una lla-
mada de atención sobre la doctrina indispensable para la
comprensión y justificación de esta oposición radical.
Segunda intervención critica (el segundo párrafo completo).
La crítica se desplaza ahora desde el fondo de la cuestión
a la historia de su formulación, y siempre en el plano de
la teoría. El desplazamiento se sitúa allí donde la historia
se ha hecho ella misma determinante, si bien todavía no
es satisfactoria (la conjunción pero marca, al igual que el
«sin embargo» ya citado, el momento crítico). Se trata de
reconocer los méritos de Rousseau (y de Fíchte) y el valor
del principio conservado para la elaboración del concepto Es-
tado. Pero también se trata de subrayar, sucesivamente, la

163
insuficiencia de su definición, lo inadecuado de su puesta
en práctica, lo nocivo de su acción. Este principio es la
voluntad2, cuya importancia se descubre ahora en todo su
alcance, mejor que en la rememoración de la doctrina,
pues aquélla, mal concebida, tiene por consecuencias, en pri-
mer lugar, la teoría del contrato social (todavía será necesa-
rio comprender por qué Hegel le es hostil), y, a continua-
ción, ¡a destrucción de la esencia divina del Estado (lo que nos
obliga a considerar las implicaciones de la filosofía políti-
ca de Hegel).
Lo anterior nos conduce (en el tercer y último párrafo) al
segundo desplaa&mknto de la critica, desde el plano de la teoría
y el pensamiento al de la práctica histórica y la experien-
cia de un gran Estado real, Francia en este caso3. ¿Cabe
preguntarse, en este sentido, si tal desplazamiento tiene
el propósito de instruir a la razón filosófica a través de
«lecciones de historia», o más bien el de verificar en el
proceso histórico la imparable lógica de la Idea? La lec-
ción sobre la que hemos de meditar es, en cualquier caso,
la siguiente: el episodio del Terror (de él se trata cuando
Hegel menciona la monstruosidad de la situación más espantosa
y cruel) manifiesta que la Razón se revuelve contra sí mis-
ma cuando practica la abstracción sin Idea como la verdad
que debemos querer realizar; esta «práctica» no es, enton-
ces, sino la peor de las violencias. Y es esto, precisamen-
te, lo que necesitamos comprender.
4. Para lograrlo es indispensable elucidar algunos
puntos doctrinales. Todo se verá con mayor nitidez si
prestamos atención a dos tesis esenciales que aparecen en
el texto, y si las situamos en el contexto más general de la
filosofía hegeliana (el de la Enciclopedia de las cienciasfilosófi-
cas'). La primera de las tesis concierne específicamente al
Estado, que es Espíritu objetivo, Idea o ser eterno y necesario en sí y

2 Ibid. Introducción a partir del $ 4. Sin embargo, y pata quienes de-

seen ahondar en el tema de la vduntad, nos remitimos para empezar a la


Enciclopedia, 3.a Parte, sección 1, C, $ 468 y ss.
1 Ver el texto paralelo: Lecciones sobre filosofía de ¡a historia, trad. José

Gaos, prólogo de J. Ortega y Gasset, Madrid, Alianza, 1985.

164
para si del Espíritu. Esto significa (en la medida en que lo
podemos explicitar brevemente) que en el Estado se unen
esos opuestos que son la subjetividad y la objetividad de la
voluntad (y, por tanto, de la libertad, que es el fin principal
de la voluntad racional), pero, además, también supone
la unión, en la propia objetividad, de lo universal inme-
diato y de lo particular. Por voluntad subjetiva o/y parti-
cular se debe entender la voluntad del individuo, movido
a la acción por sus necesidades (cuya satisfacción es diversa,
se trate de la felicidad, de la apropiación de bienes, de la
seguridad...), y, en suma, por sus intereses personales o
privados en tanto que sujeto psicológico o económico.
Sin embargo, habremos de entender por voluntad objetiva
y/o universal inmediata la voluntad que actúa inmedia-
tamente en la vida ética (la Sittlicbheit4), los usos y costum-
bres de una familia, de un pueblo; voluntad espiritual de
una comunidad y superior a la voluntad natural de los in-
dividuos, más efectiva que ésta última, al igual como el
todo, que es más efectivo que las partes. La unidad de to-
das estas determinaciones de la voluntad en el Estado
convierte al mismo en la objetivación más concreta del es-
píritu; es decir, lo más substancial que la idea efectúa, don-
de entra a la existencia la unidad, hecha asimismo con-
creta, de lo singular (el individuo) y lo universal (el
Todo), o sea, de la racionalidad en sí misma.
Llegados a este punto, nos encontramos ya con la ter-
cera tesis, muy general: ¡a racionalidad consiste, en general, en la
unión íntima de .universalidady singularidad, tesis en la que se
concentra y desde donde debe descifrarse el conjunto de
la dialéctica hegeliana. Asi, lo que supuestamente se con-
serva y, al mismo tiempo se suprime, en y por el Estado,
es la oposición entre la voluntad individual (como buen querer
y arbitro del Ubre albedrío) y la voluntad general (como deter-
minación inmanente de su fin; esto es, de la libertad ca-
racterística de toda autodeterminación). Para situar estas
indicaciones en la historia hegeliana de la razón, recorde-

4 Ver Principios de la filosofía del derecho, 5 142-156.

165
mos tan sólo que en el Estado se cumple la concreción
del derecho, o, lo que es lo mismo, el movimiento de la
voluntad hacia la libertad (ella misma substancial). Asi-
mismo, conviene traer a la memoria que la propia volun-
tad es, por así decirlo, la versión psicológica (en el Espíri-
tu) de la capacidad de autodeterminación de la Idea (en la
lógica), idea que es intemporal y, desde este mismo punto
de vista, divina. Se comprende entonces que el Estado, se-
gún Hegel, manifieste al fin en su existencia objetiva, en si
y para si, el carácter que en sí posee, al principio, la idea de
la que es su realización concreta. El Estado también pre-
senta la majestady autoridad de lo divino5. De igual modo,
podemos comprender ya que querer realizar lo racional sin
la idea (esto es, el concepto que ha alcanzado sus fines,
unificando en su objetividad los opuestos de lo subjetivo
y lo objetivo, y, en su universalidad, los contrarios de lo
universal y lo singular) supone violentar a la razón por
no haber comprendido lo que hay de concreto (en la efec-
tividad de la unidad entre determinaciones opuestas)6 en
la idea. Hay que saberla pensar íntegramente (dialéctica-
mente) para producir en la acción (política, histórica) la
verdadera libertad (substancial y universal); es necesario
además que la idea misma quiera realizarse en la voluntad
general.
5. Rousseau y Fichte no lo habían comprendido.
Para ellos, la universalidad del Estado procede del acuer-
do entre particularidades individuales (contrato), en lu-
gar de que el individuo sea el que tome su particularidad
efectiva del carácter concreto del Estado'. Ciertamente,
en la voluntad individual que quiere una asociación polí-
tica, el pensamiento ejerce su actividad, pero su finalidad,
en tanto que unilateral, continúa siendo particular, (y uni-

5 De ahí que exista, en Hegel, una teoría de las relaciones entre el Es-

tado y la religión. Acudir, por ejemplo, a ilñd., $ 552 y observación co-


rrespondiente.
6 Ver texto 19 y su comentario.
7 Ver Rousseau, Contrate tocia! (1762), I, 6; IV, I; y Fichte, Fcmdemtnts

du drott Mturel d'apres k principe de la doctrine de la ¡ritme (1797), II, § 17, tra-
ducción A. Renaut, PUF, 1984.

166
lateral porque es incapaz de asumir la unidad de contra-
rios o los dos términos de la contradicción). El «modelo»
del contrato social, concebido con el objetivo de dar ra-
zón del paso de la sociedad civil al Estado, parece sugerir que,
al resultar de la «coalición» y «adhesión» una multitud de
«átomos» sociales (los miembros de la sociedad civil) que
disponen de una razón según su libertad de elección, cada
cual será libre de convertirse o no en ciudadano. Tal es la
prueba par excelknce que demuestra la abstracción de la hipó-
tesis mencionada (como es abstracto, para Hegel, el todo
«mecanicismo» atomista), así como la razón del fracaso
de una empresa histórica que había creído poder funda-
mentar el progreso de la racionalidad política en una vo-
luntad «general» nacida de las voluntades individuales.

21. Hegel

R E A L I D A D DEL D E R E C H O , RACIONALIDAD D E LA FILOSOFÍA

También es preciso considerar que ha sido una suerte


para la ciencia (y, por otra parte, en conformidad real con
la necesidad de las cosas) que esta filosofía, que hubiera
podido desarrollarse como una doctrina académica, haya
entrado en una relación más estrecha con la realidad; en
esta última, los principios del derecho y del deber adquie-
ren cierta seriedad, y la luz de la conciencia tiene allí su
reino. Por ello, la ruptura no podía evitar hacerse mani-
fiesta. Los errores se producen a propósito de esta situa-
ción de la filosofía en relación con la realidad, y, volvien-
do a lo que ya señalé anteriormente, precisamente porque
la filosofía es el fundamento de lo racional ella es la inte-
ligencia de lo presente y lo real, y no la construcción de
un más allá situado Dios sabe dónde, o que, mejor dicho,
sabemos muy bien dónde se sitúa; está en el error, en los
razonamientos parciales y vacíos. En el curso de esta obra
he indicado que la propia República de Platón, imagen

167
proverbial de un ideal sin contenido, no llega a captar en
esencia nada más que la naturaleza de la moralidad grie-
ga. Platón era consciente de la existencia de un principio
más profundo que abría una brecha en esa misma morali-
dad, pero que, de este modo, sólo podía ser una aspira-
ción insatisfecha y, por tanto, podía aparecer como un
principio de corrupción. Platón, conmovido por una as-
piración tal, buscó un remedio contra aquel principio,
pero, como fuera que que no podía recibir otro socorro
que no descendiera de las alturas, sólo le era posible bus-
carlo en una forma exterior particular de la moralidad,
creyendo así convertirse en amo de la corrupción cuando
lo único que había logrado era atentar íntimamente con-
tra la personalidad libre infinita, lo más profundo que en
la moralidad había. Sin embargo, probó haber sido un es-
píritu muy preclaro, pues, de hecho, el principio sobre el
cual gira el núcleo fundamental de su idea es también el
eje sobre el que ha girado la revolución mundial que se
estaba preparando entonces:
Todo lo racional es real y todo lo real es racional.

Principios de la filosofía del derecho (1821), Pre-


facio.

COMENTARIO

1. Considerado en su globalidad, este texto aborda el


tema de la filosofía, su naturaleza, su posición en la reali-
dad histórica y su propia dimensión histórica.
Lo anterior nos puede ayudar a comprender tanto el
pensamiento de Hegel como la importancia que revistió
en su época para sus continuadores, los hegelianos. En
efecto, la fórmula final del fragmento que ofrecemos
constituye el resumen y, prácticamente, la consigna de
una filosofía comprometida. En este sentido, nuestro tex-
to puede ser considerado, sin riesgo a equivocarnos,
como la clarificación de esta fase clave, la más memora-
ble de todas.

168
Pero también es importante este texto en su calidad de
juicio sobre la República de Platón y de análisis del idealis-
mo en filosofía, aunque es aconsejable que este segundo
aspecto sea articulado, subordinándolo con el primero.
Si, de acuerdo con el método, partimos de la conclu-
sión, entendida como meta final a la que todo el texto se
orienta a través de diversos caminos, nos restará hacer un
balance analítico; es decir, enumerar y comparar los suce-
sivos argumentos que Hegel trae a colación desde el pun-
to de vista lógico.
La separación del texto según sus dimensiones natura-
les no tiene otra función que la ya señalada, consistiendo
ésta en examinar cómo la idea esencial, que en una pri-
mera lectura aparece como la más ambiciosa y'difícil de
establecer, es conducida mediante una serie continua de
razones que, en consecuencia, delimitan con todo dere-
cho aquellas partes del texto a las cuales confieren su uni-
dad y razón de ser.
El texto culmina, en primer lugar, con un resultado in-
termedio que la conclusión retoma a un nivel superior de
generalidad. Así, la fórmula lafílosofía... es la inteligencia 4c lo
presente y lo real pone fin al primer desarrollo y sirve de
punto de partida para el segundo (el caso de la República
de Platón) como ejemplo limite que verifica a contrario
la conexión entre lo real y la filosofía. Estamos, pues, auto-
rizados para distinguir tres partes concatenadas entre sí:
—Desde También... (comienzo del texto) —* ...en los ra-
zonamientos parcialesy vacíos: el fracaso de las filosofías rela-
tivistas ilustra una ley general, que la filosofía no está fue-
ra de la realidad, sino en su mismo fundamento.
—Desde En el curso de esta obra... ~~* ...revolución mundial
que se estaba preparando entonces: ta República de Platón trata,
en efecto, del presente real de su tiempo, pero está aboca-
da a concepciones ideales, de modo que la relación entre
realidad y racionalidad no es, por tanto, adecuada, y ello
ilustra a contrario la necesidad misma de su identidad.
—La conclusión aporta, entonces, el secreto de esta
ley, o su necesidad, en una fórmula lacónica cuya sime-
tría habremos de comentar.

169
2, Dificultad inicial: el sentido del texto no será evi-
dente hasta que hayamos identificado a «esta filosofía»,
mencionada por Hegel en las primeras líneas del texto.
La llamada doctrina académica no basta para caracterizarla,
pues se trata de un relativismo escéptko que considera
válida cualquier opinión porque «el conocimiento de la
verdad» es «una tentativa insensata»1. Una cosa es tolerar
todas las opiniones, y otra buscar la verdad más allá de la
opinión, tarea propia del filósofo, cuyo ámbito no es el de
la ligereza, como tampoco el del consentimiento.
Así, a la futilidad de una filosofía que «nivela todos los
pensamientos y todos los objetos»2, Hegel opone el espíri-
tu de seriedad característico de otra filosofía que se ocupa
de los principios del derecho y del deber. Este es el objeto de la
obra a la que el texto pertenece. Hegel describe en el pri-
mer párrafo las desventuras de los filósofos que creyeron
poder hablar de todo sin tocar jamás la realidad, en la for-
ma de una resistencia de sus principios. A su ceguera se
opone la luz de la conciencia que reina, no en los meros
derechos y deberes, sino en la realidad con ta que aquellos
filósofos, a pesar de ellos mismos, se encuentran cuando
intentan salir del círculo escolar, Al partir en busca del
mundo, lo que descubren no es la mundanidad, sino la se-
riedad de la historia. El epílogo nos dice que la ruptura se
manifiesta, y que no podía ser de otro modo; la fórmula,
pues, exige un esfuerzo de interpretación. Hegel no pre-
tende afirmar que «se produce una ruptura», sino que la
ruptura (preexistente, por tanto) no puede evitar hacerse
manifiesta. ¿En qué consiste tal ruptura? La continua-
ción del texto se ocupa de señalar el punto de ruptura,
concerniente a la forma como la mala filosofía, o doxolo-
gía, se dirige a la realidad. Sin embargo, la solución surge
a partir de un tercer término introducido por Hegel a
modo de mediación: lo racional.
Se comprende ya que nuestro autor no pretende arre-
glar cuentas con sus colegas contemporáneos, antes bien,

1 Cfr. íupra, págs. 167-168.


2 Cfr. Principios de ¡a fiiosiifía del derecho.

170
quiere ilustrar el estancamiento al que está destinada
toda concepción de la filosofía como discurso inoperan-
te, paralelo al curso de las cosas, pero sin modificarlo ni
siendo modificado por él. El caso de los agnósticos, de lo
que les ocurre en relación con la filosofía, es revelador,
pues éstos, creyendo protegerse del error mediante la sus-
pensión del juicio, separan a la filosofía de la racionali-
dad; pero, sin ellos mismos saberlo, caen en un notable
error, esto es, en la falsedad de su juicio acerca de la reali-
dad, La causa de lo anterior es que la realidad no es extraña
a la racionalidad, pues la una es la clave decisiva para la
inteligencia o comprensión de la otra, y ello porque am-
bas poseen una naturaleza común cuya señal, cuando no
su causa, son aquellas mismas desventuras que mencioná-
bamos más arriba. En efecto, al creer hablar sin conse-
cuencia, el filósofo se pierde en el presente, en la aprecia-
ción del presente histórico.
Así, pues, las relaciones establecidas entre la filosofía y
el presente histórico, a menudo consideradas como nulas e
insignificantes cuando se toma desconsideradamente el
ejercicio del pensamiento por una huida del tiempo y de
la realidad, nos remiten sin embargo a la inteligencia de
la estructura misma de la realidad, con respecto a la cual
la filosofía se presenta como su única aprehensión genui-
na. Esto es lo que las líneas siguientes del texto se ocupa-
rán de establecer.
3. Para empezar, conviene recordar una frase ante-
rior del texto: la filosofía... no es la construcción de un más allá
situado Dios sabe dónde.
Hegel ha precisado su diagnóstico sobre la doxología
en respuesta a la objeción que se hace a sí mismo en la fra-
se citada, afirmando que el lugar de esta concepción de la
filosofía es el error y no la neutralidad. No existe la uto-
pía, como tampoco existe lugar sin lugar ni pensamiento
sin historicidad; lo único que hay es la relación con la
verdad.
El problema planteado es, en consecuencia, el del idea-
lismo, componente universal de toda verdadera filosofía,
incluida la del propio Hegel. Hay que evitar por todos los

171
medios identificar la concepción hegeliana del platonis-
mo, del cual nuestro autor desarrolla aquí un solo aspec-
to, con una crítica del irrealismo filosófico como la que
ocupa la primera parte de texto. El único aspecto que po-
seen en común es el de servir de ilustración de la relación
necesaria de la razón con la realidad histórica dada. La di-
ferencia es la que Platón, aquel espíritu tan preclaro, supo
percibir en la realidad griega de su época: la tensión entre
una moralidad común y un principio superior que la
amenazaba. Sin embargo, Hegel somete a crítica la solu-
ción platónica, pues ésta, paradójicamente, muestra una
correspondencia muy débil con lo racional y demasiado
fuerte con la realidad, puesto que busca en una forma exte-
rior de ¡a moralidad algo que, de hecho, la desborda y englo-
ba: la personalidad Ubre infinita. Es preciso rechazar la tenta-
ción de considerar como una crítica de Hegel a Platón la
fórmula inicial; ...lapropia República de Platón, imagen prover-
bial de un ideal sin contenido, no llega a captar, en esencia, nada más
que la naturaleza de ¡a moralidad griega. Si leemos con aten-
ción las líneas siguientes a ésta, podremos comprender
que Hegel, de hecho, considera al platonismo como la
aprehensión filosófica de la esencia de la moralidad grie-
ga; es decir, de un presente, la realidad histórica de su
tiempo.
La relativa precariedad de esta aprehensión reside en
haberse enclaustrado en la época, sin comprender que el
principio de corrupción manifestado en su presente alberga
una revolución en su interior, y coincide con aquella par-
te de la realidad griega capaz de superar los límites histó-
ricos y hacerla pasar del ámbito de lo finito al de lo infi-
nito. La Grecia platónica experimenta la actividad que
sobre ella ejerce el Espíritu, aun cuando tome las aspira-
ciones profundas a trascender la temporalidad por una
corrupción o un desorden. No es posible comprender el
sentido del debate filosófico entre el platonismo históri-
co y su revisión hegeliana sin identificar el problema aquí
abordado, si bien de forma escueta. No es difícil encon-
trar un nombre que podamos situar bajo el título de princi-
pio de corrupción. Nos referimos a Sócrates, acusado de co-

172
(

rromper a la juventud cuando, en realidad, su única preo-


cupación era la de despertar todos los espíritus a la libre
búsqueda de la verdad, dejando de lado los ídolos del
tiempo (las costumbres, las conductas concretas). Esta es
su manera de poner de manifiesto el principio de la perso-
nalidad libre infinita en un marco histórico demasiado estre-
cho para sus miras. Cuestión más delicada es la de esta-
blecer dónde reside la culpabilidad de Platón por atentar
intimamente contra tal principio al buscar un remedio con-
tra la insatisfacción de la aspiración infinita mencionada
en una forma exterior particular de la moralidad griega.
Como sea que Hegel hace referencia explícita a su pro-
pia obra, que él prologa, conviene señalar que la cuestión
anterior aparece retomada, y desarrollada, en el párra-
fo 185, tercera parte: «La moralidad objetiva». Al suprimir
al particular, junto con sus bienes y propiedades, en la Re-
pública ideal, Platón habría ignorado el contenido espiri-
tual infinito de la personalidad, reservando al mundo ro-
mano y, sobre todo al cristianismo germánico, el desarro-
llo de este principio en el terreno del derecho. Por forma
exterior particular, por tanto, debemos entender la tenden-
cia griega a considerar lo público como substancial, y a lo
particular privado como inesencial.
4. Estamos tratando de poner en claro el entramado
orgánico establecido entre la fórmula ya mencionada más
arriba y el conjunto del texto, y no de comentarlo en sí
mismo. Una vez recordados al lector nuestros propósitos,
estamos ya en disposición de proseguir.
La filosofía ecléctica y superficial a la que Hegel se di-
rige de entrada, aprende, al entrar en una relación más estre-
cha con la realidad, a enfrentarse con la misma racionalidad
que ella escarnece igualando todas las opiniones. La reali-
dad histórica hace aquí las veces de una escuela de racio-
nalidad (seriedad y gravedad de los problemas concer-
nientes al derecho y al deber), de modo que hace necesa-
rio admitir, contra la filosofía académica, que todo lo
racional es real y no ficticio.
Platón considera la aspiración al libre examen como
una realidad importante, pero permanece cautivo del es-

173
pírjtu griego que hace de la Ciudad un orden substancial
superior al particular, precisamente desde el momento en
que procura alcanzar el cumplimiento de la aspiración a
la justicia, tal y como lo atestigua el propio Sócrates, si
bien ignorando que la realidad a la cual se dirigía es la li- ;
bertad del particular. Tal es su «idealismo», consistente
en ofrecer una respuesta vacía a una pregunta plena, y -i
creer que la idea de libertad basta para satisfacer la aspira- |
ción de realizarla en cada uno. Todo lo real (la aspiración s
a la libertad) es racional, y no subjetivo e irracional. jj

PARA SABER MÁS j

Sobre Hegel (textos 19, 20 y 21) ]

D'HONDT, J., Hegel,filósofode ¡a historia viviente, trad. Aníbal Leal, I


Buenos Aires, Amorrortu, 1971. j
GADAMER, H. G., La diaiittka de Hegel, trad. Manuel Garrido, 1
Madrid, Cátedra, Col. Teorema, 1979.
HARTMANN, N-, La filosofía del idealismo alemán, vol. II: «Hegel», |
trad. Emilio Estiú, Buenos Aires, Losada, 1960. |
HYPPOLITE, Hegel y el pensamiento moderno, trad. Ramón Salvat, I
México, Siglo X X I , 1973. ]
KAUFMANN, W., Hegel, trad. V. Sánchez de Zabala, Madrid, |
Alianza, 1972. |
MAGEE, B., Losgrandesfilósofos,trad. Amaia Barcena, Madrid, Cá- ¡
tedra, 1990, págs. 203-227.
MURE, C. R. G., La filosofía de Hegel, trad. A. Brotón Muñoz, Ma- 3
drid, Cátedra, Col. Teorema Menor, 1984. -•!

174
22. Comte

Q U E ES NECESARIO ENCADENAR LAS CIENCIAS


PARA D I R I G I R E L O R D E N SOCIAL

Querer, como se ha pretendido siempre hasta ahora,


formar una enciclopedia a la vez teológica, metafísica y
positiva es querer componer un conjunto con elemen-
tos que se excluyen mutuamente. No es de extrañar que
empresas tan mal concebidas hayan terminado por desa-
creditar tal proyecto entre los buenos espíritus. No ocu-
rrirá lo mismo, sin embargo, cuando, habiendo devenido
positiva la ciencia social y expulsado de su último asilo a
la teología junto con la metafísica, el sistema de nuestras
ideas ya no esté compuesto sino por elementos homogé-
neos. Será suficiente, entonces, resumir los conocimien-
tos relativos a los diversos órdenes de fenómenos para
descubrir de inmediato su encadenamiento natural y for-
mar así una verdadera filosofía positiva tan completa y
bien trabada como jamás pudo estarlo la filosofía metafí-
sica ni, incluso, la filosofía teológica, pues éstas, por na-
turaleza provisionales, en ninguna época fueron riguro-
samente universales.
Esta vasta empresa, que el presente siglo verá sin duda
cumplida, debe ser contemplada como el último acto y el
objetivo final de la gran revolución iniciada por Bacon,
Descartes y Galileo. Ello es indispensable como única
base espiritual posible del nuevo estado social hacia el
cual la especie humana muestra, en nuestros días, una
tendencia tan acusada, y ello porque solamente podrá lle-
gar una doctrina cualquiera a dirigir la sociedad por la
fuerza que le otorga su unidad. En tanto que las concep-
ciones positivas permanezcan aisladas entre sí, en tanto
que no se presenten al espíritu como las diversas partes de
un sistema único y completo, podrán conservar una gran
relevancia para los casos particulares, podrán incluso lu-

175
char con gran ventaja contra la autoridad política de ta
teología y la metafísica, pero no sabrán reemplazarlas en
la dirección del orden social. El perfeccionamiento de
nuestros conocimientos exige, como requisito indispen-
sable, sin duda, que se establezca en el seno de la ciencia
una división permanente del trabajo, más aún, que la es-
pecialización de las investigaciones de cada uno sea im-
pulsada lo más lejos posible. Sin embargo, es igualmente
incontestable que la masa de la sociedad, que necesita
continuamente y a un mismo tiempo los diversos resulta-
dos ya referidos y no puede, ni debe en ningún caso, in-
quietarse por este mecanismo interior, precisa, para
adoptar exclusivamente las doctrinas científicas como
guía habitual, no ver en ellas nada más que las distintas
ramas de un mismo tronco. Esta condición no es menos
indispensable, en lo relativo al propio cuerpo científico,
para la unidad y homogeneidad de su actuación política,
que será siempre demasiado débil cuando no esté concen-
trada. Así, en tanto que este estado de cosas subsista, la
teología y la metafísica, a pesar de su evidente decrepitud,
continuarán albergando, en virtud de su sola generalidad,
pretensiones legítimas a la soberanía moral.

«Consideraciones filosóficas sobre la$ ciencias


y los sabios» (1825).

COMENTARIO

1, Las Consideraciones sobre ¿as cienciasy ios sabios compo-


nen una obra que fue el fruto de circunstancias diversas.
Redactada por Auguste Comte (a la edad de 28 años)
cuando aún trabajaba de secretario de Saint-Simon, la re-
tomó más tarde para añadirle otros cinco opúsculos. El
conjunto de la obra fue publicada como complemento del
Sistema de política positiva (1851-1854). Ello indica suficien-
temente que un gran número de los temas principales de
su filosofía ya se encontraban expuestos ahí con todo su
valor sistemático, justo un año antes de comenzar a pro-

176
fesarlos en lo que llegaría a convertirse, según sus propias
palabras, en su «obra fundamental», el Curso de filosofía po-
sitiva (1830-1842). Por otra parte, la vasta empresa de cuyo
término nos informa en la primera línea del segundo pá-
rrafo, no puede ser otra que la ya comenzada en sus
opúsculos, en particular el Plan de los trabajos científicos nece-
sarios para reorganizar la sociedad (1822), título que es todo
un programa del cual todo nuestro texto hace eco.
Que Auguste Comte asuma su obra presente y la aún
por venir como una empresa, es un aspecto muy significa-
tivo para una época que fue la de los «empresarios». Entre
éstos podemos citar a Saint-Simon, uno de sus portavoces
en la era del despegue industrial y financiero (de los ban-
cos, de los sistemas de crédito), y del desarrollo de las téc-
nicas, de las ciencias y de sus aplicaciones; en suma, en la
era del carácter conquistador adoptado por cierta racio-
nalidad productora de riquezas y «progreso» (cuyo proce-
so verbal ya lo habían comenzado a formular los enciclo-
pedistas). Sin embargo, y al igual que Rousseau en su épo-
ca, Comte afronta la mayor contradicción histórica: el
progreso de la inteligencia científica y la actividad indus-
trial se corresponde con el desorden, la anarquía de la so-
ciedad y el Estado (en el plano político, religioso, moral,
espiritual). Su empresa adquiere sentido a raíz de la cons-
tatación anterior y del análisis que de ella hace Comte.
2. De atenernos al comienzo del presente extracto,
podríamos equivocarnos con respecto a las intenciones
filosóficas de Comte.
El primer párrafo, ¿no se presenta globalmente como un
discurso epistemológico, cuya finalidad sería la de volver a
fundamentar la racionalidad de la filosofía (en sus aspira-
ciones enciclopédicas) a partir de una refundación del
conjunto del saber científico, cuyo sistema mismo sería
alcanzado (y retroactivamente reorganizado) mediante el
tránsito hacia el estado positivo de la ciencia social? Tendría-
mos, así, resumida en tres puntos, la crítica constructiva
dirigida a la ambición filosófica mantenida a lo largo de
su historia:
a) Puesto que lo teológico y lo metafísico son incom-
177
patibles con la positividad, cualquier enciclopedia que les
haga coexistir estará condenada al fracaso (Querer... —•-
...buenos espíritus).
b) Ahora bien, la positividad de la ciencia social pue-
de reducir a la nada tanto lo teológico como lo metafísico
(No ocurrirá lo mismo... —* ...homgéneos.), y unificar el sistema
de nuestras ideas.
c) De este modo, es posible preveer sobre bases re-
formadas y duraderas (el encadenamiento natural\ llevado a
cabo con la física social, de las diferentes ciencias), la
constitución de una nueva filosofía que, en sí misma, será
positiva (Será, suficiente, entonces, ... —*• ...universales).
Nada más epistemológico, por lo demás, que los pro-
blemas abordados por Comte desde los conceptos de teoló-
gico, metafisico y positivo. Esta triada designa, a la vez, la
permanencia de la exigencia explicativa en el discurso filo-
sófico y, por tanto, la introducción de un principio de
causalidad eficiente, así como, por otra parte, sus trans-
formaciones, gracias a la constitución sucesiva en cien-
cias positivas de los distintos dominios del saber, en el
seno de una historia que supone el progreso de la razón.
Este progreso es análogo al que se produce con el paso de
la infancia (la edad teológica) a la fase crítica de la adoles-
cencia (la edad metafísica) y, a continuación, a la edad
adulta (o positiva). En lo que atañe, para terminar, al co-
nocimiento siempre relativo de los fenómenos naturales
que sus leyes procuran (tal es la definición del saber positi-
vo), a través de un proceso que va de lo simple a lo com-
plejo y de lo general a lo más especial, pero siempre en la
conexión cada vez más fuerte y completa de todos los he-
chos remitidos a una naturaleza común, la ciencia se de-
sembaraza de aquellos agentessobrenaturalesyfuerzas abstractas
que el pensamiento teológico y el metafísico, respecti-
vamente, creen deber convocar para dar razón de los he-
chos. La ciencia social misma —es decir, la física social, o,
mejor aún, la sociología, de la cual se afirma que Comte acu-
ñó su nombre al tiempo que le asignó su objeto— com-
pleta el sistema del saber positivo por efecto de esa ley del
desarrollo científico que quiere también que el conoci-

178
miento se extienda desde lo más alejado (la astronomía) a
lo más cercano al hombre. La empresa de Comte sería,
pues, histórica al tiempo que teórica, inscribiéndose necesa-
riamente en un proceso sometido a la ley de los tres estados
como conclusión (lógica) y conclusión (temporal) del
mismo. Nos encontramos así ante el último actoy objetivo fi-
nal de una revolución iniciada con Bacon, Descartes y
Galileo (o quienes debemos, respectivamente, según pre-
cisa Comte en otro lugar, los preceptos, las concepciones
—como la de las distintas ramas de un mismo tronco— y los
descubrimientos que han marcado el advenimiento del
espíritu positivo).
El comimw del segundo párrafo (Esta vasta empresa... *• ...
Galileo) parece, en consecuencia, prolongar o retomar el
punto de vista epistemológico de la segunda parte, des-
plazando a un lado de la historia la relación de la verdad
con respecto a la organización de las ciencias.
3. Sin embargo, ta empresa comtiana desvela ahora la
verdadera dimensión de su proyecto en la última parte
del texto: como requisito indispensable... —* ...a ¡a soberanía mo-
ral). Se trata de promover, mediante el cumplimiento de
la revolución científica, el nuevo estado social que las leyes de la
historia asignan, en la naturaleza, a la especie humana; es de-
cir, a la humanidad entendida como realidad biológica (y
ello permitirá articular la sociología en función de la bio-
logía, según una relación de dependencia que, para Com-
te, posee un estatus igualmente epistemológico). La his-
toria permite, aquí, el paso de los estadios del conoci-
miento a los del orden político, y ello desde una única y
común racionalidad. En efecto, las mismas leyes, al go-
bernar idénticas transiciones, permiten obtener la orga-
nización de clases en el estado positivo del mañana a
partirdeunaclasificacíóndelascienciasconelespíritu pos i-
tivo de hoy en día. Y la filosofía de Comte es la presenta-
ción, cada vez más argumentada, de la legitimidad de este
encadenamiento histórico en función de la necesidad del
encadenamiento natural de las ciencias —puesto que, desde
la fuerza de las doctrinas científicas a la autoridad política, nos
encontramos de lleno dentro del ámbito del poder espirí-

179
tual. No es otra cosa lo que pone en juego la filosofía
comtiana; que, en efecto, se dispone a ofrecer la dirección
de la sociedad (del orden social) a quienes poseen el saberpositi-
vo, haciendo que sea ilegítima la soberanía moral de los adver-
sarios históricamente envejecidos que son la teología y la
metafísica, junto con su decrepitud. En efecto, y ya desde
el primer párrafo, Comte hablaba de expulsarlos de su asilo, í)
anunciando el contexto de lucha que aparece expresado 1
con mayor precisión en este punto del texto. 1
El positivismo de Auguste Comte es, de hecho, una filo- 1
sofía militante ubicada en un combate histórico. La inteli- w
gencia (científica) está, en realidad, al servicio de la ac-
ción (política). Se trata de preveer para actuar (una de las
fórmulas clave del positivismo). Al «apoderarse de las
ideas sociales» (expresión de Comte), la física no puede
sino conducir a la sociedad misma a la unidad de las leyes
naturales. Es tarea de la sociología (i.e. física social) el hacer
evidente las tendencias, en las sucesivas épocas y relaciones •
mutuas, para así «descubrir e instaurar las formas prácti-
cas correspondientes» y hacer compatibles, finalmente, el |
orden y el progreso. Sin embargo, no podemos olvidar la |
ambivalencia de sentido del vocablo orden, pues, como el i
mismo Comte se molestó en señalar, puede ser entendido 1
en términos de ordenación (como la serie de conocimientos
de cada ciencia particular, y las ciencias en su clasifica-
ción), pero también en términos de mandato (que asigna a
cada individuo su lugar, esto es, su función dentro del or-
ganismo social). Así, en este segundo párrafo se dibuja, a
grandes trazos, toda una «estrategia» (algo bien distinto
de una crítica epistemológica «desinteresada») que des-
cansa sobre una apreciación de fuerzas enfrentadas. Es- j
trategia que se erige bajo el signo de lo indispensable (el lee- *
tor podrá percibir la insistencia del término, que para
Comte significa «lo que conviene al devenir del espíritu
humano», o, dicho de otro modo, la necesidad misma en
su aspecto moral, la cual se pretende hacer inevitable, sujeta i
o sometida a las leyes naturales, al aspecto físico de la ne- ¡
cesidad —de ahí, precisamente, el papel reservado
a la ciencia social). Con el objeto de ofrecer (y asegurar)

180
la dirección social al espíritu positivo, covendrá, pues:
a) Proporcionarle la fuerza de unidad que le permita
convertirse en base espiritual. (Podemos deducir del propio
texto qué debe ser lo «espiritual» si ha de servir de base al
cuerpo social extendido a la humanidad: debe comportar
generalidad, o, mejor dicho, universalidad, producida
por la extensión del espíritu positivo a todas la ramas del
saber).
b) Unificar u homogeneizar, pues, a tal efecto (En
tanto que... —*• ...orden social), las diversas concepciones positivas
en un sistema único y completo. Esta es la condición que ga-
rantiza la victoria final en la lucha (y así adquiere ahora
sentido pleno el primer párrafo).
c) En concreto (Elperfeccionamiento... ' ...concentrada),
había que sintetizar los resultados de la labor científica, a
pesar de (o a causa de) la división del trabajo sociológica-
mente necesario (ya lo es desde el punto de vista biológi-
co) para el perfeccionamiento. Lo anterior es válido tanto
para el uso de la masa (que necesita ser guiada) como para
el cuerpo científico (esa clase funcional mente especializada y
con vocación de dirigir), conciliando así una necesidad
objetiva con otra subjetiva.
4. Desde esta estrategia podemos ver con claridad
cuál es la preocupación constante y más significativa de
Auguste Comte. Para universalizar es necesario religar
progresivamente más y mejor, ya sea en la «relatividad»
de las leyes como en la religión (vocablo que, por otra
parte, recibe, etimológicamente hablando, su sentido y
función de la palabra latina religere). El punto final del
positivismo comtiano será, en este sentido, una religión
de la humanidad que exigirá un Catecismo positivista (1852).
Con respecto a la ciencia y la política, la tarea encomen-
dada a la sociología es, justamente, la de religar la sín-tesis
que debe efectuar el sabio en el orden del intelecto, y, por
otra parte, la sin-ergía, en el orden de la actividad, y la
sim-patía, en el orden de la afectividad, que deben fo-
mentar tanto el político como el educador. Sin embargo,
y desde el punto de vista filosófico, la ciencia social de Comte
se presenta como una interpretación de la historia que es-

181
tamos tentados de comparar con la de su contemporáneo
Hegel (aunque sólo sea porque ambos contemplan lo uni-
versal en términos de un proceso del Espíritu, y porque
introducen las «triadas» con el fin de jalonar el progreso
de la razón). Nos limitaremos aquí a señalar que, al fun-
dar la sociología, el filósofo positivista se ha acercado
mucho a Platón, quien, veintitrés siglos antes que él ya fl
quiso que los filósofos fueran reyes. Sin embargo, la pro-
ximidad desaparece cuando lo comparamos con el soció-
logo Max Weber, quien un siglo después1, se ocuparía de
distinguir con sumo cuidado al sabio del político, anali-
zando las contradicciones que aparecen entre la ética de
la convicción y la ética de la responsabilidad

PARA SABER MÁS

Sobre Comte

ARNAUD, P., Sociología de Comte, trad. F. Fernández Buey, Pról.


Salvador Gíner, Barcelona, Península, 1986.
ARON, R., Las etapas del pensamiento sociológico, vol, 1: «Montes-
quieu, Cotnte, Tocqueville», trad. A. Leal, Buenos Aires, Si-
glo X X , 1980, 2 vols.
MARVIN, F. S., Comte, trad. Salvador Echevarría, México,
FCE, 1941.
MILL, J. S-, Comte y el positivismo, trad. Dalmacio Negro Pavón,
Buenos Aires, Aguilar, 1977.

1 Ver Max Weber, Wisstnscbaft ais Btruf( 1919) y Politik aU BtnfiV) 19)
(EJ político y el científico, (rad. F. R. Rubio Llórente, Madrid, Alian-
za, 1987).

182
23. Schopenhauer

Q U E LA METAFÍSICA ES LA COMPRENSIÓN D E L MUNDO

El mundo y nuestra propia existencia se nos ofrecen


necesariamente como un enigma. Para Kant (advirtamos
que él admite todo esto sin demostración), no es posible
obtener la solución del enigma comprendiendo a fondo
el mundo; al contrario, se debe buscar esta solución en al-
guna cosa totalmente extraña al mismo (tal es, en efecto,
el sentido de la expresión: «más allá de toda experiencia
posible»); en la búsqueda de la solución hay que excluir
todos los datos de los que podamos tener un conocimien-
to inmediato (pues, quien dice «conocimiento inmedia-
to» dice experiencia posible interna o externa); la solu-
ción debe ser buscada de acuerdo con los datos adquiri-
dos indirectamente; esto es, deducidos de principios
generales a priori.
Ello viene a excluir la existencia de una fuente princi-
pal de todos los conocimientos y a condenar el que sólo
exista una vía que conduce a la verdad. De este modo, no
es de extrañar que las tentativas dogmáticas no alcanza-
ran su objetivo; como tampoco extraña que Kant haya sa-
bido demostrar la necesidad de su fracaso. En efecto,
habíamos previamente declarado que «metafísica»y «cono-
cimiento a priori» eran idénticos. Sin embargo, para afir-
mar lo anterior hubiera habido necesidad de comenzar
por demostrar que los elementos necesarios para resolver
el problema del mundo no deberían en ningún caso for-
mar parte del mundo mismo, sino que, al contrario, es
fuera del mundo donde tenía que proceder su búsqueda,
allí donde es imposible llegar sin el auxilio de las formas a
priori de nuestro entendimiento. En tanto que este último
punto permanezca sin demostrar, no tendremos razón al-
guna para rechazar, en lo concerniente al más importante
y grave de todos los problemas, la fuente más fecunda y

183
rica de nuestro conocimiento, quiero decir, la experien-
cia interna y externa; ni para dejar de operar en nuestras
especulaciones con la sola ayuda de formas desprovistas
de contenido. Por eso yo afirmo que la comprensión del
enigma del mundo debe de brotar de la comprensión de
este mundo mismo.

El mundo tomo voluntad y representado». Apéndice:


«Crítica a la filosofía de Kant» (1814-1844).

COMENTARIO

1. Schopenhauer quiere, en este texto, volver a deli-


mitar tanto el sentido como el campo y los medios de la
metafísica, para lo cual redefme su objeto tras las críticas
contra ella elaboradas por Kant. Con tal propósito, nues-
tro autor fija las condiciones de posibilidad de este renacimien-
to, partiendo del principio que establece que el mundo es
un enigma para el hombre.
El texto concluye, finalmente regresando a su punto de
partida: si el mundoy nuestra propia existencia se nos ofrecen nece-
sariamente como un enigma, su solución debería contar con el
requisito previo consistente en que se adquiera la compren-
sión del mundo mismo, y no el conocimiento abstracto de las
formas vacías de una experiencia de este tipo.
Con la finalidad de establecer que existe una vía ta)
será necesario volver nuestra mirada a las conclusiones
de los Prolegómenos a toda metafísicafutura que podrá presentarse
como ciencia, cuyo parágrafo 1, citado por Schopenhauer en
las líneas precedentes a nuestro texto, condena todo re-
curso a la experiencia para la edificación de una meta-
física.
La exposición de los argumentos de Kant y de sus pre-
supuestos ocupa el principio del texto ( - * ...condenar el que
sólo exista una vía que conduce a la verdad). Desde De este modo...
* ...desprovistas de contenido, Schopenhauer desmonta la re-
futación kantiana y le opone un postulado indemostrado
que sostiene el conjunto: metafísica significa a priori. Al
final, la conclusión viene seguida por la frase que afirma

184
la conveniencia de partir del principio contrario para dar
una respuesta al problema metafísico.
2. La primera fórmula condiciona el resto de la dis-
cusión: ¿en qué sentido hay que entender el enigma del
mundo y nuestra existencia, en qué sentido su necesidad
en cuanto enigma?
Que el mundo sea un enigma es algo que, ciertamente,
ningún ser vivo excepto el hombre puede comprender.
De un modo más general, todo ser definido como «natu-
ral» experimenta la existencia del mundo, y de sí mismo,
sin asombro ni perplejidad, desde un debate que lo opone
al exterior para sobrevivir. Ser un enigma supone ya ser
el objeto de una reflexión escindida del querer vivir in-
mediato. Por qué hay algo y no más bien nada, por qué
esta vida y no la nada; estas son las fórmulas más genera-
les del enigma del mundo y de nosotros mismos.
La confrontación con la versión kantiana del enigma
hace aparecer un equívoco notable: Kant hace de él un
problema de conocimiento, mientras que para Schopen-
hauer es, ante todo, «existencial». En efecto, Kant exclu-
ye de oficio cualquier tipo de conocimiento inmediato, y
no para privilegiar una forma de reflexión sobre la expe-
riencia, sino que se trata de una deducción de principios ge-
nerales a priori. La metafísica es, por consiguiente, un géne-
ro de conocimiento riguroso en lugar de una cuestión
acerca de la situación del sujeto de un conocimiento tal y
del mundo del cual es objeto, sin que ese estatus de objeto
agote el problema. Desechar el mundo como fuente de
experiencia es, entonces, más la negación del problema que
su solución.
3. Debemos, por tanto, disociar la empresa metafísi-
ca y los esfuerzos kantianos para garantizar un conoci-
miento riguroso. Con este propósito, tendremos que sa-
car a la luz los postulados implícitos su demostración.
Schopenhauer asocia, en su refutación, a los filósofos
dogmáticos con el criticismo que los denuncia, y ello
porque ambos vuelven la espalda a la única via que conduce a
la verdad. Si la experiencia es esta vía, el error más fre-
cuente es el de considerar a la metafísica fuera del campo

185
que le es propio. Resolver el enigma del mundo pasa a conver-
tirse en el asunto que aune a un sujeto implicado en una
experiencia vivida, y no a un sujeto trascendental que re-
flexione sobre las formas a priori de una experiencia tal
cuyos contenidos serian, pues, directamente considera-
dos como despreciables. Se entiende que los contenidos
mencionados constituyan la experiencia misma, ya que
ésta presenta el doble aspecto, interno y externo, de la
aprehensión de la situación del ser en un mundo privado
de su valor y significado y, por ello, erigido invencible-
mente en enigma para quienes la experimentan. Especu-
lar carece ya, para Schopenhauer, del significado de un
pseudo conocimiento amenazado por el error, porque el
error es, más bien, el punto de partida de la reflexión. En
efecto, gracias a la propia experiencia, el sujeto de la mis-
ma está avisado de que su sentido del mismo y el de la
vida le es inaccesible. Es así como hay que entender la
afirmación, ella misma indemostrada, que Schopenhauer
opone al postulado kantiano; la experiencia es lafuente más
fecunday rica de nuestro conocimiento, constituyendo además el
lugar mismo de la investigación de un conocimiento para
quien el sentido vital o «existencíal» es primordial,
4, La fórmula final provee a la metafísica su programa
y sentido, pues consiste, en verdad, en «adquirir la com-
prensión del mundo», no en aprehender las nuevas for-
mas vacías de la experiencia posible. La metafísica se
orienta hacia la solución del enigma del mundo, en el
sentido de que su misma existencia «constituye un enig-
ma» para el sujeto que reflexiona sobre su propia situa-
ción y el debate vital que mantiene consigo mismo, sien-
do esta problematícidad mayor y más honda que como
fuente de dificultades prácticas para querer vivir tal exis-
tencia. Schopenhauer subordina la metafísica a esta ubi-
cación del ideal del conocimiento en el seno del proble-
ma de la existencia.

186
PARA SABER MÁS

KANT, Prolegómenos para toda metafísica futura que podrá presentarse


como ciencia, trad. Julián Besteiro, Madrid, Sarpe, 1984.
NIETZSCHE, F „ Cay saber, Consideraciones intempestivas, cfr. tex-
tos 27 y 28.

Sobre Scbopenhauer

GARDINER, P., Scbopenhauer, trad. Angela Saiz Sáez, México,


FCE, 1975.
MAGEE, B., Scbopenhauer, trad. Amaia Barcena, Madrid, Cáte-
dra, 1991.
— Los grandes filósofos, trad. Amaia Barcena, Madrid, Cáte-
dra, 1990, págs. 227-251.

24. Marx
ATOMO Y LIBERTAD

En verdad, la singularidad inmediata no será realizada


según su concepto en tanto que se remita a otra cosa dis-
tinta de si misma, aun cuando esta cosa se presente a ella
en la forma de una existencia inmediata. Es así que el
hombre no dejará de ser un producto de la naturaleza has-
ta el momento en que la otra cosa a la que él se remite no
sea una existencia diferente, sino un hombre singular,
aunque no se trate todavía del espíritu. Mas, para que el
hombre en tanto que hombre haga de sí su único objeto
real, tiene cjue haber roto dentro de sí mismo con su exis-
tencia relativa, la fuerza del deseo y de la pura naturaleza.
La repulsión es la primera forma de la consciencia-de-sí, y corres-
ponde a la conscíencia de sí que se hace patente como
existente-inmediato, como singularidad abstracta.

187
En la repulsión se realiza, por consiguiente, el concep-
to de átomo, según el cual éste es la forma abstracta aun
siendo enteramente lo contrario, materia abstracta. Y lo
anterior es así porque aquello a lo que se refiere son, en
efecto, los átomos, pero átomos diferentes. Ahora bien, sí
me dirijo a mí mismo como si fuera algún otro-
inmedíato, mi comportamiento reviste un carácter mate-
rial. Tal es la exterioricidad más alta que pudiéramos
concebir. En la repulsión de los átomos se cumple, pues,
la síntesis de su materialidad, puesta en la caída vertical, y
de su determinación formal, puesta en el declinamiento.
En contraposición a Epicuro, Demócrito transforma en
movimiento forzado, en el acto de una necesidad ciega, lo
que, para el primero, es la realización del concepto de
átomo. Ya hemos visto que éste explica la substancia de
la necesidad por la turbulencia que engendran la repul-
sión y el entrechocamiento de los átomos. Sólo concibe,
por tanto, el aspecto material de la repulsión, la fragmen-
tación, la alteración, y no el aspecto ideal según el cual
toda relación de objeto es negada, siendo considerado el
movimiento como determinación de sí mismo. Ello re-
sulta claramente de que él se representa de forma concre-
ta un único y mismo cuerpo dividido en numerosos cuer-
pos por el espacio vacío, como el oro tras ser roto en pe-
dazos. Así, pues, no llega a comprender el Uno como al
concepto de átomo,

«Sobre la diferencia de filosofía natural en De-


mócrito y Epicuro» (1841).

COMENTARIO

1. El presente texto filosófico no puede, evidente-


• mente, ser descifrado más que a raíz de una lectura aten-
ta, pero las implicaciones y el sentido real de la discusión
inicial, en apariencia abstracta, aparecen con toda clari-
dad en las últimas líneas. En efecto, el concepto de átomo
es el Uno, concebido filosóficamente como determina-

188
ción de sí mismo. Ver tan sólo el lado material de las co-
sas y la ciega necesidad, supone, en consecuencia, perder
de vista el problema de la libertad humana individual. En
las páginas siguientes a la citada, Marx explícita esta cues-
tión al escribir que Epicuro emplea formas más concretas
de la repulsión: «El Contrato en materia política y, en
materia social, la amistad que él predica como bien su-
premo»1. Podemos, pues, establecer cierto parentesco, en
cuanto al problema tratado, con el texto de Hegel que ya
analizamos; a saber, la relación entre el orden político y
la personalidad libre. La estructuración tipográfica del
texto en tres párrafos se justifica por la articulación de
otras tantas cuestiones sucesivas, de desigual generalidad.
En primer lugar, se trata del tema de la naturaleza de la
repulsión, entendida como forma principe* de la con-
ciencia de sí y constitución de la singularidad abstracta.
En el segundo lugar, Marx aborda tan sólo el problema
del átomo. ¿Cuál es la síntesis entre su materia y su forma,
cuál es, finalmente, su concepto?
Por último, llegamos al tratamiento del núcleo de la
discusión. Epicuro da a la cuestión del dinametí, o declina-
miento, toda su relevancia filosófica y política en oposi-
ción a Demócrito, quien no alcanza a ver nada más que la
ciega necesidad. De la mano del concepto del átomo en-
tramos, con pleno derecho, en el problema de la determi-
nación de sí mismo.
2. Dos son las consideraciones que se suceden y se
responden; la realización, ella misma inmediata, de la
singularidad inmediata, y la de esta singularidad según su
concepto. A la primera forma abstracta del problema (En
verdad... —* ...existencia inmediata) le sigue su ejemplifica-
cíón, pues singularidad significa hombre y, según su concep-
to, en tanto que hombre\ En consecuencia, es legítimo
1 Cfr. «Sobre la diferencia de filosofía natural en Demócrito y Epi-

curo».
* Recordemos que la edición príncipe es la primera edición de una
obra. El autor parece haber extraído el término del lenguaje editorial.
[N. del T. ]
2 Ver texto 21.

189
concluir mediante la liberación del concepto de la re-
pulsión.
Sin embargo, esta es la noción más delicada, ya que po-
see un doble sentido: material (apartarse, ya desviarse de
o «declinar»), y humano (desprenderse, regresar a sí mis-
mo y a lo que a él concierne). En este texto se evoca, casi
palabra por palabra, el capítulo «Consciencia de sí» de la
Fenomenología del espíritu de HegeP, donde el autor identi-
fica la repulsión con la consciencia de sí en el estado
naciente.
Se trata del problema de la independencia, al igual que
en el caso de Hegel. El principio del texto no distingue
entre una singularidad (una «cosa cualquiera») depen-
diente y otra independiente, sino entre una cosa que se re-
mite a otra y una que se remite a otra cosa distinta de si misma.
El autor precisa aún más, pues la nota común a las dos
singularidades señaladas no es la inmediatez, es la común
pertenencia a un único concepto, y ello porque lo que se
busca es hallar cómo se realiza una singularidad tal según
su concepto y no de forma inmediata. El ejemplo aporta
una precisión importante acerca de qué hemos de enten-
der por remitirse. Un hombre singular se remite a otro,
también singular, pero no a una existencia diferente como,
por ejemplo, un animal o un vegetal. De ahí se infiere que
su existencia no es relativa (existir por otra cosa), sino
conforme ya a su concepto de hombre (existir por él mis-
mo). En cualquier caso, la realización de su humanidad
como tal sería imposible de llegar a remitirse a los demás
como si éstos fueran cualesquiera otros seres naturales, al
igual que ocurre cuando esta relación viene motivada por
el deseo. En efecto, en el deseo el uno se pone a sí mismo
por relación al otro, y pone igualmente al otro por rela-
ción. Marx define la repulsión en términos de ruptura
que ha de darse dentro de uno mismo con el objeto de de-
sarticular la relación anterior. No se trata, pues, de una
aversión, sino de la posición separada del uno con respec-

1 Hegel, Fenomenología de! espíritu, t. I, «La consciencia de sí», trad. W.

Roces, México, F.C.E., 1984.

190
to al otro en tanto que singularidades, a pesar de la na-
turaleza común (la humanidad) que ambos puedan
poseer.
En la repulsión el hombre no pone a la humanidad,
pone su propia singularidad en lugar de una relación
como la ya mencionada: la repulsión es, entonces, el
preámbulo para la posición del hombre en cuanto
hombre,
3. El ejemplo del hombre es substituido por el del
átomo, y a la repulsión se le asocia el declinamiento. Nos
acercamos en este punto a una evaluación filosófica de la
aportación de Epicuro. Aquí, Marx examina el concepto
mismo de átomo, mientras que considera al hombre úni-
camente como singularidad concreta. La forma abstracta
del átomo es la separación; los átomos están separados
entre sí, apartados los unos de los otros. Sin embargo,
Marx añade que su materia abstracta está sujeta a la repul-
sión, «se atiene» al desvío del resto de los átomos, que se
«dirigen» a él como sifuera algún otro-inmediato. Debido al áto-
mo, la materia aparece diferenciada tanto formal (hay
numerosos átomos idénticos) como materialmente (un
átomo «se pone» a sí mismo como otro). A este respecto,
la repulsión, en términos de acto material, se encarna en
el declinamiento. Asimismo, al ser materiales no pode-
mos decir que los átomos caen, sino que divergen de la
vertical porque son formas singulares. El declinamiento
presenta un carácter material sujeto a la repulsión, cons-
citncta de si del átomo. Ella realiza la síntesis de la caída
y el declinamiento de la materia pesante y la forma se-
parada.
4. A lo largo de las páginas que preceden a nuestro
texto, Marx no ha hecho más que precisar, sin llegar a de-
cirlo expresamente, la función de la repulsión en la con-
cepción epicúrea del átomo. El paralelismo con Demó-
crito subraya una diferencia filosófica esencial que reper-
cutirá en el problema antropológico y político.
Plutarco es el autor de un comentario acerca de Demó-
crito que Marx, al comienzo de esta obra, presenta así:
«Demócrito haría de la necesidad el destino y el derecho,

191
la providencia y el demiurgo. Pero la substancia de esta
necesidad sería la antitipia, y el movimiento y el choque
de la materia»*.
La repulsión vendría a ser, por tanto, la substancia de la
necesidad, el contenido de su idea. Todo lo que acontece,
acontece necesariamente porque, en lugar de caer eterna-
mente en el vacío según la línea vertical, los átomos en-
tran en contacto unos con otros y chocan entre sí ciega-
mente, determinando así la formación de turbulencias
que darán origen a la totalidad de las cosas particulares.
Para Demócrito, la repulsión no es sino el nombre de esa
misteriosa desviación que aleja a los átomos de la vertical.
Se produce de este modo la «dispersión» de los cuerpos en
el espacio vacío, una pura multiplicidad de lo mismo, la
substancia material dividida en fragmentos que se entre-
chocan. Marx quiere poner de manifiesto que Epicuro
pretende interrogar al concepto de átomo en sí mismo
más que forjar un mito acerca del origen material de la
necesidad del cual el átomo entraría a formar parte a títu-
lo de agente indiferente e indeterminado.
Epicuro, en contraposición al anterior, plantea el movi-
miento (de los átomos) como determinación de si mismo, de don-
de el átomo ya no es un fragmento de materia, idéntico a
cualquier otro (como el oro tras ser roto en pedáis), sino Uno,
como su concepto (la indivisibilidad) lo indica: a la divi-
sión accidental se opone la división esencial de la materia
en múltiples «Unos»5.
Hemos de anticipar algo más para comprender la fun-
ción política de las precisiones concernientes al átomo.
Marx quiere poner de manifiesto un nexo común que en-
trelace en un solo conjunto las cuestiones tratadas por
Epicuro. Si el acercamiento entre los átomos fuera aban-
donado a la pura necesidad, los contratos y aun la amis-
tad, que suponen, precisamente, una elección libre y una
libre autodeterminación de la conducta de cada uno, se-
4 Cfr. «Sobre la diferencia de filosofía natura! en Demócrito y Epi-
curo».
5 Cfr.Srítmtdt¡alogtque, t ], 1.»sección,cap. ÍII sóbrelos Unos múlti-

ples, París, Aubier, 1947.

192
rían imposibles. La concepción epicúrea de la repulsión
es, así, paradójicamente, la clave de su concepción del
nexo social.

25. Marx

E L IDEALISMO Y A L E M A N I A

La crítica de la filosofía alemana del Estado y del derecho,


que ha recibido de Hegel su forma última, la más rigurosa
y rica, es a la vez lo uno y lo otro, tanto el análisis crítico
del Estado moderno y de la realidad a la que está unida,
como la negación categórica de todo lo que hasta ahora
ha sido la eonseiencia alemana política y jurídica, cuya expre-
sión más importante, más universal, aquella que está ele-
vada al rango de ciencia, es ta propia filosofía especulativa del
derecho. Tan sólo en Alemania era posible la filosofía espe-
culativa del derecho, de esa manera abstracta y transcen-
dente de pensar el Estado moderno, cuya realidad se sitúa
en un más allá, aun cuando este más allá no se encuentre
sino al otro lado del Rhin; a la inversa, la concepción ale-
mana del Estado moderno, que hace abstracción del hom-
bre real, sólo era posible porque y en tanto que el Estado
moderno mismo hace abstracción de) hombre real, o no
da al hombre total nada más que una satisfacción imagi-
naria. En política, los alemanes han concebido lo que otros
pueblos han hecho.

«Contribución a la crítica de la filosofía del de-


recho de Hegel» (1843).

COMENTARIO

1. Nos encontramos ante uno de los raros textos


marxianos donde la crítica de la filosofía de Hegel (su fi-
losofía del derecho) se hace explícita, a igual distancia de

193
la invectiva y del flirt K Hemos de añadir a lo anterior
una determinación más precisa, que Marx se propone
arreglar cuentas (más bien una cuenta, inagotable) con
Hegel situándose, de una vez, frente por frente de la reali-
dad alemana y de la filosofía especulativa (del Derecho) soli-
daria, según él, con la anterior. La polémica se desarrolla
en estos tres órdenes. También conviene observar que
este texto «de combate», o «en la disputa», sigue siendo
programático, pues fija una crítica venidera, requerida
por la coyuntura alemana, sus objetivos y vías, situándola
entre una crítica de la religión que Marx estima acabada,
cumplida (Feuerbach) y una crítica de la economía políti-
ca que llegará a ocupar el resto de sus días.
El texto ofrece una división natural en tres partes, de
las cuales las dos últimas comentan y precisan un aspecto
que, en la primera, tan sólo aparece evocado.
Desde el comienzo hasta filosofía especulativa del derecho.
Aquí se afirma que la crítica del hegelianismo jurídico en
tanto quefilosofía alemana del Estadoy del derecho posee, nece-
sariamente, un doble aspecto: crítica de la realidad, críti-
ca de la conciencia.
La abstracción especulativa es posible en un país que
no ha realizado todavía concretamente el Estado moder-
no ya realizado en otros lugares. (Tan sólo en Alemania... —*
...al otro lado del Rhín.)
De forma recíproca, este idealismo abstracto se apoya
en la realidad insuficiente de todo Estado moderno, pues,
en concreto, hace abstracción del hombre total (... a la inver-
sa.,.-*... una satisfacción imaginaria).
La conclusión (Enpolítica... —*• ...han hecho) reúne a los
dos elementos: realidad alemana y abstracción especu-
lativa.
2, Al igual que en las tesis de Feuerbach2, que proce-
den de la misma época, Marx define las relaciones entre

1 Cfr. Miseria de ia ftiosufía, trad. Dalmacio Negro Pavón, Madrid,

Aguíhr, 1971.
2 «Tesis sobre Feuerbach», en La idtoiogia alemana, trad. julio Vera,
Buenos Aires, Caite ti, 1969.

194
la filosofía y la acción en términos dialécticos, estable-
ciendo que no es posible «suprimir la filosofía sin reali-
zarla», y que, recíprocamente, tampoco es posible «reali-
zar la filosofía sín suprimirla»3. Queda por establecer aún
el nexo que ponga en contacto la especulación con la rea-
lidad histórica, para fundamentar así la doble negación
que acabamos de señalar.
Marx descubre este nexo ya establecido como conteni-
do mismo del hegelianismo, que precede a la crítica y la
exige. Lo anterior supone tanto como decir que la crítica
general de lafilosofíaalemana del Estadoy del derecho es, prác-
ticamente, ad hominem, oportuna «idealmente» para Hegel
y nadie más. Tal es, por otra parte, el sentido del home-
naje que Marx le dirige de paso al recalcar que Hegel da a
esta filosofía (no a su crítica, por cierto) su forma más rigu-
rosa y rica, filosofía que es la «expresión más importante,
más universal... de la consciencia alemana», y en suma, cien-
cia. De ahí que la crítica dirigida a la misma suponga,
verdaderamente, traspasar los límites de la época, liberar-
se de su estrechez y, finalmente, dar a la crítica un objeto
que, sin ella, no existiría.
3. Falta aún definir de qué carece la ciencia del Dere-
cho, fiel expresión de la realidad moderna y la conscien-
cia alemana. Podemos intentar seguir el doble trazado del
pensamiento de Marx respecto a este tema. En efecto, lo
que falta al hegelianismo coincide con lo que falta al Es-
tado moderno y a la consciencia alemana, y ello porque,
en tanto que expresión de estas dos entidades históricas, es
insuperable. Resta por definir en qué términos es posible
determinar y criticar tales «carencias» que corren el ries-
go de no poder ser descubiertas por la observación empí-
rica (precisamente porque «faltan»).
Lógicamente, habrá que continuar el análisis desde los
presupuestos mismos de la filosofía hegeliana. Si lo racio-
nal es real, tendremos que encontrarlo realizado en algu-
na parte, pero ¿dónde?. Este es el punto donde la crítica

1 Cfr. supra, Contribución a ta crítica de la filosofía dtl derecho di Hegel.

195
hiere. Alemania no es, necesariamente, la tierra natal del
«presente» como síntesis de lo racional y lo real. La idea-
lización del Estado moderno, sede de lo racional, posee
sus condiciones históricas de posibilidad en una falta, en
una carencia alemana, no en algo acabado, cumplido.
Ella sitúa al Estado en una esfera transcendente, en un
más allá de las relaciones reales de los hombres concre-
tos, porque el verdadero Estado moderno está, en efecto, más
allá, geográficamente situado... ultra-Rhin. Una critica
económico-política de la realidad alemana podrá com-
pletar el panorama hasta ahora sólo esbozado. Sin embar-
go, no podemos limitarnos a esta constatación: la filoso-
fía alemana del derecho del Estado de la «época» está ca-
duca, sin negar de golpe el valor expresivo que posee y
que Marx le ha reconocido en la primera parte.
4. Es preciso, pues, «invertir», examinar la recíproca.
La posibilidad de la concepción alemana no depende de
accidentes históricos ni de contingencias reparables
(cambiar de modelo, examinar preferentemente a Fran-
cia o Inglaterra, como, por otra parte, impondrá la crítica
económica, porque la realidad y la ciencia inglesas pre-
sentan el estado más avanzado del capitalismo). Marx no
busca simplemente modernizar el derecho y el Estado
alemanes, sino elaborar la crítica del Estado moderno en
cuanto tal; la crítica dirigida a Alemania no es la última
palabra, y si esta nación no manifestara los caracteres
universales del Estado, no tendría utilidad alguna el criti-
car al hegelianismo que les da expresión dentro de esos lí-
mites. La carencia, la falta del Estado moderno como tal
es la abstracción, a pesar de que la ciencia abstracta del
derecho, como la naturaleza abstracta de las relaciones
humanas reconocidas por este mismo derecho, funda-
menta la concepción especulativa alemana y la impone a
la crítica.
Si buscamos determinar en qué sentido el Estado hace
abstracción del hombre real, no será este texto, programáti-
co y elíptico, el que nos ofrecerá la respuesta. Es preciso
considerar por sí misma la cuestión de saber qué es, exac-
tamente, aquello de lo que Hegel hace abstracción en orden a

196
fundamentar un Estado racional en una sociedad que lo
es menos que él. En el caso del funcionario, sobre todo,
habremos de olvidar al hombre privado, al miembro de
la sociedad civil inscrito en el sistema de necesidades, al
portavoz de intereses particulares y corporativos (este úl-
timo punto es problemático), si queremos asegurar «la
realización de lo racional» no por la eficacia de utopías o
ideales individualmente utilizados, sino por la «realidad
histórica» misma, de la cual el Estado es juez y parte. En
este sentido, el funcionario carece de algo, de su humani-
dad completa, debido a que al hombre privado le falta
algo correlativo a lo anterior, la dimensión racional de la
voluntad. Al hacer surgir al hombre total como fantasma
acusador ante el hombre unilateral que Hegel descubre
completamente formado en el presente, Marx indica tan
sólo los límites de la especulación, que la separan de la
crítica, sin ofrecer ningún contenido, sea empírico o ra-
cional, a esta exigencia de tan indiscutible generalidad.
5. La conclusión, lapidaria, no está exenta, sin em-
bargo, de ambigüedad ni de riqueza, y ha de ser examina-
da bajo dos aspectos. Si los alemanes vienen después, en
un segundo momento (como la filosofía misma), estarán
privados de realidad, definida como «presente». Si conci-
ben, acceden a la racionalidad de un presente que no es el
suyo, pero que llega a serlo, de una vez, en el concepto.
No es tanto que Alemania suponga una falta o carencia
de presente, como que la Historia moderna vive el divor-
cio trágico entre la realidad vivida en la inconsciencia y
la concepción abstracta imaginaria. No es posible afron-
tar la tarea de realizar la filosofía sin partir de su existen-
cia especulativa acabada, aunque el proyecto crítico de
Marx encuentra, paradójicamente en el retraso político
alemán la condición histórica de su posibilidad, aun
cuando la de su realización esté imposibilitada desde su
misma raíz.

197
PARA SABER MÁS

HEGEL, op. cit., Principios de la filosofía del derecho.

Sobre Marx (Textos 24 y 25)

ALTHUSSER, L., La revolución teórica de Marx (Prefacio a Para


Marx), trad. M. Harnecker, Buenos Aires, Siglo X X I , 1967.
ARON, R., Las etapas delpensamiento sociológico, vol. II (Marx, Durk-
heim, Weber), trad. A. Leal, Buenos Aires, Siglo X X , 1980.
B E R L Í N , I., Karl Marx, trad. Roberto Bixio, Madrid, Alian-
za, 1988.
BOTTOMORE, T,, Diccionario del pensamiento marxista, ed. T. Botto-
more et. al., colab. L. Kolakowski, trad. V. Basterrica et. al,
Madrid, Tecnos, 1984.
CORNU, Carlos Marx, Federico Ettgels, 2 vols., trad. P. Canto y
M. Alemán, Buenos Aires, Platina, 1965.
FEUERBACH, La esencia del cristianismo, trad. J. L. Iglesias, Sala-
manca, Sigúeme, Agora, 1975.
GARAUDY, R., Introducción ai estudio de Marx, trad. J, Campos, Mé-
xico, Era, 1980.
L E F É B R E , H . , Hegel, Marx, Nietzscbe, trad. Mauro Armiño, Méxi-
co, FCE, 1976.
— Síntesis del pensamiento de Marx, trad. A, Pérez González, Bar-
celona, Nova Terra, 1971,
LICHTHEIM, G., Marxismo, un estudio históricoy critico, trad. J . Cano
Tembleque, Barcelona, Anagrama, 1972.
MAGEE, B., Los grandesfilósofos,trad. Amala Barcena, Madrid, Cá-
tedra, 1990, págs. 203-227.
STEVENSON, L., Siete teorías de ia naturaleza humana, cap. V: «Marx:
la revolución comunista», trad. E. Ibáñez, Madrid, Cátedra,
Col. Teorema, 1984.

198
26. Kierkegaard

E L SABER Y E L PECADO

Sócrates fue, desde luego, un moralista (la Antigüedad


lo reivindicó siempre como tal, como inventor de la éti-
ca) y el primero de todos, como es y seguirá siendo el pri-
mero en su género; sin embargo, comienza por la igno-
rancia. Intelectualmente, tiende hacia la ignorancia,
hacia el no saber nada. Eticamente, entiende por esto
algo por completo diferente de la ignorancia, y es de ello
de donde parte. En cambio, ni que decir tiene que Sócra-
tes nada tiene de moralista religioso; todavía menos de
dogmático cristiano. He ahí por qué, en el fondo, no en-
tra en toda esta indagación por dónde, en el antecedente
señalado, se inicia el cristianismo, el cual presupone el
pecado, cuya explicación cristiana se descubre en el peca-
do original, dogma que esta investigación no hará sino
confinar.
Sócrates, en suma, no llega hasta la categoría del peca-
do, lo que, ciertamente, supone una merma para la defi-
nición del pecado. Pero ¿como es esto? Si el pecado es, en
efecto, ignorancia, su existencia, en el fondo, se derrum-
ba, pues admitirlo es creer, como Sócrates, por otra parte,
que jamás podrá ocurrir que se cometa lo injusto sabiendo
qué es lo justo, o que tal sea cometido a sabiendas de que
es injusto. Por tanto, sí Sócrates lo ha definido bien, el pe-
cado carece de existencia. Pero, ¡atenciónl, he aquí que,
llegados a este punto, todo está perfectamente en regla
desde el punto de vista cristiano, siendo, incluso, profun-
damente justo y en interés del cristianismo quod erat de-
monstrandum, El preciso concepto que introduce una dife-
rencia radical de naturaleza entre cristianismo y paganis-
mo es el pecado, la doctrina del pecado. De igual modo,
el cristianismo cree, muy lógicamente, que ni el pagano
ni el hombre natural saben qué es el pecado, y, aun más,

199
que es necesaria la Revelación para ilustrar lo que éste
sea, porque la diferencia de naturaleza entre el paganismo
y el cristianismo, en contra de lo que cabría esperar de
una visión superficial, no procede de la Redención. No,
es preciso efectuar la separación bien y en profundidad,
partir del pecado, de la doctrina del pecado, como, por
otra parte, ya hace el cristianismo. ¡Qué peligrosa obje-
ción contra este último, si el paganismo diera una defini-
ción del pecado cuya justeza hubiera de ser reconocida
por los cristianos!

Tratado de la desesperación (1849).

COMENTARIO

1. Podemos, sin recurrir a artificio alguno, poner en


conexión este texto, redactado en 1848, con el esfuerzo
filosófico en el que Marx, a pocos años de distancia, to-
maría parte activa. No se trata tan sólo de definir lo que
separa a la filosofía, entendida en sentido moderno, de su
equivalente griego más señalado, sino también de tomar
distancia con respecto a la definición hegeliana de la éti-
ca, concebida como «moralidad» subjetiva y objetiva, que
conduce a comprender la racionalidad de todó lo existen-
te. AI igual que Marx, Kierkegaard se dirige en el texto
que nos ocupa a una racionalidad en vías de realización.
No confiere a la filosofía el papel de dar expresión al es-
fuerzo de lo racional por hacerse historia. Quiere redefi-
nir las relaciones que se establecen entre el saber y la éti-
ca, tanto en el caso de Sócrates el Griego como en el del
cristianismo moderno. En suma, nos encontramos ante
el problema de saber por qué y en qué somos modernos.
Tal es el objetivo al que se orienta esta meditación.
El asunto que nos ocupa no es otro que el del pecado,
definido como no-saber o como punto de partida ante-
rior a todo saber, situado en la voluntad. El pecado es el
punto de separación entre Sócrates y el cristianismo en
general, lo que aleja al griego, más específicamente, de

200
nosotros los modernos. El texto articula, pues, las dos
eras y los dos conceptos de ética en torno al pecado,
ausente en Grecia, como punto de partida de nuestro
tiempo.
Elprimer párrafo establece que el pecado efectúa la sepa-
ración del cristianismo, antes del cual no se puede pensar
en una ética a no ser en términos de ignorancia, de modo
que Sócrates se erige en el interlocutor filosófico del pa-
sado precristiano.
El segundo párrafo invierte el punto de vista. En efecto,
ahora se trata de cómo definir el pecado como algo dis-
tinto de la ignorancia, porque, de reducirse a esta última,
se corre el riesgo de borrar la diferencia de naturales entre el
cristianismo y el paganismo y de perder de vista la mo-
dernidad. La alternativa es, en efecto, o bien Sócrates, o
bien la doctrina del pecado, sin que exista ninguna forma
de compromiso o de transición posible1. El interés de
Kierkegaard se centra en la radicalidad misma de este
problema.
2. Kierkegaard no pretende situar a Sócrates en la
historia, sino únicamente determinar si su concepción de
la ética, tan decisiva para la filosofía, entra en toda esta in-
dagación por donde se inicia el cristianismo, que define la deses-
peración como pecado.
Entendemos que el verdadero problema es el de saber
si «la filosofía», indagación con vocación de conocimien-
to y que, por tanto, guarda una relación esencial con la
ignorancia, y el cristianismo, que parte de una mancha
anterior a toda ignorancia o inocencia, pueden hallar un
mutuo entendimiento o si fundamentalmente se ignoran.
El Sócrates moralista pasa entonces a desempeñar el
papel de criterio de demarcación. Si parte de la ignoran-
cia, mucho más al no tender hacia ella (Intelectualmente,
tiende hacia ¡a ignorancia... Éticamente... es de ella de donde parte),
se aleja del dogma del pecado original punto de partida
del cristianismo, siendo este alejamiento no histórico,

1 Cfr. Lukacs, El almaj lasformasy la teoría de la novela, Barcelona, Gri-

jalbo, 1975.

201
sino radical; no como aproximación o intuición vaga de
un pensamiento en torno al pecado, sino como aleja-
miento absoluto. La segunda parte del texto se ocupa de
explicitar lo anterior.
Toda la dificultad reside en definir una ética, arte del
comportamiento justo, en relación con el saber. Es, pre-
cisamente, porque es posible hacerse con el saber de lo
justo y de lo injusto (tal es el objeto de la República de Pla-
tón) por lo que una filosofía puede llegar a ser ética y de-
finir la falta (cometer lo injusto) como ignorancia («Na-
die es voluntariamente malo»)2. La segunda parte aborda,
pues, los presupuestos más radicales de la filosofía en
cuanto tal, sin el preámbulo de la fe.
3. Para los propósitos de Kierkegaard, se hace nece-
sario definir la ausencia de la categoría del pecado como
una carencia localizada en el seno del paganismo, algo
que le impide alcanzar una dimensión decisiva, y no
como una diferencia cualquiera o alguna otra categoría
intercambiable (la ignorancia como origen de la falta)
que pudiera cumplir análogas funciones. Este es el modo
adecuado de comprender las dos primeras frases: al no
«llegar hasta» la catearía delpecado> Sócrates, quien, sin em-
bargo, es el inventor de la ética, y seguirá siendo primero en su
género, no ha llegado sencillamente tarde a una evolución
histórica ulterior, ni tampoco le falta aún algún conoci-
miento por adquirir; lo que sucede es que le es imposible
definir el pecado, y no sólo que esta definición encuentre
una merma en no haber alcanzado la categoría menciona-
da. Esto último es lo que configura el primer punto f...
...elpecado cauce de existencia). La ignorancia, causa de las
injusticias, no es una definición imperfecta del pecado,
sino su absoluta negación. Nos resta, ahora, enfrentarnos
al problema del punto de vista del propio cristianismo,
quien debe situar el punto por el cual difiere por comple-
to del paganismo y de la filosofía sin la fe, Sócrates con-
templa la falta (cometer lo injusto) como un signo de la
ausencia de la idea de justicia, manteniendo en todo mo-

2 Platón, Mtnótt, 77 d-78 a; Prvtágoras, 352 b y ss.; Corgias, 509e y ss.

202
mentó que esta idea está a) alcance de un conocimiento
umversalmente posible. El arte de la dialéctica no puede
remediar en absoluto el ocultamiento y el olvido de la
idea. Al proclamar la salvación de las almas por la Re-
dención, ¿acaso no está prometiendo al cristianismo re-
cuperar en bloque aquello que el dialéctico había media-
namente separado? Según Kierkegaard, lo anterior sería
propio de una visión superficial que vuelve la espalda a la re-
velación. Lo absolutamente primero y anterior a todo co-
nocimiento, aun cuando sea olvidadizo de sí mismo,
dado a creer (el dogma) y no a saber, tal es lo que efectúa la
separación con respecto al paganismo, entendido, por con-
siguiente, como creencia en la posibilidad de un conoci-
miento inocente.
Mediante el rechazo de todo compromiso, de alguna
lenta transición del paganismo a su contrario, Kierke-
gaard concluye en un intento de solidarizar el retorno al
fundamento último, el dogmatismo, con la negación de
la filosofía basada en la analogía de la luz. El filósofo da-
nés juzga negativamente la inocencia del paganismo: «la
intelectualidad griega era demasiado dichosa, demasiado
inocente, demasiado pecadora...»1. Así, pues, su única
utilidad es la de enfrentarse al filisteísmo filosófico, cuya
expresión, en la época de Kierkegaard, sería el hegelianis-
mo (la «virtuosidad tan frecuente»... de desarrollar inabs-
tracto «las verdades supremas»)4.

PARA SABER MÁS

Sobre Kierkegaard

ABBAGNANO, N. y otros, Estudios kierkegaardianos, Buenos Aires,


Nova, 1965.
COLLINS, }., El pensamiento de Kierkegaard, trad. E. Landázuru,
México, FCE, 1970.

' Cfr. Tratado de ¿a desesperación.


* CÍT. Tratado de ¡a desesperación.

203
HOLMES HARTSHORNE, M., Kierkegaard: el divino burlador, trad.
E. Lucena, Madrid, Cátedra, Teorema, 1992.
ROHDE, P. P., Ssren Aabje Kierkegaard (181 í-1855):padre delexis-
tenríalitmo, trad. D, Kraemer, Vibelse Pentz-Moller, Copen-
hague, Ministerio de Relaciones Exteriores de Dinamarca,
Dpto. de Prensa y Relaciones Culturales, 1983.
SARTRE, P. y otros, Kierkegaard vivo, Madrid, Alianza, 1968.

27. Nietzsche

Q U E EL ATEÍSMO FILOSÓFICO ABRE LA CUESTIÓN


D E L S E N T I D O DE LA EXISTENCIA

Schopenhauer fue, como filósofo, el primer ateo reco-


nocido e inflexible de entre nosotros, los otros alemanes;
tal era el verdadero motivo de su hostilidad contra Hegel.
El carácter no divino de la existencia era para él algo in-
mediato, tangible, indiscutible; perdía su innegable san-
gre fría y se encolerizaba cada vez que veía a alguien du-
dar sobre este asunto y perderse en rodeos. Ahí radica su
honestidad: el ateísmo absoluto y franco es, en efecto, la
condición previa de su manera de plantear los problemas,
en tanto que victoria final y duramente adquirida de la
consciencia europea, en tanto que el acto más pleno de
consecuencias de una educación doblemente milenaria
enderezada hacia el espíritu de verdad y que termina pro-
hibiéndose la mentira de la creencia en Dios... Se ye aquí
lú que ha supuesto, en suma, la victoria sobre el dios cris-
tiano: la moralidad cristiana en si misma, la noción de
veracidad tomada en un sentido cada vez más riguroso, la
sutilidad de la consciencia cristiana desarrollada por los
confesores, traducida y sublimada en consciencia cientí-
fica hasta alcanzar, a cualquier precio, la probidad inte-
lectual. Considerar la naturaleza como si fuera una prue-
ba de la bondad y protección de Dios, interpretar la his-
toria a mayor gloria de la razón divina, como testimonio

204
perpetuo de la finalidad moral del orden universal; inter-
pretar sus propias experiencias vividas en el sentido
como las gentes piadosas venían haciéndolo desde hace
mucho tiempo, como si no hubiera nada más que el don,
el signo, el anuncio de la providencia, y como si todo de-
biera concurrir infaliblemente en la salvación del alma.
He ahí lo que, a pesar de todos los pesares, se ha cumpli-
do, lo que es contrario a la conciencia; he ahí lo que las
conciencias más refinadas sienten hoy en día como des-
honestidad, mala fe, impostura, afeminamiento, debili-
dad, cobardía —y es en virtud de un rigor semejante, si es
que debe serlo en virtud de algo, que somos, en efecto,
buenos europeos, herederos de la dominación de sí más du-
radera y valerosa de la que Europa haya dado prueba.
Ahora bien, tan pronto como arrojamos lejos de nosotros
la interpretación cristiana, y tachamos su «significado»
de moneda falsa, nos asalta la cuestión sebopenhaueriana
del modo más terrible: la existencia en general, ¿tiene algún
sentido?

La gaya ciencia (1881-1882).

COMENTARIO

1. Nietzsche, interpretando la filosofía de Schopen-


hauer como novedad europea radical, concatena en este
texto cuatro proposiciones, la última de las cuales contie-
ne el sentido futuro de la empresa filosófica misma.
Schopenhauer es el filósofo ateo, condición previa de los
problemas que plantea (Schopenhauer... —* ...creencia en
Dios). El sentido de este ateísmo es la exigencia absoluta
de veracidad, exigencia recibida del propio cristianismo
(Seveaqui... * ...la probidad intelectual). La mentira inadmi-
sible por excelencia es el finalismo, la interpretación del
mundo y de la experiencia en términos de providencia
(Considerar... —*• ...haya dado prueba). Una vez descartada la
mentira del significado transcendental de la experiencia,
la cuestión sebopenhaueriana puede ya hacerse manifiesta,

205
cuestión que alude al sentido mismo de la existencia
(Ahora bien, tan pronto como... * ... algún sentido?).
2. Nietzsche quiere contemplar la novedad que su-
pone Schopenhauer desde una perspectiva histórica, y
son las Consideraciones Intempestivas* las que dejan patente su
estatura como educador. De lo anterior se sigue que su
hostilidad hacía Hegel y el hegelianismo adquiera un sen-
tido filosófico unitario, consistente en la afirmación del
carácter no divino de la existencia, idéntico al de la existencia
como tal, en su sentido moderno. En ruptura con las teo-
diceas reconocidas o hipócritas, Schopenhauer parte de
una certeza inmediata, tangible, indiscutible en lo que respecta
a la existencia y su sentido. En efecto, este último no se
encuentra predeterminado, ningún dogma dirige o regula
el modo de abordarlo. Volviendo, al igual que Kierke-
gaard, a la unión artificial de la existencia y el saber, el fi-
lósofo ateo no se detiene en el dogma del pecado, preám-
bulo de la doctrina de la redención, y no justifica ta deses-
peración ni tampoco la duda; erige en certeza la ausencia
de justificación de la existencia por un orden anterior a la
experiencia.
Para Nietzsche, se trata de un genuino punto de parti-
da de la filosofía y sus problemas (condición previa), así
como de un resultado o maduración del espíritu europeo,
pues han sido necesarios dos mil anos de filosofía para
abandonar la mentira religiosa que obstaculiza a la cues-
tión filosófica en cuanto tal. Esta cuestión filosófica
vuelve a ponerse en marcha con Schopenhauer, y co-
mienza «terminando por prohibirse», prácticamente,
aquello que la convertía en bastarda, que había ocasiona-
do su degeneración.
3. Volviendo sobre los momentos decisivos del logro
de la autonomía mencionada, Nietzsche no pretende va-
lidar retrospectivamente la cuestión socrática y su exi-
gencia de verdad, sino más bien el espíritu inquisidor de
la confesión, prolongado por una ética científica2: la ve-

1 Cfr. Consideraciones intempestivas, texto 28.


2 La gaya ciencia, § 344, L. V.

206
\
racídad no es inocente, primera, sino el triunfo de una
larca y penosa educación (disciplina). Nietzsche quiere es-
tablecer cómo ha sido posible el interés radical y absoluto
por una verdad conquistada sobre la mentira dominante.
4. El significado de este tipo de tratamiento histórico
aparece enseguida, no siendo otro que el del entero edifi-
cio del pensamiento clásico, identificado como una menti-
ra única. Este paso es necesario como preámbulo a la hora
de hacer surgir la pregunta filosófica radical de Schopen-
hauer, así como, y sobre todo, el espíritu europeo propia-
ménte moderno, desprendido de sus ilusiones.
La mentira piadosa se descompone en los tres momen-
tos de naturaleza, historia y experiencia vivida. La provi-
dencia parece dirigir los fines de la naturaleza, el sentido
de la historia, el valor de la vida. Nietzsche asume enton-
ces la consecuencia que resulta de la «revolución» llevada
a término en la Europa moderna, y ello con el propósito
de regresar, en última instancia, a la filosofía de Schopen-
hauer. En efecto, si el finalismo piadoso señalado ya ha
visto su final, ello significa que la disciplina filosófica ha
alcanzado su objetivo. En primer lugar, ésta había esta-
blecido la dominación de Dios sobre las conciencias
pero, al proceder de este modo, hizo de nosotros los here-
deros de la dominación de si más duraderay valerosa, si bien ya
no toleramos en la actualidad el engañoso (mentiroso)
consuelo del que era portadora.
5. Si la naturaleza, la historia, la vida e, incluso, el es-
píritu carecen del significado que la teología les vino atri-
buyendo desde el triunfo del cristianismo, ¿qué filosofía
podrá aún interpretar el sentido de aquéllos? En este
punto reside la importancia decisiva de Schopenhauer,
inventor del problema filosófico característico de la mo-
dernidad al que Nietzsche, por su parte, procurará adscri-
birse: la existencia en general, ¿tiene algún sentido? Podemos ad- l-
mitir, en efecto, que una filosofía no teológica tienda a
restaurar un sentido para la historia, una finalidad para la
naturaleza o un valor para la experiencia, permaneciendo
con ello en conformidad con la antigua idea de la inter-
pretación de lo existente. Anteriormente a la revolución

207
schopenhaueriana, para quien el ateísmo ¿s la condición
de condiciones y no una simple variable la filosofía ha-
bría ignorado la cuestión más radical y dolorosa, se ha-
bría situado a la cola del espíritu europeo. Schopenhauer
no renueva la cuestión del sentido sustituyendo un senti-
do por otro distinto, sino que hace de ella «la» cuestión fi-
losófica, y Nietzsche responderá investigando cómo han
sido creados los valores a partir de un mundo que, en sí
mismo, no posee ninguno.

28. Nietzsche
Q U E E L FILÓSOFO Y E L ARTISTA SON T R O P I E Z O S
D E LA NATURALEZA

La naturaleza quiere ser siempre de utilidad práctica,


mas, para cumplir este objetivo, no atiende siempre a la
búsqueda de los medios más adecuados. He aquí su gran
pesar y lo que la vuelve melancólica. Que ella quiera dar
a la existencia del hombre un significado y una importan-
cia mediante la creación del filósofo y el artista, tal es lo
que aparece como cierto, dada su aspiración a liberar. Sin
embargo, ¡qué incierto, qué débil y pobre es el efecto que
alcanza la mayoría de las veces con los filósofos y los ar-
tistas! ¡Qué raras las ocasiones en las que llega siquiera a
obtener un efecto cualquiera! Sobre todo en lo concer-
niente al filósofo, su turbación es grande cuando preten-
de darle una utilización general. Sus medios no parecen
ser nada más que ensayos vacilantes, ideas sutiles inspira- ;

das por el azar, de tal suerte que sus invenciones, por lo


general, suelen ser defectuosas y la mayoría de los filóso-
fos no pueden ser de utilidad general alguna. Los proce-
dimientos de la naturaleza toman el aspecto del derroche,
pero no se trata del derroche de una criminal exuberan-
cia, sino del propio de la inexperiencia. Es preciso admi-
tir que, si la naturaleza fuera hombre, rio podría llegar a
librarse del despecho que se ocasionaría a sí misma v de

208 :
las desdichas de él resultantes. La naturaleza envía al filó-
sofo hacia la humanidad como una flecha; no apunta,
mas espera que la flecha se clavará en algún lugar. Pero
haciendo esto se equivoca una infinidad de veces y por
ello siente despecho.

Consideraciones intempestivas (1874), t. 2

COMENTARIO

1. La cuestión inicial1 concierne al sistema educativo en


relaciórt con la vida y la influencia de los filósofos. Para
formar nuevos filósofos, para sobrevivir tan sólo, ¿cuáles
son las oportunidades con tas que puede contar un filóso-
fo, conociendo la precaria existencia de un Schopen-
hauer?
Nietzsche da cuenta del estatus actual del filósofo y el
artista a través de una prolongada metáfora, y evalúa las
posibilidades de las que aquéllos disponen a la hora de se-
ñalar cuál sea el estatus mencionado. Para ello, nuestro
autor les hace entrar en el plano de la naturaleza, dentro
de los límites de los medios de esta última que permiten
llevar a la práctica su objetivo.
Es difícil intentar dividir nuestro texto en partes autó-
nomas, pues el movimiento de la demostración conecta
entre sí diversos argumentos que convergen en la afirma-
ción de la «melancolía» de una naturaleza desbordada y
decepcionada por sus resultados. Tampoco es posible, ni
deseable, tomar al pie de la letra el punto de vista que
Nietzsche parece adoptar, en lo que respecta a la creación
artística y filosófica, al hablar «en nombre de la naturale-
za». Es esta extravagancia lo que habremos de comentar
para reestablecer la perspectiva y situar a la filosofía de la
creación que late en todo el texto.
2. La personalización de la naturaleza puede sor-
prender en un autor tan hostil al finalismo2. Es necesario

1 Cfr. texto 27.

2 Cfr. texto 27.

209
distinguir, en primer lugar, entre los finés y los medios de
la naturaleza. Nietzsche presta a esta última una preocu-
pación por los fines, pero también una pobreza de medios
para llevarlos a cabo. Sobre todo, hemos de leer la metá-
fora para adentrarnos en aquello que establece como lo
más paradójico; esto es, no sólo la impotencia, sino la me-
lancolía de una naturaleza que crea al filósofo y al artista
creyendo así hacerse útil, aunque en la práctica no llega a
alcanzar su propósito.

Nietzsche, según parece, no toma en serio la doctrina


de la utilidad práctica extendida a los productos de la na-
turaleza misma, doctrina que estuvo en boga después de
que Darwin y Spencer obtuvieran el favor del público
alemán. Asimismo, la imagen de la decepción que se apo-
dera de la naturaleza tiene algo de sarcástico, pues el artis-
ta, concebido desde el punto de vista de la utilidad prácti-
ca o social, es un producto fallido. En contraposición, lo
genuinamente nietzscheano es la idea de que la naturale-
za, con este esfuerzo, tiende al alumbramientoy quiera dar a la
existencia del hombre un significadoy una importancia. El artista y
el filósofo confieren un sentido efectivo a aquello que no
lo tiene en sí mismo; es decir, al mundo, a la vida. Al pro-
ceder de este modo, «saben», «liberan» a la naturaleza de
su insignificancia, hacen de ella algo «útil» o necesario,
pero sin que esta utilidad permita ser definida en el senti-
do práctico habitual. En su calidad de excepciones, estos
hombres creadores no entran a formar parte del orden de
fines de la especie, sino que reinventan su utilidad. Por sí
mismos no son creadores de una naturaleza divinizada,
considerada como origen del valor de la existencia. Se
comprende ahora la turbación que sentiría la naturaleza si
fuera hombre al querer dar a la filosofía una utilización gene-
ral. Desde este punto de vista, sería preciso considerar al
creador de formas o de ideas como un derroche, signo de
inexperiencia. Y, lo que es peor, la naturaleza utilitaria,
concebida según el tiempo, encuentra en los grandes
creadores no la prueba, sino la invalidación del principio %
de economía que la gobierna. El genio no es natura,

210
como creía Kant 3 , sino antinatura, manifestación de un
orden superior creado y no transcendente que se encuen-
tra del todo constituido en algunos hombres excepcio-
nales.
3. Si la naturaleza fuera hombre adquiere un significado
positivo en este punto, pues se cree que la naturaleza es
orden y, sin embargo, las excepciones que ella distribuye
torpemente y sin arte ponen de manifiesto, paradójica-
mente, la «sabiduría de sus fines» mejor que la regularidad
de sus productos ordinarios. La posibilidad de una crea-
ción procedente de un artista o un pensador confiere a la
naturaleza, recíprocamente, el carácter trágico de un es-
fuerzo orientado a dar sentido a la existencia, y el distan-
ciamiento entre fines y medios que provoca su sufrimien-
to la humaniza al hacerle presentir valores que la desbor-
dan, ¿Cuál es la utilidad del gran hombre, del genio?
Ciertamente, no es la de servir a la humanidad tal cual es,
hija de la naturaleza, sino la de humanizarla verdadera-
mente al violentarla, aun cuando esta violencia se vuelva,
en primer lugar, contra el hombre aislado que desafía el
orden establecido y lo relativiza.
Parece que, a fin de cuentas, la idea schopenhaueriana
de una voluntad que opera en la naturaleza a través de la
experiencia del arte cede aquí su puesto a la voluntad,
más moderna, de un hombre arrancado a esa misma na-
turaleza y que encuentra su sentido libremente, en una
auténtica creación de valores.

PARA SABER MÁS

' Sobre Nief&cbe (textos 27 y 28)

DELEUZE, G., Nietzsche y la filosofía, trad. C . Artal, Barcelona,


Anagrama, 1986.
F I N K , E „ La filosofía de Nietzsche, trad. A. Sánchez Pascual, Ma-
drid, Alianza, 1984.
3
I, Kant, Crítica del jukto, §46 y 547.

211
De Hegel a Nietzsche, trad. E. Estiú, Buenos Aires,
LÓWITH, K . ,
Sudamericana, 1968.
MACEE, B„ Los grandesfilósofos, trad. Amaia Barcena, Madrid, Cá-
tedra, 1990, págs. 251-273.

Otros:

DARWIN, CH„ El origen de las especies, trad. A. Froufe, Prólogo


de Faustino Cordón, Madrid, EDAF, 1985.

29. Freud

E L PSICOANÁLISIS F R E N T E A LA TRADICIÓN FILOSÓFICA


Y PSICOLÓGICA

Y he aqui que todos o casi todos coinciden en encon-


trar un carácter común para todo lo psíquico, carácter
que traduce su esencia misma. Es el carácter único e in-
descriptible de la eonscieneia, que, por otra parte, no necesi-
ta ser descrito. Todo lo consciente es psíquico y, a la in-
versa, todo lo psíquico es consciente. ¡Cómo negar una
evidencia semejante! Hemos de reconocer, sin embargo,
que este modo de ver apenas si ha clarificado en algo la
esencia del psiquismo, pues la investigación científica se
encuentra aquí frente a un muro. No descubre ninguna
vía capaz de conducirla más allá del mismo. (...) ¿Cómo
no apreciar, en efecto, que los fenómenos físicos depen-
den en alto grado de los fenómenos somáticos y que, a la
inversa, aquéllos también actúan con gran fuerza sobre
éstos? Si el espíritu humano se encontró alguna vez en un
callejón sin salida fue, sin duda, en esta ocasión. Con el
objeto de hallar un rodeo, los filósofos se vieron forzados
a admitir, al menos, la existencia de procesos orgánicos
paralelos a los psíquicos y dependientes de estos últimos
de una manera difícilmente explicable. (...) El psicoanálí-

212
sis escapa a las dificultades mencionadas negando enérgi-
camente la asimilación de lo psíquico a lo consciente.
No, la consciencia no constituye la esencia del psiquis-
mo; no es nada más que una cualidad, y una cualidad in-
constante que, por lo general, se encuentra más ausente
que presente.
(...) Pero aún resta una objeción por refutar: a pesar de
los hechos de los que acabamos de hablar, algunos pre-
tenden que no es conveniente renunciar a la idea de la
identidad entre psíquico y consciente porque los procesos
psíquicos llamados inconscientes serían, según ellos, pro-
cesos orgánicos paralelos a los procesos psíquicos. De
este modo, el problema que queremos resolver plantearía
una vana cuestión de definición. (...) ¿Es solamente por
efecto del azar que hemos llegado a elaborar una teoría
unitaria y coherente del psiquismo una vez modificada su
definición y no antes?
Por otra parte, hemos de evitar la tentación de creer
que es el psicoanálisis el que ha renovado esta teoría del
psiquismo. (...) El concepto de inconsciente había estado
llamando a las puertas de la psicología desde hacía tiem-
po, y tanto la filosofía como la literatura flirteaban con él
aun cuando la ciencia no supiera cómo utilizarlo. El psi-
coanálisis ha hecho suya esta idea, la ha considerado se-
riamente y dotado de un nuevo contenido. Las investiga-
ciones psicoanalíticas han encontrado ciertos caracteres
del psiquismo no imaginados hasta entonces y descubier-
to algunas de las leyes que lo rigen. No queremos decir
con esto que la cualidad de la consciencia haya perdido su
valor para nosotros. Ella continúa siendo la única luz que
nos ilumina y guía en las tinieblas de la vida psíquica.
Como consecuencia de la particular naturaleza de nuestro
conocimiento, nuestra tarea científica en el dominio de
la psicología consistirá en traducir los procesos incons-
cientes a procesos conscientes para, así, llenar las lagunas
de nuestra percepción consciente.
Compendio de psicoanálisis (1938),

213
COMENTARIO

1. Extraído de una obra inconclusa (Freud murió


en 1939) con pretensiones de presentación sintética del
psicoanálisis, nuestro texto pone de manifiesto una preo-
cupación por señalar el aspecto común que posee la tradi-
ción filosófica con respecto a la psicología, y ello con la
finalidad de situar con mayor precisión el punto de rup-
tura de la empresa psicoanalítica. Frente al carácter común,
que reside en la creencia en la omnipotencia de lo cons-
ciente en el ámbito de lo psíquico, el punto de ruptura im-
plica la promoción y tematización del concepto del in-
consciente. Freud procede aquí por etapas: el carácter
común, que se ofrece como unaevidencia, se apoya, precisa-
mente, en la opinión común (todos o casi todot). Sin embar-
go, cabe la posibilidad de suponer que la tradición filosó-
fica (en la estela de una cierta lectura de Descartes) se ha ;
apoderado de ella sin dificultad, pues el consciente cons-
tituye, él sólo, la totalidad del psiquismo y, en consecuen-
cia, postular la existencia de un psiquismo inconsciente
sería, desde esta perspectiva, un absurdo e, incluso, una
monstruosidad. Finalmente, la psicología con vocación
científica, al reducir el inconsciente a lo orgánico, deli-
mita las dificultades para mejor asentar la identidad de lo
psíquico y lo consciente. Et punto de ruptura marca, pues,
una recusación global inducida por cierto número de in-
dicios que atestiguan la existencia de otra tradición (lite-
raria, pero también, en nuestro caso, filosófica).
2. En el primer párrafo, parece que la asimilación ordi-
naria de lo psíquico a lo consciente y el consenso acerca
del reconocimiento de la consciencta como esencia misma
del psiquismo funcionan como verdadero obstáculo (la
imagen del muro) al rigor de una investigación de orden
científico. En efecto, la habitual afirmación de la equiva-
lencia de lo psíquico y lo consciente (todo lo consciente es psí-
quicoy, a la inversa, todo lo psíquico es consciente) desprecia, apa-
rentemente, la interacción entre lo psíquico y lo somático

214
tan corriente como obscuramente presentida por todos
nosotros. El concepto, pues, no da cuenta de lo vivido.
Tal es el callejón sin salida por el que los filósofos (y los psi-
cólogos) tratan de evitar el rodeo mediante el postulado
del paralelismo psico-fisiológico. No obstante, evitar la
dificultad no significa escapar de ella. La única salida posi-
ble es la negación, a través del psicoanálisis, de la ecuación
psiquico-consciente y la designación de residencia de la
consciencia en términos de cualidad (inconstante, por
tanto) y no de esencia del psiquismo.
La crítica, sin embargo, se desarrolla siguiendo dos ejes
complementarios cuyo telón de fondo nos será útil esta-
blecer.
a) En efecto, conviene destacar que la asimilación de
lo psíquico a lo consciente, según la cual se da una repre-
sentación inmediata de la actividad del sujeto sobre sí
mismo, conoció su forma extrema en ciertas escuelas fi-
losóficas (sobre todo, y en el siglo xix, la escuela «eclécti-
ca», que es la representación, por excelencia, de lo que los
pioneros de la psicología científica denominarían psico-
logía «clásica»). Al expresar su desconfianza con respecto
a la psicología y proclamar la separación de la conscien-
cia y del cuerpo en disciplinas esencialmente distintas en
el orden de la investigación, la escuela ecléctica (Jouffroy,
Cousin, Roger-Collard) suponía que todos recibimos
constantemente información acerca de lo que ocurre en
nuestro interior, ya sean pensamientos o sensaciones, y
que, haga lo que haga nuestro intelecto o experimente
nuestra sensibilidad, siempre tendremos consciencia in-
mediata de ello. Esta restauración de la interioridad, de la
consciencia como hecho primitivo y fundamentador de
la verdad, descansa, sin duda, según Freud, en un sofisma
consistente en confundir inmediatez y conocimiento,
evidencia y transparencia. Tal es el sofisma que deberá ser
desarticulado y destruido por el psicoanálisis.
b) Sin embargo, la crítica no se agota en esta denun-
cia, pues existe también una psicología con vocación
científica, la misma que se ha constituido en reacción
contra los apóstoles de la introspección y que, sobre todo

215
a partir de las conquistas más recientes de la físiologia, ha
topado con el muro de la omnipresente consciencia. El
modo en que ésta ha logrado evitar el callejón sin salida que
Freud menciona en el texto, se basa en la puesta en juego
del paralelismo psico-fisiológtco que estipula que nada hay en
nuestra consciencia que no esté asociado a un proceso fi-
siológico simple (como en el caso del acto reflejo) o com-
plejo (como en el caso, por ejemplo, de la memoria). No
se habla ya en términos de substancias (alma-cuerpo) ac-
tuando la una sobre la otra, sino de fenómenos que se en-
cuentran en conexión constante. La solución elaborada
por los psicólogos sigue sin ser, a los ojos de Freud, del
todo satisfactoria, y ello en la medida que no logra deste-
rrar la asimilación de lo psíquico a lo consciente, aspecto
éste que constituye, en cuanto ta), una de las caras del fe-
nómeno por estudiar'.
3. El segando párrafo representa la tercera etapa o fase
de la refutación freudiana. Una variante del paralelismo
psico-fisiológico estipula que los procesos psíquicos in-
conscientes según algunos, son pura y llanamente con-
fundidos con los procesos orgánicos paralelos; el incons-
ciente del que ellos hablan queda así reducido al ámbito
de lo somático. Es preciso, a la hora de emitir la objeción
mencionada, admitir el carácter primero de los hechos
orgánicos y la posibilidad de que existan fenómenos vita-
les que no estén acompañados de consciencia. Ahora
bien, Freud insiste en el hecho de que lo anterior no es
más que un problema de definición que, en lugar de es-
clarecer el debate entablado, lo ahoga. En efecto, me-
diante la reducción del inconsciente a lo somático y a
nada más se consigue reforzar la tesis de la identidad en-
tre lo físico y lo consciente.
Lo que resta, evidentemente, no es sino la imposibili-
dad de pensar en una interacción, cualquiera, «alma-
cuerpo», ya que el principio del paralelismo anuncia la
presencia de una concomitancia, no de una acción recíproca

' Cfr. pág. 213.

216
entre los fenómenos psíquicos y los orgánicos. No obs-
tante, el «extremismo» de ciertos psicólogos no podría
hacer resurgir el sofisma que preside la representación
habitual, filosófica y psicológica del aparato psíquico.
El golpe de gracia analítico va a consistir, precisamen-
te, en destruir el sofisma señalado a través de la promo-
ción del inconsciente como estructura psíquica, como
objeto específico de la psicología: el inconsciente será lo
propiamente psíquico y su esencial realidad. Gamo resulta-
do de lo anterior, las manifestaciones conciencíales se re-
velan lacunares (como lo atestiguan el sueño, el olvido,
etc.), y la definición de la vida psíquica deberá sufrir, por
tanto, una revisión completa y radical. Únicamente la de-
finición de lo psíquico mediante el concepto de incons-
ciente permite la elaboración de una teoría unitaria cohe-
rente que no sea un mero efecto del azar.
4, El tercer párrafo precisa en qué rompe el psicoanáli-
sis con la tradición psicológica. Al hacerse cargo de la
idea abstracta del inconsciente, y con la voluntad de to-
mar en serio esta idea, Freud le ha conferido un nuevo
contenido, transformando así la idea en concepto. Lo an-
terior quiere decir que el punto de ruptura se refiere menos
a la emergencia de la noción del inconsciente, nutrida
asimismo de una cierta tradición literaria y filosófica (po-
demos evocar, en este sentido, la tragedia griega, a los ro-
mánticos alemanes, e, igualmente, a Schopenhauer y
Nietzsche, por no hablar del sentimiento popular, que
siempre ha considerado que los sueños poseen un senti-
do), que al estatus del concepto del inconsciente, cuya
justificación viene reclamada por una infinidad de indi-
cios suministrados por la vida cotidiana (sueño, actos ol-
vidados, lapsus, palabras fruto del ingenio). Al muro y al
callejón sin salida que invitan al psicólogo a buscar un rodeo,
se opone, de este modo, la puerta por la cual puede pene-
trar el psicoanálisis, con tal de que, continuando con la
metáfora, construya la ruta adecuada (el método) y se
haga con un armazón arquitectónico capaz de dar cuenta
de la experiencia vivida por cada individuo. Tomar en se-
rio (frente a la psicología oficial) la idea del inconsciente

217
y proporcionarle un nuevo contenido es una misma cosa.
Por poner tan sólo un ejemplo, Freud, para hablar de este
nuevo contenido, acude a la expresión de «representacio-
nes inconscientes», que, al desmarcarse nítidamente de la
noción tradicional (filosófica) de representación como
simple contenido de un acto de pensamiento, sugiere
abiertamente la disociación buscada entre lo psíquico y lo
consciente. Los caracteres no imaginados del psiquismo
inconsciente nos remiten, entre otros, a los mecanismos
de represión y sublimación. Las leyes específicas que re-
gulan estos mecanismos proceden de aquello que Freud
denomina los «procesos primarios» de condensación,
desplazamiento y sobrede terminación2.
Él psicoanálisis constituye, pues, el concepto de su ob-
jeto, que viene representado por el inconsciente y sus le-
yes propias. Sin embargo, la consciencia no ha quedado
eliminada del campo científico, sino que, como cualidad
inconstante y ya no como esencia de la vida psíquica, conti-
núa siendo la luz que nos puede guiar en las tinieblas. Al
igual que Champollion frente a sus jeroglíficos, el analista
debe asumir, en términos de conjeturas, la «traducción»
de los procesos inconscientes a procesos conscientes; y el
enunciado de esa traducción sólo es posible si el concurso
de la lógica discursiva permite reanudar los hilos sólidos,
aunque dispersos, de la lógica aberrante, aunque cohe-
rente, aplicada por los procesos primarios. El concepto
de consciencia, desposeído de su hegemonía psicológica,
encuentra un lugar, su lugar, en el seno de una tópica (el
sistema inconsciente-preconsciente-consciente) que le
restituye su antigua dignidad, al tiempo que le asigna sus
límites. La destrucción de la tesis de la identidad de lo
psíquico y lo consciente no significa en modo alguno la
anulación pura y simple de la consciencia misma. Al con-
trario, al poner de manifiesto el sentido de la revolución
psicoanalítica, Freud pretende demostrar dos cosas:
— Cómo se desprende de la tradición filosófica y de la

2 Para uní explicación más precisa de estas nociones, acudir ¿1 Diccio-

nario de psicoanálisis, Barcelona, Labor, 1983.

218
psicología de su época tras haberse inscrito en una tradi-
ción diferente y, quizás, marginal.
— En qué se manifiesta el que la finalidad de la em-
presa psícoanalítíca consista en llenar las lagunas de la
percepción consciente, tarea que no se puede cumplir
más que a la luz de la propia consciencia. Todo ocurre
como si Freud estuviera oponiendo a la labor de traduc-
ción, característica del psicoanálisis, a esa especie de lectura
paralela que parecería requerir la psicología en nombre
del paralelismo psico-fisiológico.

PARA SABER MÁS

Entre la abundante literatura existente sobre Freud, el lector


podrá iniciar una primera aproximación al psicoanálisis a través
de estas obras:

Sobre Freud

JONKS,Ü , Viday obra de Sigmund Freud, trad. M. Carlisky, Buenos


Aires, Ediciones Horme, 1978.
LAPLAN<;HK/PONTALIS. Diccionario de psicoanálisis, Barcelona, La-
bor, 1983.
RICOEUR, P., Freud: una interpretación de la cultura, trad. A, Súarez,
México, Siglo X X I , 1987.
STKVHNSON, L„ Siete teorías de ¡a naturaleza humana, trad. E. Ibá-
ñez, Madrid, Cátedra, Teorema, 1984.

30. Saussure

F I L O S O F Í A D E LA E X P R E S I V I D A D
O C I E N C I A D E LOS S I G N O S

(...) Cuando esté organizada la semiología deberá pre-


guntarse si los modos de expresión basados en signos en-
teramente naturales —como la pantomima— le corres-

219
ponden por derecho. Suponiendo que los acoja, su objeto
principal dejará de ser el conjunto de los sistemas funda-
mentados en lo arbitrario del signo. En efecto, todo me-
dio de expresión recibido en una sociedad descansa, en
principio, en un hábito colectivo o, lo que viene a ser lo
mismo, en la convención. Los signos de cortesía, por
ejemplo, a menudo dotados de una cierta expresividad
natural (pensemos en el chino que saluda a su emperador
postrándose ante él nueve veces), no están menos fijados
por una regla, siendo esta regla la que obliga a emplearlos
y no su valor intrínseco. Podemos, pues, decir que los sig-
nos enteramente arbitrarios realizan mejor que los otros
el ideal del procedimiento semiológico; es por ello que el
lenguaje, el sistema de expresión más complejo y difundi-
do, es también el más característico de todos. En este sen-
tido, la lingüística puede llegar a ser patrón general de
toda posible semiología aunque el lenguaje no sea más
que un sistema particular.
Nos hemos servido de la palabra símbolo para designar
el signo lingüístico, o, más exactamente, a aquello que
denominamos significante. A causa, precisamente, de
nuestro primer principio, existen inconvenientes a la
hora de admitir lo anterior. El signo tiene por carácter el
no ser nunca del todo arbitrario; no está vacío, y hay un
rudimento de unión natural entre el significante y el sig-
nificado. El símbolo de la justicia, la balanza, no podría
ser reemplazado por cualquier otra cosa, un carro, por
ejemplo.
La palabra arbitrario requiere, asimismo, una puntuali-
zación. No debe dar la idea de que el significante depende
de la libre elección del sujeto parlante... queremos decir
que no está motivado; esto es, que es arbitrario con respecto
al significado, al cual nada le ata en la realidad.

Curso de lingüistica general (1906-1911), Primeta


parte, parágrafo 2, §2.

220
COMENTARIO

1. En muchos sentidos, este es un texto inaugural,


pues constituye el punto de partida metodológico de una
lingüística donde se plantea, a la vez, su principio y con-
cepto fundamental (lo arbitrario del signo, concebido
como simbólico e inmotivado). Asimismo, éste se halla
en el punto de partida del ulterior destino de la lingüisti-
ca de Saussure y de las aplicaciones de las que ha podido
ser objeto (el «estructuralismo»), como también de las crí-
ticas que se han venido sucediendo desde su formulación.
Saussure precisa el primer principio de una lingüística,
justifica la elección del término que designa al concepto,
desecha una interpretación falsa de lo arbitrario que lo
define. Tal es el plan general del texto, articulado en tres
párrafos. Sin embargo, también sitúa, de paso, el interés
de la disciplina como paradigma de toda semiología, sir-
viéndose para ello del principio expuesto como criterio
de demarcación entre aquello que surge de una ciencia de
los sistemas de expresión y aquello que entraría en una fi-
losofía de la expresividad.
2. Es necesario estar atento al planteamiento del pro-
blema inicial: Saussure trata, en el conjunto de su obra, de
la «lingüística general», cuyo primer principio define en
la forma de la arbitrariedad del signo. Pero, en este pun-
to, el texto comienza por una «profecía» que posee el va-
lor de definición para una ciencia más amplia, la semio-
logía como ciencia de todos los sistemas de signos; desde
el momento en que esté organizada deberá enfrentarse a
un problema de derecho: ¿qué modos de expresión en-
tran dentro de su campo? La cuestión sólo puede encon-
trar una solución en el recurso a un principio que se halle
validado por este hecho. Saussure plantea una alternati-
va: o bien cada modo de expresión está basado en signos
enteramente naturales y será, así, excluido de una posible
semiología, o bien será incluido en ella porque se reco-
nozca paradójicamente (caso de los signos de cortesía) lo

221
arbitrario del signo. Para terminar (última parte del pri-
mer párrafo), los equívocos señalados destacan, por con-
traste, el papel privilegiado de la lingüística, realización
del ideal del procedimiento semiológico, patrón de toda
semiología.
El examen de los modos de expresión no lingüísticos
hacen de piedra de toque para desgajar del signo lo arbi-
trario y disipar el mito de su naturalidad1. El autor toma,
uno tras otro, dos ejemplos, la pantomima y los signos de
cortesía; es decir, el ritual de la etiqueta. Dotados de una
fuerte expresividad, de una aceptación «sin fronteras» (el
mimo se encuentra en «todas las lenguas»), estos modos
parecen interpretar la falta de lenguaje como señal de su
carácter natural, cuando, más bien, sería artificial o pro-
ducto de una convención. La refutación consiste en sepa-
rar la parte de convención inherente no tanto a la expre-
sividad misma como a los signos recibidos como o, inclu-
so, transformados en obligatorios, SÍ el arte es libre (el
caso del mimo lo atestigua), habremos de admitir que,
para ser aceptado en una sociedad y en una sola, no puede
formar parte de la categoría de la libertad un modo de ex-
presión que pueda parecer tan natural como la gestual
mímica o el ritual que hace postrarse al súbdito nueve ve-
ces delante de su emperador en señal de humildad, y ello
porque obliga como obligaría una regla, pues nada hay
menos gratuito que la etiqueta. Ahora bien, es en virtud
de una regla aceptada por convención que el emperador
de China debe ser saludado del modo expuesto, aun, por
ejemplo, cuando se trate de diplomáticos extranjeros,
pero no porque un gesto tan expresivo revista algún sig-
nificado universal. La conclusión es, pues, que lo que
convierte en regular y obligatorio a un sistema de signos
no depende de su naturaleza intrínseca, sino de una con-
vención cuya particular importancia será objeto de análi-

1 Cfr. CratUo, de Platón, donde se enfrentan la tesis de Hermógenes,

quien supone que los nombres son fruto de una convención, y la tesis de
Cratilo, quien sostiene que los nombres, por naturaleza, son los apropia-
dos para las cosas.

222
sis en el tercer párrafo. La conclusión de este breve exa-
men es triple y concierne a tres niveles de análisis de ge-
neralidad creciente, extendiéndose desde los signos a la
lengua, a la lingüística que hace de ella su objeto de estu-
dio y, finalmente, a la entera semiología.
Existe un «procedimiento semiológico» ideal cuyos di-
ferentes signos son realizaciones más o menos perfectas.
Saussure no afirma que no hay signos naturales, sino más
bien al contrario, aunque la expresividad escape en prin-
cipio a la semiología como ciencia. Considera, sin embar-
go, que los que son enteramente arbitrarios poseen un in-
terés epistemológico especial, el de proporcionar a la se-
miología un patrón, paradigma en términos actuales, que
le permita tratar los sistemas de signos no lingüísticos se-
gún el mismo procedimiento ya probado en el caso del
lenguaje, sin manifestar con ello lo arbitrario del signo.
El tiempo ha dado la razón a nuestro autor, puesto que se
ha intentado tratar como «sistemas de signos» convencio-
nales cosas tan dispares como la natación, la marcha y los
gestos corporales (M. Mauss), la moda (R.'Barthes) o los
objetos domésticos (J. Baudrillard)2.
3. El segundo punto se refiere a la terminología
adoptada en la lingüística a la hora de designar la uní dad
de base, el signo: es preciso justificar su conformidad con
el principio de su arbitrariedad. Ahora bien, el término
de símbolo lingüístico más tradicional plantea el mismo
problema que la comparación realizada entre los signos
expresivos naturales y los arbitrarios. Solamente es arbi-
trario el signo; el símbolo no lo es. Tal es la razón por la
cual Saussure ha descartado el uso de esta palabra y rebau-
tizado con el nombre de signo la unidad de base de la lin-
güística.
En la página anterior hemos introducido tres términos
complementarios con la intención de definir el objeto
2 Cfr. M. Mauss, Sociokgm y antropología, Prólogo de Lévi-Strauss, trad.

Teresa Rubio de Martin y Retortilío, Madrid, Tecnos, 1971, págs. 337-


359. R. Barthes, Le ¡ysteme de ta mode, París, Seuil, 1967. j . Baudrillard,
El sistema dt ¿os objetas, trad. F. González Arambuto, México, Siglo
X X I , 1978,

223
elemental de la lingüistica, la «entidad psíquica de dos ca-
ras» que constituye el átomo de un sistema. Todo tiende a
la articulación de esas dos caras, a la unidad paradójica
que condiciona su distinción de principio. Este segundo
párrafo retoma la cuestión estableciendo que en el símbo-
lo existe un rudimento de unión natural entre el signifi-
cante y el significado. Esa unión natural significa identi-
dad posible (al menos, semejanza), aunque sea necesario
distinguir, por naturaleza y por principio, los dos ele-
mentos de la entidad de dos caras. No hay nada común a
una «imagen lingüistica», o vocal, y a un concepto que se
le asocia por medio de una convención, «Entender», en
sentido físico, y «entender», en sentido intelectual, son
dos cosas diferentes que no se pueden confundir sin di-
solver, al mismo tiempo, la posibilidad de una ciencia de
los sistemas de signos*. Ahora bien, el símbolo aproxima
e identifica aquello que permite que entendamos y aque-
llo que está dado a entender, el significante y el significa-
do. Una balanza se asemeja más a la idea de «justicia» de lo
que ella permite a juzgar por su solo aspecto, «pesa» en el
proceso del significado, por tanto, al hacernos adoptar
una imagen para una entidad intelectual carente de la
misma. Ser reemplazable por cualquier cosa se transfor-
ma en criterio de la pureza del signo, no siendo otra cosa
que la convención que une ese cualquier cosa a un signi-
ficado que hace las veces de procedimiento semiológico
puro, abierto a una ciencia.
4. Una interpretación psicológica engañosa podría
correr el riesgo de aferrarse a la noción mencionada de lo
arbitrario. Tradicionalmente, el término designa una li-
bertad desenfrenada, no sometida a leyes. Ahora bien, si

* La polisemia de la palabra francesa «entendre» es mucho más níti-


da en francés, y equivale a nuestro «oir» (en el sentido físico menciona-
do) y al «entender» (en sentido intelectual), coincidiendo en esta última
acepción con nuestro idioma. No obstante, el lector podrá recurrir, en el
idioma castellano, a ejemplos tales como en el caso de pedir a alguien
que repita algo que «no hemos entendido» porque «no lo hemos oído
bien». Aunque algo forzado, el ejemplo puede bastar para hacernos una
idea de to que el autor pretende decir. ¡N. dtl T, j

224
no hay nada capaz, por naturaleza, de hacer que un signo
esté representado por un significante dado, símbolo, vo-
cablo, grafismo, ¿se puede hablar de libertad?, ¿tiene al-
guna posibilidad el hablante de expresarse sin leyes y de
redefinir los términos de la lengua?
Aceptar lo anterior sería comprender mal el lazo que
une entre sí a los elementos del signo; se trata de una con-
vención en el sentido de regla obligatoria, lo que excluye
la libre elección y la modificación a voluntad. Elegir de-
signar esto mediante aquello es preferir una palabra de-
terminada para una idea o concepto, efectuar una compa-
ración y un ensayo. Pero no hay, precisamente, ninguna
razón para pronunciarse en favor de una elección en par-
ticular, ningún motivo que nos autorice a representar el
significado tal y como aparece en la lengua. Dicho de
otro modo, arbitrario significa inmotivado; es decir, pri-
mero, anterior a toda deliberación. En el caso del signifi-
cante, sería una flaqueza epistemológica buscar su funda-
mento en la naturaleza o en la razón a la hora de unirse a
un significado. Es por ello que el signo se ofrece como
objeto para una lingüística, y no como pretexto para una
libertad.

PARA SABER MÁS

Sobre Saiíssurt

GADET, F., Saussure. Une Science de lalangue, París, PUF, Col. «Phi-
losophies», núm. 31.
RLVIKRB, P., Lingüistica y nueva cultura, trad. E. Rodríguez, Ma-
drid, Maro va, 1974.

225
31. Durkheim
Q U E E L D E T E R M I N I S M O SOCIAL GARANTIZA
LA EFICACIA SOCIOLÓGICA

Fue solamente a principios del siglo xix, con Saínt-


Simon en primer lugar y, sobre todo, su discípulo Augus-
te Comte, que vio la luz, definitivamente, una concep-
ción nueva. Procediendo, en su Curso defilosofíapositiva, a
una revisión sintética de las ciencias consolidadas de su
tiempo, constató que todas ellas se basaban en el princi-
pio determinista; ésto es, en el axioma según el cual los
hechos que estudian son puestos en mutua conexión si-
guiendo leyes necesarias. De lo anterior concluía que este
principio, que había sido así verificado en los demás rei-
nos de la naturaleza, desde el reino de las magnitudes ma-
temáticas hasta el de la vida, debía ser igualmente verda-
dero en el reino social. Las resistencias mismas que se
oponen hoy en día a esta nueva extensión de la idea de-
terminista no deben detener al filósofo, pues se han veni-
do produciendo con regularidad cada vez que ha surgido
el problema de difundir a un nuevo reino este fundamen-
tal postulado, y siempre han sido vencidas. Hubo un
tiempo en que se lo rechazaba aún en el mundo de los
cuerpos brutos; pero se estableció finalmente allí. A con-
tinuación, fue negado en el caso de los seres vivos y pen-
santes; en la actualidad ya es algo incontestable.
Podemos, pues, estar seguros de que los mismos prejui-
cios contra los cuales viene a chocar cuando se trata de
aplicarlo al mundo social, no durarán más que cierto
tiempo. Por otra parte, y debido a que Comte planteaba
como una verdad evidente —verdad que, por lo demás,
nadie pone hoy en duda— que la vida mental del indivi-
duo está sometida a leyes necesarias, cabría preguntarse
cómo es que las acciones y reacciones en intercambio en-
tre las consciencias individuales, cuando se asocian entre
sí, no están sometidas a la misma necesidad.
Según este punto de vista, las sociedades dejan de pre-
sentar la apariencia de una especie de materia maleable y

226
plástica que los hombres pueden, por así decir, modelar a
voluntad; sería preciso, en adelante, ver en ellas realida-
des a través de las cuales la naturaleza se nos impone y
que no pueden ser modificadas, al igual que todas las co-
sas naturales, más que conforme a las leyes naturales que
reinan sobre ellas. Las instituciones de los pueblos no po-
dían seguir siendo consideradas como producto de la vo-
luntad, más o menos ilustrada, de los hombres de estado,
de los legisladores, sino como resultantes necesarias de
causas determinadas que las implican físicamente. Dada
la manera en que se compone un pueblo en un momento
de su historia, el estado de su civilización en esa misma
época, de ello resulta una organización social, caracteri-
zada de tal o tal forma, análogamente al modo como las
propiedades de un cuerpo resultan de su constitución
molecular. Nos encontramos, pues, frente a un orden de
cosas estable, inmutable, y, así, se hace a la vez posible y
necesaria una ciencia pura susceptible de ser descrita y
explicada, de la cual podamos decir cuáles son sus carac-
teres y de qué causas dependen (,..).
Las ciencias, al mismo tiempo que proclaman la nece-
sidad de las cosas, ponen en nuestras manos los me-
dios adecuados para dominarlas. Comte señala con insis-
tencia, incluso, que, de todos los fenómenos naturales,
los fenómenos sociales son los más maleables, los más ac-
cesibles a las variaciones, a los cambios, y ello porque
también son los más complejos. La sociología no impone
en absoluto al hombre una actitud pasivamente conser-
vadora, sino que, al contrario, amplía el campo de nues-
tra acción sólo en tanto que amplia el campo de nuestra
ciencia. Ella nos hace apartar nuestra mirada únicamente
de las empresas irreflexivas y estériles, inspiradas por la
creencia en que nos es posible cambiar, del modo como
queramos, el orden social, sin tener en cuenta los hábitos,
las tradiciones de la constitución del hombre y de las so-
ciedades.

«Sociología y ciencias sociales», en De!método en


¡as (¡enrías.

227
COMENTARIO

1. Gracias a sus tres grandes obras de finales del si-


glo xix (De ¡a división del trabajo social, 1893; Las reglas del mé-
todo sociológico, 1895; El suicidio. Estudio de sociología, 1897),E.
Durkheim se impuso como el teórico de una ciencia en
posesión de su método y sus objetivos, como el iniciador
de una auténtica escuela de pensamiento (recordemos a
algunos de sus alumnos y colaboradores, Simiand y
Mauss, por ejemplo). Sin embargo, nuestro texto muestra
claramente que este fundador es un heredero. En efecto,
al invocar su parentesco intelectual con Comte, ¿acaso no
está buscando Durkheim protegerse tras la autoridad de
un maestro? ¿Intenta reconocer una deuda o bien, más
justa y profundamente, en tanto que discípulo él mismo
del discípulo de Saint-Simon, quiere dejar patente que la
sociología no puede existir sin la memoria de su historia
—al igual que la filosofía a la que, sin embargo, se opone,
aun cuando la ciencia social todavía estaba, con Comte,
sometida a la síntesis filosófica? Volver a situarse en el
momento continuado de su nacimiento sería, para la so-
ciología, el mejor medio de redefinir la originalidad de su
singladura y, si se nos permite decirlo, su «personalidad»
propia, y ello en la medida en que, como ya vimos en el
caso de Comte1 y ya veremos en el de Durkheim, se ocu-
pa y también preocupa, a la manera y al lado de la filoso-
fía y la psicología, de la vida del Espíritu. Pero quizás se
trate igualmente, para la sociología, de hacer surgir una
tradición al reinscribirse en ella, al aplicarse a sí misma
reglas constitutivas de sus objetos, confiriéndose la reali-
dad y objetividad de todo fenómeno social.
2. Dentro de los límites de sus marcas históricas dis-
tintivas, el presente extracto es un buen ejemplo de la

1 Ver texto 22 y su comentario.

228
conciencia que de sí misma posee (de su vocación y de su
definición) la sociología hecha científica del mismo modo
que las demás ciencias de ta naturaleza. Aun cuando la
existencia de un reino de magnitudes matemáticas nos prohiba
interpretar los reinos de la naturaleza de forma ingenua-
mente «naturalista», todo prueba, en el texto, que el reino
social forma parte de la naturaleza (una parte que todavía
está relativamente inexplorada); y que la posibilidad y la
necesidad de la sociología en cuanto ciencia pura (pura-
mente teórica en su autonomía propia) proceden, junto
con la extensión al dominio de las ciencias sociales, del
principio que gobierna la racionalidad de la física (en
sentido lato), el principio del determinismo.
a) La primera parte del texto (—*• . ..verdadero en el reino so- so-
cial) rememora, pues, la innovación comtiana. Podemos
afirmar que Durkheím descubre aquí una constatación,
una convicción, y la fuerza de una inferencia lógica; sólo
hay ciencia allí donde se da una conexión de los hechos si-
guiendo relaciones necesarias (ésta es la definición del
determinismo, que nos impide confundirlo con el princi-
pio de la causalidad y todas sus implicaciones de carácter
metafísico). Cada ciencia extrae su eficacia y valor del
axioma mencionado; en una naturaleza única y universal,
lo que se verifica en cualquiera de sus regiones debe po-
der verificarse en el resto.
b) En la segunda parte nos sitúa en la historia (Las resis-
tencias mismas... ~*• ...la misma necesidad). Durkheim recuerda
las resistencias que ha debido vencer el principio del de-
terminismo para imponerse en el mundo de los cuerpos
brutos (¿alusión a las reacciones suscitadas por la revolu-
ción galileana?), en primer lugar y, a continuación, el rei-
no de lo viviente (pensemos, en este punto, en la obra de
Claude Bernard). No obstante, partiendo de la regularidad
de aparición de tales resistencias hasta la de su desapari-
ción podemos extraer, sociológicamente, el pronóstico, razo-
nable porque está razonado (Podemos, pues, estar seguros,...),
de la victoria de la sociología sobre los prejuicios que fre-
nan su desarrollo. Pero esto prueba también que el pro-
greso de una ciencia no puede reducirse a la rearticula-

229
ción puramente formal de su contenido lógico, como
tampoco a la acumulación de su saber. La «conquista» de
la verdad pasa por una lucha de ideas que la sociología apre-
hende, precisamente, en el seno de unas relaciones de fuer-
zas. Toda novedad, en cierto sentido, es subversiva, debi-
do a que altera el orden establecido, orden que, de forma
inevitable, opone una resistencia. Y, aunque el desenlace
pueda ser previsto (no por el destino de lo universal, sino
por la ley que es posible obtener de una regla), tendrá que
pasar, sin embargo, por una lucha entablada entre fuerzas
que se hallan enfrentadas. Desde este punto de vista cabe
afirmar que toda epistemología necesita una sociología
del conocimiento —que Durkheim aplica aquí a la mis-
ma sociología.
c) En este caso, pues, la novedad histórica consiste
en un nuevo objeto. Con el tercer párrafo f—* ... dependen)
da comienzo la tercera parte de nuestro texto, la más espe-
cíficamente epistemológica. Debemos entender ese nue-
vo objeto en dos sentidos: como un objeto más en el
marco de la objetividad científica, y como un objeto en sí
mismo diferente, renovado en cuanto a su concepción,
calificado de distinto modo por el hecho de hallarse «des-
calificadas» todas las representaciones anteriores. Las so-
ciedades serán, en adelante, realidades a través de las cua-
les la naturaleza se nos impone con las leyes que las rigen;
tales son las cosas que hay que caracterizar a partir de sus
causas. Este es un punto capital en la teoría sociológica
—ya volveremos sobre ello. Por el momento añadiremos
solamente que, al someter la existencia enigmática de las
sociedades (porque están instituidas en tanto que hayan
sido constituidas) a la observación, descripción y explica-
ción autorizadas por el principio determinista (aún no
había llegado, para las ciencias físicas, el momento de la
crisis), Durkheim sustrae esas mismas sociedades a las in-
terpretaciones «subjetivistas» (vol un taris tas o artificialis-
tas) y a todo lo que entrañan de tautológico, de arbitrario
o de irracional. Hemos de destacar también la concatena-
ción de causa a efecto que viene generada por la propia
teoría: de aquello de lo que la organización social será, en

230
adelante, un resultado, resulta, por su parte, la ciencia pura de
la sociología.
d) Así, para terminar, surge la necesidad del poder de
la sociología (último párrafo). La convicción racionalista
según la cual lo propio de las ciencias no es otra cosa que
la de saber transformar la necesidad de modo que favo-
rezca y beneficie al hombre, mediante los resultados que
le asegura la comprensión que de ella posee (convicción
triunfadora en nuestro fin de siglo, hasta culminar en el
«cientismo» —¿pecaremos de sociólogos si afirmamos
que, en este caso, Durkheim no escapa a ese mismo cien-
tismo?), se corresponde con la afirmación de la capacidad
de acción de la sociología, y ello en razón de la complejidad
de los hechos sociales (hay, además relaciones «en juego»,
como también puede haber «juego» en las relaciones, jue-
go que favorece la variación, idea ésta que la biología
post-darwiniana no ha cesado de verificar en sus propios
objetos). Frente a los conservadores y sus argumentos, ex-
traídos de los determinismos sociales y elaborados con la
finalidad de predicar el inmovilismo (y sostener que no
sabríamos cómo actuar con los desórdenes sociales),
Durkheim preserva, en su teoría sociológica, todas las ra-
zones y posibilidades de su eficacia práctica. Pero, frente a
los «activistas» que se imaginan que basta un decreto de la
voluntad (para cambiar el mundo, o la vida), recuerda
que hay que saber contar con la necesidad.
3. Este aspecto práctico de la sociología durkheimiana
invita a volver sobre un punto que, tal y como aparece en
nuestro texto, podría causar alguna dificultad. Nos referi-
mos a la cuestión de si los fenómenos sociales son o no
son maleables. La respuesta es afirmativa si nos remitimos
a lo que acabamos de concluir más arriba; ésto es, que son
los más maleables, aun teniendo en cuenta la totalidad de
los fenómenos naturales, y ello por una serie de razones
que ya avanzó en su momento el propio Comte. Por otra
parte, sin embargo, los hábitos, las tradiciones, la consti-
tución mental de los hombres y de las sociedades, que con
tanta fuerza resisten a los cambios y que no podemos mo-
delar a voluntad, ¿acaso no nos obligan a rechazar esa in-

231
definida maleabilidady plasticidad de la materia social? Para ver
con más claridad lo que acabamos de señalar convendrá
reconsiderar la opinión determinista que objetiva los he-
chos sociales como si fueran cosas. El propio Durkheim ha
explicado esta cuestión a lo largo de numerosas puntualí-
zaciones y revisiones. No cabe hablar, según él, de la exis-
tencia de un prejuicio «realista» que pudiera convertir en
entidades «dadas desde el exterior» las relaciones sociales,
dotadas de una realidad física, material, sometidas a la
atracción o a la repulsión como los cuerpos de la natura-
leza... «No afirmamos, en efecto, que los hechos sociales
son cosas materiales, sino que son cosas análogamente a
las cosas materiales, si bien de distinto modo.» La reivin-
dicación es de orden epistemológico, pues alude, en con-
tra de las ilusiones de la introspección y de la tentación de
inferir el sentido de las realidades sociales a partir de la in-
tuición del «yo», a la necesidad de cambiar de perspecti-
va, de hacer que el espíritu salga dé sí mismo, «por vía de
observaciones y experimentos», con la finalidad de diri-
girnos desde el exterior hacia el interior del objeto. Se tra-
ta, en suma, de cambiar de «actitud mental»: «Que el so-
ciólogo se sitúe en el estado de espíritu donde se encuen-
tran físicos, químicos y fisiólogos cuando se adentran en
una región todavía sin explorar de un dominio científi-
co»2. Tal es el precio que se debe pagar para que las reali-
dades sociales, por su parte, puedan ser cambiadas; no co-
metiendo el error de no ver la necesidad que reina sobre
ellas como hay que verla y allí donde hay que verla, nece-
sidad que es, a la vez, constreñimiento y razón.
4. Quizás ya estremos en situación de comprender
mejor cómo es preciso entender el reino social. Sus propie-
dades no son derivables de las de la materia como tal, en
una naturaleza que, en sí misma, sería reductible a una
pura materialidad. La sociología de Durkheim no tiene
nada que ver con algún tipo de «monismo materialista».

2 Todas estas citas proceden de Ja obra Las regias del método sociológico y

otros escritos de ftksofia de las ciencias sociales, trad. S. González Noriega, Ma-
drid, Alianza, 1988.

232
Pero tampoco es una psicología generalizada, pues el rei-
no social no procede de la extensión, dentro de una asocia-
ción, de la vida mental del individuo. El esfuerzo llevado a
cabo por Durkheim se orienta al establecimiento, del
modo más riguroso posible, de la autonomía metodológi-
ca y científica de la «física social», aunque sin las miras ni
el optimismo militantes de su fundador (Comte), conser-
vando, sin embargo, la «naturaleza» espiritual (la socie-
dad es «ante todo, un conjunto de ideas», según pensa-
ba Durkheím)1 de sus dos objetos principales, objetos que
Durkheim ha privilegiado en todo momento a lo largo de
su investigación, y que no son otros que el hecho moral y
el hecho religioso. Asimismo, conviene no equivocarnos
al juzgar el sentido del modelo de inteligibilidad que
nuestro autor propone cuando se trata de comprender
cómo y de qué resulta la organización social. Hemos de
advertir que no hay que tomar la analogía de la constitu-
ción molecular por un paradigma de mecanismo «ato-
mista». En efecto, la síntesis química es, precisamente, la
emergencia, en la naturaleza, de un todo cuyas propieda-
des no provienen de las de sus partes. De igual modo, la
esencia colectiva de los hechos sociales no deriva de las rea-
lidades individuales que la «componen». Durkheim reco-
gió, en efecto, la idea de que «hay más en el todo que en la
suma de las partes»; hay, pues, una especificidad irreduc-
tible del objeto sociológico que impide la posibilidad de
deducir, como hizo Spencer, lo superior a partir de lo in-
ferior.

PARA SABER MAS

Sobre Durkheim

ARON, R., Las etapas delpensamiento sociológico, vol. 2 (Marx, Durk-


heim, Weber), trad. A. Leal, Buenos Aires, Siglo X X , 1980,
2 vols.

3 Ver C. Bougíí, Prefacio a E. Durkheim, Soríologie tipbihsophie, París,

F. Alean, 1924; PUF, 1951.

233
GIDDENS, A., El capitalismo y la moderna teoría social, trad. A . Boix
Duch, Barcelona, Labor, 1977.
LUKHS, S., Emile Durkheim, su vida y su obra, trad. A. Car din
e I. Martínez, Madrid, Siglo XXI, 1984.

32. Weber

Q U E LAS SINGULARIDADES
DE LA HISTORIA REQUIEREN TIPOS IDEALES

Cualquier examen atento de los elementos conceptua-


les de una exposición histórica pone de manifiesto que la
historia, desde el momento en que busca elevarse por en-
cima de la mera constatación de relaciones concretas
para determinar el significado cultural de un acontecimien-
to singular, por simple que este sea, trabaja y debe trabajar
con conceptos que, en general, no es posible precisar de
manera rigurosa y unívoca más que en la forma de tipos
ideales.
En efecto, ¿cómo precisar el contenido de conceptos
tales como los de «individualismo», «imperialismo», «feu-
dalismo», «mercantilismo», «convencional» y de otras in-
numerables construcciones conceptuales de este género
que utilizamos para intentar dominar la realidad median-
te el pensamiento y la comprensión? ¿A través de la des-
cripción «sin presuposición» de una manifestación concre-
ta aislada cualquiera, o bien, al contrario, a través de la
síntesis abstractiva de lo que es común a varios fenómenos
concretos? El lenguaje del historiador contiene centena-
res de palabras que comportan cuadros de pensamiento
similares aunque imprecisos, porque han sido elegidos
según las necesidades de expresión propias del vocabula-
rio corriente, no elaborado por la reflexión, cuyo signifi-
cado, sin embargo, es posible demostrar concretamente
sin que sean pensados con claridad. En gran número de
casos, sobre todo en la historia política narrativa, la im-
precisión del contenido de sus conceptos no perjudica en

234
nada a la claridad de la exposición. Es suficiente, enton-
ces, que sintamos en los casos particulares aquello que el
historiador ha creído ver; aún más, podemos contentar-
nos con una precisión particular del contenido conceptual
de importancia relativa en un caso particular que se pre-
sente al espíritu como habiendo sido pensada. No obstan-
te, en el caso de que sea necesario tomar claramente cons-
ciencia, con mayor rigor, del significado de un fenómeno
cultural, urge la necesidad de operar con conceptos claros
y precisados no solamente bajo uno, sino bajo todos los
aspectos particulares. Es evidente lo absurdo de querer
dar una «definición» de tales síntesis del pensamiento his-
tórico «según el esquema»: genusproximum et differeniia spe-
cifica; no hay más que hacer la prueba. Esta última manera
de establecer el significado de las palabras tan sólo se en-
cuentra en las disciplinas dogmáticas que hacen uso de
los silogismos. (...) Cuando uno se propone dar una defi-
nición genética del contenido de un concepto, no queda
otra forma que la del tipo ideal, en el sentido indicado
más arriba. El tipo ideal es un cuadro de pensamiento, no
es la realidad histórica, ni, sobre todo, la realidad «autén-
tica», y sirve aún menos como esquema donde ordenar la
realidad a título de ejemplar. Su significado no es otro que
el de concepto limite puramente ideal con respecto al cual se
mide la realidad para clarificar el contenido empírico de
algunos de sus elementos importantes, y con el cual se la
compara. Estos conceptos son imágenes en las que cons-
truimos relaciones, mediante el empleo de la categoría de
posibilidad objetiva, que nuestra imaginación, formada y
orientada conforme a realidad, juzga como adecuadas.

«Ensayos sobre la teoría de la ciencia» (1922).

COMENTARIO

1. El presente texto posee un contenido o interés es-


pecíficamente epistemológicos (como lo indica el título
de la obra de donde procede). Sin embargo, hemos de

235
añadir, a continuación, que esta especificidad aún debe
ser especificada, puesto que no nos encontramos ante una
reflexión acerca de la cientificidad en general, de las con-
diciones que debe satisfacer todo discurso con pretensio-
nes de ser científico, sino ante problemas muy particula-
res planteados por las tiendas humanas y localizados, más
especialmente, en el plano de la inteligencia de su objeto.
Sea cual sea la manera como se la recorte, la «realidad hu-
mana» es, para las «ciencias del hombre», esencia cultural. Si
la cultura está viva, si es, al igual que la propia vida, un
flujo inagotable de fuerzas que se renuevan, que, por de-
cirlo con otras palabras, se reproducen sin repetirse,
¿dónde y cómo localizar las regularidades de las que todas
las ciencias hacen su materia? Si, en la complejidad siem-
pre original, cada hecho de la cultura es una singularidad
que procede de un contexto móvil y cambiante, ¿no será
necesario proponerse aprehender su significado? Pero, con-
forme a la exigencia científica ¿qué medio(s) permitirá(n)
objetivarlo? Estas cuestiones comprenden dificultades
muy concretas sobre las cuales volveremos más adelante.
Lo esencial, sin embargo, es la solución sugerida por Max
Weber, a saber, el tipo ideal cuya teoría estamos esbozan-
do aquí. Nuestro texto puede ser considerado como una
presentación y una justificación de este «útil» metodoló-
gico. Este es considerado en relación con la práctica ordi-
naria de los historiadores, aunque, en el caso de los soció-
logos, suplanteamientoseexpondríaentérmínossimilares.
En la obra del propio Weber, es difícil disociar lo so-
ciológico de lo histórico, sea ello lo que fuere.
2. El primer párrafo introduce, pues, la noción de
«idéaltype» (neologismo procedente del alemán que po-
dría haber sido traducido por tipo ideal, a no ser porque
se intentaba evitar un posible contrasentido con respecto
al significado de «ideal»; esto se verá más claramente en la
conclusión)*. Podemos incluso encontrar en el texto una

* Hemos transcrito la palabra francesa «idéaltype» tal y como apare-


ce en el original. En las traducciones al castellano de las obras de Weber,
al contrario que en las francesas, no se ha respetado, por lo general, el

236
primera definición del mismo: el tipo ideal es la forma
que adopta el concepto en las exposiciones históricas que
pretenden determinar, y no sólo constatar. El tipo ideal es,
pues, en sí mismo, una determinación rigurosa y unívoca
(lo contrario de equívoca) de la forma conceptual, cuya
precisión viene exigida por la particularidad de su objeto,
el acontecimiento. Ahí comienza el embarazo de la epis-
temología; esto es, ¿cómo puede la teoría hacerse cargo
del acontecimiento, si su particularidad consiste en ser
singular,? Para medir el alcance del problema, habremos de
situarlo en un contexto histórico. El modelo de toda
ciencia humana es, una vez establecidas sus bases por Au-
guste Comte, la sociología; pero, en su definición inaugu-
ral, que es, por consiguiente, positivista, su tarea es la de
formular las leyes de la estática y dinámica sociales, sin te-
ner por ello que buscar sus causas; es decir, se ocupa del
«cómo» sin preocuparse de «por qué». No obstante, lo
propio de la ley es apartar, en su generalidad, todo lo que
pueda haber de individualizante en el proceso del cual resul-
ta el acontecimiento; o, más aún, indiferenciar su cuali-
dad, eso que precisamente la distingue como tal, por la
medida y la cuantificación que le permiten establecer la
constancia y regularidad de las relaciones de las cuales
aquélla es expresión.
En suma, ¿cómo subsumir bajo una regla general el as-
pecto «cualitativo» de la singularidad? En la «materia»
histórica, sólo puede interesar al historiador, tal es la
convicción de Weber, el sentido de lo que podríamos lla-
mar sin titubeos «único en su género»; pero no lo apre-
hende (lo que lo demuestra el examen atento de la singla-
dura de esta disciplina) más que al caracterizarlo, al formar
el sistema de sus signos distintivos. A la «unicidad» del
acontecimiento le debe corresponder, como señal inteli-
gible, la univocidad de su representación intelectual (ideal).
El problema metodológico por resolver se puede resu-

neologismo indicado, por lo que lo más usual es que nos encontremos


con la expresión «tipo ideal», expresión que no impide el riesgo mencio-
nado en el texto. \N, del T.)

237
mir, pues, en los términos siguientes: los fenómenos cul-
turales, a diferencia de los naturales, extraen su sentido
de su originalidad, no de su conformidad con una ley; hacen
surgir, de este modo, ciencias que son necesariamente in-
terpretativas, siendo necesario, si se trata, en efecto, de
ciencias, conferir la mayor objetividad posible a sus inter-
pretaciones. Ello exige todo un trabajo (si hacemos nuestro el
sentido de las palabras de nuestro autor, la disciplina his-
tórica es un hecho cultural), y el objetivo de Weber es el
de restituirle su significado. En el fondo, Weber, en
cuanto epistemólogo, se aplica a reflexionar sobre la his-
toria-ciencia como el historiador a pensar sobre la histo-
ria y los acontecimientos, elaborando para ello el útil de
una determinación conceptual específica (en relación
con la especificidad cultural del objeto) y caracterizando
su significado en términos de un ideal típico.
3. Como ilustración de lo anterior, pondremos algu-
nos ejemplos. El segundo párrafo nos presenta una muestra
de las construcciones conceptuales de las que el historia-
dor se sirve para identificar sus objetos. En este punto, se
abre (segundaparte de nuestro texto: En efecto, ¿cómo ~~* ...como
habiendo sido pensada) una interrogación fundamental que
se dirige sobre aquello que se debe considerar como el
nudo gordiano de todo el asunto: ¿qué es conceptuaiizart
Cuestión epistemológica, en efecto, central y general,
porque ninguna ciencia puede dispensarse de plantearla
por su propia cuenta, y ello desde el momento en que se
inquieta por su propia singladura como ciencia, por la
validez de su trabajo teórico, por la especificidad, por la
coherencia y legitimidad de su «dominio». Hemos de ha-
cer notar que, antes de abordar la conceptualización des-
de el aspecto de su método, Weber la define por su finalidad.
El objetivo del concepto es, por consiguiente, el de domi-
nar la realidad mediante elpensamiento; una apuesta sometida a
condiciones, pero también expuesta al fracaso. De igual
modo, cabe señalar la existencia de otra determinación
del objetivo mencionado, determinación que, siendo
anodina en apariencia, es de importancia capital: se trata
de comprender; no solamente de explicar por la (o las) cau-

238
sa(s) próxima(s), sino de reunir en un todo significativo
el conjunto de los signos que contribuyen a «dar sentido»,
significando, en primer lugar, el fin en relación con el
cual se ordenan tales signos. Por otra parte, al comenzar
por la finalidad del concepto, Weber ilustra, lo más radi-
calmente posible, el sentido de la comprensión, estable-
ciéndolo a propósito del útil que da origen a la misma. Y
esto reviste una importancia capital porque a todas las
ciencias interpretativas concierne, en el esfuerzo orienta-
do hacia la comprensión, ese estatus del concepto que
acabamos de señalar. Pero también es importante porque
la «sociología weberiana» asigna con ello a las ciencias
una vía diferente (en este punto, nuestro autor se revela
heredero de Dilthey, Rickert y Simmel), distinguiéndose
así del espíritu positivista al oponerse a él.
Una vez precisado lo anterior, aún resta otra cuestión,
la de cómo conceptual izar. En efecto, ¿podemos saber
qué es lo que incluímos en el concepto de imperialismo,
o en el de feudalismo, sin preguntarnos cómo lo construi-
mos? ¿Será, quizás, describiendo sin presuposición la singula-
ridad histórica de tal imperialismo (el de los romanos en
el siglo i a.C., o el del despliegue colonial de principios o
finales del siglo xix)? ¿o sintetizando, a partir de una abs-
tracción previa, los caracteres comunes a la diversidad de
estas formas históricas? Como podemos apreciar, aquí se
están oponiendo dos métodos, uno que cabría denominar
«ingenuo», preso a la vez en la ingenuidad de lo «concre-
to» inmediato y en la creencia, por sí misma ingenua, de
que una forma cualquiera y aislada puede significar lo
universal de la cosa; y otro método que posee la convic-
ción de que es necesario cribar y seleccionar, pasar por
mediaciones, abstraerse, mediante una labor de compara-
ción, de la inmediatez «concreta», y todo ello para con-
cretar verdaderamente. No es suficiente con que el len-
guaje de los historiadores nos sea familiar, puesto que se
despliega en el seno del vocabulario corriente y porque
nuestra intuición es «sensible», de un modo inmediato, al
sentido de las palabras. Y no es suficiente para que este-
mos seguros, desde un punto de vista científico, del rigor,

239
de la precisión, del estatus realmente conceptual de las
determinaciones que aquellas producen. Las palabras son
asunto del sentido, pero en una exposición, donde la claridad
de la expresión debe estar en consonancia con las necesi-
dades de universalidad inherentes a la comunicación, no
podemos permanecer en el plano de lo narrativo. En el
sentido mismo, es necesario pasar de lo «sensible» a lo in-
teligible, reflexionado, elaborado- Un concepto histórico
es un cuadro de pensamiento, y habremos de encontrar
en la experiencia puramente descriptiva del cuadro,
cuando la exigencia de rigor se hace imperiosa, la totali-
dad del trabajo intelectual que en él se inscribe, dentro
de la búsqueda del sentido; es decir, de las causas y los
fines.
4. En la última parte del texto (No Estante, en el caso... —
como adecuadas) se ofrece una nueva definición del tipo
ideal, aunque de una manera que es, necesariamente, más
detallada porque ahora se establece a qué necesidades res-
ponde. Podemos decir que la descripción precedente de
la historia descriptiva viene seguida, en este momento, '
por la conceptúalización de la historia conceptual, de
acuerdo con las singularidades de sus objetos.
¿Cómo concebir ese concepto que es el tipo ideal? En
primer lugar, como el operador de una definición pertinen-
te en el caso de los fenómenos culturales que, desde este
punto de vista, vuelve caducos los métodos «clásicos» de
la definición. Surge entonces la cuestión de cómo aplicar
la «técnica» de la identificación por género próximo y diferen-
cia específica, practicada en aquellos dominios donde es po-
sible desplegar la racionalidad silogística a una realidad
que escapa «por naturaleza» (porque, precisamente, esta
realidad, la cultura, escapa a la naturaleza) a la posibili-
dad de una afirmación dogmática. ¿Cómo, en efecto, po-
dría ser lo individual la especie de un género? No se pue-
de hacer del capitalismo de la religión una especie de ca-
pitalismo en general, ni del protestantismo una especie
de la religión en general, sin perder de vista aquello que
constituye, no su diferencia, sino su originalidad.
El tipo ideal es, pues, un cuadro de pensamiento cuya tarea •

240 I
es, siempre, la de concentrar en el interior de su «marco»
una realidad ideal, e ideal en el sentido de que no es empí-
rica, y no en el de «ejemplar» o normativa. ¿No empírica?
En efecto, el tipo ideal no se corresponde con ninguna
forma que haya existido realmente en la historia. Está en-
teramente construido como un concepto límite donde se tra-
ta de poner de manifiesto, agrandándolos según las nece-
sidades de ta causa, o sea, de la caracterización, los trazos
típicos de tal o cual singularidad histórica, y todo ello con
el objeto de que, al permitir que sean medidos, pueda ha-
cer significativas las eventuales desviaciones que pudie-
ran presentar los fenómenos con respecto al sentido más
inteligible, siendo la función del tipo ideal la de «hacer
concreto» ese mismo sentido. Esta tarca, como se puede
apreciar, presenta un valor heurístico cuyos límites de
validez deben siempre estar circunstanciados. De este
modo, el tipo ideal es imaginario, pero la imaginación está
instruida por la realidad histórica, que según esto hace las
veces de un juicio. Podremos preguntarnos, entonces, si
lo anterior es así porque pretende testar la objetividad po-
sible de las causas desde la comprensión del sentido (al
suprimir, hipotéticamente, una causa para evaluar, por
defecto, su eficacia). ¿Será necesario añadir que la puesta
en valor metodológica del tipo ideal es una forma de in-
validar, de antemano, toda pretensión de la sociología
histórica tendente a elaborar las leyes del devenir históri-
co? En este sentido, Weber no es solamente un adversa-
rio del positivismo, sino que propone una interpretación
de la historia que se enfrenta al marxismo.

PARA SABER MÁS

Sobre Weber

ARON, R., La sociología alemana contemporánea, vol. 2 (Marx, Durk-


heim, Weber), trad. A. Leal, Buenos Aires, Siglo X X , 1980,
2 vols.

241
Otras

WEBER, M., El políticoy et científico, introd. R. Aron, trad. F. Ru-


bio Llórente, Madrid, Alianza, 1987.

33. Bergson

Q U E LA VIDA D E B E DAR PRECISIÓN A LA FILOSOFÍA

Aquello de lo que más ha carecido la filosofía ha sido


de precisión. Los sistemas filosóficos no están esculpidos
a medida de la realidad en la cual vivimos. Son demasia-
do amplios para ella. Examinad cualquiera de ellos y ve-
réis que podría aplicarse igualmente a un mundo donde
no hubiera plantas ni animales, como tampoco hombres;
donde los hombres pasaran sin beber y sin comer; donde
no durmieran, ni soñaran, ni divagaran; donde nacieran
decrépitos para acabar como niños de pecho; donde la
energía pudiera subir por la pendiente de la degradación;
donde todo fuera al revés y se mantuviera a contrapelo de
todo. Y es que un verdadero sistema es un conjunto de
concepciones tan abstractas, y, por consiguiente, tan vas-
to, que habría que mantenerlo en lo posible e, incluso, en
lo imposible, junto a lo real. La explicación que debemos
juzgar satisfactoria es aquella que se adhiere a su objeto,
sin ningún vacío entre ambos, sin intersticio alguno don-
de también pudiera alojarse una explicación diferente;
ésta no conviene más que al objeto, este último no se
presta más que a aquella explicación. Tal es, quizás, la ex-
plicación científica, pues comporta una precisión absolu-
ta y una evidencia completa o creciente. ¿Se podría decir
lo mismo de las teorías filosóficas?

El pensamientoy lo mámente (1934),

242
COMENTARIO

La estructura del texto no tiene ningún misterio: la fi-


losofía carece de precisión (Aquello de k que más ha carecido
la filosofía... —...para ella). Tal sistema conviene tanto a un
mundo posible como a un mundo real (Examinad... —*•
junto a lo real). En fin, si la ciencia realiza la adecuación del
sistema con lo que él mismo explica, ¿se podría decir lo
mismo de las teorías filosóficas? (La explicación que... *
...filosóficas).
Lo importante es comprender en qué consiste esa «pre-
cisión», esa exactitud realizada en otra parte y en la que la
filosofía debiera inspirarse, más aún teniendo en cuenta
que Bergson no tiene reputación de positivista o cienti-
ficista.
2. El comienzo del texto confecciona un balance
muy general acerca de una filosofía acusada de amar en
exceso las generalidades. ¿Cómo entender esto?
Retrospectivamente, parece que a la filosofía le deben
haber faltado muchas cosas, pues Bergson se pregunta
qué es aquello de lo que más ha carecido. Kant, en el Segundo
prefacio de la Critica de la razón pura, comparaba de igual
modo los destinos de la ciencia (la física matemática) y la
metafísica, para concluir en el estancamiento relativo de
la filosofía pretérita. Aquí, el paralelismo con lo anterior,
también negativo, prende en las filosofías existentes un
defecto de precisión, y ello en un sentido que puede resul-
tar sorprendente, ya que si estas filosofías carecen de pre-
cisión es porque no están esculpidas a medida. Pero, ¿a medida
de qué? Hemos de prestar atención a las expresiones esco-
gidas por nuestro autor, tomándolas en su aparente sim-
plicidad. La filosofía no conviene a la realidad en la cual
vivimos, pues es demasiado amplia para ella. Por otra par-
te, se encausa a la filosofía en tanto que sistemas, y en tan-
to que teorías, siempre en plural, lo cual constituye la
marca o señal suplementaria de un peculiar defecto de
adecuación con respecto a su objeto, ¿Qué objeto? La rea-

243
lidad donde vivimos, que habrá que distinguir con cuida- I
do de otra realidad no irreal (vivimos en lo irreal, el sue- I
ño, la divagación), sino muerta, inorgánica, abstracta. I
3. El ejemplo, desde su anonimato, clarifica este •
punto. De existir un aspecto que sea común a todas las ca- I
rencias que Bergson pone de relieve en ese sistema filoso- fl
fico-tipo, elegido como ejemplo de inadecuación, es el de I
olvidar la vida. En primer lugar, vida vegetal y animal, •
como si el hombre pensante no se hallara en ella desde un fl
principio. Vida del cuerpo, vida orgánica, vida de necesi- I
dades; vida entrecortada a base de ausencias que una con- M
cepción en exceso intelectualista identificaría con la H
nada: sueño, sueños, ensoñaciones. Vida orientada según •
la flecha del tiempo, vida inmortal, marcada por su final •
mucho antes de aproximarse a él; en suma, vida entrópica I
que ningún aporte de energía podrá recargar como se re- I
carga una caldera. Cuando se olvida lo anterior, la filoso- a
fía puede creer coincidir con la realidad porque, en ella, •
hace abstracción de algo que no atañe al intelecto puro. M
Sin embargo, esta realidad está al revés, a contrapelo de W
todo. Esto quiere decir que la vida es todo, incluido pen- •
Sarniento. En vista de la descripción de esta «filosofía •
cualquiera» tomada al azar, podemos preguntarnos si hay I
una sola que coincida con la caricatura ofrecida en el tex-
to. Descartes no entra en esta categoría, ni Platón, ni ;

Aristóteles, ni Spinoza . Bergson no ha descubierto la


1

muerte. No obstante, la existencia de «filosofías de la


vida», sobre todo la de Spencer, mencionada por nuestro j
autor en la obra de donde se ha extraído nuestro texto,
pone de relieve diferencias hasta entonces consideradas
como poco relevantes.
En efecto, no se trata de encausar a una «filosofía de- ¡
terminada», sino, más bien, a todo sistema en tanto que
tal, y ello desde el momento en que la vida se ofrece como
objeto. Bergson quería tomar de la idea de sistema su sen- i
tido de totalidad para permitir, así, a la filosofía abarcar

1 Todos ellos integran la finitud y la muerte en la propia situación fi-

losófica.

244
ef todo de la vida, aunque esto sólo se podría lograr re-
nunciando a «la abstracción» que constituye toda su fuer-
za. La abstracción, sintética, lo abarca todo, pero hace
coexistir en sus concepciones lo posible, lo pensable y lo
real, que prescinde del pensamiento para pasar a la exis-
tencia, pues lo real condiciona, a contrapelo, el pensa-
miento. De ahí la copresencia chocante de lo real y lo im-
posible, ese posible únicamente pensable que la realidad
excluye. Un sistema de lo real, de inspiración leibnízia-
na2, haría que lo posible fuera realizado, pero es de lo real
desde donde hay que partir para decidir acerca de lo pen-
sable.
4. Las cuatro últimas frases extraen el balance explo-
rativo de este análisis de los sistemas filosóficos. La ma-
yor virtud de los sistemas científicos, que, es cierto, no
tienen a la vida como objeto, es la de adherirse al suyo
propio, de suerte que nada llega a perturbar la relación
explicativa, como ocurre desde el instante en que una fi-
losofía explica a la vez lo real y lo que entra en contradic-
ción con él, a saber, aquello que únicamente es posible.
Tal es el ideal de la explicación científica, cuya precisión
es absoluta, y su evidencia, completa o creciente. Que las
teorías filosóficas sean inferiores a las de la ciencia no sig-
nifica necesariamente que puedan sacar provecho de es-
tas últimas convirtiéndose en su calco, sino, más bien, lo
contrario, pues, al partir de lo posible para deducir lo
real, como lo hace la verdadera ciencia, racional y no em-
pírica, la teoría científica llega con éxito a puerto, mien-
tras que la filosofía encalla. La vida le dicta el mandato de
ser, a la vez, precisa y empírica, a medida de una realidad
que prescinde del pensamiento para existir.

PARA SABER MÁS

DELEUZE, G., Le bergsomme, París, PUF, 1966.

2 Leibniz, Ttodicta.

245
Sobre Bergson

DELEUZE, Q.,Elbergsonismo, trad. Luis Perrero Carracedo, Ma-


drid, Cátedra, 1987.

Otros

PRIGOGINE, 1. y STENGERS, 1., La nueva alianza- Metamorfosis de ¡a


ciencia, trad. I. Stengers, Madrid, Alianza, 1983.

34. Husserl

Q U E LA C I E N C I A D E LA NATURALEZA
ES C I E N C I A D E L ESPIRITU

El espíritu, e incluso sólo el espíritu, existe en si mismoy para sí


mismo. Es autónomo, descansa sobre sí y puede, en el marco de esta
autonomíay solamente en este marco, ser tratado de una manera ver-
daderamente racional, genuinay radicalmente científica. En loque
respecta a la naturaleza, considerada desde la verdad que
le confieren las ciencias de la naturaleza, su autonomía
no es sino aparente, y no es sino en apariencia que accede
al conocimiento por sí mismo racional en las ciencias de
la naturaleza. Porque la verdadera naturaleza, en el senti-
do de tales ciencias, es obra del espíritu que explora y pre-
supone, en consecuencia, la ciencia del espíritu. El espíri-
tu está capacitado para ejercer el autoconocimiento y, en
tanto que espíritu científico, para emprender el autoco-
nocimiento científico, y así siempre de nuevo. Es tan
sólo en el puro conocimiento de las ciencias del espíritu
como el sabio escapa al reproche de que su actividad per-
manece velada a sus propios ojos. Es absurdo, por ello,
por parte de las ciencias del espíritu, competir con las
ciencias de la naturaleza por la igualdad de derechos entre

246
ambas. En la medida en que estas últimas reconocen una
objetividad entendida como autonomía, sucumben ellas
mismas al objetivismo.

La crisis de Jas ciencias europeas y lafenomenología tras-


cendental.

COMENTARIO

1. Husserl establece aquí un paralelismo entre el co-


nocimiento del espíritu y el de la naturaleza para afirmar
que sólo se puede hablar de rigor científico o de autono-
mía con referencia al espíritu, pero, sobre todo, rechaza
la idea de examinar ese paralelismo bajo la forma de una
rivalidad e igualdad de derechos entre las dos ciencias, las
objetivas y las otras. En última instancia, Husserl acepta
la tarea de elaborar una crítica al objetivismo.
El texto desarrolla tres argumentos a la hora de cum-
plir su crítica, argumentos que descansan en una expe-
riencia metódica común, la de partir de aquello que es en
sí mismoy para si mismo para, a continuación, fundamentar
las consecuencias de tal presupuesto ontológico y meto-
dológico. Sólo el espíritu es autónomo, susceptible, por
tanto, de un tratamiento riguroso (El espíritu.,. —*• ...cien-
tífico).
El sabio no puede acceder a la reflexión acerca de su
actividad cognoscitiva si no es a través de la propia cien-
cia del espíritu, no de la ciencia natural (El espíritu... —*•
...suspropios ojos).
Podemos concluir, en consecuencia, que aceptar la pri-
macía metodológica de las ciencias del espíritu sobre las
ciencias naturales entrañaría el absurdo de un simple «pa-
ralelismo» donde no sería posible reivindicar más que
una mera igualdad de derechos.
2. Según el bergsonismo, el espíritu no era nada
apartado de la vida, de la cual sería su forma última;
Schopenhauer diría otro tanto. En el caso que nos ocupa,
la investigación metodológica conduce a la tesis opuesta:
es posible que el espíritu consiga vencer a la naturaleza (a

247
la vida) mediante una evolución, y que «reflexione» sobre
esta historicidad, pero tal cronología no es determinante.
El espíritu no está en el tiempo como si fuera un fenóme-
i no, no es un momento perteneciente a una evolución on-
tológica, sino que sólo él «es» verdaderamente, en sí mis-
mo y para sí mismo. Desde cualquier punto de vista que
lo miremos, pues, tendremos que partir del espíritu. El es
ciencia de sí mismo y, recíprocamente, no hay más cien-
cia verdaderamente racimal, genuinay radicalmente científica que
la ciencia del espíritu. Bergson, a este respecto, opone a
las ciencias de la naturaleza (cuantificadas) una «ciencia
de la vida», cualitativa y fundamentada en la duración
real, siendo ésta la filosofía verdadera1. Husserl rechaza
toda forma de oposición que remita a diferentes grados
ontológicos de realidad o de evolución; la única ciencia
verdadera es la ciencia del espíritu, que comprende tam-
bién a la ciencia físico-matemática.
3. Este es el sentido del segundo argumento, la natu-
raleza carece de autonomía, depende del espíritu que la
fundamenta, que sí es autónomo. No queda, pues, lugar
para dos proyectos de ciencia, el uno concerniente a lo
material y el otro a lo espiritual. Husserl está convencido,
por otra parte, «de lo absurdo de,una concepción dualista
del mundo»2. O bien las ciencias de la naturaleza no son
verdaderas ciencias, y no contienen más que meras infor-
maciones despojadas de su razón de ser, o bien toda su
verdad está comprendida en la obra del espíritu que ex-
plora (i. e. la naturaleza). La única relación entre la natu-
raleza y el espíritu no es la de constituir dos substancias
contiguas o superpuestas, sino la de estar ligadas por un
supuesto de base: para que pueda ser conocida, la natura-
leza necesita de la actividad del espíritu autónomo.
4. Las aporías de las ciencias encuentran en esta fun-
damentacíón radical las condiciones de su superación. La
«crisis de fundamentos»3 hace que se estremezca el edifi-
1Bergson, El pensamiento y lo movirnte, texto 33.
2Cfr, La irisis de ¡ta demias europeas y ta ftnommoloffa trautndeníai.
5 Debate Russell-Poiricaré, cfr. R. Blanché, L 'axiomatiqut, París,

PUF, 1959, pág. 33.

248
cío de la matemática y, en consecuencia, de todas las
ciencias físicas, en tanto que buscan una autonomía don-
de jamás podrán hallarla, pues la naturaleza es depen-
diente, y el instrumento matemático es, o bien espíritu
mismo, o bien se resuelve en puro artificio sin rigor algu-
no. Este es el sentido de la enigmática afirmación que
hace más precisa la relación del espíritu que reflexiona
sobre sus propias operaciones: y así siempre de nuevo. Se
ha dicho que las matemáticas son «la única ciencia donde
nunca se llega a saber de qué se está hablando, ni si es ver-
dad lo que se dice». Ello es debido a que la idea de una
verdad únicamente extrínseca, que afecta a las relaciones
entre un conocimiento incierto y su objeto predetermina-
do, no conviene a la ciencia como tal, que es ciencia de sí.
Las ciencias se verían así desprovistas de su verdad, pu-
diendo reprocharles que la actividad (del sabio) experi-
mentará cómo ésta permanece velada a sus propios ojos, pero
esto es imposible cuando se trata de las ciencias del es-
píritu.
5. Las desventuras del espíritu científico, concebido
como «conciencia enmascarada» del sabio perplejo ante
el éxito de sus operaciones, éxito que atribuye a una auto-
determinación de su objeto (del que él es excluido en su
calidad de sujeto racional), se denominan objetivismo, la
enfermedad de la objetividad. ¿Es «subjetiva» la actividad
del sabio? ¿Encuentra esa actividad una objetividad ca-
sual que procede de las cosas mismas? Para Husserl, «las
cosas» son el espíritu, y la actividad racional del sabio es
ese mismo espíritu. Espíritu es tan sólo el nombre de una
intersubjetividad sabia, una puesta en común de sus es-
fuerzos en pos de la verdad. El objetivismo consiste en ol-
vidar el presupuesto metodológico y creer en la autono-
mía de aquello que nos es dado a conocer, mientras que
éste nos remite, necesariamente, al espíritu que conoce.
A la vista de lo anterior, apelar por una igualdad de de-
rechos entre la ciencia del espíritu y la de la naturaleza no
es pedir demasiado, sino, más bien, pedir demasiado
poco, pues no hay más ciencia de la naturaleza que en y
para el espíritu.

249
35. Husseri
P R O D U C C I O N E S FILOSÓFICAS
Y PRODUCCIONES TRADICIONALES

Para empezar, clasifiquemos la propiedad singular de


la filosofía, desarrollada en ciencias sistemáticas siempre
nuevas. Subrayemos el contraste con las otras formas cul-
turales ya presentes en el universo precientifico del hom-
bre, incluyendo entre ellas las artesanías, la civilización
agrícola, la civilización comercial. Todas estas formas de-
signan diversas categorías de productos culturales con sus
correspondientes métodos para asegurar el logro de una
producción segura (Er^fugung), Estos productos tienen,
por lo demás, una existencia pasajera en el mundo circun-
dante. En contraposición, las adquisiciones científicas,
una vez engendrado el método que asegura su produc-
ción, tienen un modo de ser enteramente distinto, una
temporalidad de todo punto diferente. No se consumen,
no son efímeras. No cabe duda de que una producción re-
petida no crea cosas idénticas; en el mejor de los casos tan
sólo es idéntico el modo de consumirlas, pero un número
cualquiera de producciones permite a un número equiva-
lente de personas crear un producto idéntico, e idéntico
en cuanto a sentido y validez. Cuando personas relacio-
nadas efectivamente con otras por una mutua compren-
sión se encuentran ante un objeto producido por algún
compañero, siguiendo un procedimiento idéntico, no
pueden evitar considerarlo semejante al que ellos mismos
han producido. En una palabra, aquello que la actividad
(Tun) científica produce (erwirbt), no es real sino ideal. Y
no sólo eso, sino que lo así engendrado, con su validez y
su verdad, deviene a continuación material para una
creación posible de ideales de nivel superior.

«La crisis de !a humanidad europea y la filoso-


fía» (1935).

250
COMENTARIO

1. Husserl emprende una comparación regulada en-


tre dos tipos de productos, correspondientes a dos cultu-
ras de espíritu antagónico: la tradición precientífica y
aquella que inaugura la filosofía con las ciencias. El autor
termina por reconocer una diferencia radical de estatus
temporal. Así, mientras que las culturas tradicionales en-
gendran lo provisional y discontinuo, la cultura científi-
ca es la única capaz de crear idealidades permanentes y en
progresión indefinida, originando, además, la reunión de
los productores en una comunidad.
Partiendo de una diferencia entre formas culturales
pre y postcientificas (Para empezar... —*• ... comercial), Hus-
serl determina una diferencia radical en cuanto a la rela-
ción que sus productos respectivos mantienen a lo largo
del tiempo (Todas estas formas... ...de todo punto diferente).
El autor desarrolla, en suma, la característica propia de
las obras científicas; más ideales que reales, ellas son las
únicas capaces de conseguir resultados a la vez intempo-
rales y en continuo progreso (No se consumen...-^*...nivelsu-
perior).
2. La comparación tiene por finalidad la de aislar la
propiedad singular de la filosofía, que constituye el objeto de
toda la argumentación, pero esta propiedad no aparece
tomada por sí misma como una obra puramente especu-
lativa, pues de hacerlo así supondría una restricción ne-
cesaria del paralelismo con las obras científicas que no
tomaría en cuenta la «civilización material». FUmofía de-
signa una forma completa de cultura, tanto material
como espiritual, y es este punto el que confiere al texto su
mayor fuerza y originalidad. Junto a las «ciencias sistemá-
ticas» que la desarrollan, la filosofía revoluciona toda la
civilización y no sólo el pensamiento puro o los puros sis-
temas de ideas, tales como los mitos, relatos, historia na-
tural, textos sagrados. Hay que considerar como signo in-
dicativo de esta aspiración lo que Husserl denomina la

251
elección tendente a «incluir», en el universoprecientífico, los
productos de la civilización artesanal, agrícola, comer-
cia); esto es, los bienes y mercancías. Es necesario, pues,
determinar una consecuencia implícita en tal elección;
existe un «modo de producción» originado por la filo-
sofía.
3. Si hacemos abstracción de toda diferencia empíri-
ca que impida fa comparación (entre bienes materiales,
por una parte, y conocimientos intelectuales, por otra),
Husserl estudia el paralelismo desde el punto de vista de
la relación con el mundo circundante y su historicidad. En
este sentido, y desde los dos aspectos de la civilización,
hemos de habérnoslas con productos cuyo método de
producción se ha visto ligado al empirismo y a las vacila-
ciones propias de los primeros pasos; son fruto de métodos
experimentales (sicber gelingend: de eficacia asegurada).
Esta propiedad, característica tanto de las diversas catearías
de productos precien tíficos como de las adquisiciones cientí-
ficas derivadas de métodos de eficacia igualmente proba-
da. Lo que diferencia a las dos épocas no depende de la
aparición de un avance metodológico productivo, sino al
tipo de productos que resultan una vez lanzados al «mun-
do circundante». Los primeros no alcanzarán nada más
que una existencia pasajera, mientras que los segundos serán
perennes. (No se consumen, no son efímeras.) Una vez más, no
puede tratarse de una simple diferencia que se refiera a su
realidad empírica (las unas, mercancías perecederas, las
verdades eternas). Se trata de la propiedad singular de la filo-
sofía que revoluciona el Unmlt mismo (el medio ambiente
vital). ¿Vivimos en un mundo finito o infinito? ¿Está li-
mitado el tiempo por la empírica duración de las cosas y
de los hombres, o está abierto sobre las idealidades y sustraí-
das al curso temporal de las cosas?
4. ¿En qué sentido los productos científicos han logra-
do sustraerse a la usura del tiempo? Husserl, en el texto
alemán, no separa la perennidad de los productos cientí-
ficos de su capacidad de escapar a una sombría repetición
que los enclaustraría en un tiempo finito. Repetir su pro-
ducción, escribe, no engendra las mismas cosas, sino,

252
como mucho, un mismo uso (gkkh Braucbbares). La con-
clusión del argumento precisa de la puesta en juego de
esta distinción; aquello que se produce de modo idéntico,
a través de una repetición tan prolongada como se desee
de sus producciones, y de un número indefinido de pro-
ductores, supone la identificación de sentido y valor (rtach
Sinn und Geítung) de los productos. En otros términos, sur-
ge un mundo que participa de su significación ideal,
puesto que la producción se realiza a partir del espíritu
científico que se prolonga en las ciencias. Este mundo no
será ya meramente «objetivo», sino intersubjetivo, y ello
porque en él lo que cuenta, en esencia, es el «sentido co-
mún» a la producción y lo producido.
Esta nueva dimensión es la comunidad, como el texto
se ocupa de señalar a continuación. A través de una doble
negación, Husserl indica, como propiedad de la experien-
cia, el reconocimiento de una comunidad de modos de
producción, y ello gracias a la similitud de los productos.
«Cuando personas relacionadas efectivamente con otras
por una mutua comprensión» (esto puede considerarse
como una definición de comunidad) «no pueden...». Esta
frase subraya que la comunidad no existe en la «inten-
ción» o como fenómeno de consciencia en el sentido psi-
cológico y superficial del término, sino que precisa el re-
conocimiento de la comunidad en la exigencia decisiva
de los productos de factura idéntica, sean científicos en
sentido restringido o en sentido lato, como artefactos
cuya producción presupone un saber científico. Tras este
importante análisis, Husserl sintetiza y concluye, resu-
miéndola en una palabra, la lección que debemos retener.
El problema es el de saber cuál es el resultado efectivo ob-
tenido por la actividad científica; es decir, aquello que
puede considerarse como su verdadero producto.
Puesto que el genuino resultado de la producción con-
siste en la emergencia de una comunidad fundada en la fi-
losofía y no en meros bienes efímeros referidos al «hom-
bre natural» que los desea, habrá que introducir entre es-
tos productos y sus productores una mediación esencial:
el rodeo en torno a la idea, aunque no en el sentido de un

253
conocimiento previo, sino en el de la visión común y
participadora que se descubre en una realización que es,
empíricamente, objeto de la experiencia. No se trata de
que las realizaciones del espíritu carezcan la realidad. Al
contrario, es, precisamente, en su calidad de idealidades
que adquieren tal realidad. Bachelard denomina a las rea-
lidades técnicas «teoremas reificados»1 con el propósito
de señalar lo que la realización de lo racional debe esen-
cialmente a la teoría. La libre contemplación que permite
la teoría hace posible que tales realizaciones, así como la
historicidad de sus productos, escapen al tiempo. En la
historia de las ciencias hay dos dimensiones: antes
de la historia, todo conocimiento es prehistórico, objeto
de recomposición retrospectiva; después de ella (tras la
«censura epistemológica»), la ciencia vive en un eterno
presente, sus adquisiciones son definitivas, si bien sus
progresos ulteriores confirman, al no impedirlo, la ver-
dad de sus comienzos2. Las geometrías no euclídeas en-
globan a la euclídea como caso particular y como funda-
dora de las otras. Michel Serres llama a este movimiento
retrógado de lo verdadero con la expresión «anamnesis
matemática»1; es en este sentido que podemos considerar
a Husserl el primero en hacer de lo así engendrado la mate-
ria prima de una creación posible de ideales de nivel su-
perior, lo cual no supone la negación de la historicidad de
la ciencia, sino la superación de un historicismo que ven-
dría a ser una forma distinta de objetivismo.

PARA SABER MÁS

De Husserl

Meditaciones Cartesianas, trad.J. L. García Baró/José Gaos, Méxi-


co, FCE, 1985.
1 G. Bzchehrd, Le ratñmaiisme apoqué, París, PUF, 1966, págs. 109-110.
1 Ver nota 1.
1 M. Serres, «Les anamnéses mathématiques», en Htrmis ou ¡a comuni-

cation, París, éd. de Minuit, 1969.

254
La idea de la fenomenología, trad. J. L. García Baró, México, FCE,
1982.

Sobre Htuserl (textos 34 y Í5)

KOLAKOWSKI, L., Husserl y la búsqueda de la certeza, trad. A. Mur-


guía Zuriarrain, Madrid, Alianza, 1977.
MAGEE, B., Los grandesfilósofos,trad. Amaia Barcena, Madrid, Cá-
tedra, 1990, págs. 273-303.

Otros

BACHELARD, G., La actividad racionalista de la ciencia contemporánea,


trad. E. Cantó, Buenos Aires, Siglo X X , 1975.
— La formación del espíritu científico, trad. J. Babini, Buenos Aires,
Siglo X X , 1972.
KOYRÉ, A., Del mundo cerrado al universo infinito, trad. C, Solís San-
tos, Madrid, Siglo X X I , 1979.
SERRES, M., Historia de las ciencias, trad. Ramón Herrera, Madrid,
Cátedra, 1991.

36. Wíttgenstein

S O B R E LAS PROPOSICIONES DE LA ÉTICA,


QUE CARECEN DE SENTIDO

ó.4 Todas las proposiciones valen lo mismo.


6.41. El sentido del mundo tiene que residir fuera de
él. E n el mundo todo es c o m o es y sucede c o m o sucede; en
él no hay valor alguno, y si lo hubiera carecería de valor.
Si hay un valor que tenga valor ha de residir fuera de
todo suceder y ser-así.
Porque todo suceder y ser-así son casuales.
L o que los hace no-casuales n o puede residir en el mun-
do; porque, de lo contrario, sería casual a su vez.
Ha de residir fuera del mundo.

255
6.42. Por eso tampoco puede haber proposiciones
éticas. Las proposiciones no pueden expresar nada más
alto.
6.421. Está claro que la ética no resulta expresable.
La ética es trascendental,
(Ética y estética son una y la misma cosa.)
(...)
6.522, Lo inexpresable, ciertamente, existe. Se mues-
tra, es lo místico.

Tractatus iogico-philosophicus (1921).

COMENTARIO

1, Ludwig Wittgenstein (1889-1951) destaca en el


panorama filosófico del siglo xx como una figura excep-
cional que goza del privilegio tan poco frecuente de ha-
ber concebido dos sistemas filosóficos incompatibles a lo
largo de su vida, cada uno de los cuales influyó en toda
una generación. No obstante, ambos sistemas giran en
torno a una misma preocupación, la de examinar el papel
del lenguaje en el pensamiento y la vida humana y trazar
los límites entre los usos válidos y no válidos del mismo,
es decir, entre el sentido y el sinsentido. Las dos concep-
ciones filosóficas aparecen reflejadas, respectivamente,
en el Tractatus iogico-philosophicus (1921) y en las Investigacio-
nesfilosóficas(publicada a título postumo en 1953). La pri-
mera obra caracteriza lo que se suele denominar primer
Wittgenstein, de cuya influencia surgieron los positivis-
tas lógicos del Círculo de Viena, La segunda, el llamado
segundo Wittgenstein, se reveló como la obra más decisi-
va de la filosofía anglosajona posterior a la segunda gue-
rra mundial, de la que es deudora la filosofía analítica que
se ha venido desarrollando hasta nuestros días.
El interés por la lógica y el lenguaje que Frege y Russell
inauguraran con el siglo xx no hizo sino recuperar mu-
chos de los problemas y preocupaciones filosóficas me-
dievales que habían sido relegados a un segundo plano

256
durante el periodo comprendido entre el Renacimiento y
la Ilustración. En efecto, la epistemología, la pregunta
acerca de cómo sabemos y podemos saber que sabemos, ha-
bía desplazado a una filosofía centrada en la lógica y desa-
rrollada fundamentalmente en análisis lingüísticos. Witt-
genstein se encuentra al redactar su primera obra con el
resurgimiento del análisis lógico del lenguaje (en esta
ocasión, desde la lógica matemática), pero también con la
agonía de la tesis fundamental de la epistemología occi-
dental, desde Platón a Kant: que el pensamiento —el
lenguaje en nuestro siglo— refleja el mundo, y que es po-
sible encontrar un número de representaciones privile-
giadas, muy pocas, con las cuales comparar la realidad y
afirmar su mutua correspondencia. En su Tractatus, Wítt-
genstein examina esta tesis en la versión heredada del
atomismo lógico de Russell-Whitehead. Enseguida vere-
mos hasta qué punto.
Podemos considerar el Tractatus iogico-philosopbkus1
como una síntesis de la teoría de las funciones de verdad
y de la idea de que el lenguaje es una figura de la realidad
(en alemán, Bildes figura, pintura, retrato —a la manera
de las ideas cartesianas— pero también modelo, patrón).
Aparte de esta síntesis se añade otro importante ingre-
diente de esta obra: la doctrina de lo que no puede ser di-
cho sino tan sólo mostrado.
Dos elementos fundamentales: por un lado, el análisis
del lenguaje; por otro, el análisis del mundo y la media-
ción entre ambos. Lógica y metafísica-epistemología que
conducen a nuestro autor a los limites de ambos territo-
rios, a lo místico. Cómo interpretar la concepción de lo
místico a la que conduce la singlatura argumentativa
del TLPnoes tarea fácil. Las proposiciones que nos toca co-
mentar están estrechamente relacionadas con lo que no
puede ser dicho. El valor de las proposiciones de la ética,
de la estética y de la religión queda diferido a ese resto
que sólo es silencio. Y la tarea del ético, del esteta, del re-

1 En adelante lo abreviaremos como TLP.

257
ligioso, irremediablemente condenada a ser extrali mi ra-
ción ilegítima de la actividad filosófica genuina.
2. Las proposiciones de nuestro comentario comien-
zan por la 6.4. Desde la 6 hasta esta última Wittgenstein
ha tratado de descubrir la estructura lógica de la ciencia.
Antes lo había hecho con el lenguaje y el mundo por se-
parado. Ahora, ambos confluyen en la ciencia. La lógica
es analizada en las proposiciones 6.1-6.2; la matemática
en 6.2-6.3 y la ciencia natural en 6.3-6.4. A partir de este
momento y como consecuencia de lo anterior, restan ám-
bitos de presunto conocimiento y problemas que no ca-
ben ser analizados desde los criterios lógicos con que pro-
cedió el análisis del lenguaje y el mundo. Así, si las únicas
proposiciones con sentido son las de la ciencia natural (se
pone en duda que lo sean las de la matemática, y las de la
lógica son tautologías), entonces la ética, la estética y la
religión (6.4-6.53), la metafísica y el mismo TLP queda-
rán fuera del análisis lógico del lenguaje.
En 6.4 se nos habla del valor de. una proposición, térmi-
no que podemos entender de dos modos. De una parte, el
valor de una proposición es su valor de verdad. A este
respecto tenemos que recordar que el sentido mismo de
una proposición es su método de verificación. El sefttido
de una proposición determina la posibilidad de su verdad
o falsedad. Es el análisis lógico del lenguaje y su sentido
donde aparece la posibilidad de un mundo (4.2) que se
supone o no como existente (3). El punto de vista es sin-
táctico y, de este modo, la verificabilidad de una proposi-
ción viene dada por su sentido, a lo que se une el hecho
de que las proposiciones con sentido son las de la ciencia
natural. Y son las propiedades formales de la lógica (tau-
tológica) las que posibilitan el que podamos decir las del
mundo y las del lenguaje.
Wittgenstein identifica pensamiento-lenguaje y mun-
do, de modo que lo pensable, imaginable e inteligible se
equipara con aquello que puede ser descrito desde un len-
guaje con sentido, y coincide con lo que podemos presen-
tar como una figura lógica. Para nuestro autor, al igual
que para Kant, la tarea de la filosofía teórica consiste en

258
identificar cuáles son los limites del discurso (teórico), de
lo que puede ser dicho en el lenguaje mostrando, a su vez,
el armazón lógico del mundo. La lógica, así, no es teoría
sino reflexión del mundo: trascendental (6.13). La filoso-
fía, por tanto, delimita el dominio del discurso científico
(4.113), delimita lo pensable y, secundariamente, lo que
no puede ser pensado, esto es, dicho. E indica lo que no
puede ser dicho exponiendo claramente lo que sí pode-
mos decir (4.114-4.115). El resto, aquello que queda fue-
ra del marco de la razón teórica, que aquí es lingüística,
pertenece al silencio de lo místico, a lo que sólo puede
mostrarse (6.522).
Según lo anterior sólo podemos hablar de verdad o fal-
sedad dentro de los límites del discurso científico, único
con sentido. Cuando Wittgenstein afirma que todas las pro-
posiciones valen lo mismo podría estar diciendo que toda pro-
posición con sentido —cuya condición de posibilidad
viene dada por la lógica— contiene ya la posibilidad misma
de su verificación. De este modo, las proposiciones de la
ética, de la estética o de la religión no serían nada más
que pseudoproposiciones sin sentido y sin posibilidad al-
guna de que podamos decir su verdad o falsedad median-
te el cálculo de tablas de verdad (6). Y si fueran verdade-
ras proposiciones estarían al mismo nivel que las propo-
siciones de la ciencia, es decir, valdrían lo mismo. Sin
embargo, hay otro tipodew/¿>/"que la ética reclama para sí.
Valor, según esta segunda acepción, equivale a lo «va-
lioso», y remite a los juicios de valor acerca de lo bueno y
de lo malo, acerca del sentido de la vida o de la inmortali-
dad, a los «valores absolutos» en función de los cuales es-
timamos qué es y qué no es nuestro deber moral. Si esto
es así, la proposición 6.4 sorprende al afirmar que todas
las proposiciones valen lo mismo, esto es, que son igual-
mente importantes o valiosas para las cuestiones más
fundamentales de la vida. Y afirmar que todas valen lo
mismo equivale a aceptar que ninguna vale nada. La si-
guiente proposición del comentario nos servirá para
adentrarnos en esta cuestión.
El primer párrafo de 6.31 introduce la expresión el seti-

259
tido del mundo, del que se afirma que ha de residir fuera de
él. De nuevo, el término parece ambivalente. En efecto,
para determinar el sentido del mundo, que tenga sentido,
tendríamos que salimos de los límites lógico-trascenden-
tales del mundo mismo y del lenguaje (pensamiento), lí-
mites que son condición de posibilidad de ambos. Este
marco lógico trascendental permite que las proposicio-
nes del lenguaje y mundo tengan sentido y puedan ser, en
consecuencia, verdaderas o falsas. Pero tratar de explorar
el sentido de ese marco lógico supondría una extralimita-
ción; pasaríamos del ámbito del discurso con sentido al
mismísimo sinsentido.
En suma, no podemos hablar en la lógica sobre cómo
se habla desde la lógica, y en ello consiste la paradoja del
atomismo lógico russellíano que Wittgensteín sólo llega a
reconocer después de haber ascendido por los peldaños
de argumentativos del TLP, en la contundente proposi-
ción 7: De lo que no se puede hablar hay que callar. La lógica,
pues, no dice nada; sólo habla de posibilidades (2.0121),
y por esta razón fundamenta todo sentido lingüístico y
todo hecho del muño. Sus proposiciones son tautologías
que la experiencia no puede rechazar ni confirmar (6,
6.1, 6.11, 5.552). Por ello podemos decir que todas las
proposiciones valen lo mismo. La lógica se limita a mos-
trar la estructura del mundo y del lenguaje (6.12, 6.121,
6.124). Es, por hablar en términos kantianos, su condi-
ción de posibilidad. La única relación de la lógica con el
mundo es que, a la hora de su aplicación a la descripción
cien tífico-natural de los hechos de ese mundo, podrán
corresponder con los nombres y proposiciones del len-
guaje analizados.
Según lo anterior, la función de la lógica consistirá en
combinar las proposiciones con sentido hasta que lo pier-
dan, transformándose en tautologías, y entonces serán
capaces de mostrar las propiedades lógicas que hacen po-
sible su decir (6.121). Pero lo que realmente importa al
hombre no es el cómo descriptivo del mundo, su estructu-
ración lógico-científica, sino precisamente el hecho de
que sea, de que tenga o no sentido (6.44, 5.552), cuestión

260
que no puede resolverse mediante análisis lógico. Así, si
el mundo tiene sentido, éste habrá de situarse fuera del
mismo, y fuera del discurso teórico-científico.
En el mundo todo es como esj sucede como sucede, lo que nos
remite a la primera proposicion del TLP: El mundo es todo
lo que es el caso. Si hay un valor en el mundo carecerá de
todo valor, pues en el mundo sólo podemos hablar de va-
lores de verdad o falsedad relativos a proposiciones y en
correspondencia con los hechos de ese mundo (o con ese
mundo considerado como un hecho susceptible de análi-
sis en hechos más simples). Dentro de tales parámetros
todo valor es funcional o, lo que es lo mismo, carece de
valor absoluto, de necesariedad, porque todo suceder y
ser-así son casuales. Casual, es decir, contingente, pues todo
suceder y ser-así lo es de eventos espacio-temporales, y las
«leyes de la naturaleza», hipótesis admitidas para el análi-
sis de los hechos del mundo. Las leyes de la causalidad
(que opone la «casualidad» a la «necesidad») quedan redu-
cidas a las leyes lógicas (6.3, 6.31). No hay más necesidad
que la necesidad lógica, y todas las proposiciones son
contingentes. De ahí que si hay algún valor genuino, éste
no podrá ser contingente, y la cuestión sobre el sentido de
la vida habrá de residir fuera de todo ese suceder y ser-así
que describen las proposiciones del lenguaje.
Wittgenstein, en su Conferencia en torno a la éticay el valor2,
distingue entre valor absoluto y valor relativo. Al segundo lo
denomina trivial y al primero ético:
De hecho el término «bueno», en el sentido relativo,
significa simplemente: adecuado a cierto estándar prede-
terminado (...). Todo juicio de valor relativo es un mero
enunciado de hechos y puede por tanto ser puesto en una
forma tal que pierda toda apariencia de juicio de valor
(..,). Ahora bien, lo que quiero sostener es que, aunque
todos los juicios de valor relativos son reducibles a meros

2 Esta obra recoge una conferencia más tas notas tomadas por Weiss-

man de conversaciones sostenidas entre Wittgenstein y Schlick, La ver-


sión castellana es de A. Salazar Bondy, publicada en Lima, Universidad
Mayor de San Marcos, Biblioteca Filosófica, 1967.

261
enunciados sobre hechos, ningún enunciado sobre he-
chos puede nunca ser o implicar un juicio de valor abso-
luto (...). No hay proposiciones que sean, absolutamente
en ningún sentido, sublimes, importantes o triviales1.

Wittgenstein no acepta más valor ético que el pura-


mente instrumental, casual, ya que lo que los hace no ca-
suales no puede residir en el mundo, porque de lo contra-
rio sería casual a su vez. Ha de residir fuera del mundo.
En efecto, nuestras palabras sólo expresan hechos, como una taza
sólo contiene una determinada cantidad de agua, aunqueyo derrame
un galón sobre ella*.
Llegamos así a las proposiciones 6.42, 5.421 y 6.522,
que consideraremos conjuntamente. Las proposiciones,
en efecto, no pueden expresar nada más alto, y es por ello que
tampoco puede haber proposiciones éticas. El filósofo
identifica la ética no sólo con «la investigación de lo que
es bueno» (Moore, Principia Ethica), sino también con la
investigación de lo que es valioso o de lo que es realmen-
te importante, o del sentido de la vida o de lo que hace a
la vida digna de ser vivida, o del modo correcto de vivir-
la. Identifica la ética no sólo con la estética, sino también
con la religión que está a la base de aquélla: La ética y la
estética son una y la misma cosa (6.421); La ética, si es algo,
es algo sobrenaturali, O Si algo es bueno es también divino. En ello,
curiosamente, se resume mi ética1,, o si hay una frase que expresa
claramente lo que pienso es esto: Bueno es lo que Dios ordena, y Dios
padre ha creado el mundo, Dios hijo (o la palabra que proviene de
Dios) es lo ético1.
Wittgenstein rechaza la posibilidad misma de una teo-
ría ética, de una ética considerada como ciencia porque,
si lo fuera, no podría aspirar más que a la descripción de
hechos, de juicios de valor relativos, pero no a una expli-

3 Conferencia..., págs. 15-17.

" Op. cit., pág. 19.


5 Op. cit., pág. 19.
6 VermsthU Bemerksmgtn, Frankfurt A. M., Suhrkamp, 1977, pág, 15.
7 Conferencia..., op, cit., pág. 31

262
cación más o menos esencialista. Cualquier modelo cien-
tífico no puede más que posibilitar un marco descriptivo,
sistemático (6.341, 6.343). Los fenómenos mundanos no
pueden explicarse porque no hay más necesidad que la
puramente lógica. Como ya señalábamos más arriba, a la
filosofía le corresponde el cómo del mundo, no el que exista
o qué exista (6.44,5.552). La ética, entonces, no puede ser
ciencia: Me parece que no se puede decir más que: «¿Vive/eUz!»*.
El resto, puro sínsentido al que, sin embargo, todos esta-
mos orientados. La ética no es sino un continuo arremeter
contra los limites del lenguaje, mero parloteo: Ningún estado de co-
sas tiene en si mismo lo que llamaría el poder coercitivo de un juez
absoluto''.
De este modo, si bien cabe hablar de un paralelismo
entre nuestro autor y Kant en cuanto a la razón teórica
—razón lingüística en el caso que nos ocupa—, ambos se
separan en el tratamiento del aspecto práctico de la ra-
zón. Desde luego Wittgenstein acepta un dominio de la
razón práctica, tal y como aparece expuesto en la última
proposición de nuestro comentario: Lo inexpresable, cierta-
mente, existe. Se muestra, es lo místico (6-522). Pero este domi-
nio reside fuera del lenguaje. No podemos, como Kant,
hablar de voluntad, pues la única voluntad de la que cabe
hablar sólo interesa a la psicología (6.423). La ética es trascen-
dental, no resulta expresable (6.421), pues en el mundo que el
lenguaje puede describir no hay valores objetivos, absolu-
tos; todas las proposiciones valen igual. Cuestiones tales
como Dios o la inmortalidad (en las proposiciones acerca
del enigma) también son desplazadas al ámbito de lo mís-
tico (6.432, 6.4312 y 6.5, respectivamente). Concluire-
mos este comentario con una última observación acerca
del concepto de lo místico en conexión con aquello que,
como la ética, no resulta expresable.
3. Lo inexpresable (das Unaussprechlkbe) coincide, desde
un punto de vista lógico, con aquello que no tiene senti-

* Diariofilosófico(1914-1916), trad. Jacobo Muñoz e Isidoro Reguera,


Ariel, Barcelona, 1982, pág. 170.
'' Confirméis..., op. tii., pág. 19.

263
do al no poder expresarse en proposiciones genuinas.
Mas, desde un punto de vista positivo se equipara con lo
inefable, con lo que no puede ser dicho porque tan sólo se
muestra. Este territorio de lo místico nos recuerda más a
la interpretación schopenhaueríana de la Dittg an sich
(cosa en sí) kantiana. Para Kant, la cosa en sí parecía tener
cierto aspecto positivo de existencia independiente de
nuestro conocer el mundo tal y como se nos aparece
(Wittgensteín estaría de acuerdo con la segunda parte de
esta oración). Pero en Schopenhauer, y quizás también en
el TLP, lo inexpresable se convierte en el símbolo de lo
trascendente, fuera de nuestro alcance.
Ahora bien, si lo místico se muestra, ¿cómo se mues-
tra? El mostrarse puede serlo en el lenguaje, y en este sen-
tido se muestra el armazón lógico-trascendental, condi-
ción de posibilidad de toda experiencia lingüística. La
conciencia del límite de todo posible conocimiento nos
conduce hacia el exterior de esa frontera sin traspasarla.
Hacia ella apunta la trascendencia del mundo, del pensa-
miento y del ser humano, y las preguntas fundamentales
de la vida comparten con la lógica el que no pueda ha-
blarse de ellas desde la misma lógica. Que la vida sea dig-
na de vivirse, que el mundo tenga un sentido, que sea y
qué sea, pertenece más bien a lo que no puede decirse ni
mostrarse en el lenguaje. ¿Cómo se muestra?
Wittgenstein parece entenderlo en términos de expe-
riencia emocional (Erlebnis) y no factual (Erfahrung). So-
bre el sentimiento habla muy pocas veces el autor en el
TLP (4.1213, 6.1223, 6.1232, 6.53). En la proposición
6.45 se dice, con palabras que nos recuerdan a Spinoza:
La visión del mundo sub specie aeterní es su visión como-todo-
limitado. El sentimiento del mundo como-todo-limitado es lo místico,
lo que podríamos interpretar en el sentido de que la con-
templación del mundo sub specie aeterní es la intuición del
mismo como todo limitado.
Y, una vez tomado conciencia de los límites, el lector
del TLP reconocerá las proposiciones que lo componen
como absurdas, cuando a través de ellas —sobre ellas— ba salido
fuera de ellas. (Tiene, por asi decirlo, que arrojar la escalera después

264
de haber subido por eiia). Tiene que superar las proposiciones; enton-
ces ve correctamente el mundo (6.54). La tarea del filósofo,
esencialmente anclada en el mundo y en el lenguaje, se re-
conoce como absurda al ser superada. El autor ejercita la
idea del fin de la filosofía, que no hace nada positivo por
el saber, sino que su labor revierte, negativamente, en su
prescindibilidad al dejar las cosas como están, aún cuan-
do deje mejor aclarado su estatus lingüístico-epistemo-
lógico.
Wittgenstein deja sin respuesta la filosofía kantiana. La
razón, en efecto, se delimita a sí misma, pero nada sabe-
mos de cómo lo hace. El presupuesto del carácter tras-
cendental de la lógica carece de sentido (el postulado on-
tológíco-gnoseológico del isomorfismo russelliano no era
susceptible de análisis, como tampoco un hecho del mun-
do). Para aprehender ese sinsentido hemos debido llegar
hasta los límites de lo expresable y más allá. Como para
Kierkegaard1" la fe religiosa, nuestra fe en la filosofía es
injustificada en último término.
Quizás ahora, al final de nuestro comentario, estemos
ya más dispuestos a entender la advertencia que el autor
dirige en una carta a Ludwíg von Ficker'1:

Quise escribir, en efecto, que mi obra se compone de


dos partes: de la que aquí aparece, y de todo aquello que
no he escrito. Y, precisamente, esta segunda parte es la
importante. Mi libro, en efecto, delimita por dentro lo
ético, por así decirlo, y estoy convencido de que, estricta-
mente, SÓLO puede delimitarse así. Creo, en una palabra,
que todo aquello sobre lo que boy muchos parlotean lo he

111 Ver Mi punto de vista, Madrid, Sarpe, 1985. Por supuesto, no preten-

demos llevar el paralelismo más allá de lo que hemos escrito. La ironía,


en Kierkegaard, era intencionada. En Wittgenstein, sin embargo, ésta
sólo parece sorprenderlo al final del TLP. Es probable que nuestro filó-
sofo conociera a Kierkegaard y, de hecho, lo cita en su Conferencia...,
op. cit., pág. 29: Kierkegaard también vio este arremeter (entra tos limites dei len-
guaje e incluso lo caracterizó de una manera similar (como arremeter centra la parado •
ja). Este arremeter contra los limites de! lenguaje es la ética.
11 Briefe an Ludwlg von Ficker, Salzburg, Otto Müller Verlag, 1969,

págs. 96-97.

265
uesto en evidencia yo en mi libro guardando silencio so-
Cpuesto
re ello (...), I^e aconsejaría ahora leer el prólogo y el final,
que son ellos los que expresan con mayor inme-
diatez el sentido.

PARA SABER MAS

KKNNY, A., Wittgnstein, versión de A. Deaño, Madrid, Alianza


Universidad, 1984.
M A G K K , B., LOS grandesfilósofos, trad. Amaia Barcena, Madrid, Cá-
tedra, 1990, págs. 347-376.
PKARS, D . , The Falte Prison, A Study of tbe Devekpement of Wittgens-
tein's pbiksopby, vol. I, Oxford, Clarendon Press, 1987.
S C H U L Z , W., Wittgnstein, La negación de ¡afilosofa, trad. J . Monto-
ya Sáenz, Madrid, G. del Toro, 1970.
S T K N I U S , E , Witigeristem's Tractatus, Oxford, Basil Blackwell,
1964.
W R I G H T . G . H. von, Wittgenstein, Oxford, Basil Blackwell, 1 9 8 2 .

37. Heidegger

PRESENCIA DEL PRESENTE EN E L PASADO,


A U S E N C I A D E LA P R E S E N C I A EN E L PRESENTE

El término aristotélico fundamental para la presencia,


etiergeia, sólo se traduce correctamente por nuestra palabra
Wirklicbheit (realidad) si, en lo que a nosotros respecta,
pensamos wirkert (obrar) a la manera de los griegos, en el
sentido de: conducir-hacia lo no-oculto, producir-en la
presencia, Wesen (ser) es la misma palabra que Wubren
(durar), permanecer. Pensamos la presencia (Anwesert)
como permanencia de lo que, una vez llegado al no-
ocultamiento, permanece allí. Sin embargo, posterior-
mente a Aristóteles, este significado de etiergeia, durar en
la obra, ha sido cubierto por otras acepciones. Los roma-
nos traducen, es decir, piensan, ergon a partir de la operatio

266
entendida como actio, y energeia actus lo expresan con una
palabra muy diferente que posee un dominio de significa-
do muy diferente también. Aquello que ha sido con-
ducido con el pro-ducto aparece ahora como resultado de
una operario. El resultado es lo que se sigue de y sigue a una
actio: la con-secuencia. Lo real es ahora el consecuente. La
consecuencia es conducida por una cosa que le precede,
por la causa (causa). Lo real aparece ahora bajo la luz de la
causalidad de la causa efficiens. Dios mismo está representa-
do en la teología, no en la fe, como causa prima, como la
primera causa. Finalmente, después de la relación causa-
efecto, la relación pasa a ocupar un primer plano, y con
ella el transcurso temporal. Kant conoce la causalidad en
términos de una regla de la sucesión. En los trabajos más
recientes de W. Heisenberg, el problema causal es un pro-
blema puramente matemático de medida del tiempo.
Sólo hay otra cosa más, y no menos esencial, que está uni-
da a este cambio (en la concepción) de la realidad de lo
real. Lo que en el sentido del consecuente había sido ob-
tenido mediante una operación, se muestra a nosotros
como algo que se ha manifestado en un «hacer»; esto es,
ahora, es una realización y un trabajo. Lo que había sido
obtenido «después de» en el hecho (Tat) de un «hacer»
tal, es lo real «de hecho» (das Tatsacblicbe). La palabra
tatsachlicb («de hecho») sirve en la actualidad para certifi-
car y decir tanto «ciertamente» como «seguramente».

«Ciencia y meditación» (1953).

COMENTARIO

En este texto Heidegger examina un «deslizamiento se-


mántico» que ha alterado el significado originario del tér-
mino realidad o presencia. Desde los griegos (Aristóteles)
a los alemanes modernos, pasando por los romanos, se
ha perdido algo que Heidegger quiere recuperar reto-
mando las palabras que designan la presencia misma del
presente.

267
Debémos partir del sentido griego de la palabra energeia
para comprender qué quiere decir realidad (El término aris-
totélico... —*• ...permanecealli). Los vocablos latinos actio, ope-
ratio, no son el equivalente de producir, e introducen la
noción de causa (Sin embargo, posteriormente... ...laprimera
causa). Los modernos (Kant, Heisenberg) piensan la cau-
salidad como sucesión temporal (Finalmente... " ...medida
del tiempo). En suma, la realidad pierde enteramente la di-
mensión esencial del presente efectivo para designar un
«consecuente» que es resultado de un trabajo (Tan sólo...
—*• ...seguramente).
2, Comprender el significado de energeia en griego su-
pondría haber resuelto ya el problema que plantea la tra-
ducción alemana Wirklicbkeit. En ambos casos se trata de
aquello que está ahí, verdaderamente presente, de forma
efectiva y no como promesa. Al igual que en el caso de la
entelequia, de la cual habla nuestro autor líneas más arri-
ba del comienzo del fragmento seleccionado, la energeia no
es ninguna «energía» cuyo uso suponga la promesa de
efectuar algo que uno espera; ella es esa misma cosa en
acto, totalmente realizada, del mismo modo que la ente-
lequia es el resultado, obtenido de forma absoluta, de un
proceso o de una evolución. Diciendo «resultado» o tér-
mino, se sugiere que el presente se refiere a su pasado
pata existir, pero Aristóteles es enérgico al plantear que el
presente que aparece como un resultado y está en acto es
absolutamente primero con relación a eso que precede o
proyecta su cumplimiento1. Tendría más verdad el decir
que el tiempo no define la presencia del pasado: lo que lo
define es su estar ahí, su ofrecerse ya cumplido y dispues-
to para ser aprehendido (teoría, contemplación).
El texto consiste, desde el principio hasta el final, en
cernir la diferencia de concepción que nos distancia del
presente en el sentido aristotélico, que es primero y autosu-
ficiente. El punto de llegada será una paradoja filosófica,
la de lamentar aquello de lo que el presente carece, mien-

' Aristóteles, Física, II, 3, 7.

268
tras que se lo plantea, por otra parte, como perfectamente
suficiente.
El problema de la traducción del griego al alemán se
basa en que añade una noción impensable desde su senti-
do primero. Wirkikbkeit es «efectividad», y wtrken, efec-
tuar u obrar. No se trata de tomar lo real por el resultado
de un trabajo que sería esencial, sino el de redescubrir el
sentido griego de la producción. No es esfuerzo o artefac-
to, es la aparición del presente, surgido de lo meramente
virtual (la mariposa surge de la oruga en tanto que verdad
de la oruga). Ser significa, por tanto, durar, mientras que
lo virtual no dura porque se metamorfosea en aquello
que en realidad era.
3. Podemos volver a delinear la historia de la altera-
ción de sentido que introduce el tiempo en la noción ple-
na del presente. El latín sitúa la acción en un presente
hasta entonces ligado a la contemplación. Actio, operario
no establecen simplemente que el presente es obra, sino
que hacen de él un resultado o una consecuencia, algo
que se sigue lógicamente de algo que, en cuanto causa, de-
viene primero. La noción misma de causa (cuya genealo-
gía aparece elaborada en Die Frage rtach der Technik, inclui-
do en el mismo volumen de donde se ha extraído nuestro
texto) reemplaza a la de un ser en el presente dotado de
sus «garantías» (aitia). Eficiente, la causa determina sus
«efectos» y les confiere la realidad que, sin ella, no ten-
drían. Ser real y estar efectuado se convierten en sinóni-
mos, y la teología concibe, entonces, al gran Dios arqui-
tecto como razón de los efectos mencionados.
4. A continuación, Heidegger reconstruye las des-
venturas sufridas por el concepto de causa cuando se lo
entiende como prioridad temporal. Kant, al hacer de él
un concepto puro del entendimiento, eterniza la relación
de causa a efecto, «donde el primer término determina al
segundo en el tiempo»2. Antes que él, Hume proponía la

2 Critica de la rasfin pura, Analítica transcendental, cap. 2, 3.a sec-

ción, 5 3.

269
sucesión regular de dos percepciones en la experiencia
como origen y contenido del concepto de causalidad1,
Heisenberg enlaza la causalidad con la medida del tiem-
po, la realidad deviene función intrínseca al curso tem-
poral, pensado como secuencia causal.
5. Sin embargo, la lengua alemana añade a estos ava-
lares algo más, una alteración juzgada como más esencial.
Causar es preceder, determinar, pero también «hacer». El
alemán marca esta dependencia de la concepción de lo
real frente a una acción fabricadora al llamar Tatsáchlich-
keii a la realidad efectiva, y ya no solamente Wirlkbkeit:
Tal es el acto hecho, y tatsüthlickkeit, «de hecho», realmen-
te o ciertamente; introduciendo la noción subjetiva de
una certeza en la noción misma de la realidad, los moder-
nos han logrado vaciar la noción griega de energeia de su
sentido originario, pleno y autosuficiente, autónomo. La
primacía se desplaza, pues, a lo «fabricado» y a lo «asegu-
rado» allí donde el griego eclipsaba al hombre ante la in-
vencible evidencia de una aparición del ser que exige una
contemplación desinteresada.
¿Estudia Heidegger únicamente desplazamientos se-
mánticos, o mutaciones conceptuales, o, más aún, evolu-
ciones históricas marcadas con el signo de la decadencia?
En el plano del lenguaje, ¿es pertinente oponer un térmi-
no aristotélico muy riguroso a un giro del lenguaje vulgar
apropiado tan sólo en el caso del alemán contemporáneo
(tatsáchtích)? Como el propio autor hace, simultanean-
do las dos actividades, plantear esta cuestión supone ya
criticar.

1 Hume, Investigación sobre el tttíendimitnlo humano, 4, 1 sección.

270
38. Heidegger

¿ O M N I P R E S E N C I A O AUSENCIA DEL PROPIO H O M B R E ?

El destino del desocultamiento no es en sí mismo un


peligro cualquiera, es el peligro. Pero, si el destino nos
gobierna según el modo de la im-posición, entonces él es
el peligro supremo. El peligro se muestra a nosotros des-
de dos aspectos diferentes. Desde el momento en que lo
no-oculto ya ni siquiera como objeto le concierne al
hombre, sino exclusivamente como fondo, y el hombre,
en el interior del sin-objeto, no es quien requiere el fondo
—y sigue entonces el hombre su camino hasta el borde
extremo del precipicio, a saber, hacia donde él mismo ha
de ser considerado como fondo. Sin embargo es justa-
mente el hombre quien se pavonea y erige en señor de la
tierra. Así se extiende la apariencia según la cual todo lo
que sale al encuentro subsiste sólo en cuanto algo hecho
por el hombre. Esta apariencia, a su vez, madura una últi-
ma ilusión engañosa: parece que, por todas partes, el
hombre no encuentra nada más que a sí mismo. Heisen-
berg ha señalado con todo derecho que al hombre actual
. lo real no puede presentarse de otro modo. (...) No obstan-
te, boy en día el hombre en ninguna parte se encuentra ya en verdad a
si mismo; es decir, a su esencia (Wesen). El hombre está tan de-
cisivamente avasallado por la provocación de la im-
posición, que no la percibe como una interpelación, que
no se ve a sí mismo como el interpelado, y así pasa por
alto también todos los modos (que le permitieran com-
prender) en qué medida ek-siste, en razón de su esencia,
en el ámbito de un llamamiento y por ello jamás puede en-
contrarse sólo a sí mismo.

«La cuestión de la técnica» (1953).

271
COMENTARIO

1. ¿Cuál es el verdadero peligro en nuestra época tec-


nocientífíca? Aquí, Heidegger1 aporta una respuesta que
pertenece al conjunto de una interpretación de la moder-
nidad como era de la técnica. En el momento en que el
hombre se cree rey de la tierra, él mismo desaparece en la
indistinción del fondo que explota,
A lo largo de la lectura del texto podemos reparar en
algunos temas familiares, tales como la ausencia del hom-
bre, su pérdida de esencia o el artificialismo universal.
Lo esencial, sin embargo, consiste más bien en poner
de manifiesto la unidad sistemática de una interpretación
fundamentada en ciertas innovaciones terminológicas o,
incluso, poéticas, que conforman el esqueleto de una ar-
gumentación invisible a fuerza de haber pasado a las cos-
tumbres filosóficas ambientes.
El texto presenta una estructura pautada en cuatro
tiempos diferenciados como momentos de una demostra-
ción. El genuino y único peligro es el «destino del des-
lumbramiento», situado éste bajo el gobierno del llama-
miento (Gestell, im-posietón en traducción castellana) (Eldes-
tino... * ...dos aspectos diferentes).
De los dos aspectos del peligro, el uno consiste en ame-
nazar al hombre mismo de caer precipitadamente en el
«fondo» explotable que ha hecho de la naturaleza en la
era técnica (Desde el momento en que... —*• ...como fondo). Una
segunda forma de peligro es la de ser víctima de una ilu-
sión que enmascara el peligro: el hombre cree ser el amo
único de la realidad exterior en el momento mismo en
que corre el riesgo de perderse (Sin embargo... —* ...por el
hombre).
Un tercer peligro (Heidegger sólo anunciaba dos) es el

1 M. Heidegger, Lt tournant (die Kibrt), Cuestiones IV, París, Galli-

mard, 1976. ( .on viene leer este texto paralelamente al ofrecido en este
comentario.

272
que el hombre se imagine omnipresente y crea descubrir
su ser allí donde no está (en la naturaleza subyugada, Esta
apariencia... —* ...de otro modo), aun cuando (último punto,
No costante... ...a sí mismo) este ser depende por comple-
to del llamamiento indefinible que lo conduce a la domi-
nación técnica del mundo. Así, sólo es él mismo en la
alienación de sí con respecto a aquello que lo llama
(pronto sabremos que se trata del ser).
2. ¿Cómo comprender que el «destino de desvela-
miento» sea el único peligro? Pese a una traducción que, a
menudo, es opaca, hemos de dar crédito a Heidegger y
admitir lo real de la inquietud que muestra esta fórmula.
Si hay un peligro, piensa nuestro autor, no procederá
del exterior, de los accidentes o avatares que acechan a la
técnica; es su misma esencia la que es peligrosa. Ahora
bien, ésta no consiste esencialmente en disponer o instru-
mentar la naturaleza, pues ella es, verdaderamente, apari-
ción o desvelamiento de aquello que es2, según una mo-
dalidad sencillamente más brutal que el caso de los grie-
gos. Así, pues, el «destino» del hombre moderno es el de
«desvelar» el ser según la modalidad técnica del Gesteti
(movilización, llamamiento, imposición). No hay nin-
gún peligro que supere en gravedad a aquel que prescribe
esta imposición, y es así como hemos de entenderlo.
Un primer aspecto del peligro supremo reside en el he-
cho de que atañe a la totalidad de la naturaleza sometida a
la técnica, si bien no es tarea fácil para el hombre escapar
al estatus universal del fondo explotable porque apenas si
es nada más que su administrador. El razonamiento de
Heidegger, en este punto, establece que, entre el prisione-
ro (el fondo) y el guardián de la prisión (el hombre que lo
«retiene»), existe una comunidad de destino tal, que la di-
ferencia podría acabar desapareciendo, lo cual no presen-
ta inconveniente alguno para el hombre (su destino es
obedecer), aunque sí para el ser, pues éste necesita de él
para conocer su propia aparición. Y si ya no cabe hablar
del hombre, tampoco del destino. Será preciso, entonces,

2 Cfr. texto 37.

273
conjurar el peligro de ver al hombre precipitarse en un
abismo que él mismo ha abierto para allí «almacenar» la
naturaleza al modo de una reserva de energía o fondo. En
lo esencial, esta es la argumentación que cuenta, aunque
expresiones tales como las de no-oculto, objeto sin-objetoyfon-
do merezcan, cada una de ellas por separado, intermina-
bles comentarios.
3. Seguidamente, la engañosa figura del señor de la
tierra pasa a tomar forma. El hombre se cree amo absolu-
to de la tierra, al igual que Dios en los cielos. Aparte, Hei-
degger caracteriza toda la ciencia cartesiana moderna
como un proyecto de dominio (y posesión) de la natura-
leza, Si todas las cosas están en sus manos, la realidad no
mantiene ya más su relativa permanenecia, a no ser en su
estatus de artefacto humano «técnico», y ello porque se
trata de cosas consideradas como naturales. Lo «engaño-
so» no reside en confundir naturaleza con artificio, sino
en creer que nos encontramos, por todas partes, con
«criaturas del hombre» (Gemáchte der Menschen). A riesgo
de caer en un ecologismo piadoso, hemos de recordar
que, según Heidegger, el «Dios fabricante» es, igualmen-
te, una ficción filosófica propia de la era técnica'. La ver-
dad es que el hombre no es más que el administrador del
fondo, y el destino común al administrador y lo adminis-
trado consiste en cumplir los mandatos del llamamiento
(Gestell).
4. El peligro es, en suma, esa última ilusión que en-
mascara, ante la mirada del hombre técnico, el sentido
profundo de su misión: puesto que ya no cabe hablar de
una independencia de los objetos (ni de objetos), sino
únicamente de sustratos, de fondos por administrar, el
hombre, al encontrar sus pseudocriaturas por todas par-
tes, concluye que tan sólo puede subsistir con entera in-
dependencia él, el hombre. Apariencia necesaria4, aun-

' Cfr. texto 37.


4 Cfr. el texto de Heisenberg comentado por Heidegger en el frag-
mento que hemos seleccionado (texto 37).

274
que ilusoria (trügerischer Schein), que vela... el desvelamien-
to mismo, entendido como destino de occidente.
5. El hombre carece de la posibilidad de volver a encon-
trarse a sí mismo como independencia o libertad autosu-
ficiente, no solamente en todas partes, sino donde quiera
que buscase. Esta tesis «existencialista» puede ser entendi-
da en términos de ausencia de una «naturaleza humana»,
de transcendencia (el hombre es algo que debe ser supera-
do). Aquí, sin embargo, se trata de la esencia de la técni-
ca, que es la que da sentido a la formulación heideggeria-
na. El texto alemán emplea, a la hora de caracterizar la si-
tuación del hombre, dos expresiones que la traducción
no ha logrado plasmar debido a una primera, y funda-
mental, confusión. La pro-vocación de la imposición,
¿qué quiere decir? ¿y en el caso de la interpelación y del
ámbito de un llamamiento en el cual él ek-siste? ¿Quién
podrá estar orgulloso de haber leído este texto en su tra-
ducción castellana?
Heidegger no ha escrito dos veces llamamiento, interpela-
ción, como tampoco im-posimn. Para el término llamamiento
cuenta el alemán con dos expresiones, Ansprucb y Zus-
pruch, y para imposición, Gestell. Ansprucb significa preten-
sión, reivindicación; Zuspruch es una palabra dirigida a al-
guien con el objeto de consolarla; en cuanto a Gestell, sería
muy prolijo intentar traducirlo, a no ser que recordemos
que Gestellung significa «llamamiento a filas» ¿Será, enton-
ces, alistamiento, asignar un papel? El destino (el ser que
quiere su aparición) llama, apela al hombre y lo moviliza
para realizarse; así, pues, se dirige a él: reivindica (Ans-
prucb), protege y cubre (Zuspruch), y esta misión hace de
él, con respecto a sí mismo, el señor de la tierra, aunque este

* «Alistamiento», en francés, se corresponde con la palabra «enróle-


ment», mientras que tiróle», que aquí hemos traducido por «asignar un
papel» presenta, al menos, dos acepciones que interesan especialmente
en nuestro caso: «papel» (que alguien o algo interpreta o juega) y «lista».
Aunque, en lo semántico, la importancia recae en el primer caso, el jue-
go de palabras «enrólementsAróle» sólo sería apreciable en nuestro
idioma si tradujésemos la segunda acepción por «lista», lo cual parece
alejarse del sentido del testo francés original. [N. del T.}

275
programa no tenga más finalidad que la parusía del ser y
no la del hombre, quien, «en orden a su esencia», sólo po-
see existencia dentro de los límites de ese alistamiento
al que ya hemos aludido. De lo anterior se sigue que
el hombre nunca se podrá encontrar solamente a
sí mismo, pues lo que encuentra es el ejército del ser.
Cabe suponer que esta concepción de la «técnica», pre-
tendidamente válida para toda la modernidad, tiene
como base unas fechas bastante más precisas de lo que
pudiera parecer a simple vista, fechas que nos remiten a
ciertas elecciones históricas propias de Heidegger, Sin
menoscabar en nada su valor, lo anterior constituye la
primera precaución que debemos tomar para compren-
der siquiera de qué habla nuestro autor.

PARA SABER MAS

Sobre Heidegger (textos 37y ÍS)

MAGEE, B., Los grandesfilósofos, trad. Ama ¡a Barcena, Madrid, Cá-


tedra, 1990, págs. 273-303.
POGGELER, O., El camino delpensaren Martin Heidegger, trad. Félix
Duque, Madrid, Alianza, 1983.
S T E Í N E R , G . , Heidegger: Ja influencia y diseminación de su pensamiento,
J. Aguilar Mora, México, FCE, 1983.

Otros

HABERMAS, J., Ciencia y técnica como «ideología», trad. M. Jiménez


Redondo y M. Garrido, Madrid, Tecnos, 1984.
HUME, D., Investigación sobre el conocimiento humano, trad. Jaime de
Salas, Madrid, Alianza, 1980.
KANT, I„ Crítica de la razón pura, trad. Pedro Ribas, Madrid, Alfa-
guara, 1978.

276
39. Sartre

S A B E R ANTROPOLÓGICO Y COMUNIDAD HUMANA

(...) En la medida en que la antropología, en un cierto


momento de su desarrollo, advierte que niega al hombre
(debido al sistemático rechazo del antropormofismo) o lo
presupone (como hace el etnólogo a cada momento), exi-
ge, implícitamente, saber cuál es el Ser de la realidad hu-
mana. Entre un etnólogo o un sociólogo —para quienes
la historia, con demasiada frecuencia, no es más que el
movimiento que altera los perfiles— y un historiador
—para quien la permanencia misma de las estructuras
supone un perpetuo cambio— la diferencia esencial y la
oposición que entre ellos se establece tienen su origen
mucho menos en la diversidad de sus métodos que en una
contradicción más profunda que alcanza al sentido mis-
mo de la realidad humana. Si la antropología debe ser un
todo organizado, deberá superar esta contradicción
—cuyo origen no reside en ningún Saber, sino en la pro-
pia realidad— y constituirse a sí misma como antropolo-
gía estructural e histórica.
Esta tarea de integración sería fácil de ser posible po-
ner al día algo así como una esencia humana, es decir, un
conjunto fijo de determinaciones a partir de las cuales se
pudiera asignar un lugar definido a los objetos estudiados.
Sin embargo, y sobre este punto coinciden la mayor parte
de los investigadores, la diversidad de grupos —aborda-
dos desde el punto de vista sincrónico— y la evolución
diacrónica de las sociedades impiden que la antropología
encuentre su fundamento en un saber conceptual. Sería
imposible hallar una «naturaleza humana» común a los
Muria, por ejemplo, y al hombre histórico de nuestras so-
ciedades contemporáneas. Inversamente, sin embargo,
en ciertas situaciones se establecen o pueden establecerse
una comunicación real y una comprensión recíproca en-

277
tre existentes también distintos (por ejemplo, entre el et-
nólogo y los jóvenes Muria que hablan de su gotbul). El
movimiento de la antropología suscita de nuevo y de una
nueva forma «la ideología» de la existencia por tener en
cuenta los dos caracteres opuestos mencionados (ninguna
naturaleza común, comunicación siempre posible).

Critica de la razón dialéctica.

COMENTARIO

1. Este texto, que versa sobre una cuestión de méto-


do, plantea el problema del estatuto de la antropología
como saber y como posibilidad independiente de ese sa-
ber. ¿Qué puede haber en común entre dos hombres tan
diferentes que el etnólogo y el «primitivo» a quien inte-
rroga? Y si no hubiera nada en común, ¿tendría su pro-
yecto algún sentido? Así se presenta la paradoja del presu-
puesto de la antropología: sin poder jamás encontrar la
esencia humana universal, suscita, debido a su propio
movimiento, la cuestión del ser del hombre.
La cuestión misma, que surge en el terreno de las cien-
cias humanas, rompe con las «ideologías de la existencia»
al suscitarlas. Sartre rehabilita el proyecto de una antro-
pología que Heidegger había reemplazado por la cuestión
de la esencia del hombre y del destino del ser. Toda su
problemática supone una vuelta a la antropología filosó-
fica, antropología que sería rechazada en bloque por ta fe-
nomenología.
El primer párrafo desarrolla la contradicción que opone
las disciplinas para las cuales su movimiento es alteración
a aquéllas para las cuales la permanencia es lo que está
por alterar (oposición entre etnología e historia). La con-
tradicción no es insuperable, pero supone que interroga-
mos al ser de la realidad humana.
El segundo párrafo hace más precisa la dificultad ante-
rior. En efecto, los investigadores no encontrarán jamás
una esencia humana común a todos, aunque postulan
una comunicación que siempre es posible.

278
2. Sartre había planteado líneas más arriba que el
existencialismo bien entendido «es la antropología mis-
ma en tanto que busca proveerse de un fundamento»1.
¿Quiere decir con esto que es una «ciencia humana»?
Para ser fiel a la lección de Husserl, Sartre admite que
toda «ciencia» del «hombre» (no está hablando en este
momento de ciencias del espíritu) carece de la dimensión
consistente en interrogarse acerca del Ser de su objeto.
Sin embargo, la antropología descubre, en su movimien-
to propio, estas cuestiones de fundamento: nuestro texto
examina el surgimiento de la cuestión crítica desde dos
formas diferentes, la negación y la presuposición, que
devienen «observables» ante la mirada del propio inves-
tigador.
¿Cómo comprender esta «negación» del hombre debi-
do al rechazo del antropomorfismo? Esto apunta, según
parece, al estatuto de la disciplina, que no «tiene como fin
desvelar» la realidad humana, sino «constituir leyes y po-
ner al día las relaciones funcionales o los procesos»2. Ne-
gar al hombre y rechazar el antropomorfismo es, en el
fondo, la antesala del método, siendo, así, entonces,
igualmente verdadero afirmar que aquello que se niega
está, del mismo modo, presupuesto en otro orden, como
algo que va de suyo (la realidad humana indefinida es el
horizonte de la investigación acerca de una cuestión et-
nológica particular).
Lo anterior permite relativizar las diferencias de méto-
do entre las ciencias humanas, sí todas ellas se encuentran
ante la cuestión implícita de) ser del hombre. La contra-
dicción más profunda que se da entre historia y sociolo-
gía, historicidad y estructuralidad, desborda la especifici-
dad de los métodos para llegar hasta el presupuesto
común no formulado. ¿Cuál es, con exactitud, esa contradic-
ción que alcanza al sentido mismo de la realidad humanal ¿Por
qué su origen no reside en un saber, sino en la propia rea-
lidad?

1 Cfr. Critica de la razó" pura.


2 Cfr. Critica de ¡a razón pura.

279
Existe una diferencia entre el modo como tratan el he-
cho humano la historia, la sociología y, sobre todo, la et-
nología. Así, mientras que la etnia objeto de la investiga-
ción puede identificarse con una estructura estable, pro-
pia de sociedades «sin historia», la historia presupone la
historicidad de su objeto, es decir, una socie dad que no es
una sociedad cualquiera, sino evolutiva. Entre el sujeto
que investiga y su objeto se instaura firmemente una dia-
léctica, y ello porque el objeto, por causa de su historici-
dad, se aproxima a la interrogación acerca de su realidad
humana: «el interrogador se descubre, precisamente,
como el interrogado». Hacer de la antropología un «todo
organizado» no implica renunciar a las estructuras, sino
integrar en ellas la dimensión de la historicidad que con-
duce a la autoreflexión, a la investigación en sí misma.
Si la realidad humana no es el objeto de ningún saber;
¿será por no estar completa, acabada, o por una imposibi-
lidad inscrita en su propio ser? «Distinguir ser de saber»,
como pretendía Kierkegaard, ¿será este el sentido último
de la experiencia antropológica? El segundo párrafo se ocu-
pa de abordar por sí mismo esta cuestión.
3. No se trata simplemente, en este punto de nuestro
texto, de recordar que «la existencia precede a la esencia»
en el hombre. La investigación preliminar de una esencia
tal como conjunto fijo de determinaciones implicaría que la an-
tropología es un saber, aunque privado de sus conceptos,
cuando esta privación posee, más bien, el valor de un
enigma que dirige al investigador, sociólogo o etnólogo, ha-
cía el proyecto existencíal que él mismo pone implícita-
mente en marcha.
No puede existir «nada común» por conceptuaüzar en-
tre el hombre de las sociedades sin historia y el nuestro,
una comunicación real y, sobre todo, una comprensión reciproca
pueden marcar la relación fundamentalmente disimétri-
ca entre el investigador y el objeto de sus estudios, ese
Muria que le informa acerca de su gotbul, Comprensión reci-
proca, tales son los términos que empleaba Husserl para
designar la característica esencial de una comunidad fun-
dada en la filosofía. Así, sin renunciar a ser investigación,

280
conocimiento, saber, que se refiera a las relaciones defini-
das y mensurables, la antropología denostada por Hei-
degger es, según Sartre, aquello que suscita de nuevoy de una
nueva forma la ideología de la existencia,

PARA SABER MÁS

De Sartre:

ElSery la Nada. Ensayo de Ontoiogia Femmenológica, trad. Juan Val-


mar, Madrid, Alianza, 1984,
El existendalismo es un humanismo, Barcelona, Orbis, 1985.
W A E L H K N S , de A., «Sartre et la raison dialectique», Revm phih-
sopbique de Louvain, vol. 60, febrero de 1962, págs. 79-99.

Sobre Sartre

ABRAHAM, T . , Pensadores bajos (Sartre, Foucault, Deleuze), Abra-


ham, T. (compilador), Buenos Aires, Catálogos Edito-
ra, 1987.
E J . D E R S , L . , / P. Sartre: el sery ¿a nada, trad. G. Novas Pekteiro,
Madrid, Magisterio Español, 1977,
FOUCAULT, M., Laspalabrasy las cosas, op. cit., 1987.
JHANSON, F , , El problema moral y el pensamiento de Sartre, trad,
A. Llanos, Buenos Aires, Siglo X X , 1968.
L A F A R G B , R., La filosofía defean Paul Sartre, trad. M. R. Zurro Ro-
dríguez, Madrid, G. del Toro, 1970.
L A I N C , R. D. Y C O O P E R , D. G., Razón y violencia. Una década de
pensamiento sartreano, trad. Marta Eguía, Buenos Aires, 1973,
Biblioteca de Psicología Social y Sociología, vol. 14.
MACEE, B,, Los grandesfilósofos,trad. Amaia Bárcena, Madrid, Cá-
tedra, 1990, págs. 273-301.
SANGUINF.TI, J . J . , / . P. Sartre. Critica de la razón dialéctica y cuestión
de método, Madrid, Magisterio Español, 1975.

281
40. Lévi-Strauss

U N A D I V E R S I D A D D E PUEBLOS SIN HISTORIA

Desde nuestra perspectiva, por consiguiente, el yo no


se opone más al otro de lo que el hombre se opone al
mundo: las verdades aprendidas a través del hombre son
«del mundo», y es por ello que son importantes. Se com-
prende, pues, que encontremos en la etnología el princi-
pio de toda investigación, si bien, para Sartre, ésta suscita
un problema que toma la forma de una dificultad por su-
perar o de una resistencia por reducir. Y, en efecto, ¿qué
hacer con los pueblos «sin historia» cuando se ha defini-
do al hombre por la dialéctica, y a la dialéctica por la his-
toria? En ocasiones, Sartre parece estar tentado de distin-
guir dos dialécticas: la «verdadera», que sería la de las so-
ciedades históricas, y una dialéctica repetitiva y a corto
plazo que él concede a las sociedades llamadas primitivas,
situándola muy cerca de la biología. Expone así el con-
junto de su sistema, puesto que, indirectamente por me-
dio de la etnografía, que es, indudablemente, una ciencia
humana consagrada al estudio de tales sociedades, el pun-
to demolido con tanto escarnecimiento entre el hombre y
la naturaleza se encontraría subrepticiamente reestableci-
do. O bien Sartre se resigna a poner del lado del hombre a
una humanidad «raquítica y deforme», aunque no sin in-
sinuar que a la humanidad no le pertenece en propiedad
su ser y que éste es función de la humanidad histórica que
lo toma a su cargo: ya sea porque, en la situación colo-
nial, la primera haya comenzado a interiorizar la historia
de la segunda; ya sea porque, gracias a la etnología, la se-
gunda dispense la bendición de un sentido a una primera
humanidad que carecía de él. En ambos casos se deja es-
capar una prodigiosa riqueza y diversidad de hábitos,
creencias y costumbres; se olvida que, ante sus propios
ojos, cada una de las decenas o de centenas de millares de

282
sociedades que han coexistido sobre la tierra después de
que el hombre hubiera hecho aparición en ella se ha de-
cantado una certeza moral —semejante a la que nosotros
mismos podemos invocar— para proclamar que en ella
—se halle reducida a un pequeño grupo de nómadas o a
una aldea perdida en el corazón de los bosques— se con-
densaban todo el sentido y la dignidad de la que es capaz
la vida humana. Pero ocurra esto entre ellos o entre noso-
tros, hace falta mucho egocentrismo e ingenuidad para
creer que el hombre se ha refugiado por completo en uno
sólo de los modos históricos o geográficos de su ser,
cuando la verdad del hombre reside en el sistema de sus
diferencias y propiedades comunes.

El pensamiento salvaje.

COMENTARIO

1. Este texto constituye una réplica de la etnología a


la Crítica de ¡a razón dialéctica, cuyas tesis reducen el papel
que desempeñan las sociedades sin historia en la dialécti-
ca histórica.
Esta defensa representa también para la etnología un
análisis de inconsecuencias y contradicciones internas a
las cuales se ha llegado desde el momento en que se ha
pretendido buscar el ser del hombre en otro lugar que no
es el diagrama comparativo de diversas formas dadas de
la humanidad, en ese «afuera» que el dialéctico minimiza
o interioriza laboriosamente.
Las diez primeras líneas plantean la pertenencia del
hombre al mundo, principio que funda la etnología y le
confiere un carácter de punto de partida. Este punto
constituye una línea de demarcación con respecto al dia-
léctico, no tanto por su metodología como por la incapa-
cidad que demuestra de dar a los pueblos sin historia el lugar
central que merecen en el marco del mundo.
La segunda parte se ocupa, por entero, de desarrollar la
multiforme contradicción en la que Sartre cae al excluir
al hombre sin historia de la dialéctica.

283
Para finalizar, el balance y la recuperación del punto
de vista etnológico ocupan las seis últimas líneas, en
términos que anuncian la ulterior crítica del etnocen-
trismo1.
2. El yo no se opone (más) al otro... constituye la consigna
de toda etnología científica. En efecto, ir a visitar al otro
no es un exotismo que nos pueda distanciar de nuestro
yo, pues ellos no son idénticos, sino reveladores de pun-
tos comunes y diferencias significativas. Para estudiar al
hombre, es necesario aprender a dirigir la mirada «a lo le-
jos», escribe Rousseau2, y el etnólogo hace de ello su mé-
todo. Atreviéndose a ir más lejos que Rousseau, Lévi-
Strauss propone anular una segunda gran oposición: no
sólo el hombre, sino el mundo, si es cierto que son significa-
tivos juntos (estudiar los gusanos es descubrir su sociedad,
su industria,...). Si la etnología aprendiera sobre el hom-
bre sin decir nada acerca del mundo, surgiría como fiador
un antropocentrismo a expensas del espíritu científico,
ese «materialismo transcendental», ese «esteticismo» que
el autor reivindica contra Sartie (ibid.).
La indiferencia del yo servía de principio a Montaigne,
quien se estudiaba a sí mismo como si se tratase de una
hormiga. Aquí, la etnología la convierte en principio de
toda investigación, pero en un sentido que no hace posi-
ble que nadie que no sea «etnólogo de sí mismo» pueda
aprender nada. Lévi-Strauss contradice a Sartre, pero no
a Rousseau.
3. Ya hemos visto con anterioridad cómo la Critica de
la razón dialéctica abordaba la paradoja de la etnología. Esta
no es «saber», y el verdadero saber es dialéctico. En este
punto, el autor resume declarando que ella es problema, un
problema resistente y molesto que plantea dificultades.
No es, sin embargo, «principio», y plantea una dificultad
para el propio Sartre.
Sartre no puede integrar en su pensamiento dialéctico

1 Claudc Lévi-Strauss, Race et hatoire, París, Gonthier, Bibliothéque

«Méditations», 1961.
2 Cfr. texto 15.

284
a quienes han sido excluidos del mismo; tal es la eviden-
cia que se extrae. Ahora bien, ser sin historia es ser «sin
dialéctica», si la una define a la otra. Así planteado, el
problema entra en un punto muerto, aunque el ensayo
de diversas soluciones aparece como una confesión de la
debilidad del principio etnológico y sin intento por de-
fenderlo.
En ocasiones, Sartre parece estar tentado de distinguir dos dialéc-
ticas... 0 bien Sartre se resigna... dos vías, dos callejones sin
salida que Lévi-Strauss reconoce sucesivamente para re-
gresar, finalmente, a su método.
En ocasiones... Pero, ¿cuándo? En una nota al pie de
página, Sartre manifiesta un sensible embarazo a propósi-
to de «la historicidad» de las sociedades sin historia1. Es-
cribe este autor: «No habría ninguna necesidad de definir
al hombre por la historicidad —puesto que existen socie-
dades sin historia—, sino por la posibilidad permanente
de vivir históricamente las rupturas que trastornan a ve-
ces a las sociedades de repetición. Esta definición es, ne-
cesariamente, aposteriori; es decir, nace en el seno de una
sociedad histórica y es, en sí misma, el resultado de trans-
formaciones sociales. Sin embargo, viene a aplicarse a las
sociedades sin historia de la misma forma que la historia
misma vuelve sobre estas últimas para transformarlas
—por el exterior, en primer lugar, y, a continuación, en y
por la interiorización de la exterioridad.»
Elpensamiento salvaje parece constituir, aquí, una réplica
a la nota de Sartre que reclamaba su intervención, asi
como también ocurre en la página 203 de la misma obra,
que presenta, como los rasgos de una «naturaleza huma-
na» adquirida, las secuelas fisiológicas de la escasez.
Expone asi el conjunto de su sistema, puede tomarse en un
sentido algo ambiguo, pues el autor lo despliega, aunque,
asimismo, lo pone en peligro porque todo él se basa en la
ruptura practicada entre naturaleza e historia, abolida en
una sola frase (elhombre es ese ser raquítico...). La alternativa

1 Cfr. Crítica Je ¡a razón dialéctica, trad. M. Lamana, Buenos Aires, Lo-

sada, 1973, pág. 225, pifis. 103-104.

285
no es mucho mejor, puesto que si el hombre es lo que aca-
bamos de señalar, y todas las sociedades se confunden a
riesgo de perder su criterio de historicidad, ello implica
que la historia toma a su cargo a las otras sociedades y las
«historiza» al colonizarlas (triste definición de historia),
sea que esta «colonización» cultural adopte la forma más
sofisticada de... la etnología misma, que da sentido a su
objeto desde el exterior (este es uno de los aspectos de la
«paradoja antropológica», a la cual Lévi-Strauss no puede
permanecer insensible).
Nos resta constatar, probar, que estas antinomias o
aporías atañen a la filosofía de Sartre y no a la historia
contemporánea en sí misma.
4, Son tres los argumentos que se suceden a la hora
de coger en falta al «materialismo dialéctico» del papa del
existencialismo: la pluralidad irreductible de los hábitos y
costumbres; el etnocentrismo universal y, por último,
una cuestión de método, el lugar que debe ocupar el com-
paratismo en la antropología.
El primer punto acentúa la diversidad sincrónica o
geográfica, cuando Sartre privilegiaba el cambio diacró-
nico característico de Ja historiad Ahí reside la riqueza de
los pobres, ocurre siempre que, sin este postulado, el pro-
pio decurso de la etnología perdería su valor.
El segundo argumento se dirige al carácter general, y
no específicamente occidental, de la creencia en la posi-
bilidad de constituir una referencia universal. En efecto,
y si se pretende ser perfectamente persuasivo, sería nece-
sario aún explicar por qué la particularidad occidental es
la única que ha de presentar un proyecto de etnología
comparada, y esto después de lo que ya, en voz baja po-
dríamos decir, el Renacimiento (Montaigne) llevara a
cabo, o, antes aún, Plinio el Viejo. La relación entre la
«razón geográfica» y el proyecto etnológico ha venido in-
teresando, posteriormente, a antropólogos procedentes
de la antropología como Jack Goody.

4 Cfr. mpra, página anterior.

286
El aspecto metodológico recapitula y concluye: no es
posible saber «qué es el hombre» (de hecho, Sartre recha-
za la cuestión) sin discernir entre los puntos comunes y
las diferencias que los seres humanos manifiestan, algo
que la zoología ya ha llegado a comprender para entender
«al viviente» o a «la especie». Volvemos asi al punto de
partida: ¿se habrá de estudiar al hombre c o m o si se tratara
de una hormiga?, y, por otra parte, ¿habrá que estudiar si-
quiera al hombre?

PARA SABER MÁS

LKVI-STRAUSS, Claude, textos de y sobre C. Lévi-Strauss reuni-


dos por R. Belloury C. Clément, París, Colt. «Idees», 1979.

Sobre Lé»i-Strauss

BAOCOCK, C. R., Lévi-Strauss: el eslructuralismoy la teoría sociológica,


trad. Juan Almela, México, FCE, 1979.
— Tristes trópicos, trad. Noelia Bastard, revisada por Elíseo Ve-
rón, Buenos Aires, EUDEBA, 1970.
I.KV I-STRAUSS, C , De cerca y de lejos, trad. Mauro Armiño, Ma-
drid, Alianza, 1980.
LKVI-STRAUSS y otros, l^a antropología como ciencia, trad. A. Des-
monts, M. Valentí y M. Uría, Barcelona, Anagrama, 1975.
PI NC; AUD, B. y otros, IJvi-Strauss: estructuralismoy dialéctica, trad.
M. Reilly de Fayard, Buenos Aires, Paidós, 1968.

287
41. Piaget

D U R A C I Ó N I N F A N T I L Y T I E M P O ESPACIAL

De una manera general, la constitución del tiempo es...


paralela a la del espacio, asi como complementaria a las
de los objetos y la causalidad. En otras palabras, procede
igualmente de la inmediatez característica del egocentris-
mo radical a un establecimiento de relaciones tal que el
espíritu se libera de su punto de vista propio para situarse
en un universo coherente. El tiempo, pues, se confunde,
en su punto de partida, con las impresiones de una dura-
ción psicológica inherente a las actitudes de atención, es-
fuerzo y satisfacción; en suma, a la actividad del sujeto
mismo. A esta duración, seguidamente, se la hace entrar
en relaciones más y más estrechas con los acontecimien-
tos del mundo exterior. En su punto de llegada, el tiempo
es promovido al rango de estructura objetiva deJ universo
como tal: la sucesión de los actos del sujeto se inserta así,
en tanto que sucesión vivida, en la serie de los aconteci-
mientos rememorados, constituyendo la historia del me-
dio ambiente, en lugar de que esta historia continúe sien-
do incoherente, como en lo precedente, y permanezcan
sus fragmentos encadenados a una acción presente que es
concebida como la única real.
Vemos que, desde la inteligencia sensoriomotriz, el
tiempo desborda necesariamente la «duración pura» y
que, si esta duración está, en efecto, en los orígenes del
tiempo, jamás habría logrado hacerse realmente temporal
sin una espacialización y una objetivación inseparables de
la entera actividad intelectual. No es, pues, arbitrario,
que para describir las etapas de la constitución del campo
temporal y la memoria podamos servirnos del marco ya
preparado de los estadios característicos de la evolución
del espacio, de los objetos y de la causalidad misma.

La construcción de lo real en el niño.

288
COMENTARIO

1. Este texto presenta una ruta demostrativa que


conduce a la definición de un método de aproximación
para comprender la constitución del campo temporal y la
memoria. El tiempo no es tanto duración vivida inme-
diata como estructura objetiva del universo: se dispone,
pues, a la hora de conceptúa fizarlo, de un marco ya pre-
parado donde el espacio, los objetos y la causalidad evolu-
cionan a través de distintos estadios.
La solidaridad del tiempo y de las categorías genéticas
es afirmada en un primer momento, permitiendo así la
periodización de ía duración en función de la actividad
objetivante del sujeto (primer párrafo). De lo anterior se
infiere la posibilidad de definir para ella misma la evolu-
ción propia de la constitución del campo temporal sin te-
ner por ello que enfrentarse a las objeciones y obstáculos
de principio que conciernen a la duración cuando se la
considera como «dato inmediato», a la manera de Bergson.
2. La primera proposición extgs una atención: ¿por
qué se habla de paralelismo y, después, de complementa-
riedad?
La categoría general del espacio está constituida a par-
tir de experiencias objetivas que conciernen a las posicio-
nes sucesivas de un mismo objeto, así como a los despla-
zamientos correlativos de numerosos objetos, de los cua-
les, unos serán percibidos como «causa», los otros como
«efectos» de los desplazamientos observados. En tanto
que categoría, la espacialídad es una adición de experien-
cias. Por ello, coincide con la temporalidad, pues suma
las experiencias de sucesión como la espacialídad suma
Jos desplazamientos. La compíementariedad alude a que
la causalidad y el objeto descubren que las categorías más
generales de espacio y tiempo presentan el significado de
estructuras del universo ligadas a las capacidades objeti-
vas del sujeto.
Lo que ha sido adquirido en el conocimiento del espa-

289
cío se traslada, así, al conocimiento del tiempo, a pesar de
la existencia de una particularidad que supone un obs-
táculo. Nos referimos a que la «consciencia del tiempo no
se exterioriza en la forma de comportamientos aislables»,
como ocurre en el caso de la «consciencia de las relacio-
nes espacíales»1. La psicología genética debe, en conse-
cuencia, añadir un desvío o rodeo que permita compren-
der la consciencia del tiempo sin caer en lo artificial o fic-
ticio. Esto se consigue con el «paralelismo».
La dirección general de la constitución del tiempo, así
como su división en etapas particulares, caen en el terre-
no de lo arch¡conocido. Esta se dirige desde lo inmediato
hasta la mediación o relación: a partir de una experiencia
que no es intercambiable, se hace posible, en conexión
con la consciencia del tiempo universal, la puesta en co-
mún de un mundo objetivo y objetivado por ta actividad
intersubjetiva. Así, el tiempo, en paralelo con el espacio,
constituye un universo coherente que podrá ser aprehendido
científicamente mediante las relaciones que lo atraviesan.
Es preciso disponer, en el punto de partida, un míni-
mo de estructuración sin el cual no encontrarían apoyo
alguno las etapas ulteriores. Es en este sentido como he-
mos de entender la «confusión» primera del tiempo con
la actividad del sujeto mismo, pues el desarrollo y la es-
tructuración de la actividad, en conexión con estructuras
racionales y relacionadas más complejas, determinarán la
aparición del «tiempo objetivo»; el tiempo netamente
«psicológico», subjetivo, experimentado como duración
pura,-cuyo ritmo está marcado por la atención y su relaja-
miento, no es, por tanto, nada más que una forma casi in-
mediata que la mediación del mundo vendrá a desplegar
en toda su amplitud.
Como ya lo hiciera M. Halbwachs en sus Marcas sociales
de la memoria, Piaget persigue una demostración filosófica
cuyos gastos corren a cargo del bergsonismo. La memoria
está objetivada en la realidad social, de la que es una de

1 Cfr. supra, en página anterior.

290
sus estructuras, y no es lícito disociar los dos aspectos,
forma y contenido. El secreto del tiempo se localiza del
lado de la organización del mundo, no ae su deconstruc-
ción, de modo que aparece como vivido cuando es lo que
permite, precisamente, pensar y actuar. Piaget no está
contradiciendo a Bergson cuando se dirige a buscar ese
secreto en las formas de la vida activa que presentan una
organización superior, en lugar de hacerlo en las formas
soñadoras de la vida separada del mundo. Estima, sin
embargo, que, privilegiado el movimiento hacia el pasa-
do, la filosofía de la duración no podrá encontrar la ver-
dad del tiempo, sino apenas un esbozo infantil de su po-
tencia organizadora.
Tal es lo que aparece al final del primer párrafo: re-
encontrar el «tiempo perdido» no significa revivir ni
«reencontrarse»; es introducir un orden y una coherencia
en la sucesión de los actos del sujeto, y, puesto que tal or-
den es objetivo, constituir así la historia del medio ambiente.
3. La idea de una duración pura resulta problemática
para la investigación genética. El indagar sobre el modo
como el tiempo se constituye en la forma de una expe-
riencia objetivante imposibilita, de antemano, el poder
privilegiar las intuiciones de formas sin contenido: puro
fluir, devenir irreprimible. Así, pues, el tiempo no es
puro, sino que compone, informa, acompaña a la expe-
riencia desde el estadio primero de la inteligencia sensoriomo-
triz. Este es el primer punto. Pero, además, el tiempo no
tiene en la duración su principio, como tampoco su ver-
dad. Razonando retroactivamente, Piaget hace surgir la
cuestión de la génesis: aquello que deviene, evoluciona y
se desarrolla debe encontrar en sus formas arcaicas de
existencia los rudimentos de lo que, posteriormente, ven-
drá a desplegar; todo el pensamiento acerca del progreso,
intelectual u orgánico, se enfrenta con esta constricción
(desde Condorcet y Lamarck a Auguste Comte, Marx,
Freud e, incluso, Bergson). De ahí que, cuando la dura-
ción se encuentra en los orígenes del tiempo, precisa, sin
embargo, ser, a la vez, adecuada a una temporalidad aca-
bada, cumplida, que adopta la forma de la estructura objeti-

291
va del universo. En este punto, el imperativo metodológico
se une al problema filosófico del tiempo. En efecto, antes
de llegar a ser realmente temporal, la duración que creemos
pura presenta en su origen una composición secreta, es
inseparable de las formas esbozadas propias de la entera
actividad intelectual, la cual implica tanto objetos como
acciones.
Nos está permitido, pues, concluir que el método ge-
nético aborda el tiempo y la memoria, de forma no arbi-
traria, mediante una vía que puede parecer la más alejada
de su naturaleza: la de la espaciaiídad, la objetualidad, la
causalidad, que determinan la articulación de su constitu-
ción en estadios.

PARA SABER MÁS

Sobre Piaget

ANDRÉ, N., Jean Piaget, trad. S. Mastrangelo, México, FCH,


1979.
B O D E N , M . , Piaget, trad. C. García Trevijano, Madrid, Cáte-
dra, 1982.
DELVAL, J . A., El animismo y el pensamiento infantil, Madrid, Siglo
X X I , 1975.
DROZ, R„ Como leer a Piaget, trad. Al Romero, México, FCH,
1984.
MAURY, L , Piaget et fenfant, París PUF, Col!. «Philosophies»,
núm. 2.

42. Foucault
CIENCIA DEL HOMBRE O FINITUD

En cierto sentido, el hombre está dominado por el tra-


bajo, la vida y el lenguaje: su existencia concreta encuen-
tra en ellos sus determinaciones; sólo se puede tener acce-
so a él a través de sus palabras, de su organismo, de los

292
objetos que fabrica —como si, de antemano, éstos (y qui-
zás únicamente éstos) detentaran la verdad; y él mismo,
desde que es capaz de pensar, sólo se desvela ante sus pro-
pios ojos en la forma de un ser que ya es, con un espesor
necesariamente sub-yacente, con una irreductible ante-
rioridad, un viviente, un instrumento de producción, un
vehículo para las palabras que le anteceden en existencia.
Todos estos contenidos que su saber le revela como exte-
riores a él y más antiguos que su nacimiento se anticipan
a él, hacen que toda su solidez se desplome y lo atraviesan
como si no fuera nada más que un objeto de la naturaleza
o un rostro que debe diluirse en la historia. La finitud del
hombre se anuncia —y de forma imperiosa— en la posi-
tividad del saber; se sabe que el hombre es finito como se
conoce la anatomía del cerebro, el mecanismo de los cos-
tes de producción o el sistema de la conjugación indoeu-
ropea, o, más bien, se anuncia en la filigrana de todas es-
tas figuras sólidas, positivas y plenas, se perciben la fini-
tud y los límites que ellas imponen, se adivina como en
blanco todo lo que ellas hacen imposible.
Pero, a decir verdad, este primer descubrimiento de la
finitud es inestable; nada permite detenerla sobre sí mis-
ma; y ¿no se podría suponer que ella promete ese mismo
infinito que rechaza, según el sistema actual?

Las palabras y las cosas.

COMENTARIO

1. El descubrimiento de la finitud, ¿se sitúa del lado


de los demás saberes, o bien los hace desplomarse hasta el
punto de llevar la atención a su principio, el de ser sabe-
res del hombre? Tal es la cuestión general que plantea este
extracto, así como la reflexión global de Foucault acerca
de las «ciencias humanas», sobre las cuales dirige su ar-
queología.
La primera parte del texto puede parecer, a primera
vista, que desarrolla el tema que resume, con mucho, el

293
hallazgo de Foucault; la «muerte del hombre» (hasta el
primer punto y aparte). Pero el principio del segundo pá-
rrafo relativiza el resultado obtenido en el primero: la fi-
nitud humana no es esto, no es un correlato de, positivida-
des, pues es más profunda que ellas.
2. Estar acabado, para el hombre, ¿significa estar do-
minado, desplomado por aquello que es de él o con respec-
to a él, pero lo supera en longevidad, consistencia y reali-
dad? El hombre, ¿es el hijo de sus obras, más joven, más
frágil? El pensamiento de las formas (Cassirer) introduce
la obra humana por encima de todo lo demás, como hori-
zonte de la historia. Una concepción de la finitud huma-
na es compatible con ella, como un humanismo que in-
tegra la precariedad de los individuos en conjuntos
interhumanos que poseen la continuidad de la que aquél
carece.
Es la miopía de estas concepciones la que aborda
esta defensa en favor de la finitud radical, del «fin» del
hombre.
En cierto sentido... el texto precisa, en primer lugar, en
qué sentidos hemos de entender esa dominación ejercida
sobre el hombre, y de qué experiencias habremos de dar
cuenta a la hora de definirlo por la subordinación a las
positividades.
Hay una idea simple y primera del hombre que debe ce-
der su puesto en tres dominios del saber que aparecen
enumerados líneas más arriba, en la misma obra (trabajo,
vida, lenguaje): la economía política, la biología y la filo-
logía constituyen la revelación de tres modalidades del
ser que preceden al hombre y lo determinan.
Tres son los puntos de vista posibles desde los cuales
contemplar esta anterioridad, pues ella es ontológica, me-
todológica y reflexiva.
Ontológica mente, el hombre concreto, individual, no
está determinado como «este hombre» más que por las lí-
neas generales de una combinatoria donde se incluyen los
criterios de la competencia y del empleo, de la edad y del
sexo, de la lengua y de la enseñanza de la misma. Su ca-
rácter determinante es evidente, aunque la ficción filosó-

294
fica de el hombre se enfrenta con el problema de la especifi-
cación, entendida como un obstáculo que obliga a pasar
por los tres saberes mencionados para acceder a «todo
hombre», a cada hombre determinado.
Para un saber objetivo, comenzar por las determinacio-
nes se convierte en una necesidad de método que posibi-
lita acceder a eso que se denomina el hombre, si bien toda
«ciencia humana», especulativa o técnica, se encuentra,
forzosamente, con las tres determinaciones y las tres dis-
ciplinas antes de tener acceso a su objeto. Está claro que ¡a
verdad del hombre está «detenida» por algo que no es él
mismo, sino ese «sí»* sin consistencia que se disuelve en
sus propiedades. La impresión de que, de antemano, éstos
(quizás únicamente éstos) detentarían la verdad es, sin duda,
algo ilusorio, aunque también inevitable.
En suma, no es tanto la experiencia del pensamiento
como sus condiciones y límites los que conducen al hom-
bre mismo, desde que es capaz de pensar, a una anterioridad
irreductible. Al pensarse, el hombre no encuentra un ser
primero, sino la forma de un ser que se desvela como vida,
trabajo y lenguaje. En este punto, sería muy clarificador
el ejemplo de un pensamiento, en sentido fuerte, que die-
ra testimonio en este sentido (recordemos a Descartes).
Esta triple experiencia, ¿refleja una verdad última, un
fin real del hombre, o bien la condición de posibilidad de
un «saber del hombre» necesariamente denudado de ver-
dad? La insistencia del texto en el estatus de saber (que su
saber le revela como exteriores... se sabe que el hombre esfinito...) ha ha
de ser entendido en el sentido del desbordamiento ulte-
rior ¿qué sería un saber del hombre, aunque finito, si se lo
reduce constantemente a determinaciones más rigurosas
que son el objeto de disciplinas específicas? Decir que el
hombre es finito corre, entonces, el riesgo de convertirse
en la mera tautología del saber. Una y la misma cosa es

* Aquí, como ya apreciará el lector, el «sí» no es una afirmación,


sino un pronombre de tercera persona como el que pueda aparecer en
expresiones tales como «para sí», como ya hemos tenido ocasión de ver
en capítulos anteriores [N. del T.j.

295
enumerar las positividades y concluir que el hombre es
ese blanco que rodea al texto que aquéllas configuran. Se
sabe que el hombre está (dominado, desplomado), se lo define
negativamente, se sabe su finitud, su finitud es, pues,
objeto de un saber. Tal es el sofisma que habrá que superar.
En efecto, el estatus que el proyecto de un saber antro-
pológico confiere a su objeto es el de no ser nada más que
una pésima definición, vaga y demasiado extensa, que
terminará desvaneciéndose completamente. El trabajo, la
vida y el lenguaje, planteados como verdaderos objetos
del saber, conducen necesariamente al anuncio imperioso de
la finitud humana, no como verdad, sino como residuo o
desecho del saber. En otras palabras, si el hombre es lo
negativo para las positividades, ello significa que el posi-
tivismo niega al hombre, aun cuando, al mismo tiempo,
afirme que es él el objeto de su ciencia (la Ciencia del
hombre de los «antropólogos» y los «observadores del
hombre», aquella que proyectara Saint-Simon, que toda
la tradición existencia lista ha tomado como objetivo últi-
mo). No hay que cargar esta aporía a la cuenta de un sa-
ber suplementario (del cual, además, ya no cabe hablar),
sino de una «inestabilidad» que obliga a una inversión
metodológica.
3. Esta última aparece condensada en el segundo pá-
rrafo, en la forma de la constatación de una insuficiencia.
En efecto, ¿qué quiere decir esa finitud? Si lo que la defi-
ne es el modo de saber propio de las positividades, no será
nada más que una interrupción del tiempo, una estabili-
zación: el hombre es ese «objeto finito» que podemos es-
tudiar porque permanece idéntico a sí mismo. Pero, ¿y si
evoluciona, y si las condiciones actuales del saber se vie-
ran trastocadas? Ocurre, en efecto, según el sistema actual,
que el hombre se ve a sí mismo rechazar el infinito, si bien
la proposición puede leerse en sentido inverso: el hombre
está dado al saber como un ser «indefinidamente» evolu-
tivo, según las tres direcciones de una biología,una eco-
nomía política y una lingüística.
Si lo que se busca es descubrir verdaderamente la fini-
tud y su verdad, concluir de lo anterior la «infinitud» del

296
hombre es imposible. Ahora bien, la continuación de
nuestro texto pone de manifiesto que la finitud es, en sí
misma, la verdad del saber, de la que está intrínsecamente
desprovisto. Así, pues, la precariedad del hombre ya no
es límite, precariedad o debilidad de las ciencias huma-
nas, sino su principio y fin.

PARA SABER MÁS

Sobre Foucault

ABRAHAM, T., Pensadores bajos (Sartre, Foucault, Deletfíp), Abra-


ham, T. (compilador), Buenos Aires, Catálogos Edito-
ra, 1987.
BLANCHOT, M,, Michel Foucault tal y como yo lo imagino, trad.
M. Arranz, Valencia, Pre-Textos, 1988.
COUZENS H O Y , D., Foucault, David Couzens Hoy (Compilador),
trad. A. Benano, Buenos Aires, Nueva Visión, 1988.
DELEUZE, G., Foucault, trad. J . Vázquez Pérez, Barcelona, Pai-
dós, 1987.
D R E Y F- USS/RA B [ N O W, Michel Foucault más allá del estructuraiismoy la
hermenéutica, trad. C. de 1 turbe, México, U. A.M., Instituto de
Investigaciones Sociales, 1988,
RASSAM, J . , Michel Foucauit: las palabras y las cosas, trad, M. Olasa-
gasti, Madrid, Magisterio Español, 1978.
VEYNE, P., Cómo se escribe la historia: Foucault revoluciona ¡a historia,
trad. J. Aguilar, Madrid, Alianza, 1984.

297
Apuntes biográficos

ANAXÁGORAS (428-300 a.C.), filósofo, matemático y astrónomo


griego, originario de Clazómenes (Asia Menor), autor de un
tratado: De la naturaleza.
ANDRÓNICO de Rodas (muerto hacia 6 0 a . C . ) , filósofo peripaté-
tico editor de las obras de Aristóteles.
ARISTÓTELES ( 3 8 4 - 3 2 2 a . C . ) , filósofo griego originario de Esta-
gira, fundador de la escuela peripatética (Liceo). Cfr. tex-
tos 4 y 5.
ARNAULD, Antoine (1612-1694), teólogo francés, jansenista,
autor de la Lógica de Port-Roya! junto con Pierre Nicole.
BACHELARD, Gastón ( 1 8 8 4 - 1 9 6 2 ) , filósofo francés, profesor au-
tor de El nuevo espíritu científico (1934), l^a formación del espíritu
científico (1938) y La filosofía de! no (1940).
BACON, Francis, barón de Verulam (1561-1626), filósofo y
hombre de estado inglés, y autor del Novum Organum Scien-
tiarum.
BERGSON, Henri ( 1 8 5 9 - 1 9 4 1 ) , filósofo francés, profesor del Co-
legio de Francia. Cfr, texto 33.
BERNARD, Claude ( 1 8 1 3 - 1 8 7 8 ) , fisiólogo francés, autor de In-
troducción ai estudio de la medicina experimental (1865).
BRAHE, Tycho (1546-1601), astrónomo danés.
BRAM HALL, John, ( 1 5 9 4 - 1 6 6 3 ) , obispo inglés, autor de Defensa de
la verdadera libertad desde la necesidad antecedente y extrínseca, en res-
puesta a una obra tardía de Mr. T. Hobbes intitulada Tratado de la li-
bertad y la necesidad.
BRUNO, Giordano (1548-1600), filósofo italiano, copernicano,
quemado vivo por orden del santo oficio, autor de El universo
infinito (1548).

299
Bu F FON , Geotges Lou i s, LECLERC,conde de(1707-1788),ri atura-
lista francés, autor de una Historia natural en 36 volúmenes.
CHERBURY, Herbert de ( 1 5 8 2 - 1 6 4 8 ) , filósofo inglés, autor de De
la verdad (1624).
CICKRÓN, Marcus Tullius Cicero ( 1 0 6 - 4 3 a . C . ) , político y filóso-
fo romano, autor de tratados de inspiración estoica (DelDes-
tino, De la Amistad, De ¡os Deberes, De ¿as Leyes).
COMTK, Auguste (1798-1857), filósofo francés, profesor.
Cfr. texto 22.
COPKRNICO, Nicolás ( 1 4 7 3 - 1 5 4 3 ) , astrónomo polaco, autor de
la primera teoría científica del heliocentrismo.
COUSIN, Víctor ( 1 7 9 2 - 1 8 6 7 ) , filósofo francés y ministro de la
instrucción pública. Director de la escuela ecléctica, autor
de De lo verdadero, de lo bello, del bien y de numerosas obras de
historia de la filosofía.
CUDWORTH, Ralph (1617-1688), filósofo inglés perteneciente a
la escuela platónica de Cambridge, autor del Verdadero sistema
intelectual del universo (1678).
DARWIN, Charles ( 1 8 0 9 - 1 8 8 2 ) , naturalista inglés, autor de El
origen de las especies por vía de la selección natural (1859).
DKMI'X.RITO ( 4 6 0 - 3 7 0 a.C.), filósofo griego defensor del ato-
mismo.
DKSCARTKS, René (1596-1650), filósofo y matemático francés.
Cfr. textos 6, 7, 8.
DIDKROT, Denis ( 1 7 1 3 - 1 7 8 4 ) , escritor y filósofo francés, funda-
dor y colaborador de l^a Enciclopedia, autor de los Pensamientos
filosóficos (1746) y de los Pensamientos sobre la interpretación de la
NATURALES ( T 7 5 3 ) .
DIÍ.THHY, Wühelm ( 1 8 3 3 - 1 9 1 1 ) , filósofo alemán, profesor, au-
tor de una Introducción a las ciencias del espíritu (1883).
DÍÓCKNKS de S inope (413-327 a.C.), filósofo cínico cuyos escri-
tos se han perdido.
DURKHKIM, Émile ( 1 8 5 8 - 1 9 1 7 ) , filósofo y sociólogo francés,
profesor, Cfr. texto 31.
KI.ISABETH de Bohemia, Princesa Palatina (1618-1680), hija de
Federico V, rey de Bohemia, mantuvo correspondencia con
Descartes.
EMPÉDOCI.ES (490-430 a.C.), filósofo griego originario de Agri-
gento y uno de los «presocráticos» más célebres.

300
T

EPICTETO (50-130), esclavo liberto, filósofo griego estoico, au-


tor de Coloquios (tratados de moral).
EPICURO (341-270 a.C.), filósofo griego, uno de los representan-
tes más ilustres del «materialismo» antiguo.
ESTRATÓN (330-268 a.C,), filósofo griego natural de Lámpsaco,
segundo escolarquio (director de una escuela) del Liceo.
EUDOXO (406-355 a.C.), astrónomo y filósofo griego.
FERMAT, Pierre de (1601-1665), magistrado y matemático fran-
cés y uno de los inventores de la geometría analítica y el
cálculo de probabilidades.
FEUERBACH, LUDWIC ( 1 8 0 4 - 1 8 7 2 ) , filósofo alemán alumno y
crítico de Hegel. Autor de una Critica dt la filosofía begeliana
(1839) y de La esencia del cristianismo.
FICHTE, Johann Gottieb (1762-1814), filósofo alemán, profe-
sor, el primero de los grandes «postkantianos» y autor de
una Wissemcbftslebre (Teoría de la ciencia) (a partir de 1794) y
del Discurso a la nación alemana (1807-1808).
FLUDD, Robert (1574-1637), téosofo inglés, autor de una Pbtlo-
sopbia Mosayca in qua sapientia et scientia creatioms et creaturarun ex-
plicatur (1638).
FOUCAULT, Míchel (1856-1939), filósofo francés, profesor del
Colegio de Francia. Cfr. texto 41.
FREUD, Sigmund (1856-1939), médico austríaco fundador del
psicoanálisis. Cfr. texto 29.
GAULEO, Galileo Galilei (1564-1642), astrónomo y físico
italiano, fundador del método experimental de la física mo-
derna.
GILBERT, William (1544-1603), médico y físico inglés, inicia-
dor de los primeros experimentos relativos a la electrostáti-
ca y el magnetismo.
GOCLENIUS, Rudolf Gockel, llamado (1547-1628), filósofo ale-
mán editor de una Psychologia (1590).
GORGIAS (485-380 a.C,), sofista y retor griego.
GROTIUS, Hugo de Groot, llamado (1583-1645), jurista, filósofo
del Detecho y diplomático holandés, autor de Dejure belli ac
pacis (1625).
HALLER, Karl Ludwig von (1768-1854), político y jurista suizo,
teórico contrarrevolucionario, autor de una Restauración de la
Ciencia política.

301
HARVEY, William (1578-1657), médico y anatomista inglés, au-
tor de la primera teoría completa de la circulación de la san-
gre (De motu coráis et sanguinis m animalibus, 1628).
HEGEL, Georg Wilhelm Friedrich (1770-1831), filósofo ale-
mán, profesor. Cf. textos 19, 20, 21.
H E I D E G G E R , Martin ( 1 8 8 9 - 1 9 7 0 ) , filósofo alemán, profesor.
Cfr. textos 36 y 37.
HEISENBERG, Werner (1901-1976), físico alemán, Premio No-
bel de 1932, autor de La naturaleza en la física contemporánea.
HERÁCLITO (545-480 a.C.), filósofo griego natura) de Éfeso y
uno de los «presocráticos» más ilustres.
HIPIAS (segunda mitad del siglo v a.C.), sofista griego natural
de Elis.
HOBBES, Thomas ( 1 5 8 8 - 1 6 7 9 ) , filósofo inglés autor de De Cive
(1642), del Leviatban (1651), de los Elementos de Filosofía (Del
Cuerpo, 1655, Del Hombre, 1657).
HUME, David (1711-1776), filósofo escocés, historiador, econo-
mista, hombre de estado, Cfr. texto 14.
HUSSERL, Edmund (1859-1938), filósofo alemán, profesor,
fundador de la fenomenología transcendental. Cfr. tex-
tos 3 4 y 3 5 .
JENÓFANES (s. VI a.C.), filósofo griego natural de Colofón, fun-
dador de la escuela de Elea.
JOUFFROY, Théodore ( 1 7 9 6 - 1 8 4 2 ) , filósofo francés que dio a co-
nocer, sobre todo a través de sus traducciones, la filosofía es-
cocesa. Autor de Miscelánea filosófica.
JUSTINO (S. II d.C.), historiador romano.
KANT, Inmanuel (1724-1804), filósofo alemán, profesor.
Cfr. textos 16, 17, 18.
KIERKEGAARD, Soren (1813-1855), filósofo danés critico de los
«sistemas»filosóficos)»(Hegel en particular). Cfr. texto 26.
LEEUWENHOEK, Antoine van (1632-1723), naturalista holan-
dés, microscopista y descubridor de entidades indefinida-
mente pequeñas.
L E I B N I Z , Gottfried Wilhelm ( 1 6 4 6 - 1 7 1 6 ) , filósofo, físico, mate-
mático, historiador, jurista y diplomático alemán. Cfr. tex-
tos 11 y 12,
LÉVI-STRAUSS, Claude ( 1 9 0 8 ) , filósofo y antropólogo francés,
profesor del Colegio de Francia. Cfr. texto 39.

302
LINNEO, Cari von ( 1 7 0 7 - 1 7 7 8 ) , naturalista sueco, autor del Sis-
tema de la naturaleza (1735), inventor de la nomenclatura bi-
naria (1753).
IXXIKK, John (1637-1704), filósofo inglés. Cfr. texto 13.
LUTKRO, Martin (1483-1546), teólogo alemán, fundador del
protestantismo alemán. Autor del tratado De servo arbitrio
(1525) en respuesta al tratado de Erasmo Del libre arbitrio,
MAQUTAVKI.O, Niccoló Machiavelli (1469-1527), político e his-
toriador florentino, autor de El Príncipe (escrito en 1513, pu-
blicado en 1532) y de los Discursos sobre la primera década de Tito
Livio,
MAINH DK BIRAN, Marie Frangois Pierre Gontier de Biran, lla-
mado (1766-1824), filósofo francés autor de De la descomposi-
ción del pensamiento (1805) y de un Ensayo sobre les fundamentos de
la psicología (1812).
M A R O DK K R O N ( . A N D , J e a n Marcus ( 1 5 9 5 - 1 6 6 7 ) , médico, mate-
mático y físico alemán, profesor en Praga.
MARX, Karl (1818-1883), filósofo, historiador y economista ale-
mán, fundador del materialismo histórico. Cfr. textos 24 y 25.
MAUSS, Marcel (1872-1950), sociólogo y etnólogo francés, pro-
fesor, autor del Ensayo sobre el don.
MKNIPO (siglos IV-IM a.C.), filósofo cínico griego, autor de
obras satíricas.
MOLINA, Luis de ( 1 5 2 5 - 1 6 0 0 ) , teólogo español, jesuíta, autor de
la Concordia del libre arbitrio con los dones de ¡a gracia, la presciencia
divina, la predestinación y la reprobación.
MONTKSQUIBU, Charles de Secondat, barón de la Bréde y de
(1689-1755), magistrado, escritor y filósofo francés, autor
del Espíritu de las leyes (1748).
MORO, Thomas (1478-1535), hombre de estado y filósofo in-
glés, autor de Utopia.
NKWTON, Isaac (1643-1727), matemático, físico y filósofo in-
glés, autor de los Naturalis pbilosopbiae Principia matbemati-
ca (1687).
NIKTZSCHK, Friedrich ( 1 8 4 4 - 1 9 0 0 ) , filósofo alemán, Cfr. tex-
tos 27 y 28.
PARACEI.SO, Philippus Aureolus Theophrastus Bombastus von
Hohenheim, llamado (1493-1541), médico suizo, profesor,
padre de la medicina denominada «hermética».

303
PARMÉNIDES (540-450? a.C.), filósofo griego natural de Elea y
uno de (os más destacados «presocráticos».
PASCAL, Blaise (1623-1662), matemático y filósofo francés, jan-
senista, inventor del cálculo de probabilidades, autor de
Cartas provinciales (1657) y de Pensamientos.
PtAGET, Jean (1896-1980), epistemólogo, psicólogo suizo fun-
dador de la escuela genética, Cfr, texto 40.
Picor, abate Claude, prior de Rouvre, amigo de Descartes, tra-
ductor de los Principia de Descartes del latín al francés.
PLATÓN (427-347 a.C.), filósofo griego natural de Atenas, fun-
dador de la Academia. Cfr, textos, 1, 2, 3.
PLUTARCO (50-125), escritor griego crítico del estoicismo y del
epicureismo. Autor de tratados. Obras morales. Vidas Paralelas.
POLO (S. V-IV a.C.), sofista y retor griego.
PRÓDICO (s, v a.C,), sofista griego natural de Ceos.
PROTACORAS (485-411 a.C,), el más ilustre, junto con Gorgias,
de los sofistas griegos, natural de Abdera.
PUFFENDORF, Samuel (1632-1694), filósofo de! Derecho e
historiógrafo alemán, profesor, autor de De jure naturae ti
gentium (1672).
RENOUVIER, Charles (1815-1903), filósofo francés, fundador
del «neocrit¡cismo» francés, autor de Ensayos de critica general
(1854-1864) y de una Filosofía analítica de la historia (1896).
RIBOT, Théodule (1839-1916), filósofo francés, profesor del
Colegio de Francia, fundador de la Revista filosófica, pione-
ro de la psicología con vocación científica.
RICKERT, Heinrich ( 1 8 6 3 - 1 9 3 6 ) , filósofo alemán, profesor, fun-
dador de una escuela de pensamiento «no kantiana», autor
de Knlturwissensschaft und Naturwissenscbaft (Ciencia de la cultu-
ra y Ciencia de la naturaleza).
ROBERVAL, Gilíes Personnier de (1602-1675), matemático
y físico francés, profesor de matemáticas en el Colegio de
Francia.
ROUSSEAU, Jean-jacques (1712-1778), escritor y filósofo gine-
bríno, autor del Discurso sobre el origeny iosfundamentos de la desi-
gualdad entre los hombres (1735), del Contrato social (1762) y de
Emilio <1762). Cfr. texto 15.
ROVÉR-COLLARO, Pierre Paul, político y filósofo francés adscri-
to a la escuela ecléctica.

304
SAINT-SÍ MON,Claude Henri de Rouvroy, conde de (1760-1825),
soldado, filósofo y economista francés, autor del Sistema in-
dustrial (1820-1823).
SARTRE, Jean Paul (1905-1980), escritor y filósofo francés, papa
de! existencialísmo. Cfr. texto 38.
SAUSSURE, Ferdinand de ( 1 8 5 7 - 1 9 1 3 ) , lingüista suizo, fundador
de la lingüística «estructural». Cfr. texto 30.
SAVIGNY, Friedtich Karl von (1779-1861), jurista y político ale-
mán, filósofo del Derecho y profesor, fundador de la escuela
histórica alemana.
SCHOPENHAUER, Arthur ( 1 7 8 8 - 1 8 6 0 ) , filósofo alemán, Cfr. tex-
to 23.
SCOTO, John Duns (Í274-1308), filósofo y teólogo escocés,
franciscano, defensor de la tesis de la libertad absoluta de
Dios.
SÉNECA (4 a.C.-65 d . C . ) , filósofo romano, preceptor de Nerón,
representante del estoicismo, autor de tratados morales y de
las Cartas a Lucillo (63-64).
SIMIANO, Francois ( 1 8 7 3 - 1 9 3 5 ) , sociólogo francés, profesor,
autor de El salario, la evolución social y la moneda (1932).
SÍMMEL, Georg (1858-1918), filósofo y sociólogo alemán,
profesor, autor de Los problemas de la Filosofía de la Historia
(1892), Filosofía del Dinero (1900), El conflicto de la civilización
moderna (1918).
SMITH, Adam (1723-1790), filósofo y economista escocés, pro-
fesor. Autor de ¡as Investigaciones sobre la naturaleza y las causas de
¿a riqueza de las naciones (1778).
SÓCRATES ( 4 6 9 - 3 9 9 a . C . ) , filósofo griego, ateniense,
SPRNCER, Herbert ( 1 8 2 0 - 1 9 0 3 ) , filósofo inglés evolucionista.
Autor de Los principios de psicología (1855) y de los Principios de
sociología (1876-1896).
SPI NOZA, Benedictus, Barueh ( 1 6 3 2 - 1 6 7 7 ) , filósofo holandés.
Cfr. textos 9 y 10.
T O R R I C E L L I , Evangelista ( 1 6 0 8 - 1 6 4 7 ) , físico y matemático ita-
liano.
VOLTAIRE, Fran^ois Marte Arouet (1694-1778), escritor y filó-
sofo francés, autor del Ensayo sobre las cmtumbrtsy el espíritu de
las naciones (1756), Cartas filosóficas sobre los ingleses (1754), Dic-
cionario filosófico (1764).

305
WEBER, Max ( 1 8 6 4 - 1 9 2 0 ) , economista y sociólogo alemán, pro-
fesor. Autor de La Ética protestante y el espíritu dei capitalismo
( 1 9 0 1 ) , y Economía y sociedad ( 1 9 2 2 ) . Cfr. texto 32.
WICLEF, Wycliffe John (1320-1384), teólogo inglés, precursor
del anglicanismo.
WOLI-'F, Christían (1679-1754), filósofo y matemático ale-
mán, profesor, principal representante del racionalismo
dogmático,
ZKNÓN ( 3 3 6 - 2 6 4 A.C), filósofo griego natural de Citio, funda-
dor del estoicismo.

306
Bibliografía

Textos originales*:

PLATÓN, Fedón, 97b-c, 98b-d, 99a-b, en El Banquete, Fedón, Fedro,


trad. Luis Gil, Barcelona, Labor, 1987.
— Gorgias, 499e-500d, en Goteas, trad. F. García Yagüe, Madrid,
Aguila r, 1974.
— El Sofista, 241d-242a, 242c-243a, en La teoríaplatónica del torna-
miento, Teiteto y El Sofista, traducción y comentario ed. F. M.
Cornford, trad. N. L. Cordero y M. D, del Carmen Ligatto, Bar-
celona, Paidós, 1983.
ARISTÓTELES, Metafísica, A , 991a9-991b5, en Metafísica, trad. Valen-
tín García Yebra, edición trilingüe (griego, latín, castellano),
Madrid, Credos, 198Z
— Política, II, 3, I26lb4-1262al, en Política, trad Julián Marías y
María Araujo, edición bilingüe (griego, castellano), Madrid, Ins-
tituto de Estudios Políticos, 1970.
DESCARTES, «Los principios de lafilosofía.Carta del autor al traduc-
tor del libro, la cual puede servir aquí de prefacio», en Obras Es-

* A la hora de traducir los textos de los filósofos que aparecen en


esta obra, nos hemos atenido tanto at texto francés del original como a
traducciones castellanas ya existentes, procurando siempre respetar las
variantes introducidas por los autores franceses, cuyo objeto suele ser el
de acercar algunos conceptos filosóficos a la comprensión de estudiantes
aún no muy versados en filosofía. En el caso de los filósofos franceses,
nuestra traducción ha pretendido ceñirse en lo posible al original. En la
siguiente lista aparecen las ediciones en castellano de las obras de donde
proceden los textos, o, en su defecto, la edición en el idioma correspon-
diente. (N. del r.J

307
cogidas (Regúlete, Discurso del método, meditaciones metafísicas, Correspon-
dencia), trad. E. de Olaso y T. Zwanck, Buenos Aires, Char-
cas, 1980.
— «Carta a Elisabeth del 18 de agosto de 1645», en Obras Escogidas
(Regulen, Discurso del método, Meditaciones metafísicas, Correspondencia),
trad. E, de Olaso y T. Zwanck, Buenos Aires, Charcas, 1980.
— Discurso del método, en Discurso del método, Meditaciones metafísicas,
trad, Manuel García Morente, Madrid, Espasa Calpe, col. Aus-
tral, 1980.
SPINOZA, Ética, en Ética. Parte Quinta, Prefacio, trad. Vidal Peña,
Madrid, Alianza, 1986.
— Tratado Político, en Tratado político, cap. 1, Sl-§2, trad, A tí laño Do-
mínguez, Madrid, Alianza, 1936.
LEIBNIZ, Sobre los principios de ¡afilosofía, en Sobre los principios de tafiloso-
fía. Observaciones criticas sobre la parte genera/ de los principios cartesianos.
Sobre la Segunda Parte, sobre el artículo 64, trad. E, López y M.
Graña, Madrid, Gredos, 1989.
— «Reflexiones en torno a la libertad, la necesidad y el azar, a pro-
pósito de la obra que Hobbes ha publicado en inglés», en Escritos
en tornea libertad, e! azar y el destino, trad. R. Rodríguez Aramayoy
C, Roldan Panadero, Madrid, Tecnos, 1990.
LOCKE, «Ensayo sobre el entendimiento humano», en Ensayo sobre el
entendimiento humano. Libro I, cap. 1, $$ 2-3-4-5, trad. M. E. Gar-
cía, 2. vols., Madrid, Editora Nacional, 1980.
HUME, «Tratado sobre la naturaleza humana», en Tratado sobre ta na-
turaleza humana. Libro tercero, parte II, sección II, trad. Félix
Duque, Madrid, Tecnos, 1988.
ROUSSEAU, «Discurso sobre el origen y los fundamentos de la desi-
gualdad entre los hombres», en Del contrato social, Sobre las cienciasy
las artes. Sobre tos fundamentos de la desigualdad entre los hombres, trad.
Mauro Armiño, Madrid, Alianza, 1990.
KANT, Prolegómenos para toda metafísica futura que podrá presentarse como
ciencia, en nuestra traducción hemos seguido la edición alemana,
Prolegmtena syt einer jtden ktínfíígtn Metaphysik, díe ais Wtssenscbaft
wird aufítetm ktinnen, Stuttgart, Reclam, 1989. En castellano, Pro-
legómenos..,, trad. Julián Besteiro, Madrid, Sarpe, 1984.
— Sobre elfracasode todo ensayofilosóficoen la teodicea, en Sobre elfracaso de
todo ensayofilosóficoen teodicea, trad. Prof. R. Rovira, Madrid, Facul-
tad de Filosofía, U.C.M., 1992.

308
— «Comienzo verosímil de la historia humana», en Filosofía de ¡a his-
toria, trad. Eugenio Díaz, México, F. C. E , 1985, 4.a ed.
H E G E L , Enciclopedia de las cienciasfilosóficas, /, Lógica, en Enciclopedia..., /,
Lógica, añadido al $ 48, trad, E Ovejero, México, Juan Pa-
blo, 1974.
— Printipios de lafilosofiadtl derecho o Derecho natural y ciencia política,
Principios de ¡afilosofíadel derecho, parte III (El Estado), § 258, Ob-
servación, trad. Juan Luis VermaJ, Barcelona, EDHASA, 1988.
— Principios de lafilosofíadel derecho o Derecho natural y ciencia política, en
Principios de la filosofía del derecho, Prefacio, trad. J. L. Vermal, Bar-
celona, EDHASA, 1988.
COMTE, «Consideraciones filosóficas sobre las ciencias y los sabios»,
en Acerca del poder espiritual, selección de textos establecida, pre-
sentada y anotada por P. Arnaud, París, Le Livre de Poche, Col.
«Pluriel», 1978.
SCHOPENHAUER, El mundo como voluntad y representación, en Et mundo
como voluntad y representación, Apéndice «Crítica a la filosofía de
Kant», trad. E Ovejero, Buenos Aires, Biblioteca Nueva, 1946.
MARX, «Sobre la diferencia de filosofía natural en Demócfíto Y Epi-
curo», en Escritos de juventud, trad. Wenceslao Rozas, México,
FCE,198Z
— «Contribución a la crítica de lafilosofíadel derecho en Hegel», en
Escritos de juventud, trad Wenceslao Rozas, México, FCE, 1982,
KJERKEGAARD, Tratado de la desesperación, en Tratado de ta desesperación,
Trad. Carlos Líacho, Buenos Aires, Santiago Rueda, ed., 1976.
NIETZSCHE, La gaya ciencia, en La gtya ciencia, trad. Pedro González
Blanco, Barcelona, José J. Oiañeta, ed., 1S84.
— Consideraciones intempestivas, en Consideraciones intempestivas, Ter-
cera Parte, J 7, trad, E. Ovejero y Maury, Buenos Aires, Aguilar,
1962.
FRKUD, Compendio de psicoanálisis, en Compendio de Psicoanálisis, trad.
Luis López Ballesteros y de Torres, Madrid, Tecnos, 1985.
SAUSSURE, Curso de lingüística general, en Curso de lingüistica general, Pri-
mera Parte, § 2, trad. Mauro Armiño, Ma< ir^, Akal, 1980.
DURKHEIM, Las reglas del método sociológico, en Las Regias del método socio-
lógico y otros escritos sobrefilosofíade ¡as ciencias sociales, trad. S. Gonzá-
lez Noriega, Madrid, Alianza, 1988.
WEBER, «Ensayos sobre teoría de la ciencia», en Sobre la teoría de las
ciencias sociales, Primer Texto: «La objetividad del conocimiento

309
en las ciencias y la política sociales», Barcelona, Península, 1972.
BERGSON, Elpensamiento y le moviente, en El pensamiento y lo moriente,
trad. M. H. Alberti, Buenos Aires, La Pléyade, 1972.
HUSSERL, «La crisis de la humanidad europea Y la filosofía», en La
crisis de las ciencias europeas y tafenomenología trascendental, trad. jacobo
Muñoz y Salvador Mas, Barcelona, Crítica, 1991,
— «La crisis de la humanidad europea y la filosofía», en La crisis de
las ciencias europeasy lafilosofíatrascendental, trad. J acobo Muñoz y S.
Mas. Barcelona, Crítica, 1991,
W I T T G E N S T E I N , Tractatus kgko-phUosophicus, trad. Jacobo Muñoz,
10 vols., Madrid, Alianza, 1989.
H E I D E G G E R , «Ciencia y meditación», en VortrUge undAufsdtm, Teil I:
«Wissenschaft und Bessinnung, Pfullingen, 1954.
— «La cuestión de la técnica», en Tecnología, Ciencia, Naturaleza y
Sociedad. Antología de autoresy textos (por el Instituto de Investiga-
ciones sobre ciencia y tecnología), trad. M. E. Vázquez García,
14/Suplementos Anthtopos.
SARTRE Crítica de la razón dialéctica, en Crítica de ¡a razón dialéctica prece-
dida de cuestiones de método, trad. M. Larnana, Buenos Aires, Losa-
da, 1963.
LÉVI-STRAUSS, El pensamiento salvaje, en El pensamiento salvaje, trad.
F. González México, FCE, 1984.
Pi AGET, La construcción de lo real en el niño, en La construcción de lo real en el
niño, trad. Mabel Arruñada, Buenos Aires, Proteo, 1970.
FOUCAULT, Las palabras y las cosas, EN Laspalabrasy las cosas: una arqueolo-
gía más alia del estructuralismo y la hermenéutica, trad. Elsa Cecilia
Frost, México, FCE, 1988.

310

1
r

Anexo
Comentarios de textos defilosofíaespañola
CARMEN GARCÍA TREVIJANO
1. Séneca: El valor del tiempo

De tal manera debes obrar, querido Lucílio, que seas


dueño de ti mismo, y recoge y conserva el tiempo que
acostumbran arrebatarte, sustraerte, o que dejas perder.
Persuádete de que te escribo cosas ciertas: nos arrebatan
parte del tiempo, nos lo sustraen o Jo dejamos perder. La
peor de todas estas pérdidas es la que ocurre por negligen-
cia propia; y, si atentamente lo consideras, verás que se
emplea considerable parte de la vida en obrar mal, tnayor
aún en no hacer nada, y toda en hacer lo contrariode loque
se debía. ¿Quién me presentaras que dé su verdadero va-
lor al tiempo? ¿que aprecie el día? ¿que comprenda que
diariamente se acerca a la muerte? Nos engañamos al
considerar que la muerte está lejos de nosotros, cuando su
mayor parte ha pasado ya, porque todo el tiempo transcu-
rrido pertenece a la muerte. Haz, pues, querido Lucí lio,
lo que me escribes que haces; emplea bien todas las horas,
y menos necesitarás del porvenir cuanto mejor trabajas
en el presente. Mientras nos detenemos, transcurre el
tiempo. Todas fas cosas nos son ajenas, querido Lucilio,
solamente es nuestro el tiempo. De esta única cosa nos
puso en posesión la naturaleza, pero es tan ligera y resba-
ladiza que nos la puede quitar cualquiera; y tal es la nece-
dad de los hombres, que agradecen las bagatelas que se les
conceden y por nada cuentan el tiempo que se les ha dado
y que, sin embargo, tan grande cosa es que ni el más gene-
roso podría pagar jamás.

Lucio Arinco Séneca (ca. 4-65), Cartas a Lim-


ito (ca. 63-65), Epístola primera.

313
COMENTARIO

Esta carta de Séneca a Lucilio tiene hoy especia) actua-


lidad porque representa, como ha escrito la filósofa espa-
ñola María Zambra no, «el descubrimiento del tiempo»
humano por un autor antiguo.
En una primera lectura este texto puede parecemos
moderno sólo porque pide que administremos bien el
tiempo y no lo perdamos y nos recuerda la máxima «el
tiempo es oro» que preside la vida del hombre de nego-
cios en la sociedad industrial.
Pero una lectura más atenta nos revela que el descubri-
miento de Séneca es bastante más profundo. Igual que el
análisis del espacio, el análisis del tiempo físico como fe-
nómeno natural que miden los relojes ha interesado des-
de la antigüedad a la ciencia y a la filosofía. Pero el tiem-
po que analiza Séneca en este texto es el tiempo vivido
por el hombre, que sólo externamente se deja medir por
el reloj. En el tiempo humano, vivido desde dentro, los
instantes no son iguales entre sí. La vivencia del presente
se entrecruza con el recuerdo del pasado y con la visión
del futuro, y en el limitado plazo que transcurre entre el
nacimiento y la muerte el hombre tiene que esforzarse
por buscar y encontrar su perfección y su felicidad.
La gran aportación de Martin Heidegger, padre del
existencialismo, en su famosísimo libro El ser y el tiempo
(1927) consistió en afirmar que el hombre es constituti-
vamente tiempo, que no sólo «está en» el tiempo sino que
«es» tiempo, porque su esencia misma es la temporalidad.
El sentimiento de angustia que normalmente acompaña
a la vida humana y al que no puede sustraerse el que trata
de olvidar el problema refugiándose en la frivolidad, está
determinado, según Heidegger, por la oculta conciencia
de la fragilidad de esa nuestra condición temporal. Un
impresionante anticipo de esta tesis se encuentra en las
palabras de Séneca en el pasaje que comentamos: «Todas
las cosas nos son ajenas, querido Lucilio, solamente es

314
nuestro el tiempo. De esta única cosa nos puso en pose-
sión la naturaleza.» La precariedad ontológica de nuestra
condición temporal es finalmente denunciada por Séneca
al escribir que el tiempo es cosa «tan ligera y resbaladiza
que nos la puede quitar cualquiera; y tal es la necedad de
los hombres, que... por nada cuentan el tiempo que se les
ha dado y que, sin embargo, tan grande cosa es que ni el
más generoso podria pagar jamás».

PARA SABER MÁS

ZAMBRANO, María, Etpensamiento vivo de Séneca, Madrid, Cátedra,


Colección Teorema, 1987.
S T E I N E R , G . , Heidegger. ta influencia y diseminación de su pensamiento, tiento,
trad. J. Aguilar Mora, México, Fondo de Cultura Económi-
ca, 1983.

2. Vives: el pacifismo

Tales son los males de la discordia; éstos, los bienes de


la concordia. ¿Quién fue el que apartó al hombre de tan
grandes bienes y le arrastró a males tamaños? Debió de
ser el genio del mal, algún fiero enemigo nuestro; acaso,
el enojo de Dios, y, sin duda, el hombre mismo que se
daña a sí queriendo dañar a otro y que, a través de males
suyos ciertos, intenta ajenos males inciertos. Dime con
franqueza: ¿Monta tanto la discordia y aquella insana e
inhumana satisfacción de la venganza que se deban por
ella sufrir tantas molestias y males tantos, preteriendo y
abandonando tantos bienes y tantas ventajas?
Ambos estados, la concordia y la discordia, han dejado
en todo tiempo sus huellas correlativas en enérgico con-
traste. La concordia congregó el humano linaje, fundó las
ciudades, las engrandeció, las conserva; introdujo las ar-
tes provechosas a la vida, el holgado bienestar, el cultivo
del espíritu; hizo grandes hombres en sabiduría, en erudi-

315
ción, en virtud. La discordia, en deplorable paralelismo,
dispersó a los hombres y los hizo errantes, los llenó de
miedos y terrores, recelosos de todo lugar, desconfiados
de cualquier otro hombre, como el pie, no bien curado de
la lesión de una espina, no asegura su huella con firmeza.
La discordia, quitadas las leyes, roto el lazo de la concor-
dia, aventó la comunidad humana, derrocó edificios, vi-
llas, ciudades; arrancó cuanto tenía sus raíces en el suelo;
trajo el hambre, la peste, la falta de todas las cosas; trajo la
ignorancia, la holganza, la inmoralidad, y en la soldades-
ca licenciada reclutó los más valientes y atraidorados
bandoleros.

Juan Luis Vives (1492-1540): Tratado de la


Concordia y de la Discordia, Libro III (1529),
en Luis Vives, Obras Completas, Aguilar, Ma-
drid, 1948, reímpr. 1992.

COMENTARIO

Ante el hecho de la guerra, los tratadistas morales y po-


líticos manifiestan opiniones contrapuestas. Sólo para
unos pocos, los llamados «pacifistas», la guerra es un mal
intolerable que es posible erradicar, y es, consecuente-
mente, derecho y deber de todo hombre de conciencia es-
forzarse por conseguir ese objetivo.
El humanista español Luis Vives ocupa, junto con su
contemporáneo y amigo Erasmo de Rotterdam, un lugar
preeminente en la historia del pensamiento pacifista. En
el pasaje de Vives que comentamos la concordia y la dis-
cordia son, respectivamente, la paz y la guerra, o si se
quiere los estados de ánimo que dan lugar a una y otra. El
segundo párrafo de este pasaje especifica que la primera
de ellas, la concordia, es la fuente de todos los bienes; y la
segunda, la discordia, el origen de todos los males. La
concordia construye y la discordia destruye. La sobrie-
dad, la prosperidad y bienestar, la cultura, la ciencia y la
virtud son frutos de la concordia. El mal que se opone a
cada uno de esos bienes: ruptura del lazo social, destruc-

316
ción de campos y ciudades, hambre y miseria, enfermeda-
des, ignorancia y crímenes son consecuencias de la guerra
ocasionadas por su instrumento que es el ejército.
Siendo inequívoco el juicio de valor que merecen la
concordia y la discordia, el problema está en cómo erra-
dicar la discordia. Pero esto presupone el análisis de su
origen, y a ese análisis se orienta el párrafo primero del
pasaje de Vives, que se inclina por la hipótesis de que la
causa de la discordia se encuentra en un desorden de la
mente y del corazón del hombre: «Debió ser el genio del
mal, algún fiero enemigo nuestro; acaso, el enojo de Dios,
y, sin duda, el hombre mismo que se daña a sí queriendo
dañar a otro y que, a través de males suyos ciertos, intenta
ajenos males inciertos.»
De este análisis se desprende, según los pacifistas, que
la guerra podría ser eliminada, pero que esa eventual eli-
minación no puede ni debe fundarse en otra violencia,
sino en la corrección del desorden del corazón y de la
mente del hombre.

PARA SABER MÁS

ORTKGA Y GASSET, J., Vives-Goethe, Madrid, Colección El Ar-


quero, Revista de Occidente, 1973. (Este libio contiene el
ensayo «Juan Luis Vives y su tiempo» y otros tres artícu-
los sobre Vives.)
MARAÑÚN, Gregorio, «Luis Vives (Un español fuera de Es-
paña)».
KANT, lnmanuel, La paz perpetua, trad. F. Rivera Pastor, Ma-
drid, Colección Austral, Espasa-Calpe.
E I N S T E I N , Albert, «Escritos sobre la paz», Barcelona, Pe-
nínsula.
FKKUD, Sigmund, «Consideraciones de actualidad sobre la gue-
rra y la muerte», trad. Luis López-Ballesteros, Obras Comple-
tas, t. VI, págs. 2101-17, Madrid, Biblioteca Nueva.

317
3. Suárez: La guerra justa

Existen dos teorías erróneas sobre la guerra. Afirma la


primera que es intrínsecamente malo hacer la guerra y
contrario a la caridad. San Agustín atribuye esta senten-
cia a los maniqueos y, según Tomás Netter, esta misma
opinión sigue Juan Wicleff.
Sostiene la segunda teoría que es ilícita la guerra parti-
cularmente a los cristianos y en especial la guerra contra
los cristianos.
Erasmo defiende que no es lícito a los cristianos hacer
la guerra después que mandó Cristo envainar la espada.
También Lulero cayó en el mismo error al condenar
la guerra contra los turcos. Porque, decía, no se puede
hacer resistencia a la voluntad de Dios, que manda estos
castigos.
Explicaremos nuestra tesis en las siguientes conclu-
siones.
Primera conclusión: la guerra, en cuanto tal, no es intrínsecamente
mala, ni está prohibida a los cristianos.
Suelen aducirse varias objeciones:
1. Cristo dijo a San Pedro; «Mete la espada en la vai-
na.» Isaías dice: «De sus espada* harán rejas de arados...,
ní se ejercitarán en la guerra.»
2. Siempre acompañan a la guerra innumerables ma-
les; y se considera que es mala y naturalmente prohibida
aquella acción que prácticamente lleva siempre anejas
circunstancias ilícitas y ocasiona daños al prójimo. Ade-
más, la guerra se opone a la paz, al amor de los enemigos
y al perdón de las injurias.
Respondemos a estas objeciones.
1. Cristo nuestro Señor habla de aquellos hombres
que pretenden echar mano a la espada por iniciativa pro-
pia, especialmente si lo intentan contra la voluntad del
soberano.
2. Replica San Agustín a los argumentos deducidos
por raciocinio: Unicamente prueban que debe evitarse la

318
guerra en cuanto sea posible; y sólo debe ser intentada en
caso de extrema necesidad, cuando no quede otro reme-
dio; la guerra absolutamente no es un mal, ya que es acci-
dental a su naturaleza los males que se sigan de ella; ade-
más de que sucederían aún mayores inconvenientes si
nunca estuviera permitida.
Segunda conclusión: La guerra defensiva es licita y a veces hasta
obligatoria.
De la conclusión precedente fluye la primera parte de
esta proposición.
Todos los códigos conceden que es lícito repeler la
fuerza usando de la fuerza. Dice Cicerón que no ha sido
hecha esta ley por los hombres, sino que es innata a la na-
turaleza humana. La razón consiste en que es natural y
necesario el derecho a la defensa propia.
Tercera conclusión: la guerra, aun agresiva, no es intrínsecamente
mala, sino que puede ser honesta y necesaria.
Lo demuestra la misma tazón. Muchas veces es necesa-
rio que el Estado haga la guerra para evitar la injusticia y
reprimir a los enemigos perversos; que sin esto no po-
drían los Estados vivir en paz.
En conclusión. Está permitida la guerra por derecho
natural y, en consecuencia, también por la ley evangélica,
que en nada deroga el derecho natural, ni contiene nue-
vos preceptos divinos, a excepción de los que se refieren a
la fe y a los sacramentos. Es ridículo, por tanto, lo que de-
cia Lutero de que no era lícito resistir al castigo divi-
no. Dios no quiere estos males, sino que los permite:
luego no prohibe que puedan soslayarse con toda jus-
ticia.
Resta que expongamos lo que significa una guerra de-
fensiva y qué se entiende por guerra agresiva, pues a veces
tiene apariencia de agresión la guerra que simplemente es
un acto de defensa.
La injusticia puede ser acción que prácticamente se
está realizando o acción ya terminada, cuya reparación se
intenta por medio de la guerra. En este segundo caso la
guerra es agresiva; en el primero tiene todas las caracte-
rísticas de una defensa, siempre que se haga con la mode-

319
ración del que se proteje a sí mismo sin excederse en sus
derechos.
Cuarta conclusión: Deben cumplirse algunas condiciones para que
la guerra se baga licitamente.
Estas condiciones pueden reducirse a tres capítulos;
primero, poder legítimo para hacer la guerra; segundo,
una causa justa o un título jurídico; tercero, que se obser-
ve un modo digno y la equidad en el comienzo de la lu-
cha, durante las hostilidades y después de la victoria.
Esta conclusión tiene una explicación general. Aun-
que no es la guerra intrínsecamente mala, es uno de esos
negocios que muchas veces llegan a ser inmorales a causa
de los muchos males que lleva consigo. Por esta razón
son necesarias muchas circunstancias para que la guerra
sea lícita.

Francisco Suárez (1548-1617), «Tratado de la


Guerra», (1621), cap. 1.

COMENTARIO

El filósofo y teólogo español Francisco Suárez resu-


mió y actualizó en su tratado de la guerra la doctrina es-
colástica sobre esta materia en el momento histórico en
que emerge el derecho internacional moderno. Para esa
doctrina la guerra no es mala en sí misma sino en sus con-
secuencias (primera conclusión del texto comentado), y
puede ser incluso buena y justa cuando los males que
combate son mayores de los que puede causar (segunda y
tercera conclusión). El fundamento de estas dos conclu-
siones es que el derecho a la guerra puede ser considerado
como una extrapolación al orden social y político del de-
recho natural de cada hombre a la defensa propia. Cual-
quier ciudadano particular de cualquier Estado puede
ejercer ese derecho, aunque normalmente el amparo de
su gobierno le permite delegar en jueces y policía la repa-
ración de las injusticias que le sean inferidas por otros
ciudadanos. Pero cuando se trata de un conflicto entre X
dos Estados que no están amparados por una instancia o I

320
tribunal superior, la guerra puede ser, según Suárez, mo-
ralmente aconsejable por conformarse al derecho natural
si no viola directamente las normas evangélicas.
Por la analogía del conflicto político y social que es la
guerra con el caso de la injusticia o afrenta inferida a un
hombre particular, se puede entender la diferencia entre
la guerra defensiva (por la que se combate una injusticia
que se está sufriendo) y la guerra ofensiva (por la que se
castiga una injusticia ya inferida).
Suárez tiene el mérito de adelantarse, con otro gran
teólogo español, Francisco de Vitoria, a la moderna doc-
trina del derecho internacional formulada por Hugo
Grocio. En la actualidad la existencia de instancias o tri-
bunales superiores a los Estados ordinarios, como sucede
con la ONU, ayuda a la normalización de las relaciones
entre los pueblos. Pero esas instancias distan de ofrecer
aún garantías suficientes de imparcialidad y eficacia.
También es un mérito de la doctrina escolástica su
insistencia en que se cumplan determinados requisitos
para que un gobernante o jefe de estado pueda decir con
seguridad: «mihi licitum est bellare» («tengo derecho a
declarar la guerra»). Estos requisitos son: poder legítimo,
causa justa y modo o método de lucha adecuado (cuarta
conclusión), y sirven de contrapeso o contrapartida a los
males que se derivan de la contienda. Las práctica? actua-
les de guerra, que suelen guiarse por mero cálculo de po-
der y políticas de alianzas, tienden aolvidarestos requisitos.
Casos recientes son la guerra de Vietnam, la del Golfo
Pérsico y la aún no acabada en Yugoslavia.

PARA SABER MÁS

SUÁREZ, Francisco, Guerra, Intervención, Paz Internación^ (Edición


del Tratado de la Guerra, con introducción de Luciano Pete-
ña), Madrid, Austral, Espasa-Calpe,
KANT, Inmanue!, La paz perpetua, trad. F. Rivera Pastor, Ma-
drid, Colección Austral, Espasa-Calpe.
E I N S T E I N , Albert, Escritos sobre la paz, Barcelona, Península.

321
F R E K D , Sigmund, «Consideraciones de actualidad sobre Ja gue-
rra y la muerte», trad. Luis López-Ballesteros, Obras completas,
t. VI, págs. 2101-17, Madrid, Biblioteca Nueva.

4. Santayana: La esencia de la poesía

¿Por qué nuestros hombres prácticos reservan un lugar


a la religión en el fondo de su mundo? ¿Por qué Platón,
después de prohibir a los poetas, poetiza el universo en su
prosa? Porque el mundo de ía ciencia y de la práctica ex-
traído de la experiencia mediante la abstracción es dema-
siado violento para satisfacer incluso al hombre más irre-
flexivo y vulgar; la idealidad de la máquina que llamamos
Naturaleza, la con venció nalídad del drama que llama-
mos mundo, son demasiado notorias para no ser precibi-
das de alguna manera por todos. Cada una de ellas hace su
aparición a veces en el fondo del alma, y el hombre hace
frente a esta aparición con el pensamiento de la muerte o
preguntándose a sí mismo por el sentido y el valor de su
vida. Entonces recordará sus amores sofocados, sentirá
que sólo sus ilusiones le dieron alguna vez un sentido de
la realidad, que sólo sus pasiones le dieron esperanza y la
visión de la paz. Se volverá a sí mismo y a duras penas en-
contrará un sentido a partir de su experiencia; y se sentirá
inclinado a creer que todo lo que ocupaba su vida era un
sueño y un símbolo, y elevará su mirada hacia la verdad
que trasciende su mundo cotidiano.
Este momento plástico de la mente, cuando nos perca-
tamos de la artificialidad e inadecuación de lo que el sen-
tido común percibe, es el verdadero momento de la opor-
tunidad poética —una oportunidad, debemos tener el
valor de confesarlo, que es generalmente desaprovecha-
da. El esfuerzo de atención, la concentración y el enfoque
del pensamiento sobre la desconocida inmediatez de las
cosas, usualmente no producen nada más que confusión.
Nos sentimos aturdidos e invadidos por un sentimiento
de algo que es inexpresable, luminoso pero indístingui-

322
ble, plural pero unitario. Y en lugar de elevarnos hacia la
imaginación, nos sumergimos en el misticismo.
Alcanzar una desintegración mística no es la función
de ninguna de las artes; si alguna parece lograrlo, el efec-
to es sólo incidental, implicado quizá en el proceso de
construcción del objeto propio de ese arte, como cuando
se cortan árboles y se cava hasta las raíces para arrancar-
los y establecer las bases de un templo. Porque cada arte
se ocupa de construir algo. Y justamente porque la ima-
gen del mundo construida por el sentido común y la cien-
cia natural es una imagen inadecuada (un esqueleto que
requiere el añadido de la sensación antes de que pueda te-
ner vida), el momento en que nos percatamos de su ina-
decuación es por tanto el momento en que las artes más
excelsas encuentran su oportunidad. Cuando el mundo
está hecho pedazos ellas pueden aparecer y construirlo más
acorde con los deseos del corazón.
La gran función de la poesía es precisamente ésta: re-
parar el material de la experiencia, recogiendo la realidad
de la sensación y de la fantasía oculta bajo la superficie de
ideas convencionales, para construir luego con ese mate-
rial, vivo pero indefinido, nuevas y mejores estructuras,
más ricas, mejor ajustadas a las tendencias primarias de
nuestra naturaleza, y más verdaderas con respecto a las
posibilidades últimas del alma. Nuestro descenso hasta
los elementos de nuestro ser queda justificado entonces
por nuestro subsiguiente y más libre ascenso hacia su
meta; nos volvemos al sentido sólo para encontrar el ali-
mento de la razón; destruimos las convenciones sólo para
construir ideales.
Tal análisis en beneficio de la creación es la esencia de toda gran
poesía. La ciencia y el sentido común son a su modo poetas
de no menor orden, puesto que toman los materiales de la
experiencia y fabrican con ellos un mundo claro, simétri-
co y bello; sin embargo, la corrección misma de este arte
lo ha convertido en común. Sus figuras se han tornado en
mera retórica y sus metáforas en prosa. Pero incluso tal
como es, una concepción científica y matemática posee
una belleza superior a la de la poesía irracional de la sen-

323
sación y el impulso, que 110 es sino un mero cosquilleo en
el cerebro, como el del alcohol, y juguetea con nuestros
fortuitos e imaginativos deseos. La imaginación de un
gran poeta, por el contrario, es tan ordenada y amplia
como la de un astrónomo; tiene la paciencia del natura-
lista, el amor del naturalista por el detalles y la vista en-
trenada para ver gradaciones sutiles y líneas esenciales;
no conoce la prisa; carece de afectación y de sentido de la
originalidad; encuentra sus efectos en su tema, en su
mundo inevitable. Se asemeja al naturalista en todo esto,
pero difiere de él en el equilibrio de sus intereses: el poeta
tiene una mente más concreta, su mundo visible viste to-
dos los colores y retiene su vida y pasión propias. En lugar
de estudiar en la experiencia sus elementos calculables, el poeta estu-
dia sus valores morales, su belleza, lasposibilidades que ofrece para el
alma; y el cosmos que construye es por tanto un teatro ideal para
el espíritu, en donde su drama potencial más noble es representadoy
su destino resuelto.

Jorge Santayana (1863-1952), «Los elemen-


tos y la función de la poesía», en Interpretaciones
de poesía y religión, trad. de Carmen García Trevi-
jano y Susana Nuccetelli, cap, X , Madrid, Cáte-
dra, Col. Teorema, 1993,
COMENTARIO

Kant pensaba que la construcción de la obra de arte era


un puro juego de la imaginación sin ningún valor de co-
nocimiento. Una cosa es imaginar que una nube tiene i
forma de caballo y otra percibir un caballo real. Hegel
sostenía, por el contrario, que la obra de arte debe trans-
mitir un mensaje moral relativo al destino de la vida hu- >
mana. Afirmaba que a través de la obra de arte se expresa I
el Espíritu Absoluto. Esta contraposición de puntos de i
vista sobre la naturaleza de la obra de arte ha tenido va- J
rias veces lugar en la historia de las ideas estéticas y de la 3
literatura.
El filósofo español Jorge Santayana se inclina en este
texto por la teoría de la obra de arte como mensaje moral,

324
n
La obra de arte supone para él, al igual que la obra cientí-
fica, un análisis de la realidad y una reconstrucción e in-
terpretación subsiguientes de esa realidad previamente
analizada. Pero el hombre de ciencia lleva a cabo su cons-
trucción utilizando sólo conceptos abstractos, con los
cuales formula leyes asimismo abstractas. En cambio el
poeta pone en la construcción de su obra el genio de la
imaginación. Después de reducir por vía de análisis,
como el hombre de ciencia, el mundo a pedazos, recom-
pone luego estos fragmentos «acercándolos más al deseo
del corazón». Este «análisis con vistas a la creación» es
para Santayana «la esencia de toda gran poesía». El gran
poeta debe ser en el uso de su imaginación tan ordenado
como un astrónomo y tan paciente como un naturalista.
Pero su diferencia con respecto al científico está en que
«en lugar de estudiar en la experiencia sus elementos cal-
culables, investiga sus valores morales, su belleza». El re-
sultado es una imagen del mundo basada en el estudio de
los caracteres de los personajes y de su entorno físico y so-
cial y plasmada en una trama que capta el interés de
quien lo contempla. El cosmos que construye el poeta es
«un teatro ideal para el espíritu en el que cobra realidad
su más noble drama potencia) y es resuelto su des-
tino».
La gran obra literaria de Homero, de Dante o de Goet-
he, la poesía romántica de Shelley, Wordsworth y Cole-
ridge, se ajustan al modelo de literatura definido en este
pasaje por Santayana. Pero las obras más importantes de
nuestro siglo, como Uiises de James Joyce o A la busca del
tiempo perdido de Marcel Proust, se ajustan más bien al mo-
delo de obra de arte defendido por Kant como pura com-
binación estructural de elementos imaginativos ausente
de mensaje moral o cognitivo. Decidir entre Mozart y
Beethoven, o entre Picasso y Van Gogh, es, en última
instancia, decidir entre modelos que son inconmensu-
rables.

325
PARA SABER MÁS

GARRIDO, Manuel, «Introducción a iaobra de Santayana», Inter-


pretaciones de poesía y religión, Madrid, Cátedra, Colección Teo-
rema, 1993.
SANTAYANA, George, Interpretaciones de poesía y religión, Madrid,
Cátedra, Col. Teorema, 1993.
B E A R D S L E Y , M. C., y H O S P E R S , J., Estética: Historia y fundamentos,
trad. Román de la Calleja, Madrid, Cátedra, Colección Teo-
rema, 1984.
O R T E G A Y G A S S E T , ]., La deshumanización del arte, Madrid, Colec-
ción El Arquero, Revista de Occidente, 1976.

5. Unamuno: La íntra-historia

Las olas de la Historia, con su rumor y su espuma que


reverbera al sol, ruedan sobre un mar continuo, hondo,
inmensamente más hondo que la capa que ondula sobre
un mar silencioso y a cuyo último fondo nunca llega el
sol. Todo lo que cuentan a diario los periódicos, la histo-
ria toda del «presente momento histórico», no es sino la
superficie del mar, una superficie que se hiela y cristaliza
en los libros y registros, y, una vez cristalizada así, una
capa dura, no mayor con respecto a la vida int ra-historie a
que esta pobre corteza en que vivimos con relación al in-
menso foco ardiente que lleva dentro. Los periódicos
nada dicen de la vida silenciosa de los millones de hom-
bres sin historia que a todas horas del día y en todos los
países del globo se levantan a una orden del sol y van a
sus campos a proseguir la oscura y silenciosa labor coti-
diana y eterna, esa labor que como la de las madréporas
suboceánicas echa las bases sobre que se alzan los islotes
de la Historia. Sobre el silencio augusto, decía, se apoya y
vive el sonido; sobre la inmensa Humanidad silenciosa se
levantan los que meten bulla en la Historia. Esa vida in-

326
tra-histórica, silenciosa y continua como el fondo mismo
del mar, es la sustancia del progreso, la verdadera tradi-
ción, la tradición eterna, no la tradición mentida que se
suele ir a buscar al pasado enterrado en libros y papeles y
monumentos y piedras.

Miguel de Unamuno (1864-1936), En tomo al


casticismo, I, «La tradición eterna», Madrid,
Aguilar.

COMENTARIO

Sobre los factores que determinan el curso de la histo-


ria hay varias teorías. La más extendida académicamente
sostiene que las decisiones de los gobernantes, generales y
políticos constituyen el verdadero motor de la historia.
En esa misma teoría creen también los hombres públicos.
Marx pensaba que son los conflictos de clase y las condi-
ciones económicas los principales factores que mueven la
historia. Unamuno sugiere aquí que «todo lo que cuentan
a diario los periódicos», que viene a coincidir con la teo-
ría académica y positivista de la historia, «no es sino la su-
perficie del mar, una superficie que se hiela y cristaliza en
los libros y registros». Los periódicos y manuales olvidan
«la vida silenciosa de los millones de hombres sin historia
que a todas horas del día y en todos los países del globo se
levantan a una orden del sol y van a sus campos a prose-
guir la oscura y silenciosa labor cotidiana y «eterna», so-
bre la cual «se alzan los islotes de la historia». Eso es lo
que llama Unamuno la «intra-historia», que es a la histo-
ria oficial como el fondo del mar a su superficie.
En la medida en que esta teoría minimiza el alcance de
las acciones y decisiones de políticos y gobernantes, tiene
una dimensión de nihilismo o de anarquismo que Una-
muno comparte con Tolstoi, cuya novela Guerray paz fue
escrita con la intención de demostrar que no fueron los
generales sino los soldados los protagonistas de las bata-
llas, y que no fueron los hombres públicos sino el pueblo
ruso quien derrotó a Napoleón.

327
Al superficial progresismo y a la estéril erudición sobre
el pasado de los historiadores positivistas opone Unamu-
no esa «vida intra-histórica», a la que sitúa en un nivel
más profundo que la lucha de clases marxista y a la que
considera como «la sustancia del progreso, la verdadera
tradición, la tradición eterna, no «la tradición mentida
que se suele ir a buscar al pasado enterrado en libros y pa-
peles y monumentos y piedras». Lo que tiene de más pro-
pio y castizo un pueblo no se obtiene con la mera erudi-
ción académica de su pasado histórico, sino buceando en
esa eterna tradición que es su intra-historia.

PARA SABER MÁS

HEGEL, G. W, F., Introducción General e Introducción Espe-


cial a las «Lecciones sobre la filosofía de la historia univer-
sal» (págs. 39-150 y 153-212, respectivamente), trad, José
Gaos, Madrid, Revista de Occidente.
UNAMUNO, Miguel de, Paz en la guerra, Madrid, Espasa-Calpe,
Colección Austral, 1980.

6. Ortega: Teoría de la vida:


el yo y la circunstancia

El animal, lo mismo en esto que la piedra, cuando em-


pieza a existir tiene ya todo su ser dado y fijo, tiene cuanto
ha menester para ser eso que es —granito o ruiseñor.
Suspendida en el aire, la piedra sabe sin saberlo y sin ha-
cerse de ello cuestión la línea que recorrerá para caer ha-
cia dentro de la tierra, y el ruiseñor sabe, ignorándolo,
que en abril empezará a cantar y en las cuerdas de su me-
nuda laringe tiene ya desde que nace la melodía de su
canto, que tanto nos encanta, como en el papel pautado
está inscrita la romanza sin palabras. El animal trae en su
cuerpo montado el maravilloso sistema de sus instintos,

328
r
el cual es doble: es un sistema de apetitos y un sistema de
mecánicos comportamientos para satisfacer esos apetitos.
De aquí que para el animal como para la piedra existir sig-
nifica simplemente dejarse ser. Pero el hombre no es su
cuerpo, que es una cosa, ni su alma, que es también una
cosa, una sutil cosa: el hombre no es en absoluto una
cosa, sino un drama; su vida. Y es ésta un drama porque
de lo que se trata en toda humana existencia es de cómo
un ente que llamamos yo, que es nuestra individual perso-
na y que consiste en un ha2 de proyectos para ser, de aspi-
raciones, en un programa de vida —acaso siempre impo-
sible— pugna por realizarse en un elemento extraño a él,
en loque I lamo la circunstancia. Esta circunstancia es siem-
pre un aquí y un ahora inexorables. Tenemos que salir
nadando vitalmente en un lugar determinado del planeta
y en un contorno social formado por los otros hombres,
el cual es distinto en cada fecha.

Todas esas cosas son —reitero— meros utensilios con


que tenemos que hacernos nuestra vida. Pero quien, ha-
blando con rigor, vive no es nada de eso q u c j a está ahí,
que ya es —mi cuerpo y mi alma son ya en todo instante
lo que son—, el que vive soy yo y yo soy el que tiene que
realizarse en su vida, el que tiene que ser, no en el ahora
que ya está aquí, que ya es sino en el mañana. Nuestro yo
es siempre un futuro, un porvenir inmediato y remoto
que hay que lograr y asegurar; en suma, el yo de cada uno
de nosotros es ese ente extraño que, en nuestra íntima y
secreta conciencia, sabe cada uno de nosotros que tiene
que ser. Esa íntima conciencia constantemente nos dice
quién es ese que tenemos que ser, esa persona o personaje
que tenemos que esforzarnos en realizar, y nos lo dice con
una misteriosa voz interior que habla y no suena, una voz
silente que no necesita palabras, que es, por rara condi-
ción, a un tiempo monólogo y diálogo, voz que, como un
hilo de agua, asciende en nosotros ele un hontanar pro-
fundo, que nos susurra el mandamiento de Píndaro: «lle-
ga a ser el que eres»; una voz que es llamada hacia nuestro
más auténtico destino; en suma, la voz de la vocación, de

329
la personal vocación. El yo auténtico de cada hombre es
su vocación.

José Onega y Gasset (1883-1955), «Juan Luis


Vives y su mundo», en Vives- Goelbe, Madrid,
Revista de Occidente, Colección El Arquero,

COMENTARIO

La filosofía ha considerado tradicionalmente al mun-


do como un conjunto de cosas reducibles a tipos y al
hombre como una cosa entre las cosas del tipo llamado
sustancia. La filosofía de la vida que emerge en la primera
mitad de nuestro siglo, inspirada en Dilthey, Nietzsche y
Scheler y ejemplarmente representada por Ortega, descri-
be al hombre como un ser que no es una cosa entre las co-
sas del mundo pero que ha de bregar con ellas para reali-
zar su vida. Ortega sintetizó el drama de la vida humana
en la fórmula: «yo y mi circunstancia».
En este pasaje contrasta con el ser mecánicamente de-
terminado de las cosas del mundo, como la piedra o el
animal, el ser problemático y libre que es la existencia del
hombre, A continuación precisa Ortega que «el hombre
no es su cuerpo, que es una cosa, ni su alma, que es tam-
bién una cosa, una sutil cosa... sino un drama: su vida».
El protagonista de este drama es nuestro «yo», «que es
nuestra individual persona y que consiste en un haz de
proyectos para ser... en un programa de vida». El entorno
geográfico e histórico que circunda al yo es lo que Ortega
llamó «la circunstancia»: «un aquí y un ahora inexo-
rables».
Evocando la llamada de Píndaro «llega a ser el que
eres», Ortega afirma que mi yo no es mi cuerpo ni mi
alma, sino una misteriosa y secreta tendencia que es a la
vez mi conciencia, mi vocación y mi destino. Para reali-
zar nuestro destino hemos de configurar nuestra circuns-
tancia, El hombre es libre de realizar o no ese destino,
pero sólo realiza su yo auténtico en la medida en que es
fiel a su vocación.

330
Cuando Ortega habla de vida se refiere sobre todo a la
vida humana. El invitaba a distinguir entre la palabra
griega dsoée o vida zoológica, que es mero mecanismo ins-
tintivo, y la palabra bios, que es la vida libre del ser huma-
no. El análisis de la vida de un hombre es, consecuente-
mente, su «biografía», que es la historia de la lucha,
fructuosa o infructuosa, de su yo o vocación con su cir-
cunstancia.

PARA SABER MÁS

ORTEGA Y GASSET, José, Vives-Goethe, Madrid, Revista de Occi-


dente, Colección El Arquero, 1973.
— «Guillermo Dilthey y la idea de la vida», ensayo contenido
en Kant, Hegel, Dilthey, Madrid, Revista de Occidente, Colec-
ción El Arquero, 1973,
— En torno a Galilea, Madrid, Revista de Occidente, Colección
El Arquero, 1976.

331
A filosofía tiene una historia. Esta historia está viva

L y se asienta sobre un proyecto común: la bús-


queda de un acuerdo entre los espíritus. Es bas-
tante natural que esta intención engendre divergencias.
Las querellas de las escuelas, los enfrenamientos en-
tre los sistemas son otras tantas manifestaciones de la
vitalidad de esta disciplina. La ambición de la presente
compilación es la de sugerir las múltiples direcciones
en las que se bifurca la filosofía. El principio que ha
dirigido los textos ha sido, pues, el de la oposición o, al
menos, el de la discusión. Cada texto entra en polémi-
ca explícita o implícita con un filósofo o una corriente
de pensamiento anterior o contemporánea. Los comen-
tarios propuestos' se orientan fundamentalmente a cap-
tar de nuevo la atención de un texto y a restituir su mo-
vimiento argumentativo. En la medida en que requiere
. la colaboración de los estudiantes, esta obra es ade-
más un instrumento de trabajo. Esta edición incluye en
Apéndice los textos y comentarios de seis filósofos es-
pañoles: Séneca, Luis Vives, Francisco Suárez, Jorge
Santayana, Miguel de Unamuno y José Ortega y Gasset.

(lili
ISBN 84-376-1170-9

0111057
9 l 788437 ll 611 709'

Potrebbero piacerti anche